Constitutional Law 2 Cases

  • Uploaded by: Feliz Xhanea Canoy
  • 0
  • 0
  • February 2021
  • PDF

This document was uploaded by user and they confirmed that they have the permission to share it. If you are author or own the copyright of this book, please report to us by using this DMCA report form. Report DMCA


Overview

Download & View Constitutional Law 2 Cases as PDF for free.

More details

  • Words: 165,185
  • Pages: 477
Loading documents preview...
RONA MAE O CANOY Constitutional Law 2 Case Digests ATTY. GONZALO D. MALIG-ON JR.

Second Semester Sat. 8:00 AM to 11:00 AM

POLICE POWER Who can exercise? MMDA vs. Garin, GR No. 130239, April 15, 2005 FACTS: The respondent was issued a traffic violation receipt (TVR) and his driver’s license was confiscated for parking illegally. Garin wrote to then MMDA Chairman Prospero Oreta requesting the return of his license and expressed his preference for his case to be filed in Court. Without an immediate reply from the chairman, Garin filed for a preliminary injunction assailing among others that Sec 5 (f) of RA 7924 violates the constitutional prohibition against undue delegation of legislative authority, allowing MMDA to fix and impose unspecified and unlimited fines and penalties. RTC rule in his favor, directing MMDA to return his license and for the authority to desist from confiscating driver’s license without first giving the driver the opportunity to be heard in an appropriate proceeding. Thus, this petition. ISSUE: “Whether or not Sec 5(f) of RA 7924 which authorizes MMDA to confiscate and suspend or revoke driver’s license in the enforcement of traffic rules and regulations constitutional?” HELD: The MMDA is not vested with police power. It was concluded that MMDA is not a local government unit of a public corporation endowed with legislative power and it has no power to enact ordinances for the welfare of the community. Police power, as an inherent attribute of sovereignty is the power vested in the legislature to make, ordain, establish all manner of wholesome and reasonable laws, statutes and ordinances either with penalties of without, not repugnant to the constitution, as they shall judge to be for good and welfare of the commonwealth and for subjects of the same. There is no provision in RA 7924 CONSTITUTIONAL LAW 2 CASE DIGESTS

that empowers MMDA or its council to “enact ordinance, approve resolutions and appropriate funds for the general welfare of the inhabitants of Metro Manila.” It is an agency created for the purpose of laying down policies and cooradinating with the various national government agencies, People’s Organizations, NGOs and private sector for the efficient and expeditious delivery of services. All its functions are administrative in nature. Hierarchy of Rights Social Justice Society, et al vs Atienza, Jr (G.R. No. 156052 March 7, 2007) Facts: On November 20, 2001, The SangguniangPanglunsod of Maynila enacted Ordinance No. 8027. Hon. Jose L. Atienza, jr. approved the said ordinance on November 28, 2001. and it became effective on December 28, 2001. Ordinance No. 8027 reclassified the area of Pandacan and Sta. Ana from industrial to commercial and directed the owners and operators of businesses disallowed under Section 1 to cease and desist from operating their businesses within six months from the date of effectivity of the ordinance. Among the businesses situated in the area are the so-called Pandacan Terminals of the oil companies Caltex, Petron and Shell. However, on June 26, 2002, the City of Manila and the Department of Energy entered into a memorandum of understanding with the oil companies in which they agreed that :scaling down of Pandacan Terminals was the most viable and practicable option. Under the memorandum of understanding, the City of Manila and the Department of Energy permits the Oil Companies to continuously operate in compliance with legal requirements, within the limited area resulting from the joint operations and the scale down program. The SangguniangPanlungsod ratified the memorandum of understanding in Resolution No. 97. In that resolution, the Sanggunian declared that the memorandum of understanding was effective only for a period of six months starting July 25, 2002. Thereafter, on January 30, 2003, the Sanggunian adopted Resolution No. 13 extending the validity of Resolution No. 97 to April 30, 2003 and authorizing Mayor Atienza to issue special business permits to the oil companies. Resolution No. 13, s. 2003 also called for a reassessment of the ordinance. CONSTITUTIONAL LAW 2 CASE DIGESTS

Issue: Whether or not respondent has the mandatory legal duty to enforce Ordinance No. 8027 and order the removal of the Pandacan Terminals. And Whether or not the June 26, 2002 memorandum of understanding and the resolutions ratifying it can amend or repeal Ordinance No. 8027. Held: The Local Government Code imposes upon respondent the duty, as City Mayor of Manila, to enforce all laws and ordinances relative to the governance of the city. One of these is Ordinance No. 8027. As the chief executive of the city, he has the duty to put into effect Ordinance No. 8027 as long as it has not been repealed by the Sanggunian or negated by the courts. On the other hand assuming that the terms of the memorandum of understanding were contradictory with Ordinance No. 8027, the resolutions which ratified it and made it binding on the City of Manila expressly gave it full force and effect only until April 30, 2003. There is nothing that legally hinders respondent from enforcing Ordinance No. 8027. Wherefore the Court Ordered Hon. Jose L. Atienza, Jr., as mayor of the city of Manila to immediately enforce Ordinance No. 8027. DIOCESE OF BACOLOD vs. COMELEC (Gr. NO. 205728, January 21,2015) FACTS: On February 2013, petitioners posted two (2) tarpaulins within the compound of San Sebastian Cathedral of Bacolod. Each tarpaulin was approximately 6×10 in size. They were posted on the front walls of the cathedral within public view. The first tarpaulin contains the message “IBASURA RH Law” referring to the Reproductive Health Law of 2012 or Republic Act No. 10354. The second tarpaulin is the subject of the present case. This tarpaulin contains the heading “Conscience Vote” and lists candidates as either “(Anti-RH)/ Team Buhay” or “(Pro-RH)/Team Patay”. The electoral candidates were classified according to their vote on the adoption of the RH Law. Those who voted for the passing of the law were CONSTITUTIONAL LAW 2 CASE DIGESTS

classified by petitioners as comprising “Team Patay,” while those who voted against it form “Team Buhay”: TEAM BUHAY TEAM PATAY Estrada, JV Angara, Juan Edgardo Honasan, Gregorio Casiño, Teddy Magsaysay, Mitos Cayetano, Alan Peter Pimentel, Koko Enrile, Jackie Trillanes, Antonio Escudero, Francis Villar, Cynthia Hontiveros, Risa *Party List Legarda, Loren Party List Buhay Gabriela, Akbayan, Bayan Muna, AnakPawis AngPamilya

Party List

Respondent Atty. Mavil V. Majarucon, as Election Officer of Bacolod City, issued a Notice to Remove Campaign Materials addressed to petitioner Most Rev. Bishop Vicente M. Navarra, otherwise, COMELEC will be constrained to file an election offense against the petitioners. ISSUE: Whether the act of the COMELEC infringes the Freedom of Religion and Freedom of Speech. HELD: On Freedom of Religion.As aptly argued by COMELEC, the tarpaulin, on its face, “does not convey any religious doctrine of the Catholic church.” That the position of the Catholic church appears to coincide with the message of the tarpaulin regarding the RH Law does not, by itself, bring the expression within the ambit of religious speech. On the contrary, the tarpaulin clearly refers to candidates classified under “Team Patay” and “Team Buhay” according to their respective votes on the RH Law. On Freedom of Speech. Embedded in the tarpaulin, are opinions expressed by petitioners. It is a specie of expression protected by our fundamental law. There are several theories and schools of thought that strengthen the need to protect the basic right to freedom of expression. CONSTITUTIONAL LAW 2 CASE DIGESTS

First, this relates to the right of the people to participate in public affairs, including the right to criticize government actions. Speech that promotes dialogue on public affairs, or airs out grievances and political discontent, should thus be protected and encouraged. Second, free speech should be encouraged under the concept of a market place of ideas. Third, free speech involves self-expression that enhances human dignity. Fourth, expression is a marker for group identity. Fifth, the Bill of Rights, free speech included, is supposed to “protect individuals and minorities against majoritarian abuses perpetrated through [the] framework [of democratic governance]. ” Lastly, free speech must be protected under the safety valve theory. In order to avoid this situation and prevent people from resorting to violence, there is a need for peaceful methods in making passionate dissent. Free speech must, thus, be protected as a peaceful means of achieving one’s goal, considering the possibility that repression of nonviolent dissent may spill over to violent means just to drive a point. In the hierarchy of civil liberties, the rights of free expression and of assembly occupy a preferred position as they are essential to the preservation and vitality of our civil and political institutions; and such priority “gives these liberties the sanctity and the sanction not permitting dubious intrusions.” PBM Employees Org. VS PBN Co. 51 SCRA 189 FACTS: Petitioners herein alleged that they informed the respondent Philippine Blooming Mills of their decision to have a mass demonstration at Malacañang, in protest against alleged abuses of the Pasig police. The company respondent pleaded to exclude the employees in the first shift to join the mass demonstration, however the petitioners still included them. As a result, the company respondent filed a case thru the city prosecutor and charged the demonstrating employees of violation of the CBA. Trial court rendered judgment in favor of the respondent company, and the petitioners failed to file a timely motion for reconsideration. CONSTITUTIONAL LAW 2 CASE DIGESTS

ISSUE: WON the case dismissal as a consequence of a procedural fault violates due process. HELD: Yes. The decision of the CIR to dismiss the petition based on technicality (being 2 days late) was rendered null and void. (The constitutional rights have dominance over procedural rules.) And, the company was directed to reinstate the eight officers with full backpay from date of separation minus the one day's pay and whatever earnings they might have realized from other sources during their separation from service. (The removal from employment of the officers were deemed too harsh a punishment for their actions) Test of Police Power Balacuit v. CFI, G.R. No. L-38429, June 30, 1988 FACTS: The Municipal Board of City of Butuan passed Ordinance No 640 on 21 April 1969, “penalizing any person, group of persons, entity or engaged in the business of selling admission tickets to any movie to require children between 7-12 years of age to pay full payment for ticket should only be charged one half.” Petitioners Carlos Balacuit and others as managers of theaters assailed the validity and constitutionality of the said ordinance. The court adjudged in favor of the respondents hence the petition for review. Petitioners contend that it violates due process clause of the Constitution for being oppressive, unfair, unjust, confiscatory and an undue restraint of trade. ISSUE: “Whether or not Ordinance 640 — prohibiting selling of theater admission tickets to children 7-12 years old at full price is constitutional or not?” HELD: CONSTITUTIONAL LAW 2 CASE DIGESTS

Yes, Ordinance 640 is not constitutional. For the assailed ordinance be held constitutional it must pass the test of police power. To invoke the exercise the police power, it must be for the interest of the public without interfering with private rights and adoptive means must be reasonably necessary for the accomplishment of the purpose and not unduly oppressive upon individuals. While it is true that a business may be regulated, it is equally true that such regulation must be within the bounds of reason, that is, the regulatory ordinance must be reasonable, and its provisions cannot be oppressive amounting to an arbitrary interference with the business or calling subject of regulation. The right of the owner to fix a price at which his property shall be sold or used is an inherent attribute of the property itself and, as such, within the protection of the due process clause. Hence, the proprietors of a theater have a right to manage their property in their own way, to fix what prices of admission they think most for their own advantage, and that any person who did not approve could stay away. Lozano vs. Matinez, 146 SCRA 323 (1986) FACTS: The defendants were prosecuted under Batas Pambansa Bilang 22 (Bouncing Check Law). The defendants contended that statute is unconstitutional being contrary Constitutional provision forbidding imprisonment for debt. BP 22 punishes a person “who makes or draws and issues any check on account or for value, knowing at the time of issue that he does not have sufficient funds in or credit with the drawer bank for the payment of said check in full upon presentment, which check is subsequently dishonored by the drawer bank for insufficiency of funds or credit or would have been dishonored for the same reason had not the drawer, without any valid reason, ordered the bank to stop payment.” RTC denied their motions apart from one declaring it unconstitutional hence filed the petition for relief. ISSUE: “Whether or not Batas Pambansa Bilang 22 (Bouncing Check Law) is unconstitutional?” HELD: Yes, BP 22 constitutional. The law is intended not to coerce the debtor to pay the debt. The offense being punished is the act of making and issuing a worthless CONSTITUTIONAL LAW 2 CASE DIGESTS

check that is dishonored upon presentation. Putting them into circulation impairs public interest. The law punishes the act not as an offense against property but against public order. It is sufficient to have reasonable connection between means and end. The facts leading to the adoption of BP 22 is cloaked with public concern. Flooding the system with worthless checks will pollute the channels of trade and commerce, injure the banking system and eventually hurt the welfare of society and public interest. Carlos Superdrug Corporation vs. DSWD, et al., FACTS: The petitioners are domestic corporations and proprietors operating drugstores in the Philippines. Petitioners assail the constitutionality of Section 4(a) of R.A. 9257, otherwise known as the “Expanded Senior Citizens Act of 2003.” Section 4(a) of R.A. 9257 grants twenty percent (20%) discount as privileges for the Senior Citizens. The provision under R.A. No. 9257, on the other hand, provides that the establishment concerned may claim the discounts under Section 4(a), (f), (g) and (h) as tax deduction from gross income, based on the net cost of goods sold or services rendered. Under this scheme, the establishment concerned is allowed to deduct from gross income, in computing for its tax liability, the amount of discounts granted to senior citizens. Effectively, the government loses in terms of foregone revenues an amount equivalent to the marginal tax rate the said establishment is liable to pay the government. This will be an amount equivalent to 32% of the twenty percent (20%) discounts so granted. The establishment shoulders the remaining portion of the granted discounts. Petitioner contends that said law is unconstitutional because it constitutes deprivation of private property. Compelling drugstore owners and establishments to grant the discount will result in a loss of profit and capital 1. Drugstores impose a mark-up of only 5% to 10% on branded medicines; and 2. The law failed to provide a scheme whereby drugstores will be justly compensated for the discount. ISSUE: “Whether or not R.A. 9257 is unconstitutional” HELD: CONSTITUTIONAL LAW 2 CASE DIGESTS

Yes, R.A. 9257 is constitutional. The law is a legitimate exercise of police power which, similar to the power of eminent domain, has general welfare for its object. Examining petitioners' arguments, it is apparent that what petitioners are ultimately questioning is the validity of the tax deduction scheme as a reimbursement mechanism for the twenty percent (20%) discount that they extend to senior citizens. In treating the discount as a tax deduction, petitioners insist that they will incur losses because, referring to the DOF Opinion, for every P1.00 senior citizen discount that petitioners would give, P0.68 will be shouldered by them as only P0.32 will be refunded by the government by way of a tax deduction. Petitioners' computation is flawed. For purposes of reimbursement, the law states that the cost of the discount shall be deducted from gross income, the amount of income derived from all sources before deducting allowable expenses, which will result in net income. Here, the petitioners tried to show a loss on a per transaction basis, which should not be the case. An income statement, showing an accounting of petitioners sales, expenses, and net profit (or loss) for a given period could have accurately reflected the effect of the discount on their income. Absent any financial statement, petitioners cannot substantiate their claim that they will be operating at a loss should they give the discount. In addition, the computation was erroneously based on the assumption that their customers consisted wholly of senior citizens. Lastly, the 32% tax rate is to be imposed on income, not on the amount of the discount. Zoning and Regulatory Ordinances: Ermita-Malate Hotel & Motel Operators v. City Mayor, 20 SCRA 849 (1967) FACTS: The petitioners filed a petition for prohibition against Ordinance No. 4760 for being violative of the due process clause, contending that said ordinance is not only arbitrary, unreasonable or oppressive but also vague, indefinite and uncertain, and likewise allege the invasion of the right to privacy and the guarantee against self-incrimination. Ordinance No. 4760 proposes to check the clandestine harboring of transients and guests of these establishments by requiring these transients and guests to fill up a registration form, prepared for the purpose, in a lobby open to public view at all times, and by introducing several other amendatory provisions calculated to shatter the privacy that characterizes the registration of transients and guests.” Moreover Moreover, the increase in the CONSTITUTIONAL LAW 2 CASE DIGESTS

licensed fees was intended to discourage “establishments of the kind from operating for purpose other than legal” and at the same time, to increase “the income of the city government.” The lower court ruled in favor of the petitioners. Hence, the appeal. ISSUE: “Whether or not Ordinance No. 4760 is unconstitutional” HELD: No, Ordinance No. 4760 is constitutional. The mantle of protection associated with the due process guaranty does not cover petitioners. This particular manifestation of a police power measure being specifically aimed to safeguard public morals is immune from such imputation of nullity resting purely on conjecture and unsupported by anything of substance. To hold otherwise would be to unduly restrict and narrow the scope of police power which has been properly characterized as the most essential, insistent and the least limitable of powers, extending as it does “to all the great public needs.” It would be, to paraphrase another leading decision, to destroy the very purpose of the state if it could be deprived or allowed itself to be deprived of its competence to promote public health, public morals, public safety and the general welfare. Negatively put, police power is that inherent and plenary power in the State which enables it to prohibit all that is hurt full to the comfort, safety, and welfare of society. On the legislative organs of the government, whether national or local, primarily rest the exercise of the police power, which, it cannot be too often emphasized, is the power to prescribe regulations to promote the health, morals, peace, good order, safety and general welfare of the people. In view of the requirements of due process, equal protection and other applicable constitutional guaranties however, the exercise of such police power insofar as it may affect the life, liberty or property of any person is subject to judicial inquiry. Where such exercise of police power may be considered as either capricious, whimsical, unjust or unreasonable, a denial of due process or a violation of any other applicable constitutional guaranty may call for correction by the courts. The Court reversed the judgment of the lower court and lifted the injunction on the Ordinance in question. Social Justice Society Vs Atienza (refer at the top) Cruz vs. Paras, 123 SCRA 569 (1983) CONSTITUTIONAL LAW 2 CASE DIGESTS

FACTS: The Local Government of Bocaue, Bulacan enacted Ordinance No. 82 which sought to prohibit the operation of nightclubs and the employment of hostesses in such night clubs. The Petitioners filed with the Court of First Instance a petition for prohibition with preliminary injunction alleging that (1) the ordinance is null and void as the municipality has no authority to prohibit a lawful business, (2) it violated the petitioners’ right to due process and equal protection of the law as their permits were withdrawn without judicial hearing, and (3) that under Presidential Decree No. 189, as amended, the power to license and regulate tourist-oriented business including night clubs has been transferred to the Department of Tourism. In answer, the municipality responded that (1) it has been authorized by law to prohibit the establishment and operation of night clubs under Section 2238 of the Revised Administrative Code, (2) it was not violative of their rights as property rights are subordinate to public interests because night clubs has been the principal cause of decadence of morality and has adverse effects to the community, and (3) Presidential Decree No. 189, as amended, did not deprive municipal councils to regulate or prohibit night clubs. The Court of First Instance upheld the constitutionality of the Ordinance. ISSUE: “Whether or not Ordinance No. 84 as enacted is a valid exercise of police power by the local government unit.” RULING: The Court ruled in favor of the petitioners. According to the Court, police power is granted to municipal corporations, which may enact such ordinances and make regulations as may be necessary to carry out its powers and duties to provide for the health and safety, promote the prosperity, improve the morals, peace, good order and convenience of the municipality. However, citing Justice Moreland, an ordinance is valid unless contravenes the fundamental law of the land, an act of national legislature, or unless it is against public policy, or is unreasonable, oppressive, discriminating, or in derogation of common right. Hence, an ordinance passed must be a reasonable exercise of the power, or it will be pronounced invalid. The general rule found in the general welfare clause must be reasonable, consonant with the general powers of the corporation, and not inconsistent with the law of the State. In the present case, it is clear that municipal corporations CONSTITUTIONAL LAW 2 CASE DIGESTS

cannot prohibit the operation of night clubs. They may be regulated, but not prevented from carrying on their business. All the petitioners would have to do is to apply once more for licenses to operate night clubs. A refusal to grant licenses, because no such businesses could legally open, would be subject to judicial correction. The purpose sought to be achieved could have been attained by reasonable restrictions rather than by an absolute prohibition. Velasco vs. Villegas, 120 SCRA 568 (1983) FACTS: In their own behalf and in representation of the other owners of barbershops in the City of Manila, petitioners challenge the constitutionality based on Ordinance No. 4964 of the City of Manila, which prohibited the business of massaging customers of a barber shop. They contend that it amounts to a deprivation of property of their means of livelihood without due process of law. ISSUE: “Whether said ordinance was unconstitutional, and therefore an improper exercise of police power” HELD: No. The attack against the validity cannot succeed. As pointed out in the brief of respondent-appellees, it is a police power measure. The objectives behind its enactment are: “(1) To be able to impose payment of the license fee for engaging in the business of massage clinic under Ordinance No. 3659 as amended by Ordinance 4767, an entirely different measure than the ordinance regulating the business of barbershops and, (2) in order to forestall possible immorality which might grow out of the construction of separate rooms for massage of customers.” The Court has been most liberal in sustaining ordinances based on the general welfare clause. As far back as U.S. v. Salaveria, 4 a 1918 decision, this Court through Justice Malcolm made clear the significance and scope of such a clause, which “delegates in statutory form the police power to a municipality. As above stated, this clause has been given wide application by municipal authorities and has in its relation to the particular circumstances of the case been liberally construed by the courts. Such, it is well to really is the progressive view of Philippine jurisprudence.” CONSTITUTIONAL LAW 2 CASE DIGESTS

Magtajas vs. Pryce Properties, 234 SCRA 255 (1994) FACTS: There was instant opposition when PAGCOR announced the opening of a casino in Cagayan de Oro City. Civic organizations angrily denounced the project. The trouble arose when in 1992, flush with its tremendous success in several cities, PAGCOR decided to expand its operations to Cagayan de Oro City. The reaction of the Sangguniang Panlungsod of Cagayan de Oro City was swift and hostile. On December 7, 1992, it enacted Ordinance No. 3353. Nor was this all. On January 4, 1993, it adopted a sterner Ordinance No. 3375-93 Pryce assailed the ordinances before the Court of Appeals, where it was joined by PAGCOR as intervenor and supplemental petitioner. Their challenge succeeded. On March 31, 1993, the Court of Appeals declared the ordinances invalid and issued the writ prayed for to prohibit their enforcement ISSUE: “Whether or not LGU Ordinance 3353 and 3375-93 are constitutional?” HELD: No, the ordinances in question are unconstitutional. Local Government Units are authorized to prevent or suppress, among others, “gambling and other prohibited games of chance.” Obviously, this provision excludes games of chance which are not prohibited but are in fact permitted by law. The rationale of the requirement that the ordinances should not contravene a statute is obvious. Casino gambling is authorized by P.D. 1869. This decree has the status of a statute that cannot be amended or nullified by a mere ordinance. Hence, it was not competent for the Sangguniang Panlungsod of Cagayan de Oro City to enact Ordinance No. 3353 prohibiting the use of buildings for the operation of a casino and Ordinance No. 3375-93 prohibiting the operation of casinos. For all their praiseworthy motives, these ordinances are contrary to P.D. 1869 and the public policy announced therein and are therefore ultra vires and void. City of Manila v. Judge Laguio, G.R. No. 118127, April 12, 2005 FACTS: Manila Ordinance No. 7738, prohibiting the establishment or operation of businesses providing certain forms of amusement, entertainment, services and facilities in the Ermita-Malate area, to include motels and inns, was enacted by CONSTITUTIONAL LAW 2 CASE DIGESTS

herein petitioners contending that the said ordinance is a valid exercise of the police power of the State in order to protect the social and moral welfare of the community. Respondent Malate Tourist Development Corporation (MTDC) assailed the ordinance as an invalid exercise of police power on the grounds that the Local Government Code grants the City Council only with the power to regulate the establishment, operation and maintenance of hotels, motels, inns, pension houses, lodging houses and other similar establishments, but not to prohibit them. ISSUE: "Whether or not Ordinance No. 7783 of the City of Manila is a valid exercise of police power" HELD: No, the ordinance in question is unconstitutional. The assailed ordinance is unreasonable and oppressive. An ordinance which permanently restricts the use of property that it cannot be used for any reasonable purpose goes beyond the regulation and must be recognized as a taking of the property without just compensation. It is an exercise of police power that is violative of the private property rights of individuals.

Ortigas vs. Feati Bank, 94 SCRA 719 FACTS: The plaintiff is engaged in real estate business, developing and selling lots to the public, particularly the Highway Hills Subdivision along EDSA. On March 4, 1952, plaintiff, as vendor, and Augusto Padilla and Natividad Angeles, as vendees, entered into separate agreements of sale on installments over two parcels of land of the Subdivision. On July 19, 1962, the said vendees transferred their rights and interests over the aforesaid lots in favor of one Emma Chavez. Upon completion of payment of the purchase price, the plaintiff executed the corresponding deeds of sale in favor of Emma Chavez. Both the agreements (of sale on installment) and the deeds of sale contained the stipulations or restrictions that: CONSTITUTIONAL LAW 2 CASE DIGESTS

1. The parcel of land shall be used exclusively for residential purposes, and she shall not be entitled to take or remove soil, stones or gravel from it or any other lots belonging to the Seller. 2. All buildings and other improvements (except the fence) which may be constructed at any time, in the said lot must be, (a) of strong materials and properly painted, (b) provided with modern sanitary installations connected either to the public sewer or to an approved septic tank, and (c) shall not be at a distance of less than two (2) meters from its boundary lines. Eventually, said lots were bought by defendant, Lot 5 directly from Chavez and Lot 6 from Republic Flour Mills by deed of exchange, with some restrictions. Plaintiff claims that restriction is for the beautification of the subdivision. Defendant claimed of the commercialization of western part of EDSA. Defendant began constructing a commercial bank building. Plaintiff demanded to stop it, which forced him to file a case, which was later dismissed, upholding police power. Motion for recon was denied, hence the appeal. ISSUE: “Whether or Not non-impairment clause violated and if the resolution is a valid exercise of police power” HELD: No, the non-impairment clause is not violated and the resolution is a valid exercise of police power. EDSA, a main traffic artery which runs through several cities and municipalities in the Metro Manila area, supports an endless stream of traffic and the resulting activity, noise and pollution are hardly conducive to the health, safety or welfare of the residents in its route. Health, safety, peace, good order and general welfare of the people in the locality are justifications for this. It should be stressed, that while non-impairment of contracts is constitutionally guaranteed, the rule is not absolute, since it has to be reconciled with the legitimate exercise of police power. Presley vs. Bel-Air Village Association FACTS: A complaint for specific performance and damages with preliminary injunction was filed by plaintiff-appellees, Bel-Air Village Association, Inc. (BAVA for short) CONSTITUTIONAL LAW 2 CASE DIGESTS

against Teofilo Almendras and Rollo Almendras (now both deceased and substituted by defendant-appellant Enedina Presley) for violation of the Deed Restrictions of Bel-Air Subdivision that the subject house and lot shall be used only for residential and not for commercial purposes and for non-payment of association dues to plaintiff BAVA amounting to P3,803.55. Presley, as lessee of the property, is the owner and operator of 'Hot Pan de Sal Store' located in the same address. At the time the Almendrases bought their property in question from Makati Development Corporation, the Deed Restrictions was already annotated in their title providing 'that the lot must be used only for residential purpose'. When BAVA came to know of the existence of the 'Pan de sal' store, it sent a letter to the defendants asking them to desist from operating the store. Under the existing Deed Restrictions aforesaid, the entire Bel-Air Subdivision is classified as a purely residential area, particularly Jupiter Road which is owned by and registered in the name of BAVA. During the pendency of the case with this Court, petitioner Enedina Fox Presley died on January 4, 1991. She was substituted by her two daughters as heirs, namely Olivia V. Pizzaro and Consuelo V. Lacson. The issues raised in the instant petition have already been dealt with in the consolidated cases decided by this Court promulgated on December 22, 1988, entitled Sangalang Doctrine. ISSUES: “Whether or not the Sangalang Doctrine is still applicable in the case at bar and whether or not it is a valid exercise of police power” HELD: Yes, it is a valid exercise of police power. The Court has carefully examined the pleadings but have found no reason to reconsider the Sangalang doctrine. In assailing the Court's decision, the private respondent has come out with mere assertions and allegations. It failed to present any proofs or convincing arguments to substantiate its claim that Jupiter Street is still classified as a residential zone. (Filinvest v. Court of Appeals, 182 SCRA 664 [1990]) No new zoning reclassification, ordinance, certification to the effect or jurisprudence for that matter was brought to the attention of this Court which would necessarily compel the court to take a second look at the Sangalang Case. The Court cannot reverse a precedent and rule favorably for the private respondent on the strength of mere inferences. The respondent court in the case at bar was not at all entirely wrong in upholding the Deed of Restrictions annotated in the title of the petitioners. It held CONSTITUTIONAL LAW 2 CASE DIGESTS

that the provisions of the Deed of Restrictions are in the nature of contractual obligations freely entered into by the parties. Undoubtedly, they are valid and can be enforced against the petitioner. However, these contractual stipulations on the use of the land even if said conditions are annotated on the Torrens title can be impaired if necessary to reconcile with the legitimate exercise of police power. (Ortigas & Co. Limited Partnership v. Feati Bank and Trust Co., 94 SCRA 533 [1979]). Administrative Rules and Regulations Knights of Rizal vs. Torre de Manila FACTS: DMCI started construction of Torre De Manila Condominium, after it was issued Building permit by the City Of Manila Office allowing it to build a 49 Storey with Basement & 2 penthouse Level Residential Condominium. However, the City Council of Manila issued Resolution No. 121 enjoining the Office of the Building Official to temporarily suspend the Building Permit of DMC citing among others, that “the Torre de Manila Condominium, based on their development plans, upon completion, will rise up high above the back of the national monument, to clearly dwarf the statue of our hero, and with such towering heights, would certainly ruin the line of sight of the Rizal Shrine from the frontal Roxas Boulevard vantage point.” Building Official Melvin Q. Balagot then sought the opinion of the City of Manila’s City Legal Officer on whether he is bound to comply with Resolution No. 121. In his letter dated 12 September 2012, City Legal Officer Renato G. Dela Cruz stated that there is “no legal justification for the temporary suspension of the Building Permit issued in favor of [DMCI-PDI]” since the construction “lies outside the Luneta Park” and is “simply too far to be a repulsive distraction or have an objectionable effect on the artistic and historical significance” of the Rizal Monument. He also pointed out that “there is no showing that the [area of] subject property has been officially declared as an anthropological or archaeological area. Neither has it been categorically designate. National Historical Commission of the Philippines Dr. Maria Serena I. Diokno maintained that the Torre de Manila project site is outside the boundaries of the Rizal Park and well to the rear of the Rizal Monument, and thus, cannot possibly obstruct the frontal view of the National Monument. On 26 November 2013, following an online petition against the Torre de Manila project that garnered about 7,800 CONSTITUTIONAL LAW 2 CASE DIGESTS

signatures, the City Council of Manila issued Resolution No. 146, reiterating its directive in Resolution No. 121 enjoining the City of Manila’s building officials to temporarily suspend DMCIPDI’s Building Permit. Manila Zoning Board of Adjustments and Appeals (MZBAA) issued Zoning Board Resolution No. 06, Series of 2013, recommending the approval of DMCI-PDI’s application for variance, which was later on amended. The City Council resolution later states that “the City Council of Manila find[s] no cogent reason to deny and/or reverse the aforesaid recommendation of the MZBAA and hereby ratifies and confirms all previously issued permits, licenses and approvals issued by the City Council of Manila for Torre de Manila.” On 12 September 2014, the Knights Of Rizal, a “civic, patriotic, cultural, non- partisan, non-sectarian and non-profit organization”18 created under Republic Act No. 646,19 filed a Petition for Injunction seeking a temporary restraining order, and later a permanent injunction, against the construction of DMCI- PDI’s Torre de Manila condominium project. The KOR argues that the subject matter of the present suit is one of “transcendental importance, paramount public interest, of overarching significance to society, or with far reaching implication” involving the desecration of the Rizal Monument. ISSUES: Whether or not the Court can issue a writ of mandamus against the officials of the City of Manila to stop the construction of DMCI-PDI’s Torre de Manila Project; and whether or not Torre De Manila is a nuisance per se. HELD: NO. There is no law prohibiting the construction of the Torre de Manila. In Manila Electric Company v. Public Service Commission, the Court held that “what is not expressly or impliedly prohibited by law may be done, except when the act is contrary to morals, customs and public order.” This principle is fundamental in a democratic society, to protect the weak against the strong, the minority against the majority, and the individual citizen against the government. In essence, this principle, which is the foundation of a civilized society under the rule of law, prescribes that the freedom to act can be curtailed only through law. Without this principle, the rights, freedoms, and civil liberties of citizens can be arbitrarily and whimsically trampled upon by the shifting passions of those who can shout the loudest, or those who can gather the biggest crowd or the most number of Internet trolls.

CONSTITUTIONAL LAW 2 CASE DIGESTS

Mandamus does not lie against the City of Manila. The Constitution states that “no person shall be deprived of life, liberty or property without due process of law x x x.”61 It is a fundamental principle that no property shall be taken away from an individual without due process, whether substantive or procedural. The is possession of property, or in this case the stoppage of the construction of a building in one’s own property, would violate substantive due process. Torre de Manila is Not a Nuisance Per Se. Article 694 of the Civil Code defines a nuisance as any act, omission, establishment, business, condition of property, or anything else which: (1) injures or endangers the health or safety of others; (2) annoys or offends the senses; (3) shocks, defies or disregards decency or morality; (4) obstructs or interferes with the free passage of any public highway or street, or any body of water; or (5) hinders or impairs the use of property. It can easily be gleaned that the Torre de Manila is not a nuisance per se. The Torre de Manila project cannot be considered as a “direct menace to public health or safety.” Not only is a condominium project commonplace in the City of Manila, DMCI-PDI has, according to the proper government agencies, complied with health and safety standards set by law. DMCI-PDI has been granted the following permits and clearances prior to starting the project. Thus, there can be no doubt that the Torre de Manila project is not a nuisance per se. The petition for mandamus lacks merit and must be dismissed. Taxicab Operators of Metro Manila vs. BOT, 119 SCRA 597 (1982) FACTS: The petitioner Taxicab Operators of Metro Manila Inc. is a domestic corporation composed of taxicab operators, who are grantees of Certificates of Public Convenience to operate taxicabs within the City of Manila and to any other place in Luzon accessible to vehicular traffic. The petitioner filed a petition which seeks the nullity of the Memorandum Circular No. 77-42, an administrative regulation phasing out taxicabs more than six years old on the ground that it is violative of their constitutional right of equal protection of the law because it is only enforced in Manila and directed solely towards the taxi industry. The respondent Board of Transportation contend that the purpose of the regulation is the promotion of safety and comfort of the riding public from the dangers posed by old and dilapidated taxis. CONSTITUTIONAL LAW 2 CASE DIGESTS

ISSUE: "Whether or not the circular issued by the Board of Transportation is a valid exercise of police power" HELD: Yes, the circular is constitutional. The Supreme Court held that there is no violation of constitutional rights. The State, in the exercise of its police power, can prescribe regulations to promote the health, morals, peace, good order, safety and general welfare of the people. It can prohibit all things hurtful to comfort, safety and welfare of society. Presidential Decree No. 101 granted the Board of Transportation the power to fix just and reasonable standards, classification, regulations, practices, measurements, or service to be furnished, imposed, observed, and followed by operators of public utility motor vehicles. The Board of Transportations reason for enforcing the Circular initially in Metro Manila is that taxicabs in this city, compared to those of other places, are subjected to heavier traffic pressure and more constant use and considering that traffic conditions are not the same in every city, a substantial distinction exists so that infringement of the equal protection clause can hardly be successfully claimed. PHILIPPINE PORTS AUTHORITY vs. CIPRES STEVEDORING & ARRASTRE, INC., G.R. No 145742 July 14, 2005 FACTS: Petitioner Philippine Ports Authority (PPA) awarded respondent Cipres Stevedoring and Arrastre, Inc. (CISAI) permits of varied durations to operate the cargo handling operations in Dumaguete City. In 1991, petitioner awarded an eight-year contract to respondent expiring on 31 December 1998. In 1990, PPA issued an administrative order which requires public bidding in the award of contracts for cargo handling services. Upon expiration of its contract, respondent were given hold-over permits by petitioner. While respondents second hold-over permit was still in effect, petitioner issued another administrative order which amended by substitution the administrative order issued in 1990. It expressly provides that all contract for cargo handling services of more than three (3) years shall be awarded through public bidding. Pursuant to the new administrative order, petitioner set the deadline for the submission of the technical and financial CONSTITUTIONAL LAW 2 CASE DIGESTS

bids. Contending that this action on the part of petitioner was in derogation of its vested right over the operation of cargo handling enterprise, respondent initiated an action for specific performance, injunction with application for preliminary mandatory injunction and temporary restraining order before the RTC. Respondent alleged in its complaint that the administrative order issued in 1990 explicitly provides that cargo handling contractors with existing or expired contracts but were able to obtain a satisfactory performance rating were entitled to a renewal of their respective cargo handling contracts with petitioner; thus, as respondent was given a rating of very satisfactory in 1998, it follows that its cargo handling agreement should have been renewed after its expiration. Respondent likewise claimed that the approval and implementation of the new administrative order was plainly arbitrary. The RTC granted respondents’ prayer for a temporary restraining order. Petitioner filed a manifestation with urgent motion for reconsideration to the aforesaid order of the trial court. Petitioner argued that the court a quo did not have the requisite jurisdiction to issue the assailed temporary restraining order. This motion was denied. Petitioner seasonably sought reconsideration. The trial court set aside the injunctive writ it previously issued. Respondent filed a motion for reconsideration but was denied. Respondent filed a petition for certiorari under Rule 65 before the Court of Appeals. The CA nullified the Order of the trial court and granted the petition ordering the RTC to issue a writ of preliminary injunction and the PPA to desist from conducting the public bidding effective until and after the case a quo shall have been finally decided. Petitioner sought the reversal of the decision of the appellate court on the following grounds: a. That P.D. No. 1818 as amended by R.A. 8975 and reiterated in Administrative Circular of the SC, bans the issuance of writs of preliminary injunctions in cases involving government infrastructure projects and service contracts which includes Arrastre and Stevedoring contracts; b. That CISAI has no clear legal right to an injunctive writ since it acquired no vested rights because its hold-over capacity could be revoked at any given time ISSUE: Whether or not the Court of Appeals erred in ordering the RTC to issue the writ of preliminary injunction to CISAI. HELD: CONSTITUTIONAL LAW 2 CASE DIGESTS

Yes. Finally, it is settled that the sole object of a preliminary injunction, may it be prohibitory or mandatory, is to preserve the status quo until the merits of the case can be heard and the final judgment rendered. The status quo is the last actual peaceable uncontested status which preceded the controversy. In the case at bar, respondent sought the issuance of a writ for preliminary injunction in order to prevent the cessation of cargo handling services in the port of Dumaguete City to the detriment and prejudice of the public, shipper, consignees and port workers. However, the factual backdrop of this case establishes that respondents eight-year contract for cargo handling was already terminated and its continued operation in the port of Dumaguete City was merely by virtue of a second hold-over permit granted by petitioner. xxx By its nature, the hold-over permit was merely temporary in nature and may be revoked by petitioner at anytime. As we declared in the case of Anglo-Fil Trading Corporation, hold-over permits are merely temporary and subject to the policy and guidelines as may be implemented by petitioner. The temporary nature of the hold-over permit should have served as adequate notice to respondent that, at any time, its authority to remain within the premises of the port of Dumaguete City may be terminated. Unlike the contract for cargo handling services previously entered into by petitioner and respondent, whose terms and conditions were agreed upon by the parties herein and which clearly provided for a specific period of effectively as well as a stipulation regarding the notice of violation, the holdover permit was unilaterally granted by petitioner pursuant to its authority under the law. Based on the foregoing, it is clear that at the time of the institution of this suit, respondent no longer possessed any contract for its continued operation in Dumaguete City and its stay in the port of said city was by virtue of a mere permit extended by petitioner revocable at any time by the latter. Obviously, the writ of preliminary injunction issued by the Court of Appeals granted respondent the authority to maintain its cargo handling services despite the absence of a valid cargo handling agreement between respondent and petitioner. For this reason, we hold that the Court of Appeals erred in ordering the court a quo to issue the writ of preliminary injunction in favor of respondent. Chavez v. Romulo, G.R. No. 157036. June 9, 2004 FACTS: GMA delivered a speech to PNP directing PNP Chief Hermogenes Ebdane to suspend the issuance of (Permit to Carry Firearms Outside of Residence PTCFOR). CONSTITUTIONAL LAW 2 CASE DIGESTS

Ebdane issued guidelines banning carrying firearms outside of residence. Petitioner, Francisco Chaves requested DILG to reconsider the implementation. The request was denied. Hence, the petition for prohibition and injunction against Executive Secretary Alberto Romulo and PNP Chief Ebdane. ISSUE: “Whether or not the banning of carrying firearms outside the residence is a valid exercise of police power?” HELD: Yes, the prohibition is a valid exercise of police power thus petition is dismissed. Just like ordinary licenses in other regulated fields, PTCFOR may be revoked any time. It does not confer an absolute right, but only a personal privilege to be exercised under existing restrictions. A licensee takes his license subject to such conditions as the Legislature sees fit to impose, and one of the statutory conditions of this license is that it might be revoked. Revocation of it does not deprive the defendant of any property, immunity, or privilege. The basis for its issuance was the need for peace and order in the society. The assailed Guidelines do not entirely prohibit possession of firearms. What they proscribe is merely the carrying of firearms outside of residence. However, those who wish to carry their firearms outside of their residences may re-apply for a new PTCFOR. This is a reasonable regulation. If the carrying of firearms is regulated, necessarily, crime incidents will be curtailed. POWER OF EMINENT DOMAIN A. Who exercises the power? City of Manila vs. Chinese Cemetery of Manila, 40 Phil 349 (1919) FACTS: The City of Manila, plaintiff herein, prayed for the expropriation of a portion private cemetery for the conversion into an extension of Rizal Avenue. Plaintiff claims that it is necessary that such public improvement be made in the said portion of the private cemetery and that the said lands are within their jurisdiction. Defendants herein answered that the said expropriation was not CONSTITUTIONAL LAW 2 CASE DIGESTS

necessary because other routes were available. They further claimed that the expropriation of the cemetery would create irreparable loss and injury to them and to all those persons owing and interested in the graves and monuments that would have to be destroyed. The lower court ruled that the said public improvement was not necessary on the particular-strip of land in question. Plaintiff herein assailed that they have the right to exercise the power of eminent domain and that the courts have no right to inquire and determine the necessity of the expropriation. Thus, the same filed an appeal. ISSUE: Whether or not the courts may inquire into, and hear proof of the necessity of the expropriation. HELD: The courts have the power of restricting the exercise of eminent domain to the actual reasonable necessities of the case and for the purposes designated by the law. The moment the municipal corporation or entity attempts to exercise the authority conferred, it must comply with the conditions accompanying the authority. The necessity for conferring the authority upon a municipal corporation to exercise the right of eminent domain is admittedly within the power of the legislature. But whether or not the Municipal Corporation or entity is exercising the right in a particular case under the conditions imposed by the general authority, is a question that the courts have the right to inquire to. The right of expropriation is not an inherent power in a municipal corporation, and before it can exercise the right some law must exist conferring the power upon it. When the courts come to determine the question, they must only find (a) that a law or authority exists for the exercise of the right of eminent domain, but (b) also that the right or authority is being exercised in accordance with the law. 2. Moday v. Court of Appeals, 268 SCRA 368 (1997) FACTS: Percival Moday is a landowner in Bunawan, Agusan del Sur. In 1989, the Sangguniang Bayan of Bunawan passed a resolution authorizing the mayor to initiate an expropriation case against a 1 hectare portion of Moday’s land. Purpose of which was to erect a gymnasium and other public buildings. The mayor approved the resolution and the resolution was transmitted to the Sangguniang CONSTITUTIONAL LAW 2 CASE DIGESTS

Panlalawigan which disapproved the said resolution ruling that the expropriation is not necessary because there are other lots owned by Bunawan that can be used for such purpose. The mayor pushed through with the expropriation nonetheless. ISSUE: Whether or not a municipality may expropriate private property by virtue of a municipal resolution which was disapproved by the Sangguniang Panlalawigan. HELD: YES. Eminent domain, the power which the Municipality of Bunawan exercised in the instant case, is a fundamental State power that is inseparable from sovereignty. It is government’s right to appropriate, in the nature of a compulsory sale to the State, private property for public use or purpose. Inherently possessed by the national legislature, the power of eminent domain may be validly delegated to local governments, other public entities and public utilities. For the taking For the taking of private property by the government to be valid, the taking must be for public use and there must be just compensation. The only ground upon which a provincial board may declare any municipal resolution, ordinance, or order invalid is when such resolution, ordinance, or order is “beyond the powers conferred upon the council or president making the same.” This was not the case in the case at bar as the SP merely stated that there are other available lands for the purpose sought, the SP did not even bother to declare the SB resolution as invalid. Hence, the expropriation case is valid. Lagcao vs. Judge Labra, GR No. 155746, October 3, 2004 FACTS: After acquiring title, petitioners tried to take possession of the lot only to discover that it was already occupied by squatters. Thus a demolition order was issued. However, when the demolition order was about to be implemented, Cebu City Mayor Alvin Garcia wrote two letters] to the MTCC, requesting the deferment of the demolition on the ground that the City was still looking for a relocation site for the squatters. Acting on the mayors request, the MTCC issued two orders suspending the demolition for a period of 120 days. Unfortunately for Petitioners, during the suspension period, the Sangguniang Panlungsod (SP) of Cebu City passed a resolution which identified Lot 1029 as a socialized housing site pursuant to RA 7279. In this appeal, petitioners argue that Ordinance No. 1843 is CONSTITUTIONAL LAW 2 CASE DIGESTS

unconstitutional as it sanctions the expropriation of their property for the purpose of selling it to the squatters, an endeavor contrary to the concept of public use contemplated in the Constitution. They allege that it will benefit only a handful of people. ISSUE: Whether or not Eminent Domain is validly exercised in this case. HELD: It is where a local government unit may, through its chief executive and acting pursuant to an ordinance, exercise the power of eminent domain for public use, or purpose, or welfare for the benefit of the poor and the landless, upon payment of just compensation, pursuant to the provisions of the Constitution and pertinent. No, it has not been validly invoked in this case due to the fact that The foundation of the right to exercise eminent domain should be a genuine necessity and that necessity must be of public character. Government may not capriciously or arbitrarily choose which private property should be expropriated. In this case, there was no showing at all why petitioners property was singled out for expropriation by the city ordinance or what necessity impelled the particular choice or selection. Ordinance No. 1843 stated no reason for the choice of petitioners property as the site of a socialized housing project. It should also be noted that, as early as 1998, petitioners had already obtained a favorable judgment of eviction against the illegal occupants of their property but Mayor Garcia requested the trial court to suspend the demolition on the pretext that the City was still searching for a relocation site for the squatters. However, instead of looking for a relocation site during the suspension period, the city council suddenly enacted Ordinance No. 1843 for the expropriation of petitioners’ lot. It was trickery and bad faith, pure and simple. The unconscionable manner in which the questioned ordinance was passed clearly indicated that respondent City transgressed the Constitution, RA 7160 and RA 7279. Jesus is Lord Christian Foundation vs. Mun. of Pasig, GR No. 155230 August 9, 2005 FACTS: The Municipality of Pasig needed an access road from E. R. Santos Street, a municipal road near the Pasig Public Market, to Barangay Sto. Tomas Bukid, Pasig. CONSTITUTIONAL LAW 2 CASE DIGESTS

The residents in the area needed the road for water and electrical outlets. The municipality then decided to acquire 51 square meters out of the 1,791 square meter property of the Ching Cuancos which is abutting E. R. Santos Street. The Sangguniang Bayan of Pasig approved an Ordinance authorizing the municipal mayor to initiate expropriation proceedings to acquire the said property and appropriate the fund therefor. The ordinance stated that the property owners were notified of the municipality’s intent to purchase the property for public use as an access road but they rejected the offer. The municipality filed a complaint, against the Ching Cuancos for the expropriation of the property under Section 19 of the Local Government Code. The plaintiff alleged therein that it notified the defendants, by letter, of its intention to construct an access road on a portion of the property but they refused to sell the same portion. The plaintiff deposited with the RTC 15% of the market value of the property based on the latest tax declaration covering the property. On plaintiff’s motion, the RTC issued a writ of possession over the property sought to be expropriated. JILCSFI filed a motion for leave to intervene as defendant-in-intervention, which motion the RTC granted. During trial, Rolando Togonon, the plaintiff’s messenger, testified on direct examination that on February 23, 1993, he served a letter of Engr. Jose Reyes, the Technical Assistant to the Mayor on Infrastructure, to Lorenzo Ching Cuanco at his store. The plaintiff offered in evidence a photocopy of the letter of Engr. Jose Reyes addressed to Lorenzo Ching Cuanco to prove that the plaintiff made a definite and valid offer to acquire the property to the co-owners. However, the RTC rejected the same letter for being a mere photocopy. RTC said plaintiff as having a lawful right to take the property in question for purposes for which the same is expropriated. CA affirmed the order of the RTC. Plaintiff substantially complied with Section 19 of R.A. No. 7160, particularly the requirement that a valid and definite offer must be made to the owner. The letter of Engr. Reyes, inviting Lorenzo Ching Cuanco to a conference to discuss with him the road project and the price of the lot, was a substantial compliance with the “valid and definite offer” requirement under said Section 19. ISSUES: Whether or not the respondent complied with the requirement, under Section 19 of the Local Government Code, of a valid and definite offer to acquire the property prior to the filing of the complaint; and Whether or not property which is already intended to be used for public purposes may still be expropriated by the respondent. CONSTITUTIONAL LAW 2 CASE DIGESTS

HELD: NO. The respondent was burdened to prove the mandatory requirement of a valid and definite offer (Art 35 IRR of LGC) to the owner of the property before filing its complaint and the rejection thereof by the latter. It is incumbent upon the condemn or to exhaust all reasonable efforts to obtain the land it desires by agreement. Failure to prove compliance with the mandatory requirement will result in the dismissal of the complaint. An offer is a unilateral proposition which one party makes to the other for the celebration of a contract. It creates a power of acceptance permitting the offeree, by accepting the offer, to transform the offeror’s promise into a contractual obligation. YES. Court rejected the contention of the petitioner that its property can no longer be expropriated by the respondent because it is intended for the construction of a place for religious worship and a school for its members.

San Roque vs Republic, GR No. 163130, Sept. 7, 2007 FACTS: The subject parcels of land are located at Lahug, Cebu City and were part of Lot No. 933. Lot No. 933 was covered by Transfer Certificate of Title No. 11946. It was originally owned by Ismael D. Rosales, Pantaleon Cabrera and Francisco Racaza. On 5 September 1938, subject parcels of land, together with seventeen (17) others, were the subject of an expropriation proceeding initiated by the then Commonwealth of the Philippines docketed as Civil Case No. 781. On 19 October 1938, Judge Felix Martinez ordered the initial deposit of P,500.00 as pre-condition for the entry on the lands sought to be expropriated. On 14 May 1940, a Decision was rendered condemning the parcels of land. However, the title of the subject parcel of land was not transferred to the government. Eventually, the land was subdivided and T.C.T. No. 11946 was cancelled and new titles were issued by the Register of Deeds of Cebu. In 1995, defendant-appellee begun construction of townhouses on the subject parcels of land. On 22 February 1996, plaintiffappellant filed the present case alleging that it is the owner of the subject parcels of land by virtue of the 1938 Decision in the expropriation case, thus, T.C.T. Nos. 128197 and 128198 are null and void. It argued that defendant-appellee, had no right to possess the subject properties because it was not its lawful owner. In its CONSTITUTIONAL LAW 2 CASE DIGESTS

Answer defendant-appellee claimed that it was a buyer in good faith and that there was no valid expropriation because it was initiated by the executive branch without legislative approval. It also alleged that the expropriation was never consummated because the government did notactually enter the land nor were the owners paid any compensation. On August 25, 1998, the RTC ruled that SRRDC's ownership is borne out by the original owner's title to Lot No. 933 and the subsequent transferees‘ respective titles all of which bore no annotation of the fact of expropriation and did not indicate the Republic's favorable lien. It also found that there was no valid expropriation since the records are bereft of a showing that consideration was paid for the subject properties. The CA reversed the RTC Decision on the finding that the appeal from the CFI Decision in the expropriation case was never perfected by the original owners of the subject properties and thus, the expropriation of Lot No. 933 became final and binding on the original owners, and SRRDC, which merely stepped into the latter's shoes, is similarly bound. The CA further held that laches and estoppel cannot work against the Republic despite its failure from 1940 to register Lot No. 933 in its name, or to record the decree of expropriation on the title. Accordingly, the CA found no necessity to rule on the applicability of Valdehueza v. Republic in the case. Hence, the instant petition. ISSUES: Whether respondent, claiming its right to eminent domain, was the dutiful owner of the subject property, despite failure to register it; and Whether petitioner was a buyer of good faith. HELD: The Supreme Court ruled in favor of petitioner on both issues. The general rule is that the State cannot be put in estoppel or laches by the mistakes or errors of its officials or agents. This rule, however, admits of exceptions. One exception is when the strict application of the rule will defeat the effectiveness of a policy adopted to protect the public, such as the Torrens system. The trial court correctly held that title registered under the Torrens system is notice to the whole world. Every person dealing with registered land may safely rely on the correctness of its certificate of title and the law will not oblige him to go beyond what appears on the face thereof to determine the condition of the property. An innocent purchaser for value is one who, relying on the certificate of title, bought the property from the registered owner, without notice that some other person has a CONSTITUTIONAL LAW 2 CASE DIGESTS

right to, or interest in such property and pays a full and fair price for the same, at the time of such purchase, or before ha has notice of the claim or interest of some other person in the property. Constitutional Limitations - Art. II, Sec. 9 What can be appropriated. RP v. PLDT, 26 SCRA 620 (1969) FACTS: Sometime in 1933, the defendant PLDT entered into an agreement with RCA Communications Inc., an American corporation, whereby telephone messages coming from the US and received by RCA’s domestic station, could automatically be transferred to the lines of PLDT, and vice versa.The plaintiff through the Bureau of Telecommunications, after having set up its own Government Telephone System, by utilizing its own appropriation and equipment and by renting trunk lines of the PLDT, entered into an agreement with RCA for a joint overseas telephone service. Alleging that plaintiff is in competition with them, PLDT notified the former and receiving no reply, disconnected the trunk lines being rented by the same; thus, prompting the plaintiff to file a case before the CFI praying for judgment commanding PLDT to execute a contract with the Bureau for the use of the facilities of PLDT’s telephone system, and for a writ of preliminary injunction against the defendant to restrain the severance of the existing trunk lines and restore those severed. ISSUE: Whether or not the defendant PLDT can be compelled to enter into a contract with the plaintiff. HELD: The Republic may, in the exercise of the sovereign power of eminent domain, require the telephone company to permit interconnection of the government telephone system and that of the PLDT, as the needs of the government service may require, subject to the payment of just compensation to be determined by the court. Nominally, of course, the power of eminent domain results in the taking or appropriation of title to, and possession of, the expropriated property; but no cogent reason appears why the said power may not be availed of to impose only a CONSTITUTIONAL LAW 2 CASE DIGESTS

burden upon the owner of condemned property, without loss of title and possession. It is unquestionable that real property may, through expropriation, be subjected to an easement of right of way. City of Manila vs. Chinese Community, 40 Phil 349 FACTS: The City of Manila, plaintiff herein, prayed for the expropriation of a portion private cemetery for the conversion into an extension of Rizal Avenue. Plaintiff claims that it is necessary that such public improvement be made in the said portion of the private cemetery and that the said lands are within their jurisdiction. Defendants herein answered that the said expropriation was not necessary because other routes were available. They further claimed that the expropriation of the cemetery would create irreparable loss and injury to them and to all those persons owing and interested in the graves and monuments that would have to be destroyed. The lower court ruled that the said public improvement was not necessary on the particular-strip of land in question and absolved each and all of the defendants from all liability under the complaint, without any finding as to costs. Plaintiff herein assailed that they have the right to exercise the power of eminent domain and that the courts have no right to inquire and determine the necessity of the expropriation. Section 2429 of Act No. 2711 (Charter of the city of Manila) provides that "the city(Manila) . . . may condemn private property for public use. Thus, the same filed an appeal. ISSUE: Whether or not the City of Manila can condemn private property for public use. HELD: NO. The right of expropriation is not an inherent power in a municipal corporation, and before it can exercise the right some law must exist conferring the power upon it. When the courts come to determine the question, they must only find (a) that a law or authority exists for the exercise of the right of eminent domain, but (b) also that the right or authority is being exercised in accordance with the law. The courts have the power of restricting the exercise of eminent domain to the actual reasonable necessities of the case and for the purposes designated by the law. The moment the municipal corporation or entity attempts to exercise the authority conferred, it must comply with the conditions CONSTITUTIONAL LAW 2 CASE DIGESTS

accompanying the authority. The necessity for conferring the authority upon a municipal corporation to exercise the right of eminent domain is admittedly within the power of the legislature. But whether or not the municipal corporation or entity is exercising the right in a particular case under the conditions imposed by the general authority, is a question that the courts have the right to inquire to. Where Expropriation Suit is Filed Barangay San Roque v. Heirs of Pastor, GR 138896 June 20, 2000 FACTS: Petitioner Brgy. San Roque, Talisay, Cebu filed before the MTC of Talisay, Cebu a complaint to expropriate a property of the respondents heirs of Francisco Pastor. The MTC dismissed the complaint on the ground of lack of jurisdiction. It reasoned that "eminent domain is an exercise of the power to take private property for public use after payment of just compensation. In an action for eminent domain, therefore, the principal cause of action is the exercise of such power or right. The fact that the action also involves real property is merely incidental. An action for eminent domain is therefore within the exclusive original jurisdiction of the RTC and not with this Court." The RTC also dismissed the Complaint when filed before it, holding that the action for eminent domain or condemnation of real property is a real action affecting title to or possession of real property, hence, it is the assessed value of the property involved which determines the jurisdiction of the court. Section 3, paragraph (3), of Republic Act No. 7691, provides that all civil actions involving title to, or possession of, real property with an assessed value of less than P20,000.00 are within the exclusive original jurisdiction of the MTCs. The Tax Declaration shows that the assessed value of the land involved is only P1,740.00. Hence, it is the MTC which has jurisdiction. Petitioner thus appealed directly to the SC, raising a pure question of law. ISSUE: Whether or not MTC or RTC has jurisdiction over cases for eminent domain or expropriation where the assessed value of the subject property is below Twenty Thousand (P20,000.00) Pesos. HELD: CONSTITUTIONAL LAW 2 CASE DIGESTS

An expropriation suit is incapable of pecuniary estimation. The test to determine whether it is so was laid down by the Court in this wise: A review of the jurisprudence of this Court indicates that in determining whether an action is one the subject matter of which is not capable of pecuniary estimation, this Court has adopted the criterion of first ascertaining the nature of the principal action or remedy sought. If it is primarily for the recovery of a sum of money, the claim is considered capable of pecuniary estimation, and whether jurisdiction is in the municipal courts or in the courts of first instance would depend on the amount of the claim. However, where the basic issue is something other than the right to recover a sum of money, or where the money claim is purely incidental to, or a consequence of, the principal relief sought, like in suits to have the defendant perform his part of the contract (specific performance) and in actions for support, or for annulment of a judgment or to foreclose a mortgage, this Court has considered such actions as cases where the subject of the litigation may not be estimated in terms of money, and are cognizable exclusively by courts of first instance (now RTC). The rationale of the rule is plainly that the second class cases, besides the determination of damages, demand an inquiry into other factors which the law has deemed to be more within the competence of courts of first instance, which were the lowest courts of record at the time that the first organic laws of the Judiciary were enacted allocating jurisdiction. What is Taking Requisites of Taking Republic vs. Castelvi, 58 SCRA 336 (1974) FACTS: In 1947, the republic, through the Armed Forces of the Philippines (AFP), entered into a lease agreement with Castelvi on a year-to-year basis. When Castelvi gave notice to terminate the lease in 1956, the AFP refused. She then instituted an ejectment proceeding against the AFP. In 1959, however, the republic commenced the expropriation proceedings for the land in question. After the owner of a parcel of land has been rented and occupied by the government in 1947 refused to extend the lease, the latter commenced expropriation proceedings in 1959. During the assessment of just compensation, the government argued that it had taken the property when the contract of lease commenced and not when the proceedings begun. The owner maintains that the disputed land was not taken CONSTITUTIONAL LAW 2 CASE DIGESTS

when the government commenced to occupy the said land as lessee because the essential elements of the “taking” of property under the power of eminent domain, namely (1) entrance and occupation by condemn or upon the private property for more than a momentary period, and (2) devoting it to a public use in such a way as to oust the owner and deprive him of all beneficial enjoyment of the property, are not present. ISSUE: Whether or Not the compensation should be determined as of 1947 or 1959. HELD: NO. The property was deemed taken only when the expropriation proceedings commenced in 1959. The Supreme Court ruled that the “taking” should not be reckoned as of 1947, and that just compensation should not be determined on the basis of the value of the property as of that year. Taking' under the power of eminent domain may be defined generally as entering upon private property for more than a momentary period, and, under the warrant or color of legal authority, devoting it to a public use, or otherwise informally appropriating or injuriously affecting it in such a way as substantially to oust the owner and deprive him of all beneficial enjoyment thereof. The requisites for taking are: 1) the expropriator must enter a private property, 2) the entry must be for more than a momentary period, 3) it must be under warrant or color of authorities, 4) the property must be devoted for public use or otherwise informally appropriated or injuriously affected, and 5) the utilization of the property for public use must be such a way as to oust the owner and deprive him of beneficial enjoyment of the property. City Govt. of Quezon City vs. Ericta, 122 SCRA 759 (1983) FACTS: Quezon City enacted an ordinance entitled “ORDINANCE REGULATING THE ESTABLISHMENT, MAINTENANCE AND OPERATION OF PRIVATE MEMORIAL TYPE CEMETERY OR BURIAL GROUND WITHIN THE JURISDICTION OF QUEZON CITY AND PROVIDING PENALTIES FOR THE VIOLATION THEREOF”. The law basically provides that at least six (6) percent of the total area of the memorial park cemetery shall be set aside for charity burial of deceased persons who are paupers and have been residents of Quezon City for at least 5 years prior to their death, to be determined by competent City Authorities. QC justified the law by invoking police CONSTITUTIONAL LAW 2 CASE DIGESTS

power. Petitioners argued that the taking of the respondent’s property is a valid and reasonable exercise of police power and that the land is taken for a public use as it is intended for the burial ground of paupers. They further argued that the Quezon City Council is authorized under its charter, in the exercise of local police power, ” to make such further ordinances and resolutions not repugnant to law as may be necessary to carry into effect and discharge the powers and duties conferred by this Act and such as it shall deem necessary and proper to provide for the health and safety, promote the prosperity, improve the morals, peace, good order, comfort and convenience of the city and the inhabitants thereof, and for the protection of property therein.” On the other hand, respondent Himlayang Pilipino, Inc. contended that the taking or confiscation of property was obvious because the questioned permanently restricts the use of the property such that it cannot be used for any reasonable purpose and deprives the owner of all beneficial use of his property. ISSUE: Whether or not the ordinance is valid. HELD: NO. The SC held the law as an invalid exercise of police power. There is no reasonable relation between the setting aside of at least six (6) percent of the total area of all private cemeteries for charity burial grounds of deceased paupers and the promotion of health, morals, good order, safety, or the general welfare of the people. The ordinance is actually a taking without compensation of a certain area from a private cemetery to benefit paupers who are charges of the municipal corporation. Instead of building or maintaining a public cemetery for this purpose, the city passes the burden to private cemeteries. Deprivation of Use Republic vs. Fajardo , 104 Phil.443 (1958) FACTS: The municipal council of Baao, Camarines sur stating among others that construction of a building, which will destroy the view of the plaza, shall not be allowed and therefore be destroyed at the expense of the owner, enacted an CONSTITUTIONAL LAW 2 CASE DIGESTS

ordinance. Herein appellant filed a written request with the incumbent municipal mayor for a permit to construct a building adjacent to their gasoline station on a parcel of land registered in Fajardo's name, located along the national highway and separated from the public plaza by a creek. The request was denied, for the reason among others that the proposed building would destroy the view or beauty of the public plaza. Defendants reiterated their request for a building permit, but again the mayor turned down the request. Whereupon, appellants proceeded with the construction of the building without a permit, because they needed a place of residence very badly, their former house having been destroyed by a typhoon and hitherto they had been living on leased property. Thereafter, defendants were charged in violation of the ordinance and subsequently convicted. Hence this appeal. ISSUE: Whether or Not the ordinance is a valid exercise of police power. HELD: NO. It is not a valid exercise of police power. The ordinance is unreasonable and oppressive, in that it operates to permanently deprive appellants of the right to use their own property; hence, it oversteps the bounds of police power, and amounts to a taking of appellant’s property without just compensation. We do not overlook that the modern tendency is to regard the beautification of neighbourhood as conducive to the comfort and happiness of residents. As the case now stands, every structure that may be erected on appellants' land, regardless of its own beauty, stands condemned under the ordinance in question, because it would interfere with the view of the public plaza from the highway. The appellants would, in effect, be constrained to let their land remain idle and unused for the obvious purpose for which it is best suited, being urban in character. To legally achieve that result, the municipality must give appellants just compensation and an opportunity to be heard. Napocor v. San Pedro, G.R. 170945, September 26, 2006 FACTS: The National Power Corporation (NPC) is a government-owned-and controlled corporation created to undertake the development of hydro-electric generation of power and the production of electricity from any and all sources; and particularly CONSTITUTIONAL LAW 2 CASE DIGESTS

the construction, operation, and maintenance of power plants, auxiliary plants, dams, reservoirs, pipes, mains, transmission lines, power stations and substations, and other works for the purpose of developing hydraulic power from any river, lake, creek, spring and waterfalls in the Philippines and supplying such power to the inhabitants thereof .# Under Republic Act No. 6395, as amended, the NPC is authorized to enter private property provided that the owners thereof shall be indemnified for any actual damage caused thereby. For the construction of its San Manuel-San Jose 500 KV Transmission Line and Tower No. SMJ-389, NPC negotiated with Maria Mendoza San Pedro, then represented by her son, Vicente, for an easement of right of way over her property, Lot No. 2076. The property, which was partly agricultural and partly residential land, was located in Barangay Partida, Norzagaray, Bulacan and covered by Tax Declaration No. 00386. On June 19, 1997, Maria executed a Right of Way Grant# in favor of NPC over the lot for P1,277,886.90. The NPC paid her P524,635.50 for the damaged improvements thereon. The payment voucher for the residential portion of the lot valued at P6,000,000.00 (at P600.00 per sq m) was then processed. However, the NPC Board of Directors approved Board Resolution No. 97-246 stating that it would pay only P230.00 per sq m for the residential portion and P89.00 per sq m for the agricultural portion. On July 12, 1999, Atty. Baltazar and Engr. Cruz submitted their report, recommending as payment for just compensation P800.00 per sq m for the residential lot and P700.00 per sq m for the agricultural lot. On October 28, 1999, the RTC rendered judgment, declaring as well-grounded, fair and reasonable the compensation for the property as recommended by Atty. Baltazar and Engr. Cruz. ISSUE: Whether or not the just compensation was achieved with regards to the fair market value of the residential and agricultural property. HELD: YES. Such valuation of the trial court as affirmed by the CA is reasonable as it is and supported by the evidence on record, we find no compelling reason to disturb the same. In this case, the commissioners' observation on the reported constant loud buzzing and exploding sounds emanating from the towers and transmission lines, especially on rainy days; the constant fear on the part of the landowners that the large transmission lines looming not far above their land and the huge tower in front of their lot will affect their safety and health; and the slim chance CONSTITUTIONAL LAW 2 CASE DIGESTS

that no one would be interested to buy the remaining portions on each side of the residential lot affected by the project, to the damage of the landowners, both as to future actual use of the land and financial gains to be derived there from, makes the instant case fall within the ambit of expropriation. U.S. vs. Causby, 328 U.S. 256 (1946) FACTS: Respondents own 2.8 acres near an airport outside of Greensboro, North Carolina. Respondents’ property contained a house and a chicken farm. The end of one of the runways of the airport was 2,220 feet from Respondents’ property, and the glide path passed over the property at 83 feet, which is 67 feet above the house, 63 feet above the barn, and 18 feet above the highest tree. The use by the United States of this airport is pursuant to a lease beginning June 1, 1942, and ending June 30, 1942, with provisions for renewal until June 30, 1967, or six months after the end of the national emergency, whichever is earlier. The United States’ four motored bombers make loud noises when flying above the property, and have very bright lights. Respondents’ chicken farm production had to stop, because 150 chickens were killed by flying into walls from fright. In the Court of Claims, it was found that the United States had taken an easement over the property on June 1, 1942, and that the value of the property depreciation as the result of the easement was $2,000.00. Respondents claim that their property was taken, within the meaning of the Fifth Amendment, by the regular army and navy aircraft flights over their house and chicken farm. The United States petitioned for certiorari, which was granted. ISSUE: Whether respondents' property was taken within the meaning of the Fifth Amendment by frequent and regular flights of army and navy aircraft over respondents' land at low altitudes. HELD: YES. The airspace is a public highway, but if the landowner is to have the full enjoyment of his land, he must have exclusive control over the immediate reaches of the enveloping atmosphere. The Constitution entrusts Congress with full power to control all navigable airspace. Congress has already acted under that power. It has by statute, 44 Stat. 568, 52 Stat. 973, provided that 'the United States of CONSTITUTIONAL LAW 2 CASE DIGESTS

America is ... to possess and exercise complete and exclusive national sovereignty in the [328 U.S. 256, 272] air space (over) the United States.' navigable airspace which Congress has placed in the public domain is 'airspace above the minimum safe altitudes of flight prescribed by the Civil Aeronautics Authority. Airspace, apart from the immediate reaches above the land, is part of the public domain. Supreme Court agree that, in those circumstances (USAF planes taking off and landing near property), there would be a taking. Though it would be only an easement of flight which was taken, that easement, if permanent and not merely temporary, normally would be the equivalent of a fee interest. It would be a definite exercise of complete dominion and control over the surface of the land. The fact that the planes never touched the surface would be as irrelevant as the absence in this day of the feudal livery of seizing on the transfer of real estate. The owner's right to possess and exploit the land -- that is to say, his beneficial ownership of it -- would be destroyed.

PPI v. Comelec, 244 SCRA 272 (1995) FACTS: Respondent Comelec promulgated Resolution No. 2772 directing newspapers to provide free Comelec space of not less than one-half page for the common use of political parties and candidates. The Comelec space shall be allocated by the Commission, free of charge, among all candidates to enable them to make known their qualifications, their stand on public Issue and their platforms of government. The Comelec space shall also be used by the Commission for dissemination of vital election information. Petitioner Philippine Press Institute, Inc. (PPI), a non-profit organization of newspaper and magazine publishers, asks the Supreme Court to declare Comelec Resolution No. 2772 unconstitutional and void on the ground that it violates the prohibition imposed by the Constitution upon the government against the taking of private property for public use without just compensation. On behalf of the respondent Comelec, the Solicitor General claimed that the Resolution is a permissible exercise of the power of supervision (police power) of the Comelec over the information operations of print media enterprises during the election period to safeguard and ensure a fair, impartial and credible election. ISSUE: CONSTITUTIONAL LAW 2 CASE DIGESTS

Whether Section 2 of Resolution No. 2772 constitute a valid exercise of the power of eminent domain. HELD: NO. The Supreme Court declared the Resolution as unconstitutional. The taking of private property for public use is, of course, authorized by the Constitution, but not without payment of "just compensation" (Article III, Section 9). And apparently the necessity of paying compensation for "COMELEC space" is precisely what is sought to be avoided by respondent Commission, whether Section 2 of Resolution No. 2772 is read as petitioner PPI reads it, as an assertion of authority to require newspaper publishers to "donate" free print space for COMELEC purposes, or as an exhortation, or perhaps an appeal, to publishers to donate free print space, as Section 1 of Resolution No. 2772-A attempts to suggest. There is nothing at all to prevent newspaper and magazine publishers from voluntarily giving free print space to COMELEC for the purposes contemplated in Resolution No. 2772. Section 2 of Resolution No. 2772 does not, however, provide a constitutional basis for compelling publishers, against their will, in the kind of factual context here present, to provide free print space for COMELEC purposes. Section 2 does not constitute a valid exercise of the power of eminent domain. City of Mandaluyong v. Francisco, G.R. No. 137152, January 29, 2001 FACTS: Antonio, Francisco, Thelma, Eusebio, and Rodolfo N. Aguilar, constructed residential houses several decades ago on a portion of the 3 lots located at 9 de Febrero Street, Barangay Mauwag, City of Mandaluyong. The Aguilars had since leased out these houses to tenants until the present. On the vacant portion of the lots, other families constructed residential structures which they likewise occupied. In 1983, the lots were classified by Resolution 125 of the Board of the Housing and Urban Development Coordinating Council as an Area for Priority Development for urban land reform under Proclamation 1967 and 2284 of then President Marcos. As a result of this classification, the tenants and occupants of the lots offered to purchase the land from the Aguilars, but the latter refused to sell. On 7 November 1996, the Sangguniang Panlungsod of Mandaluyong, upon petition of the Kapitbisig, an association of tenants and occupants of the subject land, adopted Resolution 516, Series of 1996 authorizing Mayor Benjamin Abalos CONSTITUTIONAL LAW 2 CASE DIGESTS

of the City of Mandaluyong to initiate action for the expropriation of the subject lots and construction of a medium-rise condominium for qualified occupants of the land. On 10 January 1996, Mayor Abalos allegedly sent a letter to the Aguilars offering to purchase the said property at P3,000.00 per square meter. On 4 August 1997, the City filed with the Regional Trial Court (RTC), Branch 168, Pasig City a complaint for expropriation, seeking to expropriate 3 adjoining parcels of land with an aggregate area of 1,847 square meters in the names of the Aguilars, and praying that the fixing of just compensation at the fair market value of P3,000.00 per square meter. In their answer, the Aguilars, except Eusebio who died in 1995, denied having received a copy of Mayor Abalos' offer to purchase their lots. They alleged that the expropriation of their land is arbitrary and capricious, and is not for a public purpose; that the subject lots are their only real property and are too small for expropriation, while the City has several properties inventoried for socialized housing; and that the fair market value of P3,000.00 per square meter is arbitrary because the zonal valuation set by the Bureau of Internal Revenue is P7,000.00 per square meter. As counterclaim, the Aguilars prayed for damages of P21 million. On 5 November 1997, the City filed an Amended Complaint and named as an additional defendant Virginia N. Aguilar and, at the same time, substituted Eusebio Aguilar with his heirs. The City also excluded from expropriation TCT N59870 and thereby reduced the area sought to be expropriated from three (3) parcels of land to two (2) parcels totaling 1,636 square meters. The Amended Complaint was admitted by the trial court on 18 December 1997. On 17 September 1998, the trial court issued an order dismissing the Amended Complaint after declaring the Aguilars as "small property owners" whose land is exempt from expropriation under Republic Act 7279. The court also found that the expropriation was not for a public purpose for the City's failure to present any evidence that the intended beneficiaries of the expropriation are landless and homeless residents of Mandaluyong. The City moved for reconsideration. On 29 December 1998, the court denied the motion. The City filed a petition for review with the Supreme Court. ISSUE: Whether the City has exhausted all means to acquire the land under the hands of private persons, but which is within the Areas for Priority Development (APD). HELD: CONSTITUTIONAL LAW 2 CASE DIGESTS

Presidential Decree (PD) 1517, the Urban Land Reform Act, was issued by then President Marcos in 1978. The decree adopted as a State policy the liberation of human communities from blight, congestion and hazard, and promotion of their development and modernization, the optimum use of land as a national resource for public welfare. Pursuant to this law, Proclamation 1893 was issued in 1979 declaring the entire Metro Manila as Urban Land Reform Zone for purposes of urban land reform. This was amended in 1980 by Proclamation 1967 and in 1983 by Proclamation 2284 which identified and specified 245 sites in Metro Manila as Areas for Priority Development and Urban Land Reform Zones. The acquisition of lands for socialized housing is governed by several provisions in the law. Pursuant to Section 9 of RA 7279, Lands for socialized housing are to be acquired in the following order: (1) government lands; (2) alienable lands of the public domain; (3) unregistered or abandoned or idle lands; (4) lands within the declared Areas for Priority Development (APD), Zonal Improvement Program (ZIP) sites, Slum Improvement and Resettlement (SIR) sites which have not yet been acquired; (5) BLISS sites which have not yet been acquired; and (6) privately owned lands. Section 9, however, is not a single provision that can be read separate from the other provisions of the law. It must be read together with Section 10 of RA 7279. Thus, lands for socialized housing under RA 7279 are to be acquired in several modes. Among these modes are the following: (1) community mortgage; (2) land swapping, (3) land assembly or consolidation; (4) land banking; (5) donation to the government; (6) joint venture agreement; (7) negotiated purchase; and (8) expropriation. The mode of expropriation is subject to two conditions: (a) it shall be resorted to only when the other modes of acquisition have been exhausted; and (b) parcels of land owned by small property owners are exempt from such acquisition. The acquisition of the lands in the priority list must be made subject to the modes and conditions set forth in the next provision. In other words, land that lies within the APD may be acquired only in the modes under, and subject to the conditions of, Section 10. The law states "expropriation shall be resorted to when other modes ofacquisition have been exhausted." The City alleged only one mode of acquisition,i.e., by negotiated purchase. The City, through the City Mayor, tried to purchase the lots from the Aguilars but the latter refused to sell. As to the other modes of acquisition, no mention has been made. Not even Resolution 516, Series of 1996 of the Sangguniang Panlungsod authorizing the Mayor of Mandaluyong to effect the expropriation of the subject property states whether the city government tried to acquire the same by community mortgage, land CONSTITUTIONAL LAW 2 CASE DIGESTS

swapping, land assembly or consolidation, land banking, donation to the government, or joint venture agreement under Section 9 of the law. Lagcao vs. Judge Labra- GR No. 155746, October 13, 2004 FACTS: In 1965 petitioners purchased a lot (1029 ) on installment basis from the province of Cebu. Consequently, the province through the City of Cebu tried to annul the sale which prompted the petitioner to file civil action in the court of first instance. On July 9, 1986, the court of first instance ruled that the province execute a deed of sale in favor of the petitioner. On June 11, 1992, the Court of Appeals affirmed the decision of the trial court wherein this ruling was affirmed by the CA. After the title was acquired, petitioners discovered that the property was occupied by squatters which prompted them to institute ejectment proceedings. In this regard, The Municipal Trial Court in Cities (MTCC), Branch 1, Cebu City, rendered a decision on April 1, 1998, ordering the squatters to vacate the lot. On appeal, the RTC affirmed the MTCC's decision and issued a writ of execution and order of demolition. The writ of execution and order of demolition was suspended for 120 days when Cebu City Alvin Garcia wrote a request for the deferment of the ejection order on the ground that the City was still looking for a relocation site for the squatters. During the effect of the suspension order, the Sangguniang Panlungsod of Cebu City passed a resolution and an Ordinance on Feb 22, 1999 and June 30, 1999 respectively, which identified lot 1029 as part of the socialized housing in pursuant of RA 7279. On July 19, 2000, Ordinance No. 1843 was enacted authorizing the mayor of Cebu City to initiate expropriation proceedings on petitioners’ property. ISSUES: Whether or not Cebu City Ordinance No. 1843 contravenes the Constitution and other applicable laws and whether or not Eminent Domain was validly exercised in this case. HELD: YES. The enactment of Ordinance 1843 contravenes the Constitution and other applicable laws. First, Ordinance 1843 contravenes the constitution because condemnation of private lands in an irrational or piecemeal fashion or the random expropriation of small lots to accommodate no more than a few tenants or CONSTITUTIONAL LAW 2 CASE DIGESTS

squatters is certainly not the condemnation for public use contemplated by the Constitution. This is depriving a citizen of his property for the convenience of a few without perceptible benefit to the public. Second, the ordinance is violative of the petitioners’ right to due process since petitioners had already obtained a favorable judgment of eviction against the illegal occupants of their property. The judgment in this ejectment case had, in fact, already attained finality, with a writ of execution and an order of demolition. But Mayor Garcia requested the trial court to suspend the demolition on the pretext that the City was still searching for a relocation site for the squatters. However, instead of looking for a relocation site during the suspension period, the city council suddenly enacted Ordinance No. 1843 for the expropriation of petitioners' lot. It was trickery and bad faith, pure and simple. Third, RA 7160 itself explicitly states that local appropriation of property must comply with the provisions of the Constitution and pertinent laws. Relatively, RA 7279 mandates that local expropriation of property must comply on the order of the priorities on the expropriation of property under section 10 for which private property ranks last in the order of priorities provided under section 9. What should be the order of priority? Filstream International v. CA, 284 SCRA 716 (1998) Filstream filed ejectment suit before MTC against occupants on the grounds of termination of contact and non-payment of rentals. MTC decided in favor of Filstream. This was appealed in RTC and CA and both upheld existing decision. During the pendency of ejectment proceedings, City of Manila approved Ordinance 7813 authorizing Mayor Lim to initiate the acquisition by negotiation, expropriation, purchase, or other legal means certain parcels of land that covers properties of Filstream. City of Manila filed complaint for eminent domain to expropriate Filstream properties. Filstream filed a motion to dismiss the complaint for eminent domain as well as a motion to quash the writ of possession on the ground of no valid cause of action, the petition does not satisfy the requirements of public use and maneuver to circumvent the ejectment suit, violation of the constitutional guarantee against non-impairment, price offered was too low violating just compensation. RTC denied the petition and declared the property condemned in favor of City of Manila. CONSTITUTIONAL LAW 2 CASE DIGESTS

ISSUE: Whether or not Filstream was deprived of due process on the ground of noncompliance with priority in expropriation. HELD: NO. There is no dispute as to the existence of a final and executor judgment in favour of petitioner Filstream ordering the ejectment of private respondents from the properties. The City of Manila has an undeniable right to exercise its power of eminent domain within its jurisdiction specifically in pursuit of its urban land reform and housing program. Private lands rank last in the order of priority for purposes of socializedhousing. In the same vein, expropriation proceedings are to be resorted to only when the other modes of acquisition have been exhausted. Compliance with these conditions must be deemed mandatory because these are the only safeguards in securing the right of owners of private property to due process when their property is expropriated for public use. What is public use? Heirs of Juancho Ardona vs. Reyes, 125 SCRA 220 (1983) FACTS: This is a petition for certiorari with preliminary injunction challenging the constitutionality of Presidential Decree No. 564, the Revised Charter of the Philippine Tourism Authority, and Proclamation No. 2052 declaring the barangays of Sibugay, Malubog, Babag and Sirao including the proposed Lusaran Dam in the City of Cebu and in the municipalities of Argao and Dalaguete in the province of Cebu as tourist zones. The petitioners ask that we restrain respondent Court of First Instance of Cebu and the Philippine Tourism Authority (PTA) from enforcing and implementing the writs of possession issued in four (4) expropriation cases filed by PTA against the petitioners. The Philippine Tourism Authority filed four (4) Complaints with the Court of First Instance of Cebu City for the expropriation of some 282 hectares of rolling land situated in barangays Malubog and Babag, Cebu City, under PTA's express authority "to acquire by purchase, by negotiation or by condemnation proceedings any private land within and without the tourist zones" for the purposes indicated in Section 5, paragraph B(2), of its Revised Charter (PD 564), more specifically, for the development into integrated resort complexes of CONSTITUTIONAL LAW 2 CASE DIGESTS

selected and well-defined geographic areas with potential tourism value. The defendants filed their respective Opposition Dismiss and/or Reconsideration. The defendants, now petitioners, had a common allegation in that the taking is allegedly not impressed with public use under the Constitution. They further alleged, in addition to the issue of public use, that there is no specific constitutional provision authorizing the taking of private property for tourism purpose. ISSUE: Whether expropriation of several barangays for provocation of tourism and construction of sports and hotel complexes constitutes expropriation for public use. HELD: YES. The petitioners' contention that the promotion of tourism is not "public use" because private concessioners would be allowed to maintain various facilities such as restaurants, hotels, stores, etc. inside the tourist complex is impressed with even less merit. The expropriation of private land for slum clearance and urban development is for a public purpose even if the developed area is later sold to private homeowners, commercial firms, entertainment and service companies, and other private concerns. Private bus firms, taxicab fleets, roadside restaurants, and other private businesses using public streets and highways do not diminish in the least bit the public character of expropriations for roads and streets. The lease of store spaces in underpasses of streets built on expropriated land does not make the taking for a private purpose. Airports and piers catering exclusively to private airlines and shipping companies are still for public use. The expropriation of private land for slum clearance and urban development is for a public purpose even if the developed area is later sold to private homeowners, commercial firms, entertainment and service companies, and other private concerns. Province of Camarines Sur vs. CA, 222 SCRA 170 (1993) FACTS: Sangguniang Panlalawigan of CamSur authorized the provincial governor to purchase or expropriate property contiguous to the provincial capitol site in order to establish a pilot farm for non-food and non-traditional agricultural crops and a CONSTITUTIONAL LAW 2 CASE DIGESTS

housing project for provincial government employees. Pursuant to the resolution, Gov. Villafuerte filed two separate cases for expropriation against Ernesto San Joaquin and Efren San Joaquin. Upon motion for the issuance of writ or possession, San Joaquin failed to appear at the hearing. San Joaquin later moved to dismiss the complaints on the ground of inadequacy of the price offered for their property. The court denied the motion to dismiss and authorized the province to take possession of the properties. San Joaquin filed for motion for relief, but denied as well. In their petition. Asked by the CA, Solicitor General stated that there is no need for the approval of the president for the province to expropriate properties, however, the approval of the DAR is needed to convert the property from agricultural to non-agricultural (housing purpose). The Court of Appeals set aside the order of the trial court, suspending the possession and expropriation of the property until the province has acquired the approval of DAR. Hence, this petition. ISSUE: Whether the expropriation of property intended for the establishment of a pilot development center and housing project of the Province of Camarines Sur is in consonance with the public purpose requirement of the Constitution. HELD: YES. The expropriation of the property authorized by the questioned resolution is for a public purpose. The establishment of a pilot development center would insure to the direct benefit and advantage of the people of the Province of Camarines Sur. To sustain the Court of Appeals would mean that the local government units can no longer expropriate agricultural lands needed for the construction of roads, bridges, schools, hospitals, etc, without first applying for conversion of the use of the lands with the Department of Agrarian Reform, because all of these projects would naturally involve a change in the land use. In effect, it would then be the Department of Agrarian Reform to scrutinize whether the expropriation is for a public purpose or public use. Manosca v. Court of Appeals, 252 SCRA 412 (1996) FACTS: A petition for review on certiorari, from the decision of the Court of Appeals, dated 15 January 1992, in CA-G.R. SP No. 24969 (entitled ―Alejandro Manosca, et al. v. Hon. Benjamin V. Pelayo, et al.ǁ). Wherein, Petitioners inherited a piece of CONSTITUTIONAL LAW 2 CASE DIGESTS

land located at P. Burgos Street, Calzada, Taguig, Metro Manila, with an area of about four hundred ninety-two (492) square meters. When the parcel was ascertained by the NHI to have been the birthsite of Felix Y. Manalo, the founder of Iglesia Ni Cristo, it passed Resolution No. 1, Series of 1986, pursuant to Section 4 of Presidential Decree No. 260, declaring the land to be a national historical landmark. The resolution was, on 06 January 1986, approved by the Minister of Education, Culture and Sports. Later, the opinion of the Secretary of Justice was asked on the legality of the measure. Thus the assailment of this petition. ISSUE: Whether or not the public use requirement of Eminent Domain is extent in the attempted expropriation by the Republic of a 492-square-meter parcel of land so declared by the National Historical Institute (NHI) as a national historical landmark. HELD: YES. The taking to be valid must be for public use. The Court is asked to resolve whether or not the "public use" requirement of Eminent Domain is extant in the attempted expropriation by the Republic of a 492-squaremeter parcel of land so declared by the National Historical Institute ("NHI") as a national historical landmark. x x x (the birthsite of Felix Y. Manalo, the founder of Iglesia Ni Cristo) x x x The validity of the exercise of the power of eminent domain for traditional purposes is beyond question; it is not at all to be said, however, that public use should thereby be restricted to such traditional uses. The idea that "public use" is strictly limited to clear cases of "use by the public" has long been discarded. Reyes vs. NHA, GR No. 147511, January 20, 2003 FACTS: Respondent National Housing Authority (NHA) filed complaints for the expropriation of sugarcane lands belonging to the petitioners. The stated public purpose of the expropriation was the expansion of the Dasmariñas Resettlement Project to accommodate the squatters who were relocated from the Metropolitan Manila area. The trial court rendered judgment ordering the expropriation of these lots and the payment of just compensation. The Supreme Court affirmed the judgment of the lower court. A few years later, petitioners contended that respondent NHA violated the stated public purpose for the expansion of the Dasmariñas Resettlement Project when it failed to relocate the squatters from the CONSTITUTIONAL LAW 2 CASE DIGESTS

Metro Manila area, as borne out by the ocular inspection conducted by the trial court which showed that most of the expropriated properties remain unoccupied. Petitioners likewise question the public nature of the use by respondent NHA when it entered into a contract for the construction of low cost housing units, which is allegedly different from the stated public purpose in the expropriation proceedings. Hence, it is claimed that respondent NHA has forfeited its rights and interests by virtue of the expropriation judgment and the expropriated properties should now be returned to herein petitioners. ISSUE: Whether or not the judgment of expropriation was forfeited in the light of the failure of respondent NHA to use the expropriated property for the intended purpose but for a totally different purpose. HELD: The Supreme Court held in favor of the respondent NHA. Accordingly,petitioners cannot insist on a restrictive view of the eminent domain provision of the Constitution by contending that the contract for low cost housing is a deviation from the stated public use. It is now settled doctrine that the concept of public use is no longer limited to traditional purposes. The term "public use" has now been held to be synonymous with "public interest," "public benefit," "public welfare," and "public convenience." Thus, whatever may be beneficially employed for the general welfare satisfies the requirement of public use." In addition, the expropriation of private land for slum clearance and urban development is for a public purpose even if the developed area is later sold to private homeowners, commercials firms, entertainment and service companies, and other private concerns. Moreover, the Constitution itself allows the State to undertake, for the common good and in cooperation with the private sector, a continuing program of urban land reform and housing which will make at affordable cost decent housing and basic services to underprivileged and homeless citizens in urban centers and resettlement areas. The expropriation of private property for the purpose of socialized housing for the marginalized sector is in furtherance of social justice. MCWD v. J. King and Sons Co., Inc. 175983, April 16, 2009 FACTS: CONSTITUTIONAL LAW 2 CASE DIGESTS

The Metropolitan Cebu Water District (MCWD), a public corporation, appealed the decision rendered in favor of Margarita A. Adala (Adala) by the National Water Resources Board (NWRB), granting her a franchise permit to supply water to three sitios in Bulacao. MCWD was the exclusive distributor of water in the district. MCWD contended that the proposed waterworks would interfere with their water supply which it has the right to protect, and the water needs of the residents in the subject area was already being well served by petitioner. They also contend that they were granted by Section 47 of Presidential Decree 198, granting exclusive franchise only to public utilities. Engineer Paredes, the general manager of MCWD, filed Certificate of Public Convenience by the National Water Resources Board (NWRB), which permitted the company to operate and maintain waterworks supply services. MCWD alleged that the Board of Directors of MCWD did not give consent to the issuance of the franchise applied for. ISSUE: Whether or not Section 47 of Presidential Decree 198 grants exclusive franchise to public utilities. HELD: MWCD‘s position that an overly strict construction of the term ―franchise as used in Section 47 of P.D. 198 would lead to an absurd result impresses. If franchises, in this context, were strictly understood to mean an authorization issuing directly from the legislature, it would follow that, while Congress cannot issue franchises for operating waterworks systems without the water district‘s consent, the NWRB may keep on issuing CPCs authorizing the very same act even without such consent. In effect, not only would the NWRB be subject to less constraints than Congress in issuing franchises. The exclusive character of the franchise provided for by Section 47 would be illusory. While the prohibition in Section 47 of P.D. 198 applies to the issuance of CPCs for the reasons discussed above, the same provision must be deemed void ab initio for being irreconcilable with Article XIV Section 5 of the 1973 Constitution which was ratified on January 17, 1973 – the constitution in force when P.D. 198 was issued on May 25, 1973. That the legislative authority – in this instance, then President Marcos – intended to delegate its power to issue franchises in the case of water districts is clear from the fact that, pursuant to the procedure outlined in P.D. 198, it no longer plays a direct role in authorizing the formation and maintenance of water districts, it CONSTITUTIONAL LAW 2 CASE DIGESTS

having vested the same to local legislative bodies and the Local Water Utilities Administration (LWUA). Can the Government Withdraw? NHA v. Heirs of Isidro Guivelondo, G.R. No. 154411. June 19, 2003 FACTS: NHA filed with RTC of Cebu Branch 11 a complaint as amended regarding the eminent domain against Heirs of Guivelondo docketed as civil case. The petitioner alleged that defendant heirs et. al were the rightful private owners of the land which the petitioner intends to develop a socialized housing project. The respondent heirs filed a manifestation of waiving their objections to petitioners power to expropriate their properties, thereafter trial court declares plaintiff has a right to expropriate the properties of the defendant heirs and appointed 3 commissioners who ascertain the just compensation of the said properties be fixed at 11, 200.00 PHP per square meter. Petitioner NHA filed 2 motion for reconsideration that assails inclusion of lots 12, 13 and 19 as well as the amount of just compensation, however the respondents filed a motion for reconsideration of the trial courts partial judgment . But the trial court issued an omnibus order to deny the motion of respondent granting the petitioner’s motion and of just compensation. Petitioner filed with the Court of Appeals a petition for certiorari. Thereafter, heirs filed a motion for execution since the trial court move for the entry of the partial judgment as modified by the omnibus order. The Court of Appeals rendered dismissal of the petition for certiorari on the ground of partial judgment and omnibus order became a final and executor when petitioner failed to appeal. The petitioner filed a motion for reconsideration but then it was denied by the court. The courts of appeals serve on petitioner for a notice of levy pursuant to writ of Execution and a Notice of third garnishment from the Land bank of the Philippines. ISSUES: Whether or not the state can be compelled and coerced by the courts to continue with its inherent power of eminent domain and whether or not writs of execution and garnishment may be issued against the state in an expropriation where in the exercise of power of eminent domain will not serve public use or purpose. CONSTITUTIONAL LAW 2 CASE DIGESTS

HELD: The state as represented by the NHA for housing project can continue its inherent power of eminent domain provided that the just compensation for the property sought is taken. After the rendition of such order the plaintiff shouldn’t be permitted to dismiss or discontinue such proceedings except on such terms of the court be equitable. The order was final after the non-appealing of the petitioner as the lawful right to expropriate the properties of respondent heirs of Guivelondo. Petitioner NHA are not exempt from garnishment or execution, although it is public in character since it is arbitrary and capricious for a government entity to initiate expropriation proceedings that seize a private owner’s property. Petition was DENIED and the trial court’s decision denying petitioner’s motion to dismiss expropriation proceeding was AFFIRMED. Its injunctive relief against the levy and garnishment of its funds and personal properties was also DENIED. The temporary Restraining Order was LIFTED. NPC & Pobre v. CA, G.R. No. 106804. August 12, 2004 FACTS: At the height of the typhoon “Kading”, a flash flood covered the towns near the Angat Dam, causing deaths and destructions to residents and their properties. Respondents blamed the tragedy to the reckless and imprudent opening of the 3 floodgates by petitioner, without prior warning to the residents within the vicinity of the dam. Petitioners denied the allegations and contended that they have kept the water at a safe level, that the opening of floodgates was done gradually, that it exercises diligence in the selection of its employees, and that written warnings were sent to the residents. It further contended that there was no direct causal relationship between the damage and the alleged negligence on their part, that the residents assumed the risk by living near the dam, and that what happened was a fortuitous event and are of the nature of damnum absque injuria. ISSUES: Whether or not the petitioner can be held liable even though the coming of the typhoon is a fortuitous event; Whether or not a notice was sent to the residents; And whether or not the damage suffered by respondents is one of damnum absque injuria. HELD: CONSTITUTIONAL LAW 2 CASE DIGESTS

NO. The obligor cannot escape liability, if upon the happening of a fortuitous event or an act of God, a corresponding fraud, negligence, delay or violation or contravention in any manner of the tenor of the obligation as provided in Article 1170 of the Civil Code which results in loss or damage. Even if there was no contractual relation between themselves and private respondents, they are still liable under the law on quasi-delict. Article 2176 of the Civil Code explicitly provides "whoever by act or omission causes damage to another there being fault or negligence is obliged to pay for the damage done." Act of God or force majeure, by definition, are extraordinary events not foreseeable or avoidable, events that could not be foreseen, or which, though foreseen, are inevitable. It is therefore not enough that the event should not have been foreseen or anticipated, as is commonly believed, but it must be one impossible to foresee or to avoid. The principle embodied in the act of God doctrine strictly requires that the act must be occasioned solely by the violence of nature. Human intervention is to be excluded from creating or entering into the cause of the mischief. When the effect is found to be in part the result of the participation of man, whether due to his active intervention or neglect or failure to act, the whole occurrence is then humanized and removed from the rules applicable to the acts of God. In the case at bar, although the typhoon "Kading" was an act of God, petitioners cannot escape liability because their negligence was the proximate cause of the loss and damage. NO. The letter itself, addressed merely "TO ALL CONCERNED", would not strike one to be of serious importance, sufficient enough to set alarm and cause people to take precautions for their safety's sake. The notices were not delivered, or even addressed to responsible officials of the municipalities concerned who could have disseminated the warning properly. They were delivered to ordinary employees and policemen. As it happened, the said notices do not appear to have reached the people concerned, which are the residents beside the Angat River. The plaintiffs in this case definitely did not receive any such warning. Indeed, the methods by which the defendants allegedly sent the notice or warning was so ineffectual that they cannot claim, as they do in their second assignment of error, that the sending of said notice has absolved them from liability. NO. We cannot give credence to petitioners' third assignment of error that the damage caused by the opening of the dam was in the nature of damnum absque injuria, which presupposes that although there was physical damage, there was no legal injury in view of the fortuitous events. There is no question that petitioners have the right, duty and obligation to operate, maintain and preserve the facilities of Angat Dam, but their negligence cannot be countenanced, however noble their intention may CONSTITUTIONAL LAW 2 CASE DIGESTS

be. The end does not justify the means, particularly because they could have done otherwise than simultaneously opening the spillways to such extent. Needless to say, petitioners are not entitled to counterclaim. Recovery of Expropriated Land ATO v. Gopuco, G.R. No. 158563, June, 30 2005 FACTS: Respondent was the owner of Cadastral Lot No. 72 consisting of 995 square meters located in the vicinity of the Lahug Airport in Cebu City. The Lahug Airport had been turned over by the US Army to RP sometime in 1947 through the Surplus Property Commission, which accepted it in behalf of the Philippine Government. In 1947, the Surplus Property Commission was succeeded by the Bureau of Aeronautics, which office was supplanted by the National Airport Corporation (NAC). The NAC was in turn dissolved and replaced with the Civil Aeronautics Administration (CAA).Sometime in 1949, the NAC informed the owners of the various lots surrounding the Lahug Airport, including the herein respondent, that the government was acquiring their lands for purposes of expansion. Some landowners were convinced to sell their properties on the assurance that they would be able to repurchase the same when these would no longer be used by the airport. Others, including Gopuco, refused to do so. Thus, on 16 April 1952, the CAA filed a complaint with the CFI of Cebu for the expropriation of Lot No. 72 and its neighboring realties. CFI promulgated a Decision declaring the expropriation of [the subject lots, including Lot No. 72] justified and in lawful exercise of the right of eminent domain and declaring a balance of P1,990 in favor of Apolonio Go Puco, Jr. with legal interest from November 16, 1947 until fully paid. After the payment of the foregoing financial obligation to the landowners, directing the latter to deliver to the plaintiff the corresponding Transfer Certificates of Title to their respective lots; and upon the presentation of the said titles to the Register of Deeds, ordering the latter to cancel the same and to issue, in lieu thereof, new Transfer Certificates of Title in the name of the plaintiff. No appeal was taken from the above Decision on Lot No. 72, and the judgment of condemnation became final and executory. Thereafter, on 23 May 1962, absolute title to Lot No. 72 was transferred to the Republic of the Philippines under TCT No. 25030. Subsequently, when the Mactan International Airport commenced operations, the Lahug Airport was ordered closed by then CONSTITUTIONAL LAW 2 CASE DIGESTS

President Corazon C. Aquino in a Memorandum of 29 November 1989. Lot No. 72 was thus virtually abandoned. On 16 March 1990, Gopuco wrote the Bureau of Air Transportation, seeking the return of his lot and offering to return the money previously received by him as payment for the expropriation. This letter was ignored. In the same year, Congress passed RA 6958 creating MCIAA and in part providing for the transfer of the assets of the Lahug Airport thereto. Consequently, on 08 May 1992, ownership of Lot No. 72 was transferred to MCIAA under TCT No. 120356 Respondent filed an amended complaint for recovery of ownership with RTC. He alleged that the original purpose for which the property was expropriated had ceased or otherwise been abandoned, and title to the property had therefore reverted to him. And that when the original judgment of expropriation had been handed down, and before they could file an appeal thereto, the CAA offered them a compromise settlement whereby they were assured that the expropriated lots would be resold to them for the same price as when it was expropriated in the event that the Lahug Airport would be abandoned. Respondent claims to have accepted this offer. However, he failed to present any proof on this matter, and later admitted that insofar as the said lot was concerned, no compromise agreement was entered into by the government and the previous owners. The trial court rendered a Decision dismissing the complaint. Respondent appealed to the Court of Appeals, which overturned the RTC decision, ordered the herein petitioners to reconvey Lot No. 72 to Gopuco upon payment of the reasonable price as determined by it. Motion for Reconsideration was denied. Hence the appeal. ISSUE: Whether or not the court of appeals erred in holding that respondent has the right to Reclaim ownership over the subject expropriated lot. HELD: YES. SC resolve to grant the petition. When private land is expropriated for a particular public use, and that particular public use is abandoned, does its former owner acquire a cause of action for recovery of the property? Gopuco argues that there is present, in cases of expropriation, an "implied contract" that the properties will be used only forthe public purpose for which they were acquired. No such contract exists. Eminent domain is generally described as "the highest and most exact idea of property remaining in the government" that may be acquired for some public purpose through a method in the nature of a forced CONSTITUTIONAL LAW 2 CASE DIGESTS

purchase by the State. Also often referred to as expropriation and, with less frequency, as condemnation, it is, like police power and taxation, an inherent power of sovereignty and need not be clothed with any constitutional gear to exist; instead, provisions in our Constitution on the subject are meant more to regulate, rather than to grant, the exercise of the power. It is a right to take or reassert dominion over property within the state for public use or to meet a public exigency and is said to be an essential part of governance even in its most primitive form and thus inseparable from sovereignty. In fact, "all separate interests of individuals in property are held of the government under this tacit agreement or implied reservation. Notwithstanding the grant to individuals, the eminent domain, the highest andmost exact idea of property, remains in the government, or in the aggregate body of people in their sovereign capacity; and they have the right to resume the possession of the property whenever the public interest so requires it. Republic v. Lim, G.R. 161656, June 29, 2005 FACTS: In 1938, the Republic instituted a special civil action for expropriation of a land in Lahug, Cebu City for the purpose of establishing a military reservation for the Philippine Army. The said lots were registered in the name of Gervasia and Eulalia Denzon. The Republic deposited P9,500 in the PNB then took possession of the lots. Thereafter, on May 1940, the CFI rendered its Decision ordering the Republic to pay the Denzons the sum of P4,062.10 as just compensation. The Denzons appealled to the CA but it was dismissed on March 11, 1948. An entry of judgment was made on April 5, 1948. In 1950, one of the heirs of the Denzons, filed with the National Airports Corporation a claim for rentals for the two lots, but it "denied knowledge of the matter." On September 6, 1961, Lt. Cabal rejected the claim but expressed willingness to pay the appraised value of the lots within a reasonable time. For failure of the Republic to pay for the lots, on September 20, 1961, the Denzons successors-in-interest, Valdehueza and Panerio, filed with the same CFI an action for recovery of possession with damages against the Republic and AFP officers in possession of the property. On July 1962, the CFI promulgated its Decision in favor of Valdehueza and Panerio, holding that they are the owners and have retained their right as such over lots because of the Republic's failure to pay the amount of 4,062.10, PHP adjudged in the expropriation proceedings. However, in view of the annotation on their land titles, they were ordered to execute a deed of sale in favor of the Republic. They appealed the CFI's decision to CONSTITUTIONAL LAW 2 CASE DIGESTS

the SC. The latter held that Valdehueza and Panerio are still the registered owners of Lots 932 and 939, there having been no payment of just compensation by the Republic. SC still ruled that they are not entitled to recover possession of the lots but may only demand the payment of their fair market value. Meanwhile, in 1964, Valdehueza and Panerio mortgaged Lot 932 to Vicente Lim , herein respondent, as security for their loans. For their failure to pay Lim despite demand, he had the mortgage foreclosed in 1976. The lot title was issued in his name. On 1992, respondent Lim filed a complaint for quieting of title with the RTC against the petitioners herein. On 2001, the RTC rendered a decision in favour of Lim, declaring that he is the absolute and exclusive owner of the lot with all the rights of an absolute owner including the right to possession. Petitioners elevated the case to the CA. In its Decision dated September 18, 2003, it sustained the RTC Decision. Petitioner, through the OSG, filed with the SC a petition for review alleging that they remain as the owner of Lot 932. ISSUE: Whether the Republic has retained ownership of Lot 932 despite its failure to pay respondent's predecessors-in-interest the just compensation there for pursuant to the judgment of the CFI rendered as early as May 14, 1940. HELD: YES. The Court of Appeals is correct in saying that Republic’s delay is contrary to the rules of fair play. In jurisdictions similar to ours, where an entry to the expropriated property precedes the payment of compensation, it has been held that if the compensation is not paid in a reasonable time, the party may be treated as a trespasser ab initio. In summation, while the prevailing doctrine is that "the nonpayment of just compensation does not entitle the private landowner to recover possession of the expropriated lots, however, in cases where the government failed to pay just compensation within five (5) years from the finality of the judgment in the expropriation proceedings, the owners concerned shall have the right to recover possession of their property. This is in consonance with the principle that "the government cannot keep the property and dishonor the judgment." To be sure, the five-year period limitation will encourage the government to pay just compensation punctually. This is in keeping with justice and equity. After all, it is the duty of the government, whenever it takes property from private persons against their will, to facilitate the payment of just compensation. In Cosculluela v. Court of Appeals, we defined just compensation as not only the correct determination of the amount to be paid to CONSTITUTIONAL LAW 2 CASE DIGESTS

the property owner but also the payment of the property within a reasonable time. Without prompt payment, compensation cannot be considered "just." Vda. De Ouano v. Republic, GR No. 168770, February 9, 2011 FACTS: In 1949, the National Airport Corporation (NAC), MCIAAs predecessor agency, pursued a program to expand the Lahug Airport in Cebu City. Through its team of negotiators, NAC met and negotiated with the owners of the properties situated around the airport, which included Lot of the Banilad Estate. As the landowners would later claim, the government negotiating team, as a sweetener, assured them that they could repurchase their respective lands should the Lahug Airport expansion project do not push through or once the Lahug Airport closes or its operations transferred to Mactan-Cebu Airport. Some of the landowners accepted the assurance and executed deeds of sale with a right of repurchase. Others, however, including the owners of the aforementioned lots, refused to sell because the purchase price offered was viewed as way below market, forcing the hand of the Republic, represented by the then Civil Aeronautics Administration (CAA), as successor agency of the NAC, to file a complaint for the expropriation. (Republic v. Damian Ouano, et al.) In view of the adverted buy-back assurance made by the government, the owners of the lots no longer appealed the decision of the trial court. [8] Following the finality of the judgment of condemnation, certificates of title for the covered parcels of land were issued in the name of the Republic were subsequently transferred to MCIAA. At the end of 1991, or soon after the transfer of the aforesaid lots to MCIAA, Lahug Airport completely ceased operations, Mactan Airport having opened to accommodate incoming and outgoing commercial flights. On the ground, the expropriated lots were never utilized for the purpose they were taken as no expansion of Lahug Airport was undertaken. This development prompted the former lot owners to formally demand from the government that they be allowed to exercise their promised right to repurchase. Uy, an employee of the CAA, testified that he was a member of the team which negotiated for the acquisition of certain lots in Lahug for the proposed expansion of the Lahug Airport. He recounted that, in the course of the negotiation, their team assured the landowners that their landholdings would be re conveyed to them in the event the Lahug Airport would be abandoned or if its operation were transferred to the Mactan Airport. Some landowners opted to \ sell, while others were of a different bent owing to the inadequacy of the offered price. Inocian CONSTITUTIONAL LAW 2 CASE DIGESTS

testified that he and his mother, Isabel Lambaga, attended a meeting called by the NAC team of negotiators sometime in 1947 or 1949 where he and the other landowners were given the assurance that they could repurchase their lands at the same price in the event the Lahug Airport ceases to operate. He further testified that they rejected the NACs offer. However, he said that they no longer appealed the decree of expropriation due to the repurchase assurance adverted to. The MCIAA presented Michael Bacarizas (Bacarizas), who started working for MCIAA as legal assistant in 1996. He testified that, in the course of doing research work on the lots subject of Civil Case No. CEB-18370, he discovered that the same lots were covered by the decision in Civil Case No. R-1881. He also found out that the said decision did not expressly contain any condition on the matter of repurchase. The RTC directed defendant Mactan Cebu International Airport Authority (MCIAA) to reconvey (free from liens and encumbrances) to plaintiffs after plaintiffs shall have paid MCIAA the sums indicated in the decision in Civil Case No. R-1881. CA: (Appeal) Affirmed the RTC ruling. ISSUE: Whether or not abandonment of the public use for which the subject properties were expropriated entitles petitioners to reacquire them. HELD: YES. If, for example, land is expropriated for a particular purpose, with the condition that when that purpose is ended or abandoned the property shall return to its former owner, then of course, when the purpose is terminated or abandoned, the former owner reacquires the property so expropriated. If, upon the contrary, however the decree of expropriation gives to the entity a fee simple title, then, of course, the land becomes the absolute property of the expropriator and in that case the non-user does not have the effect of defeating the title acquired by the expropriation proceedings. Genuine Necessity Mun. of Meycauayan vs. IAC, 157 SCRA 640 (1988) FACTS: Respondent Philippine Pipes and Merchandising Corporation filed with the Office of the Municipal Mayor of Meycauayan, Bulacan, an application for a permit to CONSTITUTIONAL LAW 2 CASE DIGESTS

fence a parcel of land. The fencing of said property was allegedly to enable the storage of the respondent's heavy equipment and various finished products. The Municipal Council of Meycauayan, passed Resolution No. 258, Series of 1975, manifesting the intention to expropriate the respondent's parcel of land. It was opposed by the respondent with the Office of the Provincial Governor. The Special Committee recommended that the Provincial Board of Bulacan disapprove or annul the resolution in question because there was no genuine necessity for the Municipality of Meycauayan to expropriate the respondent's property for use as a public road. Then the Provincial Board of Bulacan passed Resolution No. 238, Series of 1976, disapproving and annulling Resolution No. 258, Series of 1975, of the Municipal Council of Meycauayan. However, the Municipal Council of Meycauayan, filed with the Regional Trial Court a special civil action for expropriation and upon deposit of the amount of P24,025.00, which is the market value of the land with the Philippine National Bank, the trial court issued a writ of possession in favor of the petitioner. The respondent went to the Intermediate Appellate Court on petition for review, which the appellate court affirmed the trial court's decision. However, upon motion for reconsideration by the respondent, the decision was re examined and reversed and held that there is no genuine necessity to expropriate the land for use as a public road as there were several other roads for the same purpose and another more appropriate lot for the proposed public road. ISSUE: Whether the Municipality of Meycauayan was right to exercise its power of eminent domain to expropriate the respondent's property for use as a public road. HELD: YES. The Court held that the foundation of the right to exercise the power of eminent domain is genuine necessity and that necessity must be of a public character. Condemnation of private property is justified only if it is for the public good and there is a genuine necessity of a public character. Consequently, the courts have the power to inquire into the legality of the exercise of the right of eminent domain and to determine whether there is a genuine necessity therefore. There is absolutely no showing in the petition why the more appropriate lot for the proposed road which was offered for sale has not been the subject of the CONSTITUTIONAL LAW 2 CASE DIGESTS

petitioner's attempt to expropriate assuming there is a real need for another connecting road. Republic vs. De Knecht, G.R. 87351, February 12, 1990 FACTS: A Complaint for expropriation was filed by respondent Municipality of Meycauayan, Bulacan against the property of petitioners Amos Francia, Cecilia Francia and Benjamin Francia. The Municipality of Meycauayan seeks to use the said property in order to establish a common public terminal for all public utility vehicles. The Regional Trial Court (RTC) ruled that the expropriation was for public purpose and issued an Order of Expropriation. On appeal, the Court of Appeals partially granted the petition. It nullified the Order of Expropriation except with regard to the writ of possession. It upheld the decision of the RTC that in issuance of writ of possession, prior determination of the existence of public purpose is necessary. ISSUE: Whether or not prior determination of existence of public purpose is necessary before the issuance of writ of possession. HELD: From all the foregoing, the facts of record and recommendations of the Human Settlements Commission, it is clear that the choice of Fernando Rein — Del Pan Streets as the line through which the Epifanio de los Santos Avenue should be extended to Roxas Boulevard is arbitrary and should not receive judicial approval. The respondent judge committed a grave abuse of discretion in allowing the Republic of the Philippines to take immediate possession of the properties sought to be expropriated. Before a local government unit may enter into the possession of the property sought to be expropriated, it must (1) file a complaint for expropriation sufficient in form and substance in the proper court and (2) deposit with the said court at least 15% of the property's fair market value based on its current tax declaration. The law does not make the determination of a public purpose a condition precedent to the issuance of a writ of possession. De la Paz Masikip v. Judge Legaspi, G.R. No. 136349, January 23, 2006 CONSTITUTIONAL LAW 2 CASE DIGESTS

FACTS: Lourdes Dela Paz Masikip is the registered owner of a parcel of land, which the City of Pasig sought to expropriate a portion thereof for the “sports development and recreational activities” of the residents of Barangay Caniogan. This was in January 1994. Masikip refused. On March 23, 1994, City of Pasig sought again to expropriate said portion of land for the alleged purpose that it was “in line with the program of the Municipal Government to provide land opportunities to deserving poor sectors of our community.” Petitioner protested, so City of Pasig filed with the trial court a complaint for expropriation. The Motion to Dismiss filed by Masikip was dismissed by the trial court on the ground that there was genuine necessity to expropriate the property. Case was elevated to the Court of Appeals, which dismissed petition for lack of merit. Hence, this petition. ISSUE: Whether or not there was genuine necessity to expropriate the property. HELD: Respondent City of Pasig has failed to establish that there is a genuine necessity to expropriate petitioner’s property. Our scrutiny of the records shows that the Certification issued by the Caniogan Barangay Council dated November 20, 1994, the basis for the passage of Ordinance No. 42 s. 1993 authorizing the expropriation, indicates that the intended beneficiary is the Melendres Compound Homeowners Association, a private, non-profit organization, not the residents of Caniogan. It can be gleaned that the members of the said Association are desirous of having their own private playground and recreational facility. Petitioner’s lot is the nearest vacant space available. The purpose is, therefore, not clearly and categorically public. The necessity has not been shown, especially considering that there exists an alternative facility for sports development and community recreation in the area, which is the Rainforest Park, available to all residents of Pasig City, including those of Caniogan Eminent domain is “the right of a government to take and appropriate private property to the public use, whenever the public exigency requires it, which can be done only on condition of providing a reasonably compensation therefor.” It is the power of the State or its instrumentalities to take private property for public use and is inseparable from sovereignty and inherent in government. This power is lodged in the legislative branch of government. It delegates the power thereof to the LGUs, other public CONSTITUTIONAL LAW 2 CASE DIGESTS

entities and public utility corporations, subject only to constitutional limitations. LGUs have no inherent power of eminent domain and may exercise it only when expressly authorized by statute. What is just compensation? Eslaban v. De Onorio, G.R. No. 146062, June 28, 2001 FACTS: Clarita Vda. De Onorio is the owner of the land in Barangay M. Roxas, Sto. Nino, South Cotabato. Such land is the subject for the construction of an irrigation canal of the National Irrigation Administration (NIA). Mr. Santiago Eslaban Jr. is the project manager of NIA. The parties agreed to the construction of the canal provided that the government will pay for the area that has been taken. A right-ofway agreement was entered into by the parties in which respondent was paid the amount of P4, 180.00 as right of way damages. Subsequently, respondent executed an Affidavit of Waiver of Rights and Fees which waives her rights for the damage to the crops due to construction of the right of way. After which, respondent demands that petitioner pay P111, 299.55 for taking her property but the petitioner refused. Petitioner states that the government had not consented to be sued and that the respondent is not entitled for compensation by virtue of the homestead patent under CA no. 141. The RTC held that the NIA should pay respondent the amount of P107, 517.60 as just compensation for the 24,660 sq meters that have been used for the construction of the canal. The Court of Appeals also affirmed the decision of the RTC. ISSUE: Whether or Not the CA erred in affirming the decision of the RTC. HELD: YES. The CA is correct in affirming the decision of the RTC but modifications shall be made regarding the value of the just compensation. The following are the points to be considered in arriving in this decision. With respect to the compensation which the owner of the condemned property is entitled to receive, it is likewise settled that it is the market value which should be paid or "that sum of money which a person, desirous but not compelled to buy, and an owner, willing but not compelled to sell, would agree on as a price to be given and CONSTITUTIONAL LAW 2 CASE DIGESTS

received therefor." Further, just compensation means not only the correct amount to be paid to the owner of the land but also the payment of the land within a reasonable time from its taking. Without prompt payment, compensation cannot be considered "just" for then the property owner is made to suffer the consequence of being immediately deprived of his land while being made to wait for a decade or more before actually receiving the amount necessary to cope with his loss. RP vs. Gingoyon, December 19, 2005 FACTS: NAIA 3, a project between the Government and the Philippine International Air Terminals Co., Inc (PIATCO) was nullified. Planning to put NAIA 3 facilities into immediate operation, the Government, through expropriation filed a petition to be entitled of a writ of possession contending that a mere deposit of the assessed value of the property with an authorized government depository is enough for the entitlement to said writ (Rule 67 of the Rules of Court). However, respondents avers that before an entitlement of the writ of possession is issued, direct payment of just compensation must be made to the builders of the facilities, citing RA No. 8974 and a related jurisprudence (2004 Resolution). ISSUE: Whether Rule 67 of the Rules of Court or Rep. Act No. 8974 governs the expropriation proceedings in this case. HELD: YES. Rep. Act No. 8974 applies in this case, particularly insofar as it requires the immediate payment by the Government of at least the proffered value of the NAIA 3 facilities to PIATCO and provides certain valuation standards or methods for the determination of just compensation. In expropriation proceedings, entitlement of writ of possession is issued only after direct payment of just compensation is given to property owner on the basis of fairness. The same principle applied in the 2004 Jurisprudence Resolution and the latest expropriation law (RA No. 8974). Office of the SolGen v. Ayala Land Inc., GR No. 177056, September 18, 2009 CONSTITUTIONAL LAW 2 CASE DIGESTS

FACTS: This is a petition for review on certiorari seeking the reversal and setting aside of the decision of the court of appeals which affirmed the decision of the Makati RTC in two civil cases and the resolution of the appellate court in the same case which denied the motion for reconsideration filed by the OSG. Respondents herein are operators of shopping malls in various locations in Metro Manila that have parking facilities (inside the main buildings, in separate buildings and/or in adjacent lots solely provided for parking use). The respondents are also the one which maintains the parking spaces and in turn, they collect parking fees subject to their imposed parking rates. The Senate Committee on Trade and Commerce and on Justice and Human Rights conducted a joint investigation to inquire on the legality of the parking fees and to find out the basis and reasonableness of the parking rates. More importantly, to determine the legality of the policy of the shopping malls denying liability in cases of theft, robbery or carnapping by invoking the waiver clause at the back of the parking tickets. After the public hearings, the Senate Committees jointly concluded that the collection parking fee is contrary to the National Building Code and that the reasonable interpretation of the code is that the parking spaces are for free; thus, the Committee recommended that the Office of the Solicitor General should institute the necessary action to enjoin the collection of parking fees as well as to enforce the penal sanctions of the National Building Code. Two civil cases arise and by being of the same subject matter, the RTC Makati issued an order to consolidate the cases. The court ruled that the respondents are not obligated to provide parking spaces that are free of charge, compelling them to do so would be an unlawful taking of property right without just compensation. The petitioners sought for relief by filing a Motion for Reconsideration in the Court of Appeals but the appellate court denied the appeal and affirmed the joint decision by the RTC. Hence, this present petition with a single assignment of error that the Court of Appeals erred in affirming the ruling of the lower court. ISSUE: Whether or not the property right of the respondents can be taken so as to provide free parking spaces for the general public welfare. HELD: YES. The court affirmed the previous decision that the respondents are not obliged to provide free parking spaces. There is no pertaining provision in the CONSTITUTIONAL LAW 2 CASE DIGESTS

National Building Code that expressly provides the same. The law is clear and unequivocal that it needs no further interpretation, it only provides for measurement requirements of the parking spaces. The OSG cannot rely on their invoked provisions; they even failed to consider the substantial differences and legal backgrounds on the jurisprudence they are insisting. Wherefore, the instant petition is hereby DENIED. Previous ruling AFFIRMED. No Costs. Internal Revenue vs. Bicolandia Drug Corp., GR No. 148083, July 21, 2006 FACTS: Bicolandia Drug Corporation, a corporation engaged in the business of retailing pharmaceutical products under the business style of "Mercury Drug," granted the 20% sales discount to qualified senior citizens purchasing their medicines in compliance with R.A. No. 7432 It then alleged error that they should have tax credit so it claimed for refund. CTA: Rev. Reg. No. 2-94 is null and void for being inconsistent with Sec. 4 of RA 7432 that states the discount is claimed as credit But, it computed the tax credit as cost of sales / gross income x 20% It also excluded those sales without pre-marked cash slips. Both CIR and petitioner appealed. CTA modified its decision to issue a certificate of tax credit to petitioner. ISSUE: Whether or not the discount granted is based on the acquisition cost rather than actual discount granted and whether or not petitioner can claim its refund. HELD: YES. Cost refers to the amount extended to senior citizens. It shall be applied as tax credit and may be deducted from tax liability. If no current tax due or net loss for the period, the credit may be carried over to the succeeding taxable year. Petition is hereby DENIED. No. The words of statute are clear and free from ambiguity. It must be given literal meaning. Thus, can only claim as tax credit the law (R.A. No. 7432) allows the discounts as a tax credit but its implementing regulations (RR No. 2-94) only allows the same as deductions from gross income. The SC ruled that in cases of conflict between the law and the rules and regulations implementing the law, the law shall always prevail. It bears emphasis that R.A. No. 7432 is now repealed by R.A. 9257 which was approved into law on February 26, 2004. Under this later law, discounts given to senior citizens are treated as deduction from gross income and no longer allowed as tax credit. The CONSTITUTIONAL LAW 2 CASE DIGESTS

tax treatment thereby under the present law was made consistent with financial accounting treatment thereby making the tax system more attuned to the principle of administrative feasibility LBP v. Honeycomb Farms Corp., GR No. 169903, February 29, 2012 FACTS: Honeycomb Farms Corporation (Honeycomb Farms) was the registered owner of two parcels of agricultural land in Cataingan, Masbate. Honeycomb Farms voluntarily offered these parcels of land, with a total area of 495.1374 hectares, to the Department of Agrarian Reform (DAR) for coverage under the Comprehensive Agrarian Reform Law (CARL). From the entire area offered, the government chose to acquire only 486.0907 hectares. The Land Bank of the Philippines (LBP), as the agency vested with the responsibility of determining the land valuation and compensation for parcels of land acquired pursuant to the CARL, and using the guidelines set forth in DAR Administrative Order (AO) No. 17, series of 1989, as amended by DAR AO No. 3, series of 1991, fixed the value of these parcels of land. When Honeycomb Farms rejected the LBPs and the DARs valuation for being too low, Honeycomb Farms filed a case with the RTC, acting as a Special Agrarian Court (SAC), against the DAR Secretary and the LBP, praying that it be compensated for its landholdings in the amount of P12,440,000.00, with damages and attorney's fees. The RTC constituted a Board of Commissioners to aid the court in determining the just compensation for the subject properties. Since the Board of Commissioners could not reach a common valuation for the properties, the RTC made its own valuation. First, the RTC took judicial notice of the fact that a portion of the land, measuring approximately 10 hectares, is commercial land, since it is located a few kilometers away from Sitio Curvada, Pitago, Cataingan, Masbate, which is a commercial district. The lower court thus priced the 10 hectares at P100,000.00 per hectare and the remaining 476 hectares at P32,000.00 per hectare. Both parties appealed to the CA. The LBP argued that the RTC committed a serious error when it disregarded the formula for fixing just compensation embodied in DAR AO No. 6, series of 1992, as amended by DAR AO No. 11, series of 1994. The LBP also argued that the RTC erred in taking judicial notice that 10 hectares of the land in question is commercial land. In contrast, Honeycomb Farms maintains that the DAR AOs were issued merely to serve as guidelines for the DAR and the LBP in administratively fixing the valuation to be offered by the DAR to the landowner for acceptance or rejection. However, it is CONSTITUTIONAL LAW 2 CASE DIGESTS

not mandatory for courts to use the DAR AOs to fix just compensation as this would amount to an administrative imposition on an otherwise purely judicial function and prerogative of determination of just compensation for expropriated lands specifically reserved by the Constitution to the courts. The CA affirmed with modification the assailed RTC judgment with respect to the computation of the amount fixed by the trial court and the award of attorneys fees is deleted. ISSUES: Whether or not CA commit a serious error of law when it failed to apply the mandatory formula for determining just compensation fixed in DAR AO No. 11, series of 1994 and whether or not RTC correctly take judicial notice of the nature of the subject land. HELD: It is the RTC, sitting as a SAC, which has the power to determine just compensation for parcels of land acquired by the State, pursuant to the agrarian reform program. In Land Bank of the Philippines v. Sps. Banal, the DAR, as the administrative agency tasked with the implementation of the agrarian reform program, already came up with a formula to determine just compensation which incorporated the factors enumerated in Section 17 of RA 6657. In Landbank of the Philippines v. Celada, the Court emphasized the duty of the RTC to apply the formula provided in the applicable DAR AO to determine just compensation, stating that: While RTC is required to consider the acquisition cost of the land, the current value of like properties, its nature, actual use and income, the sworn valuation by the owner, the tax declaration and the assessments made by the government assessors to determine just compensation, it is equally true that these factors have been translated into a basic formula by the DAR pursuant to its rule-making power under Section 49 of R.A. No. 6657. As the government agency principally tasked to implement the agrarian reform program, it is the DAR's duty to issue rules and regulations to carry out the object of the law. RTC was at no liberty to disregard the formula which was devised to implement the said provision. These rulings plainly impose on the RTC the duty to apply the formula laid down in the pertinent DAR administrative regulations to determine just compensation. Clearly, the CA and the RTC acted with grievous error when they disregarded the formula laid down by the DAR, and chose instead to come up with their own basis for the valuation of the subject land. How determined? CONSTITUTIONAL LAW 2 CASE DIGESTS

EPZA vs. Dulay, 149 SCRA 305 (1987) FACTS: The four parcels of land which are the subject of this case is where the Mactan Export Processing Zone Authority in Cebu (EPZA) is to be constructed. Private respondent San Antonio Development Corporation (San Antonio, for brevity), in which these lands are registered under, claimed that the lands were expropriated to the government without them reaching the agreement as to the compensation. Respondent Judge Dulay then issued an order for the appointment of the commissioners to determine the just compensation. It was later found out that the payment of the government to San Antonio would be P15 per square meter, which was objected to by the latter contending that under PD 1533, the basis of just compensation shall be fair and according to the fair market value declared by the owner of the property sought to be expropriated, or by the assessor, whichever is lower. Such objection and the subsequent Motion for Reconsideration were denied and hearing was set for the reception of the commissioner’s report. EPZA then filed this petition for certiorari and mandamus enjoining the respondent from further hearing the case. ISSUE: Whether or Not the exclusive and mandatory mode of determining just compensation in PD 1533 is unconstitutional. HELD: YES. The Supreme Court ruled that the mode of determination of just compensation in PD 1533 is unconstitutional. It is violative of due process to deny to the owner the opportunity to prove that the valuation in the tax documents is unfair or wrong. And it is repulsive to basic concepts of justice and fairness to allow the haphazard work of a minor bureaucrat or clerk to absolutely prevail over the judgment of a court promulgated only after expert commissioners have actually viewed the property, after evidence and arguments pro and con have been presented, and after all factors and considerations essential to a fair and just determination have been judiciously evaluated. x x x P.D. No. 1533, which eliminates the court's discretion to appoint commissioners pursuant to Rule 67 of the Rules of Court, is unconstitutional and void. CONSTITUTIONAL LAW 2 CASE DIGESTS

Belen vs. CA, 195 SCRA 59 FACTS: A small portion of land (Lot No. 10, Block 18 at Sunog Apog, Tondo, Manila) measuring a hundred (100) square meters, more or less, belonging to the Manotoc Services, Inc., was leased to Pedro M. Belen, which the latter has built a house. Respondents Alfredo Juliano and his family occupied a portion of the said land and later on bought a house standing thereon, not belonging to Belen and moved in without the latter's knowledge. On learning of this, Belen had a talk with Juliano, and they came to an agreement that Juliano could continue staying on the land temporarily and would pay one-half of the rental to Manotok Realty, Inc. Later a fire razed both Belen's and Juliano's houses to the ground. Belen told Juliano not to build anything on the land any more. However, on Juliano's pleas, Belen acceded to Juliano's continued stay on the land on the explicit condition that his occupancy should not be longer than two and a half (2 1/2) years. Juliano failed to leave after the stipulated term. Metropolitan Trial Court ordered defendant to vacate the land. While Regional Trial Court reversed the judgment of MTC by virtue of Presidential Decree No. 1670. Hence, the appeal. ISSUE: Whether or not the Manotok Realty, Inc. divested its title to the National Housing Authority and was there an appropriate action of eminent domain. HELD: The petition was hereby granted and the challenged judgment of the Court of Appeals, reversed. Presidential Decree No. 1670 was struck down as "unconstitutional and therefore, null and void." The decrees do not by themselves, provide for any form of hearing or procedure by which the petitioners can question the propriety of the expropriation of their properties or the reasonableness of the just compensation. Having failed to provide for a hearing, the Government should have filed an expropriation case under Rule 67 of the Revised Rules of Court but it did not do so. Obviously, it did not deem it necessary because the enactment of the questioned decrees which rendered, by their very passage, any questions with regard to the expropriation of the properties, moot and academic. In effect, the properties under the decrees were "automatically expropriated." This becomes more evident when the NHA wrote the Register of Deeds and requested her to cancel the certificate of titles of the petitioners, CONSTITUTIONAL LAW 2 CASE DIGESTS

furnishing said Register of Deedsonly with copies of the decrees to support its request. This is hardly the due process of law which the state is expected to observe when it exercises the power of eminent domain. The Court found that both the decrees, being "violative of the petitioners' (owners') right to due process of law," failed "the test of constitutionality," and that, additionally, they were tainted by another infirmity as regards "the determination of just compensation." This Court further observed that contrary to Rule 67 and established precedents, the decrees provided for the determination of just compensation at a time earlier than that "of the actual taking of the government or at the time of the judgment by the court, whichever came first." Apart from this, the fixing of the value of the property was left by the decrees to the City Assessor. PD 1670 being void ab initio, all acts done in reliance thereon and in accordance therewith must also be deemed void ab initio, including particularly the taking of possession of the property by the National Housing Authority and its attempts to convert the same into a housing project and the selection of the beneficiaries thereof. Republic vs. CA, 227 SCRA 401 FACTS: James Hughes, a natural born citizen of the United States of America, married Lenita Mabunay, a Filipino Citizen, who herself was later naturalized as a citizen of that country. The spouses jointly filed a petition with the RTC to adopt the minor niece and nephews of Lenita, who had been living with the couple even prior to the filing of the petition. The minors, as well as their parents, gave consent to the adoption. The RTC rendered a decision granting the petition. ISSUE: Whether or not the spouses can adopt the minors. HELD: NO. While James Anthony unquestionably is not permitted to adopt under any of the exceptional cases enumerated in paragraph (3) of the aforequoted article, Lenita, however, can qualify pursuant to paragraph (3)(a). Lenita may not thus adopt alone since Article 185 requires a joint adoption by the husband and the wife, a condition that must be read together along with Article. 184. Art 185 provides that "Husband and wife must jointly adopt, except in the following cases: CONSTITUTIONAL LAW 2 CASE DIGESTS

(1) When one spouse seeks to adopt his own illegitimate child; or (2) When one spouse seeks to adopt the legitimate child of the other." As amended by Executive Order 91, Presidential Decree No. 603, had thus made it mandatory for both the spouses to jointly adopt when one of them was an alien. The law was silent when both spouses were of the same nationality. The Family Code has resolved any possible uncertainty. Article 185 thereof now expresses the necessity for joint adoption by the spouses except in only two instances: (1)When one spouse seeks to adopt his own legitimate child; or (2) When one spouse seeks to adopt the legitimate child of the other. It is in the foregoing cases when Article 186 of the Code, on the subject of parental authority, can aptly find governance. Article 186 provides that "In case husband and wife jointly adoptor one spouse adopts the legitimate child of the other, joint parental authority shall be exercised by the spouses in accordance with this Code." Sps. Lee vs. LBP, GR No. 170422, March 7, 2008 FACTS: Petitioners-spouses Edmond Lee and Helen Huang (petitioners) the registered owners of parcels of situated in Mambog, Hermosa, Bataan. The subject property was compulsorily acquired by the Department of Agrarian Reform (DAR) in accordance with Republic Act No. 6657, as amended, otherwise known as the “Comprehensive Agrarian Reform Law of 1988.” DAR offered the sum of ₱109,429.98 as just compensation for the 1.5073- ha portion of the subject property. Rejecting the valuation, petitioners instead filed the present petition for determination of just compensation against Provincial Adjudicator of the Department of Agrarian Reform Adjudication Board (DARAB). In defense, respondent claimed that its valuation was based on DAR Administrative Order (AO) No. 11, and also contended that petitioners’ appraisal was biased. After due proceedings, the RTC, rendered a Decision dated January 17, 2002 rejecting the valuation given by respondent and setting the just compensation for petitioners’ 1.5073 has. at ₱250.00 per square meter, or a total amount of ₱3,768,250.00. It took judicial notice of the fact that the lots within the vicinity of the subject property are valued between ₱200.00 to ₱500.00 per square meter. Respondent’s motion for reconsideration was denied. Several years later, in September 2006, petitioners filed a motion for execution of the RTC’s January 17, 2002 Decision, alleging that while they received a copy of respondent’s Notice of Appeal dated June 19, 2002, upon verification, no such appeal was actually filed before the RTC. CONSTITUTIONAL LAW 2 CASE DIGESTS

Respondent denied petitioners’ claim and asserted that it filed a Notice of Appeal in accordance with the rules and therefore, perfected its appeal. As such, the RTC’s January 17, 2002 Decision was not yet final and executory. Finding that respondent had perfected its appeal the RTC, in an Order dated June 7, 2007, gave due course to respondent’s appeal and directed that the entire records thereof be transmitted to the CA. Petitioners moved for reconsideration, which the RTC denied in an Order dated August 27, 2008. The RTC clarified that respondent was able to file its Notice of Appeal within the prescribed period and that a postal money order in the amount of ₱520.00 had been issued by respondent in favor of the Clerk of Court of the RTC of Balanga City, Bataan, representing the payment of the appeal fee. Petitioners filed a motion for execution of the RTC’s Decision, alleging that, upon verification, no such appeal was actually filed before the RTC. Respondent denied petitioners’ claim and asserted that it filed a Notice of Appeal in accordance with the rules and has, therefore, perfected its appeal. Five (5) years later, petitioners filed a motion to dismiss the appeal of respondent for failure to prosecute, asseverating that from the time the RTC gave due course to its appeal, respondent had not made any further action on its appeal, particularly with regard to the payment of the prescribed appeal fees. Respondent argued that the RTC no longer had jurisdiction to entertain petitioners’ motion and that the motion should have been filed not before the RTC, but before the CA. Upon a meticulous inspection of the records, the RTC found that respondent failed to pay the prescribed appeal fees. Therefore, held that respondent’s appeal was not duly perfected and such did not lose jurisdiction over the case. Respondent’s motion for reconsideration was denied hence, the matter was elevated before the CA. The CA found grave abuse of discretion on the part of the RTC in dismissing respondent’s appeal for failure to prosecute, holding that the validity of the latter’s appeal had already been passed upon in the RTC’s earlier Orders that gave due course to the appeal and directed the transmittal of the records to the CA. It also ruled that upon the perfection of respondent’s appeal, the RTC had already lost jurisdiction over the case. Thus, any orders subsequently issued by the RTC after the filing of respondent’s Notice of Appeal on June 19, 2002 were of no force and effect. Aggrieved, petitioners filed a motion for reconsideration, which the CA denied in a Resolution; hence, this petition. ISSUE: Whether or not the CA erred in finding grave abuse of discretion on the part of the RTC when it dismissed respondent’s appeal for failure to prosecute. CONSTITUTIONAL LAW 2 CASE DIGESTS

HELD: YES. The petition has merit. The Court notes that if it were true that respondent actually paid the appellate docket fees, it could have easily produced proof of payment if only to dispel any doubts thereon and consequently, prove compliance with the rules on the perfection of appeals. Unfortunately, no such evidence was forthcoming. Indubitably, the dismissal of respondent’s appeal was in order, and the RTC’s January 17, 2002 Decision, as a result, had attained finality. When Determined? Ansaldo vs. Tantuico, G.R. 50147 August 3, 1990 FACTS: Two lots of private ownership were taken by the Government and used for the widening of a road more than forty-three years ago, without benefit of an action of eminent domain or agreement with its owners, albeit without protest by the latter. The lots belong to the petitioners, Jose Ma. Ansaldo and Maria Angela Ansaldo, are covered by title in their names and have an aggregate area of 1,041 square meters. These lots were taken from the Ansaldos sometime in 1947 by the Department of Public Work Transportation and Communication and made part of what used to be Sta. Mesa Street and is now Ramon Magsaysay Avenue at San Juan, Metro Manila. Said owners made no move whatever until twenty-six years later. They wrote to ask for compensation for their land on January 22, 1973. 2 Their claim was referred to the Secretary of Justice who in due course rendered an opinion dated February 22, 1973, 3 that just compensation should be paid in accordance with Presidential Decree No. 76. The Commission on Audit, however, declined to adopt the recommendation. In a decision handed down on September 26, 1973. The Acting Chairman ruled that "the amount of compensation to be paid to the claimants is to be determined as of the time of the taking of the subject lots. ISSUE:

CONSTITUTIONAL LAW 2 CASE DIGESTS

Whether or not just compensation should be fixed as of the time of actual taking of possession by the expropriating entity or only after conveyance of title to the expropriator pursuant to expropriation proceedings duly institute. HELD: YES. Clearly, then, the value of the Ansaldos' property must be ascertained as of the year 1947, when it was actually taken, and not at the time of the filing of the expropriation suit, which, by the way, still has to be done. The value, once fixed, shall earn interest at the legal rate until full payment is effected, conformably with other principles laid down by case law. In the context of the State's inherent power of eminent domain, there is a "taking" when the owner is actually deprived or dispossessed of his property; when there is a practical destruction or a material impairment of the value of his property or when he is deprived of the ordinary use thereof. There is a "taking" in this sense when the expropriator enters private property not only for a momentary period but for a more permanent duration, for the purpose of devoting the property to a public use in such a manner as to oust the owner and deprive him of all beneficial enjoyment thereof. For ownership, after all, "is nothing without the inherent rights of possession, control and enjoyment. Where the owner is deprived of the ordinary and beneficial use of his property or of its value by its being diverted to public use, there is taking within the Constitutional sense. Under these norms, there was undoubtedly a taking of the Ansaldos' property when the Government obtained possession thereof and converted it into a part of a thoroughfare for public use. NAPOCOR v. Tiangco, G.R. No. 170846, February 6, 2007 FACTS: Respondents are owners of a parcel of land with an area of 152,187 square meters at Barangay Sampaloc, Tanay, Rizal. NPC requires 19,423 square meters of the respondents‘ aforementioned property, across which its 500Kv Kalayaan-San Jose Transmission Line Project will traverse. NPC‘s Segregation Plan# for the purpose shows that the desired right-of way will cut through the respondents‘ land. Within the portion sought to be expropriated stand fruit-bearing tress, such as mango, avocado, jackfruit, casuy , santol , calamansi , sintones and coconut trees. After repeated unsuccessful negotiations, NPC filed an expropriation complaint against the land of the respondent in the RTC of Tanay, Rizal. The RTC issued a writ of possession in favor of NPC after paying the deposit requirement. The trial court CONSTITUTIONAL LAW 2 CASE DIGESTS

rendered its decision on the value of the property using the 1984 tax declaration. (which is incorrect as stated in the decision of the supreme court). The respondents filed a motion for recon. but it was denied by RTC. So They filed an appeal and the CA gave merit to the contention of the respondents and made its revised valuation using the 1993 tax declaration (increasing the value of the property). The case went up to the SC. ISSUES: Whether or not the property should be valued using the 1984 or the 1993 tax declarations and whether or not Sec. 3-A of R.A. No. 6395, as amended by P.D. 938 will apply. HELD: In eminent domain cases, the time of taking is the filing of the complaint, if there was no actual taking prior thereto. Hence, in this case, the value of the property at the time of the filing of the complaint on November 20, 1990 should be considered in determining the just compensation due the respondents. Normally, the time of taking coincides with the filing of complaint for expropriation as ruled in the case of Power Corporation v. Court of Appeals, et al. The expropriation proceedings in this case having been initiated by NPC on November 20, 1990, property values on such month and year should lay the basis for the proper determination of just compensation. It should not apply in the case at bar, the acquisition of such easement is not gratis. The limitations on the use of the property taken for an indefinite period would deprive its owner of the normal use thereof. For this reason, the latter is entitled to payment of a just compensation, which must be neither more nor less than the monetary equivalent of the land taken. Manner of Payment Assoc. of Small Landowners v. DAR, 175 SCRA 343 (1989) FACTS: These are four consolidated cases questioning the constitutionality of the Comprehensive Agrarian Reform Act (R.A. No. 6657 and related laws i.e., Agrarian Land Reform Code or R.A. No. 3844). Brief background: Article XIII of the Constitution on Social Justice and Human Rights includes a call for the adoption by CONSTITUTIONAL LAW 2 CASE DIGESTS

the State of an agrarian reform program. The State shall, by law, undertake an agrarian reform program founded on the right of farmers and regular farmworkers, who are landless, to own directly or collectively the lands they till or, in the case of other farmworkers, to receive a just share of the fruits thereof. RA 3844 was enacted in 1963. P.D. No. 27 was promulgated in 1972 to provide for the compulsory acquisition of private lands for distribution among tenant-farmers and to specify maximum retention limits for landowners. In 1987, President Corazon Aquino issued E.O. No. 228, declaring full land ownership in favor of the beneficiaries of PD 27 and providing for the valuation of still unvalued lands covered by the decree as well as the manner of their payment. In 1987, P.P. No. 131, instituting a comprehensive agrarian reform program (CARP) was enacted; later, E.O. No. 229, providing the mechanics for its (PP131’s) implementation, was also enacted. Afterwhich is the enactment of R.A. No. 6657, Comprehensive Agrarian Reform Law in 1988. This law, while considerably changing the earlier mentioned enactments, nevertheless gives them suppletory effect insofar as they are not inconsistent with its provisions. [Two of the consolidated cases are discussed below] G.R. No. 78742: (Association of Small Landowners vs Secretary) The Association of Small Landowners in the Philippines, Inc. sought exception from the land distribution scheme provided for in R.A. 6657. The Association is comprised of landowners of ricelands and cornlands whose landholdings do not exceed 7 hectares. They invoke that since their landholdings are less than 7 hectares, they should not be forced to distribute their land to their tenants under R.A. 6657 for they themselves have shown willingness to till their own land. In short, they want to be exempted from agrarian reform program because they claim to belong to a different class. G.R. No. 79777: (Manaay vs Juico) Nicolas Manaay questioned the validity of the agrarian reform laws (PD 27, EO 228, and 229) on the ground that these laws already valuated their lands for the agrarian reform program and that the specific amount must be determined by the Department of Agrarian Reform (DAR). Manaay averred that this violated the principle in eminent domain which provides that only courts can determine just compensation. This, for Manaay, also violated due process for under the constitution, no property shall be taken for public use without just compensation. Manaay also questioned the provision which states that landowners may be paid CONSTITUTIONAL LAW 2 CASE DIGESTS

for their land in bonds and not necessarily in cash. Manaay averred that just compensation has always been in the form of money and not in bonds. ISSUE: Whether or not there was a violation of the equal protection clause; Whether or not there is a violation of due process; and Whether or not just compensation, under the agrarian reform program, must be in terms of cash. HELD: NO. The Association had not shown any proof that they belong to a different class exempt from the agrarian reform program. Under the law, classification has been defined as the grouping of persons or things similar to each other in certain particulars and different from each other in these same particulars. To be valid, it must conform to the following requirements: (1) it must be based on substantial distinctions; (2) it must be germane to the purposes of the law; (3) it must not be limited to existing conditions only; and (4) it must apply equally to all the members of the class. Equal protection simply means that all persons or things similarly situated must be treated alike both as to the rights conferred and the liabilities imposed. The Association have not shown that they belong to a different class and entitled to a different treatment. The argument that not only landowners but also owners of other properties must be made to share the burden of implementing land reform must be rejected. There is a substantial distinction between these two classes of owners that is clearly visible except to those who will not see. There is no need to elaborate on this matter. In any event, the Congress is allowed a wide leeway in providing for a valid classification. Its decision is accorded recognition and respect by the courts of justice except only where its discretion is abused to the detriment of the Bill of Rights. In the contrary, it appears that Congress is right in classifying small landowners as part of the agrarian reform program. NO. It is true that the determination of just compensation is a power lodged in the courts. However, there is no law which prohibits administrative bodies like the DAR from determining just compensation. In fact, just compensation can be that amount agreed upon by the landowner and the government – even without judicial intervention so long as both parties agree. The DAR can determine just compensation through appraisers and if the landowner agrees, then judicial intervention is not needed. What is contemplated by law however is that, the just CONSTITUTIONAL LAW 2 CASE DIGESTS

compensation determined by an administrative body is merely preliminary. If the landowner does not agree with the finding of just compensation by an administrative body, then it can go to court and the determination of the latter shall be the final determination. This is even so provided by RA 6657: Section 16 (f): Any party who disagrees with the decision may bring the matter to the court of proper jurisdiction for final determination of just compensation. NO. Money as [sole] payment for just compensation is merely a concept in traditional exercise of eminent domain. The agrarian reform program is a revolutionary exercise of eminent domain. The program will require billions of pesos in funds if all compensation have to be made in cash – if everything is in cash, then the government will not have sufficient money hence, bonds, and other securities, i.e., shares of stocks, may be used for just compensation. Trial with Commissioners Meralco v. Pineda, 206 SCRA 196 (1992) FACTS: MERALCO wanted to construct a 230 KV Transmission line from Barrio Malaya to Tower at Pililla, Rizal in the reat estate property of Teofilo Arayon Sr., Gil de Guzman, Lucito Santiago and Teresa Bautista. Parties attempt negotiations but failed to reach an agreement and offers to pay compensation. Respondents then filed a complaint for eminent domain with the trial court and won the case granting t hem rights to expropriate the property needed. Unfortunately, the petitioner sol d to NAPOCOR its power plant and transmission lines including the property disputed herein. The court then ordered the appraisal of the land in answer for the motion for withdrawal of deposit which respondents were entitled to. However, the appraisal by the commissioners were never completed due to the suspension when petitioners failed a Motion to Dismiss due to the said sale to NAPOCOR. Later, respondents file another motion for payment and the court granted them another sum for the compensation. Petitioner then filed a Motion for Reconsideration that at this stage the respondents are not yet entitled to payment of just compensation as there is no appraisal yet and that court, upon awarding a fraction of sum, based it only on a witness of a credible real estate broker instead of employing the assistance of three commissioners to determine just compensation. ISSUE: CONSTITUTIONAL LAW 2 CASE DIGESTS

Whether or not the employment of three commissioners in the ascertainment of just compensation is dispensable. HELD: NO. The judge's act of determining and ordering the payment of just compensation without the assistance of a Board of Commissioners is a flagrant violation of MERALCO's constitutional right to due process and is a gross violation of the mandate d rule established by the Revised Rules of Court. In an expropriation case where the principal issue is the determination of just compensation, a trial before the Commissioners is indispensable to allow the par ties to present evidence on the issue of just compensation. The appointment of at least 3 competent persons as commissioners to ascertain just compensation for the property sought to be taken is a mandatory requirement in expropriation cases. While it is true that the findings of commissioners may be disregarded and the court may substitute its own estimate of the value, the latter may only do so for valid reasons: 1. where the Commissioners have applied illegal principles to the evidence submitted to them; 2. where they have disregarded a clear preponderance of evidence; 3. where the amount allowed is either grossly inadequate or excessive. Thus, trial with the aid of the commissioners is a substantial right that may no t be done away with capriciously or for no reason at all. Moreover, in such instances, where the report of the commissioners may be disregarded, the trial court may make its own estimate of value from competent evidence that may be gathered from the record. Leca Realty v. Republic, G.R. No. 155605, September 27, 2006 FACTS: Manuela Corporation (Manuela) is a duly registered domestic corporation, principally engaged in the business of leasing commercial spaces in shopping malls to retailers. At the time, respondent owned and operated M Star One, M Star, Starmall, Metropolis Star, and Pacific Mall. Manuela obtained several loans from two syndicates of lenders to finance the costs of two of its buildings. Aside from its Php2.174 billion loan from banks, the company also had Php1.476billion indebtedness to Hero Holdings, Inc. and its trade suppliers, and other parties. The region was then beset by the 1997 Asian financial crisis which prompted banks to stop their lending activities. This severely affected Manuela whose malls did not operate sufficiently, causing serious losses to the company. The adjusted interest CONSTITUTIONAL LAW 2 CASE DIGESTS

rates on Manuela's loans were around 18% to 30%, which contributed to its liquidity problems. The company, however, exerted all efforts to cushion the financial blow by “closing down non-income generating businesses, concentrating on its business of leasing commercial spaces, intensifying collection efforts, reducing personnel, negotiating for restructuring of loan with creditors, and working out a viable payment scheme without giving undue preference to any creditor.” In spite of all these initiatives, Manuela still failed to pay its financial obligations. This forced the company to ask the court to issue a Stay Order and approve its proposed Rehabilitation Plan, which if successfully implemented will “enable it to settle its remaining obligations in an orderly manner, restore its financial viability, and allow it to resume its normal operations.” The trial court subsequently issued the Stay Order, which stated: ‟ a) a stay in the enforcement of all claims, whether for money or otherwise and whether such enforcement is by court action or otherwise, against petitioner MANUELA, its guarantors and sureties not solidarily liable with it; e) directing the payment in full of all administrative expenses incurred after the issuance of this Stay Order. The trial court appointed Marilou Adea as rehabilitation receiver. Adea recommended the approval of Manuela's Rehabilitation Plan and convened with Manuela's creditors for the latter to air their concerns. Leca Realty Corporation (Leca) filed its Comment and/or Formal Claim against Manuela amounting to Php193.7 million, comprised of unpaid rentals, security deposits, interests, and penalty charges. After Leca's receipt of Adea's Report and Recommendation, petitioner questioned the reduction of Manuela's liability, “considering its contractual nature which cannot be impaired during the process of rehabilitation.” The trial court eventually approved the Rehabilitation Plan. Leca's appeal to the Court of Appeals was dismissed for lack of merit. The disagreement is grounded on the fact that the rental rates agreed upon by Leca and Manuela were reduced in the Rehabilitation Plan. There was a gross discrepancy between the amounts of rent agreed upon by the parties and those provided in the Rehabilitation Plan. Leca filed another petition before the appellate court alleging violation of its constitutional right to non-impairment contract and the Interim Rules of Procedure on Corporate Rehabilitation. The Court of Appeals, in denying the petition, ruled: The pendency of the rehabilitation proceedings cannot be interpreted to impair the contractual obligations previously entered into by the contracting parties because the automatic stay of all actions is sanctioned by P.D. [No.] 90-A which provides that "all actions for claims against corporations, partnerships or associations under management or receivership pending before any court, tribunal, board or body CONSTITUTIONAL LAW 2 CASE DIGESTS

shall be suspended accordingly.” Thus, Leca filed a petition for review on certiorari before the Supreme Court. ISSUE: Whether the pendency of the rehabilitation proceedings can justify impairment of contractual obligations previously entered into by the parties. HELD: NO. The pendency of the rehabilitation plan can no justify the impairment of contractual obligations. The amount provided in the rehabilitation plan is null and void. Petitioner, in support of its contention, cites in its Memorandum the treatises of Ateneo Law Dean Cesar L. Villanueva and former SEC Commissioner Danilo L. Concepcion, both known authorities on Corporation Law. In his Article which appeared in the Ateneo Law Journal, Dean Villanueva said: The nature and extent of the power of the SEC to approve and enforce a rehabilitation plan is certainly an important issue. Often, are habilitation plan would require a diminution, if not destruction, of contractual and property rights of some, if not most of the various stakeholders in the petitioning corporation. In the absence of clear coercive legal provisions, the courts of justice and much less the SEC would have no power to amend or destroy the property and contractual rights of private parties, much less relieve a petitioning corporation from its contractual commitments. The Supreme Court, in agreeing with Leca, cited its ruling in The Insular Life Assurance Company, Ltd. v. Court of Appeals, which provides: When the language of the contract is explicit leaving no doubt as to the intention of the drafters thereof, the courts may not read into it any other intention that would contradict its plain import. The Court would be rewriting the contract of lease between Insular and Sun Brothers under the guise of construction were we to interpret the option to renew clause as Sun Brothers propounds it, despite the express provision in the original contract of lease and the contracting parties subsequent acts. As the Court has held in Riviera Filipina, Inc. vs. Court of Appeals, a court, even the Supreme Court, has no right to make new contracts for the parties or ignore those already made by them, simply to avoid seeming hardships. Neither abstract justice nor the rule of liberal construction justifies the creation of a contract for the parties which they did not make themselves or the imposition upon one party to a contract of an obligation not assumed. The Court voided the Rehabilitation Plan insofar as it amends the rental rates agreed upon by the parties. It opined that the change is not justified as the amount of rent is an CONSTITUTIONAL LAW 2 CASE DIGESTS

"essential condition of any lease contract;" thus, any alteration on the rate is tantamount to impairment of stipulation of the parties. Legal Interest for Expropriation Cases NPC v. Angas, 208 SCRA 542 (1992) FACTS: The petitioner National Power Corporation, a government-owned and controlled corporation and the agency through which the government undertakes the ongoing infrastructure and development projects throughout the country, filed two complaints for eminent domain against private respondents with the Court of First Instance (now RTC). The complaint which sought to expropriate certain specified lots situated at Limogao, Saguiaran, Lanao del Sur was for the purpose of the development of hydro-electric power and production of electricity as well as the erection of such subsidiary works and constructions as may be necessarily connected therewith. Both cases were jointly tried upon agreement of the parties. After responsive pleadings were filed and issues joined, a series of hearings before court-designated commissioners were held. After a series of hearings, a consolidated decision was rendered by the lower court, declaring and confirming that the lots mentioned and described in the complaints have entirely been lawfully condemned and expropriated by NAPOCOR, and ordering the latter to pay the landowners certain sums of money as just compensation for their lands expropriated "with legal interest thereon until fully paid. Two consecutive motions for reconsideration of the consolidated decision were filed by NAPOCOR. The same were denied by the court. NAPOCOR did not appeal on the consolidated decision, which became final and executory. Thus, Sittie Sohra Batara, one of the landowners filed an exparte motion for the execution of the decision, praying that petitioner be directed to pay her the unpaid balance for the lands expropriated from her, including legal interest which she computed at 6% per annum. The said motion was granted by the lower court. Thereafter, the lower court directed the petitioner to deposit the sums of money as adjudged in the joint decision. NAPOCOR complied with said order. Pangonatan Cosna Tagol, another landowner filed with the trial court an ex parte motion praying, for the first time, that the legal interest on the just compensation awarded to her by the court be computed at 12% per annum as allegedly "authorized under and by virtue of Circular 416 of the Central Bank issued pursuant to Presidential Decree 116. The lower court CONSTITUTIONAL LAW 2 CASE DIGESTS

granted the said motion allowing 12% interest per annum. Subsequently, the other landowners filed motions also praying that the legal interest on the just compensation awarded to them be computed at 12% per annum, on the basis of which the lower court issued March and August 1981 orders bearing similar import. NAPOCOR moved for the reconsideration of the lower court's last order, which the court denied. NAPOCOR filed a petition for certiorari and mandamus with the Supreme Court. ISSUE: Whether, in the computation of the legal rate of interest on just compensation for expropriated lands, the rate applicable as legal interest is 6% (Article 2209 of the Civil Code) or 12% (Central Bank Circular 416). HELD: YES. The SC declared that the computation of legal interest at 6% per annum is the correct and valid legal interest allowed in payments of just compensation for lands expropriated for public use to herein private respondents by the Government through the National Power Corporation. The Central Bank circular applies only to loan or forbearance of money, goods or credits and to judgments involving such loan or forbearance of money, goods or credits. Herein, the transaction involved is clearly not a loan or forbearance of money, goods or credits but expropriation of certain parcels of land for a public purpose, the payment of which is without stipulation regarding interest, and the interest adjudged by the trial court is in the nature of indemnity for damages. The legal interest required to be paid on the amount of just compensation for the properties expropriated is manifestly in the form of indemnity for damages for the delay in the payment thereof. Therefore, since the kind of interest involved in the joint judgment of the lower court sought to be enforced in this case is interest by way of damages, and not by way of earnings from loans, etc. Article 2209 of the Civil Code shall apply. Wycoco v. Judge Caspillo, G.R. No. 146733, January 13, 2004 FACTS: This case is a consolidated petition of one seeking review of the decision of CA modifying the decision of RTC acting as a Special Agrarian Court, and another for mandamus to compel the RTC to issue a writ of execution and to direct Judge Caspillo to inhibit. Feliciano F. Wycoco is the registered owner of a 94.1690 hectare land. Wycoco voluntarily offered to sell the land to the Department of CONSTITUTIONAL LAW 2 CASE DIGESTS

Agrarian Reform for P14.9 million. DAR offered P2,280,159.82. The area which the DAR offered to acquire excluded idle lands, river and road located therein. Wycoco rejected the offer, prompting the DAR to indorse the case to the Department of Agrarian Reform Adjudication Board (DARAB) for the purpose of fixing the just compensation in a summary administrative proceeding. Thereafter, the DARAB requested LBP to open a trust account in the name of Wycoco and deposited the compensation offered by DAR. In the meantime, the property was distributed to farmer-beneficiaries. On April 13, 1993, Wycoco filed the instant case for determination of just compensation with the Regional Trial Court of Cabanatuan City against DAR and LBP. On March 9, 1994, the DARAB dismissed the case on its hand to give way to the determination of just compensation by the RTC. Meanwhile, DAR and LBP filed their respective answered that the valuation of Wycoco’s property was in accordance with law and that the latter failed to exhaust administrative remedies by not participating in the summary administrative proceedings before the DARAB which has primary jurisdiction over determination of land valuation. On November 14, 1995, the trial court rendered a decision in favor of Wycoco. It ruled that there is no need to present evidence in support of the land valuation in as much as it is of public knowledge that the prevailing market value of agricultural lands sold in Licab, Nueva Ecija is from P135,000.00 to 150,000.00 per hectare. The court thus took judicial notice thereof and fixed the compensation for the entire 94.1690 hectare land at P142,500.00 per hectare or a total of P13,428,082.00. It also awarded Wycoco actual damages for unrealized profits plus legal interest. The DAR and the LBP filed separate petitions before the Court of Appeals. The petition brought by DAR on jurisdictional and procedural issues was dismissed. This prompted Wycoco to file a petition for mandamus before this Court praying that the decision of the Regional Trial Court of Cabanatuan City be executed, and that Judge Caspillo be compelled to inhibit himself from hearing the case. The petition brought by LBP on both substantive and procedural grounds was likewise dismissed by the Court of Appeals. However, the Court of Appeals modified its decision by deducting from the compensation due to Wycoco the amount corresponding to the 3.3672 hectare portion of the 94.1690 hectare land which was found to have been previously sold by Wycoco to the Republic. LBP contended that the Court of Appeals erred in its ruling. ISSUES: ISSUES: Whether or not the RTC acquired jurisdiction over the case acting as Special Agrarian Court; Assuming that it acquired jurisdiction, whether or not the CONSTITUTIONAL LAW 2 CASE DIGESTS

compensation arrived at supported by evidence; Whether or not Wycoco can compel DAR to purchase the entire land; and Whether or not the awards of interest and damages for unrealized profits is valid. HELD: YES. The RTC acting as Special Agrarian Court, acquired jurisdiction of the case. Sections 50 and 57 of Republic Act No. 6657 (Comprehensive Agrarian Reform Law of 1988) provides: Section 50.Quasi-judicial Powers of the DAR. – The DAR is hereby vested with primary jurisdiction to determine and adjudicate agrarian reform matters and shall have exclusive original jurisdiction over all matters involving the implementation of agrarian reform, except those falling under the exclusive jurisdiction of the Department of Agriculture (DA) and the Department of Environment and Natural Resources (DENR)….Section 57.Special Jurisdiction. – The Special Agrarian Court shall have original and exclusive jurisdiction over all petitions for the determination of just compensation to landowners, and the prosecution of all criminal offenses under this Act. NO. The compensation arrived is not supported by evidence. In arriving at the valuation of Wycoco’s land, the trial court took judicial notice of the alleged prevailing market value of agricultural lands without apprising the parties of its intention to take judicial notice thereof. Section 3, Rule 129 of the Rules on Evidence provides: Sec. 3. Judicial Notice. When Hearing Necessary. – During the trial, the court, on its own initiative, or on request of a party, may announce its intention to take judicial notice of any matter and allow the parties to be heard thereon. After trial and before judgment or on appeal, the proper court, on its own initiative, or on request of a party, may take judicial notice of any matter and allow the parties to be heard thereon if such matter is decisive of a material issue in the case. The trial court should have allowed the parties to present evidence thereon instead of practically assuming a valuation without basis. Only the market value was taken into account in determining the just compensation. Since other factors were not considered, the case was remanded for determination of just compensation. NO. DAR cannot be compelled to purchase the entire property voluntarily offered by Wycoco. The power to determine whether a parcel of land may come within the coverage of the Comprehensive Agrarian Reform Program is essentially lodged with the DAR. Tha t Wycoco will suffer damages by the DAR’s non -acquisition of the approximately 10 hectare portion of the entire land which was found to be not suitable for agriculture is no justification to compel DAR to acquire the whole area. YES. Wycoco’s claim for payment of interest is partly meritorious. The trust CONSTITUTIONAL LAW 2 CASE DIGESTS

account opened as the mode of payment of just compensation should be converted to a deposit account. The conversion should be retroactive in application in order to rectify the error committed by the DAR in opening a trust account and to grant the landowners the benefits concomitant to payment in cash or LBP bonds. Otherwise, petitioner’s right to payment of just and valid compensation for the expropriation of his property would be violated. The interest earnings accruing on the deposit account of landowners would suffice to compensate them pending payment of just compensation. Writ of Possession City of Manila v. Oscar Serrano, G.R. No. 142304, June 20, 2001 FACTS: The City Council of Manila enacted Ordinance No. 7833 authorizing the expropriation of certain properties in Manila’s First District in Tondo. One of the properties sought to be expropriated was that supposedly owned by respondents. Petitioner City of Manila filed an amended complaint for expropriation, with the RTC of Manila, against the supposed owners of the lots, which included herein respondents Oscar, Felicitas, Jose, Benjamin, Estelita, Leonora, Adelaida, all surnamed Serrano. Respondents filed a consolidated answer, in which they alleged that their mother, the late Demetria De Guia, that they had been the bona fide occupants of the said parcel of land for more than 40 years; that the expropriation of Lot 1-C would result in their dislocation, it being the only residential land left to them by their deceased mother; and that the lot was exempt from expropriation because dividing the said parcel of land among them would entitle each of them to only about 50 square meters of land. They prayed that judgment be rendered declaring Lot 1-C exempt from expropriation and ordering the cancellation of the notice, regarding the pendency of Civil Case for eminent domain filed by petitioner. Upon motion by petitioner, the trial court issued an order, dated October 9, 1998, directing petitioner to deposit the amount of P1,825,241.00 equivalent to the assessed value of the properties. After petitioner had made the deposit, the trial court issued another order, dated December 15, 1998, directing the issuance of a writ of possession in favor of petitioner. Respondents filed a petition for certiorari with the Court of Appeals, alleging that the expropriation of Lot 1-C would render respondents, who are CONSTITUTIONAL LAW 2 CASE DIGESTS

actual occupants thereof, landless because R.A. No. 7279 provides that properties consisting of residential lands not exceeding 300 square meters in highly urbanized cities are exempt from expropriation; that respondents would only receive around 49 square meters each after the partition of Lot 1-C; and that R.A. No.7279 was not meant to deprive an owner of the entire residential land but only that in excess of 300 square meters. On November 16, 1999, the Court of Appeals rendered a decision holding that Lot 1-C is not exempt from expropriation because it undeniably exceeds 300 square meters which is no longer considered a small property within the framework of R.A. No.7279. However, it held that in accordance with the ruling in Filstream International Inc. v. Court of Appeals, the other modes of acquisition of lands enumerated in the law must first be tried by the city government before it can resort to expropriation. As petitioner failed to show that it had done so, the Court of Appeals gave judgment for respondents and enjoined petitioner from expropriating Lot1-C. ISSUE: Whether or not the CA erred in concluding that the Order of the RTC which authorizes the immediate entry of the City as the expropriating agency into the property sought to be expropriated upon the deposit thereof as tantamount to condemnation of the property. HELD: YES. Rule 67, (2) provides: Upon the filing of the complaint or at any time thereafter and after due notice to the defendant, the plaintiff shall have the right to take or enter upon the possession of the real property involved if he deposits with the authorized government depositary an amount equivalent to the assessed value of the property for purposes of taxation to be held by such bank subject to the orders of the court. A writ of execution may be issued by a court upon the filing by the government of a complaint for expropriation sufficient in form and substance and upon deposit made by the government of the amount equivalent to the assessed value of the property subject to expropriation. Upon compliance with these requirements, the issuance of the writ of possession becomes ministerial. In this case, these requirements were satisfied and, therefore, it became the ministerial duty of the court to issue the writ of possession. However, this does not amount to an order of condemnation. Whether petitioner has complied with these provisions requires the presentation of evidence, although in CONSTITUTIONAL LAW 2 CASE DIGESTS

its amended complaint petitioner did allege that it had complied with the requirements. The determination of this question must await that hearing on the complaint for expropriation, particularly the hearing for the condemnation of the properties sought to be expropriated. Expropriation proceedings consist of two stages: first, condemnation of the property after it is determined that its acquisition will be for a public purpose or public use and, second, the determination of just compensation to be paid for the taking of the private property to be made by the court with the assistance of not more than three commissioners. Rep. v. Holy Trinity Realty Dev. Corp., 551 SCRA 303 FACTS: The Republic of the Philippines, represented by the Toll Regulatory Board (TRB), filed with the RTC a Consolidated Complaint for Expropriation against landowners whose properties would be affected by the construction, rehabilitation, and expansion of the North Luzon Expressway. The Holy Trinity Reality and Development Corporation was one of the affected landowners. TRB filed an Urgent Ex-Parte Motion for the Issuance of a Writ of Possession, manifesting that it deposited a sufficient amount to cover the payment of 100% of the zonal value of the affected properties (in the total amount of 28,406,700 pesos) with the Land Bank of the Philippines, South Harbor Branch (LBP-South Harbor), an authorized government depository. TRB maintained that since it had already complied with the provisions of Sec. 4 of RA 8974 in relation to Sec. 2 of Rule 67 of the Rules of Court, the issuance of the writ of possession becomes ministerial on the part of the RTC. RTC issued an Order for the Issuance of the Writ of Possession as well as the Writ of Possession itself. Holy Trinity moved for reconsideration. The Sheriff filed with the RTC a Report on Writ of Possession stating that since none of the landowners voluntarily vacated the properties subject of the expropriation proceedings, the assistance of the PNP would be necessary in implementing the Writ of Possession. Accordingly, TRB, through OSG, filed with the RTC an Omnibus Motion praying for an Order directing the PNP to assist the Sheriff in the implementation of the Writ of Possession. The Holy Trinity filed with the RTC a Motion to Withdraw Deposit, praying that it be allowed to withdraw 22,968,000 out of 28,406,700, including the interest which accrued thereon. RTC granted the motion (except as to the interest) since Holy Trinity already proved its absolute ownership over the properties and paid the taxes due to the government. RTC conducted a hearing on the accrued interest, after which it directed the issuance CONSTITUTIONAL LAW 2 CASE DIGESTS

of an Order of Expropriation, and granted TRB a period of 30 days to inquire from LBP-South Harbor whether the deposit made by DPWH with the bank relative to the expropriation proceedings is earning interest or not. TRB submitted a Manifestation to which was attached the letter by Atty. Osoteo stating that the DPWH Expropriation Account was an interest bearing current account. RTC resolved the issue by ruling that the interest earnings from the deposit of 22,968,000 (under the principle of accession) are considered as fruits and should properly pertain to the property owner (in this case, Holy Trinity). Upon motion of TRB, it issued an Order of Expropriation. But later on, it reversed itself stating that the issue as to who is entitled to the payment of interest should be ventilated before the Board of Commissioners. The CA reversed ISSUE: Whether or not Holy Trinity is only entitled to the amount equivalent to the zonal value of the expropriated property and not to the accrued interest. HELD: NO. Holy Trinity is also entitled to the accrued interest. TRB failed to distinguish between the expropriation procedures under RA 8974 and Rule 67. The former specifically governs expropriation proceedings for national government infrastructure projects. In the case of Republic vs. Gingoyon, the SC ruled that under RA 8974, the government is required to make immediate payment to the property owner upon the filing of the complaint to be entitled to a writ of possession, whereas in Rule 67, the government is authorized only to make an initial deposit with an authorized government depositary. In the case at bar, the proceedings deal with the expropriation of properties intended for a national government infrastructure project. Thus, the RTC was correct in applying the procedure laid out in RA 8974, by requiring the deposit of the amount equivalent to 100% of the zonal value of the properties sought to be expropriated. The controversy though arises not from the amount of the deposit but as to the ownership of the interest that had since accrued on the deposited amount. The SC agrees with the ruling of the CA. The critical factor in the different modes of effecting delivery which gives legal effect to the act is the actual intention to deliver on the part of the party making such delivery. The intention of the TRB in depositing such amount through DPWH was clearly to comply with the requirement of immediate payment in RA 8974, so that it could already secure a writ of possession over the properties subject of the expropriation and commence CONSTITUTIONAL LAW 2 CASE DIGESTS

implementation of the project. Nonetheless, the SC finds it necessary to emphasize that Holy Trinity is determined to be the owner of only a part of the amount deposited in the expropriation account, in the sum of P22,968,000.00. Hence, it is entitled by right of accession to the interest that had accrued to the said amount only.

Expropriation of Utilities, Landed Estates and Municipal Property City of Baguio vs. Nawasa, 106 Phil. 114 (1959) FACTS: Plaintiff a municipal corporation filed a complaint against defendant a public corporation, created under Act.1383. It contends that the said act does not include within its purview the Baguio Water Works system, assuming that it does, is unconstitutional because it deprives the plaintiff ownership, control and operation of said water works without just compensation and due process of law. The defendant filed a motion to dismiss ion the ground that it is not a proper exercise of police power and eminent domain. The court denied the motion and ordered the defendants to file an answer. The court holds that the water works system of Baguio belongs to private property and cannot be expropriated without just compensation. Sec. 8 of R.A.1383 provides for the exchange of the NAWASA assets for the value of the water works system of Baguio is unconstitutional for this is not just compensation. Defendants motion for reconsideration was denied hence this appeal. ISSUE: Whether or not there is a valid exercise of police power or power of eminent domain. HELD: NO. R.A. 1383 does not constitute a valid exercise of police power. The act does not confiscate, destroy or appropriate property belonging to a municipal corporation. It merely directs that all water works belonging to cities, municipalities and municipal districts in the Philippines to be transferred to the CONSTITUTIONAL LAW 2 CASE DIGESTS

NAWASA. The purpose is placing them under the control and supervision of an agency with a view to promoting their efficient management, but in so doing does not confiscate them because it directs that they be paid with equal value of the assets of NAWASA. The Baguio water works system is not like a public road, the park, street other public property held in trust by a municipal corporation for the benefit of the public. But it is a property of a municipal corporation, water works cannot be taken away except for public use and upon payment of just compensation. Judgment affirmed.

Zamboanga del Norte vs. City of Zamboanga, 22 SCRA 1334 (1968) Facts: Prior to its incorporation as a chartered city, the Municipality of Zamboanga used to be the provincial capital of the then Zamboanga Province. On October 12, 1936, Commonwealth Act 39 was approved converting the Municipality of Zamboanga into Zamboanga City. Sec. 50 of the Act also provided that “Buildings and properties which the province shall abandon upon the transfer of the capital to another place will be acquired and paid for by the City of Zamboanga at a price to be fixed by the Auditor General.” Such properties include lots of capitol site, schools, hospitals, leprosarium, high school playgrounds, burleighs, and hydroelectric sites. On June 6, 1952, Republic Act 711 was approved dividing the province of Zamboanga into two (2): Zamboanga del Norte and Zamboanga del Sur. As to how the assets and obligations of the old province were to be divided between the two new ones, Sec. 6 of that law provided “Upon the approval of this Act, the funds, assets and other properties and the obligations of the province of Zamboanga shall be divided equitably between the Province of Zamboanga del Norte and the Province of Zamboanga del Sur by the President of the Philippines, upon the recommendation of the Auditor General.” However, on June 17, 1961, Republic Act 3039 was approved amending Sec. 50 of Commonwealth Act 39 by providing that, “All buildings, properties and assets belonging to the former province of Zamboanga and located within the City of Zamboanga are hereby transferred, free of charge, in favor of the said City of Zamboanga.” This constrained Zamboanga del Norte to file on March 5, 1962, a complaint against defendants-appellants Zamboanga City; that, among others, Republic Act 3039 be declared unconstitutional for depriving Zamboanga del Norte of property without CONSTITUTIONAL LAW 2 CASE DIGESTS

due process and just compensation. Lower court declared RA 3039 unconstitutional as it deprives Zamboanga del Norte of its private properties. Hence the appeal. ISSUE: Whether RA 3039 is unconstitutional on the grounds that it deprives Zamboanga del Norte of its private properties. HELD: NO. RA 3039 is valid. The properties petitioned by Zamboanga del Norte is a public property. The validity of the law ultimately depends on the nature of the 50 lots and buildings thereon in question. For, the matter involved here is the extent of legislative control over the properties of a municipal corporation, of which a province is one. The principle itself is simple: If the property is owned by the municipality (meaning municipal corporation) in its public and governmental capacity, the property is public and Congress has absolute control over it. But if the property is owned in its private or proprietary capacity, then it is patrimonial and Congress has no absolute control. The municipality cannot be deprived of it without due process and payment of just compensation. The classification of properties other than those for public use in the municipalities as patrimonial under Art. 424 of the Civil Code — is “… without prejudice to the provisions of special laws.” For purpose of this article, the principles, obtaining under the Law of Municipal Corporations can be considered as “special laws”. Hence, the classification of municipal property devoted for distinctly governmental purposes as public should prevail over the Civil Code classification in this particular case. TAXATION Tax Exemptions CIR v CA & YMCA GR No 124043, October 14, 1998 FACTS: CONSTITUTIONAL LAW 2 CASE DIGESTS

In 1980, YMCA earned an income of 676,829.80 from leasing out a portion of its premises to small shop owners, like restaurants and canteen operators and 44,259 from parking fees collected from non-members. On July 2, 1984, the CIR issued an assessment to YMCA for deficiency taxes which included the income from lease of YMCA’s real property. YMCA formally protested the assessment but the CIR denied the claims of YMCA. On appeal, the CTA ruled in favor of YMCA and excluded income from lease to small shop owners and parking fees. However, the CA reversed the CTA but affirmed the CTA upon motion for reconsideration. ISSUE: Whether the rental income of YMCA is taxable RULING: Yes. The exemption claimed by YMCA is expressly disallowed by the very wording of then Section 27 of the NIRC which mandates that the income of exempt organizations (such as the YMCA) from any of their properties, real or personal, be subject to the tax imposed by the same Code. While the income received by the organizations enumerated in Section 26 of the NIRC is, as a rule, exempted from the payment of tax in respect to income received by them as such, the exemption does not apply to income derived from any of their properties, real or personal or from any of their activities conducted for profit, regardless of the disposition made of such income. LLADOC VS. COMMISSIONER OF INTERNAL REVENUE [14 SCRA 292; NO.L-19201; 16 JUN 1965] Facts: Sometime in 1957, M.B. Estate Inc., of Bacolod City, donated 10,000.00 pesos in cash to Fr. Crispin Ruiz, the parish priest of Victorias, Negros Occidental, and predecessor of Fr. Lladoc, for the construction of a new Catholic church in the locality. The donated amount was spent for such purpose. On March 3, 1958, the donor M.B. Estate filed the donor's gift tax return. Under date of April 29, 1960. Commissioner of Internal Revenue issued an assessment for the donee's gift tax against the Catholic Parish of Victorias of which petitioner was the parish priest. CONSTITUTIONAL LAW 2 CASE DIGESTS

Issue: Whether or not the imposition of gift tax despite the fact the Fr. Lladoc was not the Parish priest at the time of donation, Catholic Parish priest of Victorias did not have juridical personality as the constitutional exemption for religious purpose is valid. Held: Yes, imposition of the gift tax was valid, under Section 22(3) Article VI of the Constitution contemplates exemption only from payment of taxes assessed on such properties as Property taxes contra distinguished from Excise taxes The imposition of the gift tax on the property used for religious purpose is not a violation of the Constitution. A gift tax is not a property by way of gift inter vivos. The head of the Diocese and not the parish priest is the real party in interest in the imposition of the donee's tax on the property donated to the church for religious purpose. Abra Valley College vs Aquino G.R. No. L-39086 FACTS: Petitioner, an educational corporation and institution of higher learning duly incorporated with the Securities and Exchange Commission in 1948, filed a complaint to annul and declare void the “Notice of Seizure’ and the “Notice of Sale” of its lot and building located at Bangued, Abra, for non-payment of real estate taxes and penalties amounting to P5,140.31. Said “Notice of Seizure” by respondents Municipal Treasurer and Provincial Treasurer, defendants below, was issued for the satisfaction of the said taxes thereon. The parties entered into a stipulation of facts adopted and embodied by the trial court in its questioned decision. The trial court ruled for the government, holding that the second floor of the building is being used by the director for residential purposes and that the ground floor used and rented by Northern Marketing CONSTITUTIONAL LAW 2 CASE DIGESTS

Corporation, a commercial establishment, and thus the property is not being used exclusively for educational purposes. Instead of perfecting an appeal, petitioner availed of the instant petition for review on certiorari with prayer for preliminary injunction before the Supreme Court, by filing said petition on 17 August 1974. ISSUE: Whether or not the lot and building are used exclusively for educational purposes. HELD: Section 22, paragraph 3, Article VI, of the then 1935 Philippine Constitution, expressly grants exemption from realty taxes for cemeteries, churches and parsonages or convents appurtenant thereto, and all lands, buildings, and improvements used exclusively for religious, charitable or educational purposes.ン Reasonable emphasis has always been made that the exemption extends to facilities which are incidental to and reasonably necessary for the accomplishment of the main purposes. The use of the school building or lot for commercial purposes is neither contemplated by law, nor by jurisprudence. In the case at bar, the lease of the first floor of the building to the Northern Marketing Corporation cannot by any stretch of the imagination be considered incidental to the purpose of education. The test of exemption from taxation is the use of the property for purposes mentioned in the Constitution. The decision of the CFI Abra (Branch I) is affirmed subject to the modification that half of the assessed tax be returned to the petitioner. The modification is derived from the fact that the ground floor is being used for commercial purposes (leased) and the second floor being used as incidental to education (residence of the director). Double Taxation PUNSALAN VS. MUNICIPAL BOARD OF MANILA 95 PHIL 46; NO.L-4817; 26 MAY 1954 Facts: Petitioners, who are professionals in the city, assail Ordinance No. 3398 together with the law authorizing it (Section 18 of the Revised Charter of the City of CONSTITUTIONAL LAW 2 CASE DIGESTS

Manila). The ordinance imposes a municipal occupation tax on persons exercising various professions in the city and penalizes non-payment of the same. The law authorizing said ordinance empowers the Municipal Board of the city to impose a municipal occupation tax on persons engaged in various professions. Petitioners, having already paid their occupation tax under section 201 of the National Internal Revenue Code, paid the tax under protest as imposed by Ordinance No. 3398. The lower court declared the ordinance invalid and affirmed the validity of the law authorizing it. Issue: Whether or Not the ordinance and law authorizing it constitute class legislation, and authorize what amounts to double taxation. Held: The Legislature may, in its discretion, select what occupations shall be taxed, and in its discretion may tax all, or select classes of occupation for taxation, and leave others untaxed. It is not for the courts to judge which cities or municipalities should be empowered to impose occupation taxes aside from that imposed by the National Government. That matter is within the domain of political departments. The argument against double taxation may not be invoked if one tax is imposed by the state and the other is imposed by the city. It is widely recognized that there is nothing inherently terrible in the requirement that taxes be exacted with respect to the same occupation by both the state and the political subdivisions thereof. Judgment of the lower court is reversed with regards to the ordinance and affirmed as to the law authorizing it. License Fees Physical Therapy Organization vs Municpal Board GR 10448 30 August 1957 Facts: Municipal Board of Manila enacted Ordinance 3659 regulating the operations of massage clinics in Manila penalizing and enforcing permit fee for its operation. Petitioner appealed for the dismissal of the ordinance. They contend that City of Manila is without authority to regulate the operation of massagists and the operation of massage clinics and that the fee is unreasonable and unconscionable. Trial court dismissed the petition. CONSTITUTIONAL LAW 2 CASE DIGESTS

Issue: Whether or not license fee enforced by the Municipal Board is valid? Decision: Decision affirmed. The end sought to be attained in the Ordinance is to prevent the commission of immorality and the practice of prostitution in an establishment masquerading as a massage clinic where the operators thereof offer to massage or manipulate superficial parts of the bodies of customers for hygienic and aesthetic purposes. The permit fee is made payable by the operator of a massage clinic who may not be a massagist himself. Compared to permit fees required in other operations, P100.00 may appear to be too large and rather unreasonable. Manila Municipal Board considered the practice of hygienic and aesthetic massage not as a useful and beneficial occupation which will promote and is conducive to public morals, and consequently, imposed the said permit fee for its regulation. DUE PROCESS Meaning of Life, Liberty and Property Libanan v Sandiganbayan; G.R. No. 112386; 14 Jun 1994; 233 SCRA 163 FACTS: Petitioner Marcelino C, Libanan, the incumbent Vice-Governor of Eastern Samar, was a member of the Sangguniang Panlalawigan of that province prior to the 11 May 1992 elections. He was suspended from office after he was charged before the Sandiganbayan with having violated Section 3(e) of Republic Act No. 3019. ISSUE(S): Whether or not Libanan’s suspension violated his constitutional right to due process. HELD: NO. The suspension order cannot amount to a deprivation of property without due process of law. Public office is “a public agency or trust,” and it is not the property envisioned by the Constitutional provision which petitioner invokes. CONSTITUTIONAL LAW 2 CASE DIGESTS

Petition is DISMISSED.

Substantive Due Process Villegas vs. Hu Chong Tsai Pao Ho 86 SCRA 275 (1978) G.R. No. L-29646, November 10, 1978 Fact: September 17, 1968 of respondent Judge rendered in favor of the respondent and against the petitioner, declaring Ordinance No. 6 37 of the City of Manila null and void. The Ordinance No. 6537 was passed by the Municipal Board of Manila on February 22, 1968 and signed by the herein petitioner who prohibits aliens from being employed or to engage or participate in any position or occupation or business enumerated therein, whether permanent, temporary or casual, without first securing an employment permit from the Mayor of Manila and paying the permit fee of P50.00 except persons employed in the diplomatic or consular missions of foreign countries, or in the technical assistance programs of both the Philippine Government and any foreign government, and those working in their respective households, and members of religious orders or congregations, sect or denomination, who are not paid monetarily or in kind. Violations of this ordinance is punishable by an imprisonment of not less than three (3) months to six (6) months or fine of not less than P100.00 but not more than P200.00 or both such fine and imprisonment, upon conviction. Private respondent who was employed in Manila, filed a petition with the praying for the issuance of the writ of preliminary injunction and restraining order to stop the enforcement of Ordinance No. 6537 as well as for a judgment declaring said Ordinance No. 6537 null and void. Adrian Avilado Antazo Issue: Whether the Petitioner’s ordinance is arbitrary, oppressive and unreasonable, being applied only to aliens who are thus, deprived of their rights to life, liberty and property and therefore, violates the due process and equal protection clauses of the Constitution.Adrian Avilado Antazo CONSTITUTIONAL LAW 2 CASE DIGESTS

Held: Yes, While it is true that the first part which requires that the alien shall secure an employment permit from the Mayor involves the exercise of discretion and judgment in the processing and approval or disapproval of applications for employment permits and therefore is regulatory in character the second part which requires the payment of P50.00 as employee’s fee is not regulatory but a revenue measure. There is no logic or justification in exacting P50.00 from aliens who have been cleared for employment. It is obvious that the purpose of the ordinance is to raise money under the guise of regulation. It is unreasonable not only because it is excessive but because it fails to consider valid substantial differences in situation among individual aliens who are required to pay it. The Ordinance does not lay down any criterion or standard to guide the Mayor in the exercise of his discretion. It has been held that where an ordinance of a municipality fails to state any policy or to set up any standard to guide or limit the mayor’s action, expresses no purpose to be attained by requiring a permit, enumerates no conditions for its grant or refusal, and entirely lacks standard, thus conferring upon the Mayor arbitrary and unrestricted power to grant or deny the issuance of building permits, such ordinance is invalid, being an undefined and unlimited delegation of power to allow or prevent an activity per se lawful. Requiring a person before he can be employed to get a permit from the City Mayor of Manila who may withhold or refuse it at will is tantamount to denying him the basic right of the people in the Philippines to engage in a means of livelihood. While it is true that the Philippines as a State is not obliged to admit aliens within its territory, once an alien is admitted, he cannot be deprived of life without due process of law. This guarantee includes the means of livelihood. The shelter of protection under the due process and equal protection clause is given to all persons, both aliens and citizens. Rubi vs Provincial Board of Mindoro G.R. No. L-14078; March 7, 1919; 39 Phil 660 FACTS: The case is an application for habeas corpus in favor of Rubi and other Manguianes of the Province of Mindoro. It is alleged that the Maguianes are being illegally deprived of their liberty by the provincial officials of that province. Rubi CONSTITUTIONAL LAW 2 CASE DIGESTS

and his companions are said to be held on the reservation established at Tigbao, Mindoro, against their will, and one Dabalos is said to be held under the custody of the provincial sheriff in the prison at Calapan for having run away from the reservation. The provincial governor of Mindoro and the provincial board thereof directed the Manguianes in question to take up their habitation in Tigbao, a site on the shore of Lake Naujan, selected by the provincial governor and approved by the provincial board. The action was taken in accordance with section 2145 of the Administrative Code of 1917, and was duly approved by the Secretary of the Interior as required by said action. Section 2145 of the Administrative Code of 1917 reads as follows: SEC. 2145. Establishment of non-Christian upon sites selected by provincial governor. — With the prior approval of the Department Head, the provincial governor of any province in which non-Christian inhabitants are found is authorized, when such a course is deemed necessary in the interest of law and order, to direct such inhabitants to take up their habitation on sites on unoccupied public lands to be selected by him an approved by the provincial board. Petitioners, however, challenge the validity of this section of the Administrative Code. ISSUE: Does section 2145 of the Administrative Code of 1917 constitute an unlawful delegation of legislative power by the Philippine Legislature to a provincial official and a department head, therefore making it unconstitutional? HELD: No. The Philippine Legislature has here conferred authority upon the Province of Mindoro, to be exercised by the provincial governor and the provincial board. In determining whether the delegation of legislative power is valid or not, the distinction is between the delegation of power to make the law, which necessarily involves a discretion as to what it shall be, and conferring an authority or discretion as to its execution, to be exercised under and in pursuance of the law. The first cannot be done; to the later no valid objection can be made. Discretion may be committed by the Legislature to an executive department or official. The CONSTITUTIONAL LAW 2 CASE DIGESTS

Legislature may make decisions of executive departments of subordinate official thereof, to whom it has committed the execution of certain acts, final on questions of fact. The growing tendency in the decision is to give prominence to the "necessity" of the case. In enacting the said provision of the Administrative Code, the Legislature merely conferred upon the provincial governor, with the approval of the provincial board and the Department Head, discretionary authority as to the execution of the law. This is necessary since the provincial governor and the provincial board, as the official representatives of the province, are better qualified to judge “when such as course is deemed necessary in the interest of law and order”. As officials charged with the administration of the province and the protection of its inhabitants, they are better fitted to select sites which have the conditions most favorable for improving the people who have the misfortune of being in a backward state. Hence, Section 2145 of the Administrative Code of 1917 is not an unlawful delegation of legislative power by the Philippine Legislature to provincial official and a department head.

Void Due Process/ Overbreadth ESTRADA v SANDIGANBAYAN G.R. No. 148560, November 19, 2001 Facts: Petitioner Joseph Estrada prosecuted An Act Defining and Penalizing the Crime of Plunder, wishes to impress upon the Court that the assailed law is so defectively fashioned that it crosses that thin but distinct line which divides the valid from the constitutionally infirm. His contentions are mainly based on the effects of the said law that it suffers from the vice of vagueness; it dispenses with the "reasonable doubt" standard in criminal prosecutions; and it abolishes the element of mens rea in crimes already punishable under The Revised Penal Code saying that it violates the fundamental rights of the accused. CONSTITUTIONAL LAW 2 CASE DIGESTS

The focal point of the case is the alleged “vagueness” of the law in the terms it uses. Particularly, this terms are: combination, series and unwarranted. Because of this, the petitioner uses the facial challenge on the validity of the mentioned law. Issue: Whether or not the petitioner possesses the locus standi to attack the validity of the law using the facial challenge. Ruling: On how the law uses the terms combination and series does not constitute vagueness. The petitioner’s contention that it would not give a fair warning and sufficient notice of what the law seeks to penalize cannot be plausibly argued. Void-for-vagueness doctrine is manifestly misplaced under the petitioner’s reliance since ordinary intelligence can understand what conduct is prohibited by the statute. It can only be invoked against that specie of legislation that is utterly vague on its face, wherein clarification by a saving clause or construction cannot be invoked. Said doctrine may not invoked in this case since the statute is clear and free from ambiguity. Vagueness doctrine merely requires a reasonable degree of certainty for the statute to be upheld, not absolute precision or mathematical exactitude. On the other hand, overbreadth doctrine decrees that governmental purpose may not be achieved by means which sweep unnecessarily broadly and thereby invade the area of protected freedoms. Doctrine of strict scrutiny holds that a facial challenge is allowed to be made to vague statute and to one which is overbroad because of possible chilling effect upon protected speech. Furthermore, in the area of criminal law, the law cannot take chances as in the area of free speech. A facial challenge to legislative acts is the most difficult challenge to mount successfully since the challenger must establish that no set of circumstances exists. Doctrines mentioned are analytical tools developed for facial challenge of a statute in free speech cases. With respect to such statue, the established rule is that one to who application of a statute is constitutional will not be heard to attack the statute on the ground that impliedly it might also be taken as applying to other persons or other situations in which its application might be unconstitutional. On its face invalidation of statues results in striking them down CONSTITUTIONAL LAW 2 CASE DIGESTS

entirely on the ground that they might be applied to parties not before the Court whose activities are constitutionally protected. It is evident that the purported ambiguity of the Plunder Law is more imagined than real. The crime of plunder as a malum in se is deemed to have been resolve in the Congress’ decision to include it among the heinous crime punishable by reclusion perpetua to death. Supreme Court holds the plunder law constitutional and petition is dismissed for lacking merit.

Ong vs Sandiganbayan G.R. No. 126858 Sept. 16, 2005 FACTS: Congressman Bonifacio H. Gillego executed a Complaint-Affidavit, claiming that petitioner Jose U. Ong, then Commissioner of the BIR, has amassed properties worth disproportionately more than his lawful income. Ong submitted an explanation and analysis of fund sourcing, reporting his net worth covering the calendar years 1989 to 1991 and showing his sources and uses of funds, the sources of the increase in his net worth and his net worth as of December 13, 1991. Ong filed a Counter-Affidavit, submitting his Statement of Assets and Liabilities for the years 1988-1990, income tax return for 1988, bank certificate showing that he obtained a loan from Allied Bank, certificate from SGV & Co. showing that he received retirement benefits from the latter, a document entitled Acknowledgement of Trust showing that he acquired one of the questioned assets for his brother-in-law, and other documents explaining the sources of funds with which he acquired the questioned assets. ISSUE: Does the presumption of innocence apply to forfeiture proceedings? CONSTITUTIONAL LAW 2 CASE DIGESTS

RULING: No. The presumption of innocence clause of the Constitution refers to criminal prosecutions and not to forfeiture proceedings which are civil actions in rem. The Constitution is likewise not violated by RA 1379 because statutes which declare that as a matter of law a particular inference follows from the proof of a particular fact, one fact becoming prima facie evidence of another, are not necessarily invalid, the effect of the presumption being merely to shift the burden of proof upon the adverse party. The presumption of innocence clause is not violated by Sec. 2 of RA 1379 which states that property acquired by a public officer or employee during his incumbency in an amount which is manifestly out of proportion to his salary as such public officer or employee and to his other lawful income and the income from legitimately acquired property shall be presumed prima facie to have been unlawfully acquired. As elaborated by Fr. Joaquin Bernas, under the principle of presumption of innocence, it is merely required of the State to establish a prima facie case, after which the burden of proof shifts to the accused.

DUE PROCESS Publication Requirement Republic vs Extelcom, 373 SCRA 316; GR 147096, January 15, 2002 Facts: National Telecommunications Commission (NTC) granted Bayantel the provisional authority to operate a Cellular Mobile Telephone System/Service (CMTS) on its own initiative applying Rule 15, Section 3 of its 1987 Rules of Practice and Procedures. Respondent Extelcom contends that the NTC should have applied the Revised Rules which were filed with the Office of the National Administrative Register CONSTITUTIONAL LAW 2 CASE DIGESTS

where the phrase “on its own initiative” were deleted and since the 1993 Revised Rules were filed with the UP Law Center. Issue: WON the 1993 Revised Rules which was filed in the UP Law Center is the law in force and effect in granting provisional authority. Held: No. There is nothing in the Administrative Code of 1987 which implies that the filing of the rules with the UP Law Center is the operative act that gives the rules force and effect. The National Administrative Register is merely a bulletin of codified rules. Publication in the Official Gazette or a newspaper of general circulation is a condition sine qua non before statutes, rules and regulations can take effect.

Impartial Court or Tribunal Tumey vs. Ohio [273 US 510, 7 March 1927] Facts: Tumey was arrested at White Oak, and was brought before Mayor Pugh, of the village of North College Hill, charged with unlawfully possessing intoxicating liquor. He moved for his dismissal because of the disqualification of the mayor to try him under the 14th Amendment. The mayor denied the motion, proceeded to the trial, convicted Tumey of unlawfully possessing intoxicating liquor within Hamilton county as charged, fined him $100, and ordered that he be imprisoned until the fine and costs were paid. Tumey obtained a bill of exceptions and carried the case on error to the court of common pleas of Hamilton county. That court heard the case and reversed the judgment, on the ground that the mayor was disqualified as claimed. The state sought review by the Court of Appeals of the First Appellate District of Ohio, which reversed the common pleas and affirmed the judgment of the mayor. On 4 May 1926, the state Supreme Court refused Tumey’s application to require the Court of Appeals to certify its record in the case. Tumey then filed a petition in error in that court as of right, asking that the judgment of the mayor’s court and of the appellate court be reversed on constitutional grounds. On 11 CONSTITUTIONAL LAW 2 CASE DIGESTS

May 1926, the Supreme Court adjudged that the petition be dismissed for the reason that no debatable constitutional question was involved in the cause. The judgment was then brought to the US Supreme Court upon a writ of error allowed by the Chief Justice of the state Supreme Court, to which it was rightly directed. Issue: Whether the pecuniary interest of the Mayor and his village, and the system of courts in prosecuting violations of the Prohibition Act, renders the mayor disqualified from hearing the case. Held: All questions of judicial qualification may not involve constitutional validity. Thus matters of kinship, personal bias, state policy, remoteness of interest would seem generally to be matters merely of legislative discretion. But it certainly violates the 14th Amendment and deprives a defendant in a criminal case of due process of law to subject his liberty or property to the judgment of a court, the judge of which has a direct, personal, substantial pecuniary interest in reaching a conclusion against him in his case. Herein, the mayor has authority, which he exercised in the case, to order that the person sentenced to pay a fine shall remain in prison until the fine and costs are paid. The mayor thus has a direct personal pecuniary interest in convicting the defendant who came before him for trial, in the $12 of costs imposed in his behalf, which he would not have received if the defendant had been acquitted. This was not exceptional, but was the result of the normal operation of the law and the ordinance. The system by which an inferior judge is paid for his service only when he convicts the defendant has not become so embedded by custom in the general practice, either at common law or in this country, that it can be regarded as due process of law, unless the costs usually imposed are so small that they may be properly ignored as within the maxim “de minimis non curat lex.” The Court cannot regard the prospect of receipt or loss of such an emolument in each case as a minute, remote, trifling, or insignificant interest. It is certainly not fair to each defendant brought before the mayor for the careful and judicial consideration of his guilt or innocence that the prospect of such a prospective loss by the mayor should weigh against his acquittal. But the pecuniary interest of the mayor in the result of his judgment is not the only reason for holding that due process of law is denied to the defendant here. The statutes were drawn to stimulate small municipalities, in the country part of counties in which there are large cities, to organize and maintain courts to CONSTITUTIONAL LAW 2 CASE DIGESTS

try persons accused of violations of the Prohibition Act everywhere in the county. The inducement is offered of dividing between the state and the village the large fines provided by the law for its violations. The trial is to be had before a mayor without a jury, without opportunity for retrial, and with a review confined to questions of law presented by a bill of exceptions, with no opportunity by the reviewing court to set aside the judgment on the weighing of evidence, unless it should appear to be so manifestly against the evidence as to indicate mistake, bias, or willful disregard of duty by the trial court. Thus, no matter what the evidence was against him, the defendant had the right to have an impartial judge. He seasonably raised the objection, and was entitled to halt the trial because of the disqualification of the judge, which existed both because of his direct pecuniary interest in the outcome, and because of his official motive to convict and to graduate the fine to help the financial needs of the village. There were thus presented at the outset both features of the disqualification. The judgment of the Supreme Court of Ohio is reversed, and the cause remanded for further proceedings not inconsistent with the present opinion. Tabuena vs Sandiganbayan GR 103501 17 February 1997 Facts: Luis Tabuena as General Manager of MIAA received direct order from Marcos to pay directly to his office sum of 55mio in cash to pay for MIAAs liability to PNCC. He then received Presidential Memorandum from Fe Gimenez (secretary). The money was delivered in cash in three withdrawals, no vouchers prepared to support the disbursement although Gimenez issued a receipt on the third delivery for the entire amount. Tabuena was accused and convicted of the crime of malversation by Sandiganbayan for defrauding the government, taking and misappropriating money when there is no outstanding obligation between MIAA and PNCC. Petitioner contended that he was acting in good faith when the office of the president directed him to deliver the said amount to his office – “person who acts in obedience to an order issued by a superior for some lawful purpose.” Issue: Whether or not Sandiganbayan violated due process on the ground of departing from that common standard of fairness and impartiality? CONSTITUTIONAL LAW 2 CASE DIGESTS

Decision: Sandiganbayan decision reversed and set aside. Tabuena and Peralta are acquitted of the crime of malversation. The majority believes that the interference by the Sandiganbayan Justices was just too excessive that it cannot be justified under the norm applied to a jury trial, or even under the standard employed in a non-jury trial where the judge is admittedly given more leeway in propounding questions to clarify points and to elicit additional relevant evidence. It is never proper for a judge to discharge the duties of a prosecuting attorney. However anxious a judge may be for the enforcement of the law, he should always remember that he is as much judge in behalf of the defendant accused of crime, and whose liberty is in jeopardy, as he is judge in behalf of the state, for the purpose of safeguarding the interests of society. Tejano v. Ombudsman G.R. No. 159190, June 30, 2005 Fact: The report of Resident Auditor Alexander A. Tan implicated petitioner as persons involved in the irregular withdrawal of P2.2 million of PNB funds. The Office of the Deputy Ombudsman for the Visayas ordered petitioner to file their respective counter-affidavits. Graft Investigation Officer Edgardo G. Canton recommended the filing of the proper information against petitioner and was thereafter referred for review to the Office of the Special Prosecutor who affirmed the resolution of Graft Investigation Officer, Deputy Special Prosecutor recommended the approval of the memorandum of Special Prosecution Officer.Aniano A. Desierto, then the Special Prosecutor, concurred in the approval. Ombudsman concurred thereto. Subsequently, on 24 November 1994, an Information for violation of Section 3(e) of Rep. Act No. 3019, as amended, was filed before the Sandiganbayanpetitioner filed with the Sandiganbayan an Urgent Motion for a Period of Time to File Motion for Reinvestigation.The Sandiganbayan granted the motion for reinvestigation. Petitioner filed his motion for reinvestigation in the Office of the Special Prosecutor. The Sandiganbayan ordered the Office of the Special Prosecutor to conduct the reinvestigation. The reinvestigation was assigned to Special CONSTITUTIONAL LAW 2 CASE DIGESTS

Prosecution Officer. Convinced that no probable cause existed to indict petitioner Special Prosecutor Micael recommended the dismissal of the case. The recommendation was approved by Deputy Special Prosecutor Kallos and concurred in by Special Prosecutor Tamayo. Ombudsman Aniano A. Desierto, who earlier participated in the initial preliminary investigation as Special Prosecutor, disapproved the recommendation for the dismissal of the case with the marginal note “assign the case to another prosecutor to prosecute the case aggressively.” Special Prosecutor Micael filed a Manifestation, to which was attached a copy of his memorandum, informing the Sandiganbayan of the disapproval by Ombudsman Desierto of his recommendation to dismiss the case. On 10 February 2000, petitioner filed a Motion for Reconsideration of the disapproval by Ombudsman Desierto of the recommendation of Micael. Apparently, petitioner’s motion for reconsideration was not resolved on the merits because on 27 June 2000, Special Prosecution Officer III Joselito R. Ferrer filed a Motion to Set the Case for Arraignment alleging therein that the prosecution did not give due course to the motion for reconsideration on the ground that it was the second motion which is prohibited under the Ombudsman Act of 1989. He added that the results of the reinvestigation were already submitted to the respondent court before receiving the motion for reconsideration.Petitioner manifested before the Sandiganbayan the Office of the Special Prosecutor’s failure to resolve his motion for reconsideration. Thus, in a resolution13 dated 24 March 2003, the respondent court directed the Office of the Ombudsman to resolve the said motion.In a memorandum14 dated 09 June 2003, Special Prosecutor Joselito R. Ferrer recommended the denial of the motion for reconsideration filed by petitioner. Deputy Special Prosecutor Robert E. Kallos changed his previous position and recommended that the memorandum for the dismissal of the motion for reconsideration be approved, with Special Prosecutor Dennis M. Villa-Ignacio concurring in the denial. Ombudsman Simeon V. Marcelo, who succeeded Ombudsman Desierto when he retired, approved Joselito Ferrer’s memorandum recommending the denial of the motion for reconsideration. Petitioner thus filed the instant petition with prayer for the issuance of a temporary restraining order to enjoin the Sandiganbayan from taking further action in Criminal Case. The First Division of this Court issued the temporary restraining order prayed for.The instant petition was transferred to the Second Division of this Court. Issue: Where Ombudsman Desierto committed grave abuse of discretion? CONSTITUTIONAL LAW 2 CASE DIGESTS

Held: Yes, attributes partiality on the part of Ombudsman Desierto for having participated in the reinvestigation of the instant case despite the fact that he earlier participated in the initial preliminary investigation of the same when he was a Special Prosecutor by concurring in the recommendation for the filing of the information before the Sandiganbayan. Having participated in the initial preliminary investigation of the instant case and having recommended the filing of appropriate information, it behooved Ombudsman Desierto to recuse himself from participating in the review of the same during the reinvestigation. He should have delegated the review to his Deputies. Prejudicial Publicity HUBERT J. P. WEBB, VS. HONORABLE RAUL E. DE LEON G.R. No. 121234, August 23, 1995 FACTS: On June 19, 1994, the National Bureau of Investigation (NBI) filed with the Department of Justice a letter-complaint charging petitioners Hubert Webb, Michael Gatchalian, Antonio J. Lejano and six (6) other persons with the crime of Rape and Homicide of Carmela N. Vizconde, her mother Estrellita NicolasVizconde, and her sister Anne Marie Jennifer in their home at Number 80 W. Vinzons, St., BF Homes Paranaque, Metro Manila on June 30, 1991. Forthwith, the Department of Justice formed a panel of prosecutors headed by Assistant Chief State Prosecutor Jovencio R. Zuno to conduct the preliminary investigation. ARGUMENTS: Petitioners fault the DOJ Panel for its finding of probable cause. They assail the credibility of Jessica Alfaro as inherently weak and uncorroborated due to the inconsistencies between her April 28, 1995 and May 22, 1995 sworn statements. CONSTITUTIONAL LAW 2 CASE DIGESTS

They criticize the procedure followed by the DOJ Panel when it did not examine witnesses to clarify the alleged inconsistencies. Petitioners charge that respondent Judge Raul de Leon and, later, respondent Judge Amelita Tolentino issued warrants of arrest against them without conducting the required preliminary examination. Petitioners complain about the denial of their constitutional right to due process and violation of their right to an impartial investigation. They also assail the prejudicial publicity that attended their preliminary investigation. ISSUES: 1. Whether or not the DOJ Panel likewise gravely abused its discretion in holding that there is probable cause to charge them with the crime of rape and homicide 2. Whether or not respondent Judges de Leon and Tolentino gravely abused their discretion when they failed to conduct a preliminary examination before issuing warrants of arrest against them 3. Whether or not the DOJ Panel denied them their constitutional right to due process during their preliminary investigation 4. Whether or not the DOJ Panel unlawfully intruded into judicial prerogative when it failed to charge Jessica Alfaro in the information as an accused. HELD: 1. NO. 2. NO. 3. NO. There is no merit in this contention because petitioners were given all the opportunities to be heard. 4. NO. REASONS: 1. The Court ruled that the DOJ Panel did not gravely abuse its discretion when it found probable cause against the petitioners. A probable cause needs only to rest on evidence showing that more likely than not, a crime has been committed and was committed by the suspects. Probable cause need not be based on clear and CONSTITUTIONAL LAW 2 CASE DIGESTS

convincing evidence of guilt, neither on evidence establishing guilt beyond reasonable doubt and definitely, not on evidence establishing absolute certainty of guilt. 2. The Court ruled that respondent judges did not gravely abuse their discretion. In arrest cases, there must be a probable cause that a crime has been committed and that the person to be arrested committed it. Section 6 of Rule 112 simply provides that “upon filing of an information, the Regional Trial Court may issue a warrant for the accused. Clearly the, our laws repudiate the submission of petitioners that respondent judges should have conducted “searching examination of witnesses” before issuing warrants of arrest against them. 3. The DOJ Panel precisely ed the parties to adduce more evidence in their behalf and for the panel to study the evidence submitted more fully. 4. Petitioner’s argument lacks appeal for it lies on the faulty assumption that the decision whom to prosecute is a judicial function, the sole prerogative of the courts and beyond executive and legislative interference. In truth, the prosecution of crimes appertains to the executive department of government whose principal power and responsibility is to see that our laws are faithfully executed. A necessary component of this power is the right to prosecute their violators (See R.A. No. 6981 and section 9 of Rule 119 for legal basis). With regard to the inconsistencies of the sworn statements of Jessica Alfaro, the Court believes that these have been sufficiently explained and there is no showing that the inconsistencies were deliberately made to distort the truth. With regard to the petitioners’ complaint about the prejudicial publicity that attended their preliminary investigation, the Court finds nothing in the records that will prove that the tone and content of the publicity that attended the investigation of petitioners fatally infected the fairness and impartiality of the DOJ Panel. Petitioners cannot just rely on the subliminal effects of publicity on the sense of fairness of the DOJ Panel, for these are basically unbeknown and beyond knowing.

CONSTITUTIONAL LAW 2 CASE DIGESTS

PEOPLE VS SANCHEZ 313 SCRA 254 Facts: Appellants Antonio Sanchez, the mayor of Calauan, Laguna, Averion, Peradillas and Corcolon were found guilty of the complex crime of double murder for the killing of Nelson Peñalosa and his son, Rickson. The aggravating circumstances of conspiracy, evident premeditation, and conspiracy were appreciated. The shooting was carried out upon the order of former Mayor Sanchez, using armalites in automatic firing mode. Witnesses say that three bursts of gunfire were heard. Issue: Whether the court was correct in convicting the accused of the complex crime of double murder. Held: Three bursts of gunfire were heard. Although each burst of shots was caused by one single act of pressing the trigger, in view of its special mechanism, the person firing it has only to keep pressing the trigger with his finger and it would fire continuously. Hence, it is not the act of pressing the trigger which should be considered as producing the several felonies, but the number of bullets which actually produced them. Thus the accused are liable for as many offenses resulting from pressing the trigger. They are liable for two counts of murder and not the complex crime of double murder

NOTICE OF HEARING SUMMARY DISMISSAL BOARD VS TORCITA 330 SCRA 445 FACTS: CONSTITUTIONAL LAW 2 CASE DIGESTS

On 26 April 1994, a red Cortina Ford, driven by C/Insp. Lazaro Torcita, with his aide, PO2 Java, in the front seat and his wife with two ladies at the backseat, were overtaken by a Mazda pick-up owned by Congressman Manuel Puey and driven by one Reynaldo Consejo with four (4) passengers in the persons of Alex Edwin del Rosario, Rosita Bistal, Carmen Braganza and Cristina Dawa. After the Mazda pick-up has overtaken the red Cortina Ford, and after a vehicular collision almost took place, it accelerated speed and proceeded to Hacienda Aimee, a sugarcane plantation owned by the congressman. The red Cortina Ford followed also at high speed until it reached the hacienda where Torcita and Java alighted and the confrontation with del Rosario and Jesus Puey occurred. Torcita identified himself but the same had no effect. PO2 Java whispered to him that there are armed men around them and that it is dangerous for them to continue. That at this point, they radioed for back-up. Torcita, upon the arrival of the back-up force of PNP Cadiz City, proceeded to the place where Capt. Jesus Puey and Alex Edwin del Rosario were. On 6 July 1994, 12 verified administrative complaints were filed against Torcita for Conduct Unbecoming of a Police Officer, Illegal Search, Grave Abuse of Authority and Violation of Domicile, and Abuse of Authority and Violation of COMELEC Gun Ban.The 12 administrative complaints were consolidated into 1 major complaint for conduct unbecoming of a police officer. The Summary Dismissal Board, however, did not find sufficient evidence to establish that Torcita threatened anybody with a gun, nor that a serious confrontation took place between the parties, nor that the urinating incident took place, and held that the charges of violation of domicile and illegal search were not proven. Still, while the Board found that Torcita was "in the performance of his official duties" when the incident happened, he allegedly committed a simple irregularity in performance of duty (for being in the influence of alcohol while in performance of duty) and was suspended for 20 days and salary suspended for the same period of time.

CONSTITUTIONAL LAW 2 CASE DIGESTS

Torcita appealed his conviction to the Regional Appellate Board of the Philippine National Police (PNP, Region VI, Iloilo City), but the appeal was dismissed for lack of jurisdiction. Whereupon, Torcita filed a petition for certiorari in the Regional Trial Court of Iloilo City (Branch 31), questioning the legality of the conviction of an offense for which he was not charged (lack of procedural due process of law). The Board filed a motion to dismiss, which was denied. The RTC granted the petition for certiorari and annulled the dispositive portion of the questioned decision insofar as it found Torcita guilty of simple irregularity in the performance of duty. The Board appealed from the RTC decision, by petition of review to the Court of Appeals, which affirmed the same for the reason that the respondent could not have been guilty of irregularity considering that the 12 cases were eventually dismissed. The Board filed the petition for review on certiorari before the Supreme Court. ISSUE: Whether Torcita may be proceeded against or suspended for breach of internal discipline, when the original charges against him were for Conduct Unbecoming of a Police Officer, Illegal Search, Grave Abuse of Authority and Violation of Domicile, and Abuse of Authority and Violation of COMELEC Gun Ban. HELD: NO. Notification of the charges contemplates that the respondent be informed of the specific charges against him. The absence of specification of the offense for which he was eventually found guilty is not a proper observance of due process. There can be no short-cut to the legal process. While the definition of the more serious offense is broad, and almost all-encompassing a finding of guilt for an offense, no matter how light, for which one is not properly charged and tried cannot be countenanced without violating the rudimentary requirements of due process. CONSTITUTIONAL LAW 2 CASE DIGESTS

Herein, the 12 administrative cases filed against Torcita did not include charges or offenses mentioned or made reference to the specific act of being drunk while in the performance of official duty. There is no indication or warning at all in the summary dismissal proceedings that Torcita was also being charged with breach of internal discipline consisting of taking alcoholic drinks while in the performance of his duties. The omission is fatal to the validity of the judgment finding him guilty of the offense for which he was not notified nor charged. Further, the cursory conclusion of the Dismissal Board that Torcita "committed breach of internal discipline by taking drinks while in the performance of same" should have been substantiated by factual findings referring to this particular offense. Even if he was prosecuted for irregular performance of duty, he could not have been found to have the odor or smell of alcohol while in the performance of duty because he was not on duty at the time that he had a taste of liquor because he was on a private trip fetching his wife. Hence , the decision of the petitioners Board was rendered without or in excess of jurisdiction. Petition dismissed.

Banco Español vs. Palanca, 37 Phil 921 Facts: Engracio Palanca Tanquinyeng secured a debt with various parcels of real property in Manila. The debt   amounted to P218,294.10 at 8% per annum, payable quarterly. Property's estimated value was about   P292,558. After the instrument's execution, mortgagor returned to Amoy, China and died on January 29, 1810. The foreclosure proceeding needed publication pursuant to section 399 of the Code of Civil Procedure.   Publication was made in a newspaper of Manila and an order of the court deposited in the post office in   a stamped envelope of the summons and complaint directed to defendant.   The clerk, however, failed to comply with the mail publication requirement. CONSTITUTIONAL LAW 2 CASE DIGESTS

The bank was able to foreclose the property without the defendant. After seven years, the administrator of the estate, Vicente Palanca, appeared and requested the court   to set aside the order of default of July 2, 1908, and the judgment rendered upon July 3, 1908, and to   vacate all the proceedings subsequent thereto. The basis of this application, as set forth in the motion itself, was that the order of default and the judgment rendered thereon were void because the court   had never acquired jurisdiction over the defendant or over the subject of the action. His appeal was denied by the lower court, hence the appeal. His appeal was denied by the lower court, hence the appeal. Issue:  Whether or not the procedural aspect of the right to due process has been prejudiced. Held: 1. There must be a COURT or TRIBUNAL clothed with judicial power to hear and determine the matter before it; 2. JURISDICTION must be lawfully acquired over the person of the defendant or over the property which is the subject of the proceeding; 3. The defendant must be given the OPPORTUNITY to be heard; and 4. Judgment must be rendered upon lawful HEARING. The essentials of procedural fairness inn judicial proceedings are: Conclusions stated by the court indicated that the judgment appealed from is without error, and the same is accordingly affirmed. PROSPERO A. PICHAY v. OFFICE OF DEPUTY EXECUTIVE SECRETARY FOR LEGAL AFFAIRS INVESTIGATIVE, GR No. 196425, 2012-07-24 Facts: On April 16, 2001, then President Gloria Macapagal-Arroyo issued Executive Order No. 12 (E.O. 12) creating the Presidential Anti-Graft Commission (PAGC) and vesting it with the power to investigate or hear administrative cases or complaints CONSTITUTIONAL LAW 2 CASE DIGESTS

for possible graft and corruption,... among others, against presidential appointees and to submit its report and recommendations to the President. On November 15, 2010, President Benigno Simeon Aquino III issued Executive Order No. 13 (E.O. 13), abolishing the PAGC and transferring its functions to the Office of the Deputy Executive Secretary for Legal Affairs (ODESLA), more particularly to its newly-established Investigative and Adjudicatory Division (IAD). On April 6, 2011, respondent Finance Secretary Cesar V. Purisima filed before the IAD-ODESLA a complaint affidavit[2] for grave misconduct against petitioner Prospero A. Pichay, Jr., Chairman of the Board of Trustees of the Local Water Utilities Administration (LWUA), as well as the incumbent members of the LWUA Board of Trustees, namely, Renato Velasco, Susana Dumlao Vargas, Bonifacio Mario M. Pena, Sr. and Daniel Landingin, which arose from the purchase by the LWUA of Four Hundred Forty-Five Thousand Three Hundred Seventy Seven (445,377) shares of stock of Express Savings Bank, Inc. On April 14, 2011, petitioner received an Order[3] signed by Executive Secretary Paquito N. Ochoa, Jr. requiring him and his co-respondents to submit their respective written explanations under oath. In compliance therewith, petitioner filed a Motion to Dismiss Ex Abundante Ad Cautelam manifesting that a case involving the same transaction and charge of grave misconduct entitled, "Rustico B. Tutol, et al. v. Prospero Pichay, et al.", and docketed as OMB-C-A-10-0426-I, is already pending before the Office of the Ombudsman. Now alleging that no other plain, speedy and adequate remedy is available to him in the ordinary course of law, petitioner has resorted to the instant petition for certiorari and prohibition CONSTITUTIONAL LAW 2 CASE DIGESTS

Issues: I. E.O. 13 IS UNCONSTITUTIONAL FOR USURPING THE POWER OF THE LEGISLATURE TO CREATE A PUBLIC OFFICE. II. E.O. 13 IS UNCONSTITUTIONAL FOR USURPING THE POWER OF THE LEGISLATURE TO APPROPRIATE FUNDS. III. E.O. 13 IS UNCONSTITUTIONAL FOR USURPING THE POWER OF CONGRESS TO DELEGATE QUASI-JUDICIAL POWERS TO ADMINISTRATIVE AGENCIES. IV. E.O. 13 IS UNCONSTITUTIONAL FOR ENCROACHING UPON THE POWERS OF THE OMBUDSMAN. V. E.O. 13 IS UNCONSTITUTIONAL FOR VIOLATING THE GUARANTEE OF DUE PROCESS. VI. E.O. 13 IS UNCONSTITUTIONAL FOR VIOLATING THE EQUAL PROTECTION CLAUSE. In assailing the constitutionality of E.O. 13, petitioner asseverates that the President is not authorized under any existing law to create the Investigative and Adjudicatory Division, Office of the Deputy Executive Secretary for Legal Affairs (IAD-ODESLA) and that by creating a... new, additional and distinct office tasked with quasi-judicial functions, the President has not only usurped the powers of congress to create a public office, appropriate funds and delegate quasi-judicial functions to administrative agencies but has also encroached upon the... powers of the Ombudsman. Petitioner, however, goes on to assert that the President went beyond the authority granted by E.O. 292 for him to reorganize the executive department since his issuance of E.O. 13 did not merely involve the abolition of an office but the creation of one as well. Petitioner next avers that the IAD-ODESLA was illegally vested with judicial power which is reserved to the Judicial Department and, by way of exception through an CONSTITUTIONAL LAW 2 CASE DIGESTS

express grant by the legislature, to administrative agencies. He points out that the name Investigative and Adjudicatory Division is proof itself that the IAD-ODESLA wields quasi-judicial power. Ruling: The President has Continuing Authority... to Reorganize the Executive Department... under E.O. 292. Section 31 of Executive Order No. 292 (E.O. 292), otherwise known as the Administrative Code of 1987, vests in the President the continuing authority to reorganize the offices under him in order to achieve simplicity, economy and efficiency. E.O. 292 sanctions the following... actions undertaken for such purpose: (1) Restructure the internal organization of the Office of the President Proper, including the immediate Offices, the Presidential Special Assistants/Advisers System and the Common Staff Support System, by abolishing, consolidating, or merging units thereof or... transferring functions from one unit to another; (2) Transfer any function under the Office of the President to any other Department or Agency as well as transfer functions to the Office of the President from other Departments and Agencies; and (3) Transfer any agency under the Office of the President to any other Department or Agency as well as transfer agencies to the Office of the President from other departments or agencies.[4] Clearly, the abolition of the PAGC and the transfer of its functions to a division specially created within the ODESLA is properly within the prerogative of the President under his continuing "delegated legislative authority to reorganize" his own office pursuant to E.O. Generally, this authority to implement organizational changes is limited to transferring either an office or a function from the Office of the President to another Department or Agency, and the other way around.[7] Only Section 31(1) CONSTITUTIONAL LAW 2 CASE DIGESTS

gives the President a... virtual freehand in dealing with the internal structure of the Office of the President Proper by allowing him to take actions as extreme as abolition, consolidation or merger of units, apart from the less drastic move of transferring functions and offices from one unit to... another. The Reorganization Did not Entail... the Creation of a New, Separate and Distinct Office. The abolition of the PAGC did not require the creation of a new, additional and distinct office as the duties and functions that pertained to the defunct anti-graft body were simply transferred to the ODESLA, which is an existing office within the Office of the President Proper. The reorganization required no more than a mere alteration of the administrative structure of the ODESLA through the establishment of a third division the Investigative and Adjudicatory Division through which ODESLA could take on the additional functions it has... been tasked to discharge under E.O. 13. The Reorganization was Pursued in Good Faith. A valid reorganization must not only be exercised through legitimate authority but must also be pursued in good faith. A reorganization is said to be carried out in good faith if it is done for purposes of economy and efficiency.[13] It appears in this case... that the streamlining of functions within the Office of the President Proper was pursued with such purposes in mind. In its Whereas clauses, E.O. 13 cites as bases for the reorganization the policy dictates of eradicating corruption in the government and... promoting economy and efficiency in the bureaucracy. Indeed, the economical effects of the reorganization is shown by the fact that while Congress had initially appropriated P22 Million for the PAGC's operation in the 2010 annual budget,[14] no separate... or added funding of such a considerable amount was ever required after the transfer of the PAGC functions to the IAD-ODESLA.

CONSTITUTIONAL LAW 2 CASE DIGESTS

Apparently, the budgetary requirements that the IAD-ODESLA needed to discharge its functions and maintain its personnel would be sourced from the following year's appropriation for the President's Offices under the General Appropriations Act of 2011.[15] Petitioner asseverates, however, that since Congress did not indicate the manner by which the appropriation for the Office of the President was to be distributed, taking therefrom the operational funds of the IAD-ODESLA would amount to an illegal appropriation by the President. The contention is without legal basis. There is no usurpation of the legislative... power to appropriate public funds. In the chief executive dwell the powers to run government. Placed upon him is the power to recommend the budget necessary for the operation of the Government, [16] which implies that he has the necessary authority to evaluate and determine the structure that... each government agency in the executive department would need to operate in the most economical and efficient manner.[17] Hence, the express recognition under Section 78 of R.A. 9970 or the General Appropriations Act of 2010 of the President's authority to "direct changes in the organizational units or key positions in any department or agency." The aforecited provision, often and consistently included in the general appropriations laws, recognizes the extent of the President's power to reorganize the executive offices and... agencies under him, which is, "even to the extent of modifying and realigning appropriations for that purpose."[18] And to further enable the President to run the affairs of the executive department, he is likewise given constitutional authority to augment any item in the General Appropriations Law using the savings in other items of the appropriation for his office.[19] In fact, he is explicitly allowed by law to transfer any fund appropriated for the different departments, bureaus, offices and agencies of the Executive Department which is included in the General Appropriations Act, to any program, project or CONSTITUTIONAL LAW 2 CASE DIGESTS

activity of any department, bureau... or office included in the General Appropriations Act or approved after its enactment.[20] Thus, while there may be no specific amount earmarked for the IADODESLA from the total amount appropriated by Congress in the annual budget for the Office of the President, the necessary funds for the IADODESLA may be properly sourced from the President's own office budget... without committing any illegal appropriation. After all, there is no usurpation of the legislature's power to appropriate funds when the President simply allocates the existing funds previously appropriated by Congress for his office. The IAD-ODESLA is a fact- finding... and recommendatory body not vested... with quasi- judicial powers. while the term "adjudicatory" appears part of its appellation, the IAD-ODESLA cannot try and resolve cases, its authority being limited to the conduct of investigations, preparation of... reports and submission of recommendations. E.O. 13 explicitly states that the IAD-ODESLA shall "perform powers, functions and duties xxx, of PAGC."[22] Under E.O. 12, the PAGC was given the authority to "investigate or hear administrative cases or complaints against all presidential appointees in the government"[23] and to "submit its report and recommendations to the President."[24] The IAD-ODESLA is a fact-finding and recommendatory body to the President, not having the power to settle controversies and adjudicate cases. The President's authority to issue E.O. 13 and constitute the IAD ODESLA as his fact-finding investigator cannot be doubted. After all, as Chief Executive, he is granted full control over the Executive Department to ensure the enforcement of the laws. Section 17, Article VII of... the Constitution provides: Section 17. The President shall have control of all the executive departments, bureaus and offices. He shall ensure that the laws be faithfully executed. The obligation to see to it that laws are faithfully executed necessitates the corresponding power in the President to conduct investigations into the conduct of officials and employees in the executive department.[27] CONSTITUTIONAL LAW 2 CASE DIGESTS

The IAD-ODESLA does not encroach... upon the powers and duties of the Ombudsman. The primary jurisdiction of the Ombudsman to investigate and prosecute cases refers to criminal cases cognizable by the Sandiganbayan and not to administrative cases. It is only in the exercise of its primary jurisdiction that... the Ombudsman may, at any time, take over the investigation being conducted by another investigatory agency. Section 15 (1) of R.A. No. 6770 or the Ombudsman Act of 1989, empowers the Ombudsman to (1) Investigate and prosecute on its own or on complaint by any person, any act or omission of any public officer or employee, office or agency, when such act or omission appears to be illegal, unjust, improper or inefficient. It has primary jurisdiction over... cases cognizable by the Sandiganbayan and, in the exercise of its primary jurisdiction, it may take over, at any stage, from any investigatory agency of government, the investigation of such cases. (Emphasis supplied) Since the case filed before the IAD-ODESLA is an administrative disciplinary case for grave misconduct, petitioner may not invoke the primary jurisdiction of the Ombudsman to prevent the IAD-ODESLA from proceeding with its investigation. In any event, the Ombudsman's authority... to investigate both elective and appointive officials in the government, extensive as it may be, is by no means exclusive. It is shared with other similarly authorized government agencies.[28] While the Ombudsman's function goes into the determination of the existence of probable cause and the adjudication of the merits of a criminal accusation, the investigative authority of the IAD-ODESLA is limited to that of a fact-finding investigator whose determinations and... recommendations remain so until acted upon by the President. As such, it commits no usurpation of the Ombudsman's constitutional duties. Executive Order No. 13 Does Not Violate Petitioner's Right to Due CONSTITUTIONAL LAW 2 CASE DIGESTS

Process and the Equal Protection... of the Laws. Presidential appointees come under the direct disciplining authority of the President. This proceeds from the well settled principle that, in the absence of a contrary law, the power to remove or to discipline is lodged in the same authority on which the power to appoint is... vested.[32] Having the power to remove and/or discipline presidential appointees, the President has the corollary authority to investigate such public officials and look into their conduct in office.[33] Petitioner is a presidential... appointee occupying the high-level position of Chairman of the LWUA. Necessarily, he comes under the disciplinary jurisdiction of the President, who is well within his right to order an investigation into matters that require his informed decision. Also, contrary to petitioner's assertions, his right to due process was not violated when the IAD-ODESLA took cognizance of the administrative complaint against him since he was given sufficient opportunity to oppose the formal complaint filed by Secretary Purisima. In... administrative proceedings, the filing of charges and giving reasonable opportunity for the person so charged to answer the accusations against him constitute the minimum requirements of due process,[35] which simply means having the opportunity to explain... one's side.[36] Hence, as long as petitioner was given the opportunity to explain his side and present evidence, the requirements of due process are satisfactorily complied with because what the law abhors is an absolute lack of opportunity to be... heard.[37] The records show that petitioner was issued an Order requiring him to submit his written explanation under oath with respect to the charge of grave misconduct filed against him. His own failure to submit his explanation despite notice defeats his... subsequent claim of denial of due process. Finally, petitioner doubts that the IAD-ODESLA can lawfully perform its duties as an impartial tribunal, contending that both the IAD-ODESLA and respondent Secretary Purisima are connected to the President. The mere suspicion of partiality will not suffice to invalidate the... actions of the IADODESLA. Mere allegation is not equivalent to proof. Bias and partiality cannot be presumed.[38] Petitioner must present substantial proof to show that the lAD-ODES LA had unjustifiably sided against him in the conduct of the investigation.

CONSTITUTIONAL LAW 2 CASE DIGESTS

No such evidence has been presented as to defeat the presumption of regularity m the perfonnance of the fact-finding investigator's duties. The assertion, therefore, deserves scant consideration. Every law has in its favor the presumption of constitutionality, and to justify its nullification, there must be a clear and unequivocal breach of the Constitution, not a doubtful and argumentative one.[39] Petitioner has failed to discharge the burden of... proving the illegality of E.O. 13, which IS indubitably a valid exercise of the President's continuing authority to reorganize the Office of the President. WHEREFORE, premises considered, the petition is hereby DISMISSED. Principles: The equal protection of the laws is a... guaranty against any form of undue favoritism or hostility from the government.[29] It is embraced under the due process concept and simply requires that, in the application of the law, "all persons or things similarly situated should be treated alike, both... as to rights conferred and responsibilities imposed."[30] The equal protection clause, however, is not absolute but subject to reasonable classification so that aggrupations bearing substantial distinctions may be treated differently from each other. The equal protection of the law clause is against undue favor and individual or class privilege, as well as hostile discrimination or the oppression of inequality. It is not intended to prohibit legislation which is limited either in the object to which it is... directed or by territory within which it is to operate. It does not demand absolute equality among residents; it merely requires that all persons shall be treated alike, under like circumstances and conditions both as to privileges conferred and liabilities enforced. The... equal protection clause is not infringed by legislation which applies only to those persons falling within a specified class, if it applies alike to all persons within such class, and reasonable grounds exist for making a distinction between those who fall within such class and... those who do not. (Emphasis supplied) Substantial distinctions clearly exist between elective officials and appointive officials. The former occupy their office by virtue of the mandate of the electorate. CONSTITUTIONAL LAW 2 CASE DIGESTS

They are elected to an office for a definite term and may be removed therefrom only upon... stringent conditions. On the other hand, appointive officials hold their office by virtue of their designation thereto by an appointing authority. Some appointive officials hold their office in a permanent capacity and are entitled to security of tenure while others serve... at the pleasure of the appointing authority. x

x

x

x

An election is the embodiment of the popular will, perhaps the purest expression of the sovereign power of the people. It involves the choice or selection of candidates to public office by popular vote. Considering that elected officials are put in office by their constituents... for a definite term, x x x complete deference is accorded to the will of the electorate that they be served by such officials until the end of the term for which they were elected. In contrast, there is no such expectation insofar as appointed officials are concerned. Chavez vs. Romulo G.R. No. 157036, June 9, 2004 FACTS: This case is about the ban on the carrying of firearms outside of residence in order to deter the rising crime rates. Petitioner questions the ban as a violation of his right to property. ISSUE: Whether or not the revocation of permit to carry firearms is unconstitutional Whether or not the right to carry firearms is a vested property right HELD: Petitioner cannot find solace to the above-quoted Constitutional provision. In evaluating a due process claim, the first and foremost consideration must be whether life, liberty or property interest exists. The bulk of jurisprudence is that a license authorizing a person to enjoy a certain privilege is neither a property nor CONSTITUTIONAL LAW 2 CASE DIGESTS

property right. In Tan vs. The Director of Forestry, we ruled that “a license is merely a permit or privilege to do what otherwise would be unlawful, and is not a contract between the authority granting it and the person to whom it is granted; neither is it property or a property right, nor does it create a vested right.” In a more emphatic pronouncement, we held in Oposa vs. Factoran, Jr. that: “Needless to say, all licenses may thus be revoked or rescinded by executive action. It is not a contract, property or a property right protected by the due process clause of the Constitution.” xxx In our jurisdiction, the PNP Chief is granted broad discretion in the issuance of PTCFOR. This is evident from the tenor of the Implementing Rules and Regulations of P.D. No. 1866 which state that “the Chief of Constabulary may, in meritorious cases as determined by him and under such conditions as he may impose, authorize lawful holders of firearms to carry them outside of residence.” Following the American doctrine, it is indeed logical to say that a PTCFOR does not constitute a property right protected under our Constitution. Consequently, a PTCFOR, just like ordinary licenses in other regulated fields, may be revoked any time. It does not confer an absolute right, but only a personal privilege to be exercised under existing restrictions, and such as may thereafter be reasonably imposed. A licensee takes his license subject to such conditions as the Legislature sees fit to impose, and one of the statutory conditions of this license is that it might be revoked by the selectmen at their pleasure. Such a license is not a contract, and a revocation of it does not deprive the defendant of any property, immunity, or privilege within the meaning of these words in the Declaration of Rights. The US Supreme Court, in Doyle vs. Continental Ins. Co, held: “The correlative power to revoke or recall a permission is a necessary consequence of the main power. A mere license by the State is always revocable.”

Secretary of Justice vs Lantion Doctrine of Incorporation SECRETARY OF JUSTICE, petitioner, CONSTITUTIONAL LAW 2 CASE DIGESTS

vs. HON. RALPH C. LANTION, Presiding Judge, Regional Trial Court of Manila, Branch 25, and MARK B. JIMENEZ, respondents. Facts: This is a petition for review of a decision of the Manila Regional Trial Court (RTC). The Department of Justice received a request from the Department of Foreign Affairs for the extradition of respondent Mark Jimenez to the U.S. The Grand Jury Indictment. The warrant for his arrest, and other supporting documents for said extradition were attached along with the request. Charges include: Conspiracy to commit offense or to defraud the US Attempt to evade or defeat tax Fraud by wire, radio, or television False statement or entries Election contribution in name of another The Department of Justice (DOJ), through a designated panel proceeded with the technical evaluation and assessment of the extradition treaty which they found having matters needed to be addressed. Respondent, then requested for copies of all the documents included in the extradition request and for him to be given ample time to assess it. The Secretary of Justice denied request on the following grounds: He found it premature to secure him copies prior to the completion of the evaluation. At that point in time, the DOJ is in the process of evaluating whether the procedures and requirements under the relevant law (PD 1069 Philippine Extradition Law) and treaty (RP-US Extradition Treaty) have been complied with by the Requesting Government. Evaluation by the DOJ of the documents is not a preliminary investigation like in criminal cases making the constitutionally guaranteed rights of the accused in criminal prosecution inapplicable. The U.S. requested for the prevention of unauthorized disclosure of the information in the documents.

CONSTITUTIONAL LAW 2 CASE DIGESTS

The department is not in position to hold in abeyance proceedings in connection with an extradition request, as Philippines is bound to Vienna Convention on law of treaties such that every treaty in force is binding upon the parties. Mark Jimenez then filed a petition against the Secretary of Justice. RTC presiding Judge Lantion favored Jimenez. Secretary of Justice was made to issue a copy of the requested papers, as well as conducting further proceedings. Thus, this petition is now at bar. Issue/s: Whether or not respondent’s entitlement to notice and hearing during the evaluation stage of the proceedings constitute a breach of the legal duties of the Philippine Government under the RP-US Extradition Treaty. Discussions: The doctrine of incorporation is applied whenever municipal tribunals are confronted with situations in which there appears to be a conflict between a rule of international law and the provisions of the constitution or statute of a local state. Efforts should be done to harmonize them. In a situation, however, where the conflict is irreconcilable and a choice has to be made between a rule of international law and municipal law, jurisprudence dictates that municipal law should be upheld by the municipal courts. The doctrine of incorporation decrees that rules of international law are given equal standing, but are not superior to, national legislative enactments. Ruling/s: No. The human rights of person, Filipino or foreigner, and the rights of the accused guaranteed in our Constitution should take precedence over treaty rights claimed by a contracting state. The duties of the government to the individual deserve preferential consideration when they collide with its treaty obligations to the government of another state. This is so although we recognize treaties as a source of binding obligations under generally accepted principles of international law incorporated in our Constitution as part of the law of the land. CONSTITUTIONAL LAW 2 CASE DIGESTS

PEOPLE vs. ESTRADA G.R. No. 164368-69 FACTS: An Information for plunder was filed with the Sandiganbayan against respondent Estrada, among other accused. A separate Information for illegal use of alias was likewise filed. The Amended Information reads: “…to conceal the ill-gotten wealth he acquired during his tenure and his true identity as the President, represents himself as JOSE VELARDE in several transactions and use and employ the said alias Jose Velarde which is neither his registered name at birth nor his baptismal name, in signing documents with Equitable PCI Bank and/or other corporate entities. ISSUES: 1. Whether or not Joseph Estrada’s use of his alias Jose Velarde was not public despite the presence of Messrs. Aprodicio Laquian and Fernando Chua on 4 February 2000 (YES, not public) 2. Whether or not Joseph Estrada’s use of his alias Jose Velarde was allowable under banking rules, despite the clear prohibition under Commonwealth Act No. 142; (YES, allowable when the act was committed) 3. Whether or not the court a quo gravely erred and abused its discretion in limiting the coverage of the amended Information in Crim. Case No. 26565 to the use of the alias Jose Velarde by respondent Joseph Estrada on February 4, 2000; (Not limitative [procedural question]) 4. Whether or not the court a quo gravely erred and abused its discretion in departing from its earlier final finding on the non-applicability of Ursua v. Court of Appeals and forcing its application to the instant case. (NO) HELD: The petition has no merit. The Law on Illegal Use of Alias and the Ursua Ruling Ursua definition of an alias: a name or names used by a person or intended to be used by him publicly and habitually usually in business transactions in addition to CONSTITUTIONAL LAW 2 CASE DIGESTS

his real name by which he is registered at birth or baptized the first time or substitute name authorized by a competent authority. There must be a sign or indication that the user intends to be known by this name (the alias) in addition to his real name, and there must be habituality. The repeated use of an alias within a single day cannot be deemed habitual, as it does not amount to a customary practice or use. Following the doctrine of stare decisis, we are guided by the Ursua ruling on how the crime punished under CA No. 142 may be committed. The court found no merit in the argument that the Sandiganbayan erred when it resurrected the application of Ursua, resulting in the reversal of its earlier final ruling. First, the cited Sandiganbayan resolution is a mere interlocutory order. Second, in the earlier motion to quash, the Sandiganbayan solely looked at the allegations of the Information to determine the sufficiency of these allegations and did not consider any evidence aliened. What is the coverage of the indictment? (Regarding the limitative coverage) The court found no merit on the argument of the People that the Sandiganbayan abused its discretion in limiting the coverage of the amended Information to Estrada's use of the alias Jose Velarde on February 4, 2000, considering that there were other transactions covered by the phrase prior to or subsequent thereto. The date of the commission of the offense need not be precisely stated in the complaint or information except when the precise date is a material ingredient of the offense. Under this analysis, the several transactions involving the signing of documents with Equitable PCI Bank and/or other corporate entities all had their reference to February 4, 2000; they were all made on or about or prior or subsequent to that date, thus plainly implying that all these transactions took place only on February 4, 2000 or on another single date sometime before or after February 4, 2000. To be sure, the Information could have simply said on or about February 4, 2000 to capture all the alternative approximate dates, so that the phrase sometime prior or subsequent thereto would effectively be a surplusage that has no meaning separately from the on or about already expressed. This consequent uselessness of the prior or subsequent thereto phrase cannot be denied, but it is a direct and CONSTITUTIONAL LAW 2 CASE DIGESTS

necessary consequence of the use of the OR between the two phrases and the THERETO that referred back to February 4, 2000 in the second phrase. Of course, the reading would have been very different (and would have been clearly in accord with the Peoples present interpretation) had the Information simply used AND instead of OR to separate the phrases; the intent to refer to various transactions occurring on various dates and occasions all proximate to February 4, 2000 could not be disputed. Unfortunately for the People, the imprecision in the use of OR is the reality the case has to live with. To act contrary to this reality would violate Estradas right to be informed of the nature and cause of accusation against him; the multiple transactions on several separate days that the People claims would result in surprise and denial of an opportunity to prepare for Estrada, who has a right to rely on the single day mentioned in the Information. The issues of publicity, and the application of CA No. 142, R.A. No. 1405, and R.A. No. 9160 The rule in the law of libel that mere communication to a third person is publicity does not apply to violations of CA No. 142. The use of the alias, to be considered public, must be made openly, or in an open manner or place, or to cause it to become generally known, in other words, the intent to publicly use the alias must be manifest. The enactment of R.A. No. 9160 clearly manifests that prior to its enactment, numbered accounts or anonymous accounts were permitted banking transactions, whether they be allowed by law or by a mere banking regulation. To be sure, an indictment against Estrada using this relatively recent law cannot be maintained without violating the constitutional prohibition on the enactment and use of ex post facto laws. Mariveles Shipyard Corp v. Court of Appeals G.R. No. 144134, Novemeber 11, 2003, 415 SCRA 513 FACTS: Petitioner submits that respondent Court of Appeals (CA) erred in its decisions in the previous cases where the petitioner was involved. The latter contend that, among other issues, CA gravely erred in its affirmation on the National Labor CONSTITUTIONAL LAW 2 CASE DIGESTS

Relations Commission‘s (NLRC) decision that the petitioner together with ‘Longest Force’, a security agency, are jointly and severally liable for the payment of back wages and overtime pay to private respondents. The petitioner invokes that it has already paid all the necessary compensation to the private respondents. ISSUE: Whether or not the petitioner should be held jointly and severally liable, together with ‘Longest Force’ in the payment of back wages to the private respondents as affirmed by respondent CA? HELD: Yes. REASONING: Under Article 106, par. 2 of the Labor Code, ‘in the event that the contractor or subcontractor fails to pay wages of his employees…the employer shall be jointly and severally liable with his contractor or subcontractor xxx’. Also, in Article 107 of the same Code, the law states that ‘…the preceding Article shall likewise apply to person, partnership, association or corporation which, not being an employer, contracts with an independent contractor…’. Pursuant to the mentioned provisions of the Labor Code, the Court said that, in this case, the petitioner as an indirect employer, shall truly be liable jointly and severally with ‘Longest Force’ in paying backwages and overtime pay to the private respondents. Moreover, the Court emphasized that ‘Labor standard are enacted by the legislature to alleviate the plight of workers whose wages barely meet the spiraling costs of their basic needs. Labor laws are considered written in every contract. Stipulations in violation thereof are considered null‘. Therefore, the petitioner should be held jointly and severally liable, together with ‘Longest Force’ to the private respondents as earlier decided by NLRC, as affirmed by the CA. OPPORTUNITY TO BE HEARD Budiongan v. De la Cruz G.R. No. 170288, September 22, 2006 Fact: By virtue of Municipal Ordinance, the Municipality of Carmen, Bohol appropriated an amount for the purchase of a road roller for the municipality. However, the Municipal Development Council recommended that the amount be realigned and CONSTITUTIONAL LAW 2 CASE DIGESTS

used for the asphalt laying of a portion of a Street. Thereafter, it was discovered that there was yet no ordinance approving the realignment of the funds. Thus, the Sangguniang Bayan passed Ordinance, approving the realignment of the fund. Malmis was paid the contract price. private respondents filed a complaint against the petitioners before the Office of the Deputy Ombudsman for Visayas alleging illegality in the conduct of the bidding, award and notice to commence work since there was no fund appropriated for the purpose. the Office of the Deputy found probable cause and recommended the filing of an information for violation of Article 2207 of the Revised Penal Code against the petitioners. Upon review, the Case Assessment, Review and Reinvestigation Bureau of the Office of the Special Prosecutor, issued the assailed Memorandum modifying the charge against petitioners for allegedly giving unwarranted benefit to Malmis and violation of Section 3(h) of R.A. No. 3019 against petitioner Budiongan for allegedly “directly or indirectly having financial or pecuniary interest in a contract or transaction in connection with which he intervenes or takes part in his official capacity.” Petitioners filed a Motion for Reconsideration with the Office of the Special Prosecutor which was denied for lack of merit in the Resolution dated October 19, 2005. adrianantazo.wordpress.com Issue: Whether the refusal or failure to conduct a re-investigation has violated petitioners’ right to due process? adrianantazo.wordpress.com Held: No, The right to a preliminary investigation is not a constitutional right, but is merely a right conferred by statute. The absence of a preliminary investigation does not impair the validity of the Information or otherwise render the same defective. It does not affect the jurisdiction of the court over the case or constitute a ground for quashing the Information. If absence of a preliminary investigation does not render the Information invalid nor affect the jurisdiction of the court over the case, then the denial of a motion for reinvestigation cannot likewise invalidate the Information or oust the court of its jurisdiction over the case. Petitioners were not deprived of due process because they were afforded the opportunity to refute the charges by filing their counter-affidavits. The modification of the offense charged did not come as a surprise to the petitioners because it was based on the same set of facts and the same alleged illegal acts. Moreover, petitioners failed to aver newly discovered evidence nor impute CONSTITUTIONAL LAW 2 CASE DIGESTS

commission of grave errors or serious irregularities prejudicial to their interest to warrant a reconsideration or reinvestigation of the case as required under Section 8, Rule III of the Rules of Procedure of the Office of the Ombudsman. Thus, the modification of the offense charged, even without affording the petitioners a new preliminary investigation, did not amount to a violation of their rights. Marohombsar v. Judge Adiong A.M. RTJ-02-1674, January 22, 2004 Fact: Complainant Marohombsar was the defendant in Civil Case for “injunction with prayer for preliminary injunction.” The case was filed by Yasmira Pangadapun, daughter of Judge Yusoph Pangadapun of RTC Branch 10, Marawi City. In the said complaint, Pangadapun questioned the legality of Marohombsar’s appointment by DSWD Regional Secretary Salic-Malna as provincial social welfare officer V of the DSWD-ARMM. Upon the filing of the said complaint, respondent judge issued a TRO and set the hearing on the application for the issuance of a writ of preliminary injunction. Summons, together with a copy of the complaint and a notice indicating that a preliminary conference would be held was also served on both parties. Marohombsar filed an ex parte urgent motion to dissolve the TRO. Pangadapun was asked to comment and, pending the filing of the same, the TRO was extended. Respondent issued an order stating that a preliminary conference had been held and that both parties had waived the raffle of the case. He reset the hearing on the application for the issuance of a writ of preliminary injunction. Respondent extended the TRO to let Pangadapun submit her comment. During the hearing on the application for the issuance of a writ of preliminary injunction, none of the lawyers appeared. Hence, respondent considered it submitted for resolution and issued the preliminary injunction the following day. In his partial Comment respondent denied that: (1) he issued the TRO in favor of Pangadapun without benefit of a hearing; (2) in his order dated March 22, 1999, he made it appear that a preliminary conference was held where the parties agreed to waive the raffle of the case, when in fact there was none; (3) he falsified the records of the case and (4) he granted the preliminary injunction without a hearing. He alleged that the complaint was purely a harassment case filed by a disgruntled party because of the latter’s failure to obtain a favorable resolution from him. Although respondent judge admitted that Judge Yusoph Pangadapun and Judge CONSTITUTIONAL LAW 2 CASE DIGESTS

Abdulhakim Ibrahim were his distant relatives and townmates, he stressed that “never in our careers in the judiciary have we interfered nor influenced one another on any pending case before our courts.” Issue: Whether the complainant was denied due process because the preliminary injunction was issued without hearing.adrianantazo.wordpress.com Held: No, In applications for preliminary injunction, the dual requirement of prior notice and hearing before injunction may issue has been relaxed to the point that not all petitions for preliminary injunction need undergo a trial-type hearing, it being doctrinal that a formal or trial-type hearing is not, at all times and in all instances, essential to due process. The essence of due process is that a party is afforded a reasonable opportunity to be heard and to present any evidence he may have in support of his defense. In the present case, complainant was able to move for a reconsideration of the order in question; hence her right to due process was not in anyway transgressed. The court ruled that a party cannot claim that he has been denied due process when he has availed of the opportunity to present his position. Even assuming for the sake of argument that respondent judge erred in ordering the issuance of the writ of preliminary injunction, in Equatorial Realty vs. Anunciacion, Jr. that, as a matter of public policy, the acts of a judge in his official capacity are not subject to disciplinary action even though such acts are erroneous, provided he acts in good faith and without malice. Respondent judge, or any other member of the bench for that matter, is presumed to have acted regularly and in the manner that preserves the ideal of the cold neutrality of an impartial judge implicit in the guarantee of due process. EXCEPTION TO NOTICE AND HEARING PHILIPPINE COMMUNICATIONS SATELLITE CORPORATION, petitioner, vs. JOSE LUIS A. ALCUAZ, as NTC Commissioner, and NATIONAL TELECOMMUNICATIONS COMMISSION, respondents. G.R. No. 84818 December 18, 1989 Facts: CONSTITUTIONAL LAW 2 CASE DIGESTS

The petition before us seeks to annul and set aside an Order 1 issued by respondent Commissioner Jose Luis Alcuaz of the National Telecommunications Commission Herein petitioner is engaged in providing for services involving telecommunications. Charging rates for certain specified lines that were reduced by order of herein respondent Jose AlcuazCommissioner of the National Telecommunications Commission. The rates were ordered to be reduced by fifteen percent (15%) due to Executive Order No. 546 which granted the NTC the power to fix rates. Said order was issued without prior notice and hearing. Under Section 5 of Republic Act No. 5514, petitioner was exempt from the jurisdiction of the then Public Service Commission, now respondent NTC. However, pursuant to Executive Order No. 196 issued on June 17, 1987, petitioner was placed under the jurisdiction, control and regulation of respondent NTC Issue: Whether or Not E.O. 546 is unconstitutional. Held: In Vigan Electric Light Co., Inc. vs. Public Service Commission the Supreme Court said that although the rule-making power and even the power to fix rates- when such rules and/or rates are meant to apply to all enterprises of a given kind throughout the Philippines-may partake of a legislative character. Respondent Alcuaz no doubt contains all the attributes of a quasi-judicial adjudication. Foremost is the fact that said order pertains exclusively to petitioner and to no other The respondent admits that the questioned order was issued pursuant to its quasi-judicial functions. It, however, insists that notice and hearing are not necessary since the assailed order is merely incidental to the entire proceedings and, therefore, temporary in nature but the supreme court said that While respondents may fix a temporary rate pending final determination of the application of petitioner, such rate-fixing order, temporary though it may be, is not exempt from the statutory procedural requirements of notice and hearing CONSTITUTIONAL LAW 2 CASE DIGESTS

The Supreme Court Said that it is clear that with regard to rate-fixing, respondent has no authority to make such order without first giving petitioner a hearing, whether the order be temporary or permanent. In the Case at bar the NTC didn’t scheduled hearing nor it did give any notice to the petitioner Var-Orient Shipping Co vs Achacoso GR 81805 31 May 1988 Facts: Var-Orient Shipping Co filed a complaint with POEA against crewmembers for having allegedly violated their Contracts of Employment with the petitioners that supposedly resulted in damages. the case was heard and the parties agreed to submit their respective position papers and thereafter the case would be submitted for decision. On the basis of the pleadings and memoranda, Tomas Achacoso, POEA Administrator rendered decision in favour of the crew. A copy of the decision was sent by registered mail and delivered by the postman to the petitioners’ counsel. Petitioners allegedly learned about the decision only when the writ of execution was served on them by the Sherriff. Petitioner filed filed an ‘urgent Motion to Recall Writ of Execution’ on the ground that the decision had not been received by the petitioners, hence, it was not yet final and executory. Issue: Whether or not petitioner was denied due process of law because the respondent Administrator resolved the case without any formal hearing? Decision: Petition denied. Equally unmeritorious is the petitioners ‘allegation that they were denied due process because the decision was rendered without a formal hearing. The essence of due process is simply an opportunity to be heard or, as applied to administrative proceedings, an opportunity to explain one’s side or an opportunity to seek a reconsideration of the action or ruling complained of. The fact is that at the hearing of the case it was agreed by the parties that they would file their respective memoranda and thereafter consider the case submitted for decision. This procedure is authorized by law to expedite the settlement of labor disputes. CONSTITUTIONAL LAW 2 CASE DIGESTS

ADMINISTRATIVE & QUASI-JUDICIAL DUE PROCESS G.R. NO. L-46496 FEBRUARY 27, 1940 ANG TIBAY, REPRESENTED BY TORIBIO TEODORO, MANAGER AND PROPIETOR, AND NATIONAL WORKERS BROTHERHOOD, PETITIONERS, VS. THE COURT OF INDUSTRIAL RELATIONS AND NATIONAL LABOR UNION, INC., RESPONDENTS. 69 Phil. 635 – Political Law – Constitutional Law – Due Process in Administrative Bodies Facts: Teodoro Toribio owns and operates Ang Tibay, a leather company which supplies the Philippine Army. Due to alleged shortage of leather, Toribio caused the lay off of a number of his employees. However, the National Labor Union, Inc. (NLU) questioned the validity of said lay off as it averred that the said employees laid off were members of NLU while no members of the rival labor union National Workers Brotherhood (NWB) were laid off. NLU claims that NWB is a company dominated union and Toribio was merely busting NLU. The case reached the Court of Industrial Relations (CIR) where Toribio and NWB won. Eventually, NLU went to the Supreme Court invoking its right for a new trial on the ground of newly discovered evidence. The Supreme Court agreed with NLU. The Solicitor General, arguing for the CIR, filed a motion for reconsideration. ISSUE: Whether or not the National Labor Union, Inc. is entitled to a new trial. HELD: Yes. The records show that the newly discovered evidence or documents obtained by NLU, which they attached to their petition with the SC, were evidence so inaccessible to them at the time of the trial that even with the exercise of due diligence they could not be expected to have obtained them and offered as evidence in the Court of Industrial Relations. Further, the attached documents and exhibits are of such far-reaching importance and effect that their admission would CONSTITUTIONAL LAW 2 CASE DIGESTS

necessarily mean the modification and reversal of the judgment rendered (said newly obtained records include books of business/inventory accounts by Ang Tibay which were not previously accessible but already existing). The SC also outlined that administrative bodies, like the CIR, although not strictly bound by the Rules of Court must also make sure that they comply to the requirements of due process. For administrative bodies, due process can be complied with by observing the following: The right to a hearing which includes the right of the party interested or affected to present his own case and submit evidence in support thereof. Not only must the party be given an opportunity to present his case and to adduce evidence tending to establish the rights which he asserts but the tribunal must consider the evidence presented. While the duty to deliberate does not impose the obligation to decide right, it does imply a necessity which cannot be disregarded, namely, that of having something to support its decision. A decision with absolutely nothing to support it is a nullity, a place when directly attached. Not only must there be some evidence to support a finding or conclusion but the evidence must be “substantial.” Substantial evidence is more than a mere scintilla It means such relevant evidence as a reasonable mind might accept as adequate to support a conclusion. The decision must be rendered on the evidence presented at the hearing, or at least contained in the record and disclosed to the parties affected. The administrative body or any of its judges, therefore, must act on its or his own independent consideration of the law and facts of the controversy, and not simply accept the views of a subordinate in arriving at a decision. The administrative body should, in all controversial questions, render its decision in such a manner that the parties to the proceeding can know the various issues involved, and the reasons for the decisions rendered. The performance of this duty is inseparable from the authority conferred upon it. Montemayor vs Araneta University Foundation GR 44251 31 May 1977 Facts: Felix Montemayor was a faculty of Araneta University Foundation serving as Head of Humanities and Psychology Department. The Chaplain filed a complaint of CONSTITUTIONAL LAW 2 CASE DIGESTS

immorality against him. A committee was created to investigate the allegation. With the assistance of counsel, he filed a motion to dismiss or to hold the hearing in abeyance. The committee found him responsible of the act complained of and recommended for his demotion by one degree. The President adopted such recommendation and thereafter referred the same to the Board of Trustees of private respondent for appropriate action. Subsequently new charges was filed by different faculty members against him and a new committee was formed to investigate the allegations. Montemayor asked for postponement of the hearing and was denied. The hearing proceeded without him and found him guilty of the same charges and recommended for the discontinuance of his service. He then filed a complaint with NLRC. NLRC decided in favour of the Foundation. Hence the present petition. Issue: Whether or not the proceeding relating to Montemayor’s dismissal was done in violation of due process? Decision: Petition dismissed. In Montemayor’s absence the matter was heard and was sufficiently found by the committee to be guilty of his conduct unbecoming and recommended his removal. Such deficiency was remedied when Montemayor was able to present his case with the Labour Commission. Records will show that after all efforts on conciliation had failed parties agreed to submit their dispute for compulsory arbitration. Several hearings were conducted. he legal aspect as to the procedural due process having been satisfied was then summarized by the Solicitor General thus: “All the foregoing clearly shows that petitioner was afforded his day in court. Finally, and more significant, is the fact that petitioner claims denial of due process in the proceeding had before the investigating committees and not in the proceedings before the NLRC wherein, as shown heretofore, he was given the fullest opportunity to present his case. SOPHIA ALCUAZ, ET AL., petitioners vs. PHILIPPINE SCHOOL OF BUSINESS ADMINISTRATION Quezon City Branch ET AL, respondents May 2, 1988 FACTS: CONSTITUTIONAL LAW 2 CASE DIGESTS

In 1986, some PSBA students, herein petitioners Alcuaz et. al. staged demonstrations in the premises of the school. In order for the demonstration to be settled, an agreement was entered into among others the regulations for the conduct of protest action. In spite of the agreement, it was alleged that the petitioners, committed tumultuous and anarchic acts within the premises of the school, fanned by the cooperation of the intervening professors, causing disruption of classes to the prejudice of the majority students. The school took administrative sanctions upon them in view of their participation in the demonstration. The students and the intervening professors were sanctioned. They were dismissed and terminated. ISSUE: Whether or not there has been a deprivation of constitutional rights of expression and assembly and of due process of law of the students who have been barred from re-enrollment. HELD: The Supreme Court held that due process in disciplinary cases such as the case at bar does not entail proceedings and hearings similar to those prescribed for actions and proceedings in the courts of justice. The Court has already recognized the right of the school to refuse re-enrollment of students for academic delinquency and violation of disciplinary regulations. In the school’s administrative process, both students and professors were given three (3) days from receipt of letter to explain in writing why the school should not take administrative sanction against them. With respect to the academic activities of the students and the teaching loads of the teachers, the respondent school has created new class for the petitioners and the intervening professors during and when the investigation was going on. The Court then upheld that there is no denial of due process where all requirements of administrative due process were met by the school and the students were given the opportunity to be heard and that the right of expression and assembly are not absolute especially when parties are bound to certain rules under a contract. CONSTITUTIONAL LAW 2 CASE DIGESTS

EQUAL PROTECTION SEXUAL DISCRIMINATION PASEI v. Drilon G.R. No. 81958 June 30, 1988, Sarmiento, J. FACTS: Phil association of Service Exporters, Inc., is engaged principally in the recruitment of Filipino workers, male and female of overseas employment. It challenges the constitutional validity of Dept. Order No. 1 (1998) of DOLE entitled “Guidelines Governing the Temporary Suspension of Deployment of Filipino Domestic and Household Workers.” It claims that such order is a discrimination against males and females. The Order does not apply to all Filipino workers but only to domestic helpers and females with similar skills, and that it is in violation of the right to travel, it also being an invalid exercise of the lawmaking power. Further, PASEI invokes Sec 3 of Art 13 of the Constitution, providing for worker participation in policy and decision-making processes affecting their rights and benefits as may be provided by law. Thereafter the Solicitor General on behalf of DOLE submitting to the validity of the challenged guidelines involving the police power of the State and informed the court that the respondent have lifted the deployment ban in some states where there exists bilateral agreement with the Philippines and existing mechanism providing for sufficient safeguards to ensure the welfare and protection of the Filipino workers. ISSUE: Whether or not D.O. No. 1 of DOLE is constitutional as it is an exercise of police power. RULING: “[Police power] has been defined as the "state authority to enact legislation that may interfere with personal liberty or property in order to promote the general welfare." As defined, it consists of (1) an imposition of restraint upon liberty or property, (2) in order to foster the common good. It is not capable of an exact CONSTITUTIONAL LAW 2 CASE DIGESTS

definition but has been, purposely, veiled in general terms to underscore its allcomprehensive embrace. “The petitioner has shown no satisfactory reason why the contested measure should be nullified. There is no question that Department Order No. 1 applies only to "female contract workers," but it does not thereby make an undue discrimination between the sexes. It is well-settled that "equality before the law" under the Constitution does not import a perfect Identity of rights among all men and women. It admits of classifications, provided that (1) such classifications rest on substantial distinctions; (2) they are germane to the purposes of the law; (3) they are not confined to existing conditions; and (4) they apply equally to all members of the same class. The Court is satisfied that the classification made-the preference for female workers — rests on substantial distinctions. ADMINISTRATION OF JUSTICE People v Hernandez; G.R. Nos. L-6025-26; 18 Jul 1956; 99 Phil 515 Published on 15 September 2017 in Legal Chyme by Claudine FACTS: Amado Hernandez and several others were accused of the crime of rebellion with multiple murder, arsons and robberies. They were convicted of the crime and sentenced to suffer the penalty of life imprisonment. ISSUE(S): Whether or not there is a complex crime of rebellion with murder. HELD: NO. Under the allegations of the amended information, the murders, arsons and robberies described therein are mere ingredients to the crime of rebellion allegedly committed by said defendants as means “necessary” for the perpetration of said offense of rebellion. The crime, therefore, is simple rebellion. Motion for bail is GRANTED. CONSTITUTIONAL LAW 2 CASE DIGESTS

ANTONIO F. TRILLANES IV v. HON. OSCAR PIMENTEL, SR., IN HIS CAPACITY AS PRESIDING JUDGE, REGIONAL TRIAL COURT- BRANCH 148, MAKATI CITY, et al. 556 SCRA 471 (2008), EN BANC All persons, except those charged with offenses punishable by reclusion perpetua when evidence of guilt is strong, shall, before conviction, be bailable by sufficient sureties, or be released on recognizance as may be provided by law. FACTS: On July 27, 2003, more than 300 heavily armed soldiers led by junior officers of the Armed Forces of the Philippines (AFP) stormed into the Oakwood Premier Apartments in Makati City and publicly demanded the resignation of the President and key national officials. After a series of negotiations, military soldiers surrendered that evening. In the aftermath of such event dubbed as the Oakwood Incident, petitioner Antonio F. Trillanes IV was charged with coup d’état before the Regional Trial Court of Makati. Four years later, Trillanes remained in detention and won a seat in the Senate. Before starting his term, Trillanes filed with RTC an Omnibus Motion for Leave of Court to be Allowed to Attend Senate Sessions and Related Requests. Trillanes requested to be allowed to attend senate sessions and fulfill his functions as senator. The RTC however denied his motion. Thus, he filed Petition for Certiorari with the Supreme Court to set aside orders of the RTC. ISSUES: Whether or not Trillanes‘ case is different from that of the Jalosjos case Whether or not Trillanes‘ election as senator provides legal justification to allow him to work and serve his mandate as senator Whether or not there are enough precedents that allows for a liberal treatment of detention prisoners who are held without bail HELD: No distinction between Trillanes’ case and that of Jalosjos case

CONSTITUTIONAL LAW 2 CASE DIGESTS

The distinctions cited by petitioner were not elemental in the pronouncement in Jalosjos that election to Congress is not a reasonable classification in criminal law enforcement as the functions and duties of the office are not substantial distinctions which lift one from the class of prisoners interrupted in their freedom and restricted in liberty of movement. The Constitution provides: All persons, except those charged with offenses punishable by reclusion perpetua when evidence of guilt is strong, shall, before conviction, be bailable by sufficient sureties, or be released on recognizance as may be provided by law. The Rules also state that no person charged with a capital offense, or an offense punishable by reclusion perpetua or life imprisonment, shall be admitted to bail when evidence of guilt is strong, regardless of the stage of the criminal action. That the cited provisions apply equally to rape and coup d’état cases, both being punishable by reclusion perpetua, is beyond cavil. Within the class of offenses covered by the stated range of imposable penalties, there is clearly no distinction as to the political complexion of or moral turpitude involved in the crime charged. In the present case, it is uncontroverted that petitioner's application for bail and for release on recognizance was denied. The determination that the evidence of guilt is strong, whether ascertained in a hearing of an application for bail or imported from a trial court's judgment of conviction, justifies the detention of an accused as a valid curtailment of his right to provisional liberty. This accentuates the proviso that the denial of the right to bail in such cases is "regardless of the stage of the criminal action." Such justification for confinement with its underlying rationale of public selfdefense applies equally to detention prisoners like Trillanes or convicted prisoners-appellants like Jalosjos. The Court in People v. Hon. Maceda said that all prisoners whether under preventive detention or serving final sentence can not practice their profession nor engage in any business or occupation, or hold office, elective or appointive, while in detention. This is a necessary consequence of arrest and detention. Trillanes’ election as Senator not a legislative justification to allow him to serve his mandate CONSTITUTIONAL LAW 2 CASE DIGESTS

The case against Trillanes is not administrative in nature. And there is no "prior term" to speak of. In a plethora of cases, the Court categorically held that the doctrine of condonation does not apply to criminal cases. Election, or more precisely, re-election to office, does not obliterate a criminal charge. Petitioner's electoral victory only signifies pertinently that when the voters elected him to the Senate, "they did so with full awareness of the limitations on his freedom of action [and] x x x with the knowledge that he could achieve only such legislative results which he could accomplish within the confines of prison. It is opportune to wipe out the lingering misimpression that the call of duty conferred by the voice of the people is louder than the litany of lawful restraints articulated in the Constitution and echoed by jurisprudence. The apparent discord may be harmonized by the overarching tenet that the mandate of the people yields to the Constitution which the people themselves ordained to govern all under the rule of law. The performance of legitimate and even essential duties by public officers has never been an excuse to free a person validly in prison. The duties imposed by the "mandate of the people" are multifarious. The accusedappellant asserts that the duty to legislate ranks highest in the hierarchy of government. The accused-appellant is only one of 250 members of the House of Representatives, not to mention the 24 membersof the Senate, charged with the duties of legislation. Congress continues to function well in the physical absence of one or a few of its members. x x x Never has the call of a particular duty lifted a prisoner into a different classification from those others who are validly restrained by law. Trillanes’ case fails to compare with the species of allowable leaves Emergency or compelling temporary leaves from imprisonment are allowed to all prisoners, at the discretion of the authorities or upon court orders. That this discretion was gravely abused, petitioner failed to establish. In fact, the trial court previously allowed petitioner to register as a voter in December 2006, file his certificate of candidacy in February 2007, cast his vote on May 14, 2007, be proclaimed as senator-elect, and take his oath of office on June 29, 2007. In a seeming attempt to bind or twist the hands of the trial court lest it be accused of taking a complete turn-around, petitioner largely banks on these prior grants to him and insists on unending concessions and blanket authorizations. CONSTITUTIONAL LAW 2 CASE DIGESTS

PEOPLE OF THE PHILIPPINES, plaintiff-appellee, vs. ROMEO G. JALOSJOS, accused-appellant. Facts: The victim of rape in this case was a minor below twelve (12) years of age, who herself narrated the shameful details of the dastardly act against her virtue. The victim was peddled for commercial sex by her own guardian whom she treated as a foster father. Because the complainantwas a willing victim, the acts of rape were preceded by several acts of lasciviousness on distinctlyseparate occasions..The accused was then CongressmanRomeo Jalosjoswho, inspite of his having been charged and convicted by the trial court for statutory rape, was stillre-elected to his congressional office. On December 16, 1996, two (2) informations for the crime ofstatutory rape and twelve (12) for acts of lasciviousness, were filed against accused-appellant The victim, Maria Rosilyn, grew up in a two-storey apartment in Pasay City under the care of SimplicioDelantar, whom she treated as her own father. Simplicio was a fifty-six year old homosexual whose ostensible source of income was selling longganiza and tocino and accepting boarders at his house.He, however, was also engaged in the skin trade as a pimp. Rosilyn ran away from home with the help of one of their boarders. They went tothe Pasay City Police where she executed a sworn statement against SimplicioDelantar. Rosilynwas thereafter taken to the custody of the Department of Social Welfare and Development (DSWD).The National Bureau of Investigation (NBI) conducted an investigation, which eventually led to thefiling of criminal charges against accused-appellant He was also convicted on six (6) counts of acts of lasciviousness. Issue/s 1. WHETHER OR NOT THE TRIAL COURT GRIEVOUSLY ERRED IN CONVICTING THE ACCUSED-APPELLANT BASED ON TESTIMONY OF THE PRIVATE COMPLAINANT, CONSIDERING THE ATTENDANT INDICIA OF INCONSISTENCIES AND UNTRUTHS.

CONSTITUTIONAL LAW 2 CASE DIGESTS

2. WHETHER OR NOT THE TRIAL COURT GRIEVOUSLY ERRED IN DISREGARDING THE SIGNIFICANCE OF THE CONFLICTING STATEMENTS GIVEN BY THE PRIVATE COMPLAINANT. 3. WHETHER OR NOT THE TRIAL COURT GRIEVOUSLY ERRED IN DISREGARDING THE SIGNIFICANCEOF PRIVATE COMPLAINANT'S FAILURE TO IDENTIFY THE ACCUSED-APPELLANT. 4. WHETHER OR NOT THE TRIAL COURT GRIEVOUSLY ERRED IN RULING THAT THE PRIVATECOMPLAINANT WAS A MINOR LESS THAN TWELVE YEARS OF AGE WHEN THECLAIMED INCIDENTS ALLEGEDLY TOOK PLACE. 5. WHETHER OR NOT THE TRIAL COURT GRIEVOUSLY ERRED IN FINDING THAT RAPE WASCOMMITTED AGAINST THE PRIVATE COMPLAINANT. Ruling The Supreme Court affirmed the decision of the RTC Makati with modification of penalty. 1. TESTIMONY OF VICTIM; DOCTRINE OFFALSUS IN UNO FALSUS IN OMNIBUS; APPLICATION THEREOF NOT AN ABSOLUTERULE OF LAW; CASE AT BAR. The contention is without merit. Falsus in unofalsus inomnibus is not an absolute rule of law and is in fact rarely applied in modernjurisprudence.Trier of facts are notbound to believe all that any witness has said; they may accept some portions of his testimony and reject other portions, according to what seems to them, upon other facts and circumstances to be thetruth . . . Even when witnesses are found to have deliberately falsified in some material particulars,the jury are not required to reject the whole of their uncorroborated testimony, but may credit suchportions as they deem worthy of belief. 2. CREDIBILITY OF WITNESSES; NOT AFFECTED BY SOMEAMBIGUOUS ANSWERS ON WITNESS STAND, WHICH REFERS TO MINOR ANDPERIPHERAL DETAILS; CASE AT BARA reading of the pertinent transcript ofstenographic notes reveals that Rosilyn was in fact firm and consistent on the fact of rape andlascivious conduct committed on her by accused-appellant. She answered in clear, simple andnatural words customary of children of her age. CONSTITUTIONAL LAW 2 CASE DIGESTS

3. IDENTIFICATION OF THE ACCUSED; DEFECT IN OUT-OF-COURT IDENTIFICATION OF THE ACCUSED CAN BE CURED BY AN IDENTIFICATIONSUBSEQUENTLY MADE IN COURT; APPLICATION IN CASE AT BAR.Contrary to thecontentions of accused-appellant, the records reveal that Rosilyn positively and unhesitatingly identified accused-appellant at the courtroom. Such identification during the trial cannot be diminished by the fact that in her sworn statement, Rosilyn referred to accusedappellant as her abuser based on the name she heard from the person to whom she was introduced and on the name she saw and read in accused-appellant's office. 4. AGE OF THE VICTIM IN RAPE CASES MAY BE ESTABLISHED BY DOCUMENTARY EVIDENCE OTHER THAN THE BIRTH CERTIFICATE; PRESENT INCASE AT BAR. — It is settled that in cases of statutory rape, the age of the victim may be provedby the presentation of her birth certificate. In the case at bar, accusedappellant contends that the birth certificate of Rosilyn should not have been considered by the trial court because said birthcertificate has already been ordered cancelled and expunged from the records by the Regional TrialCourt of Manila, Branch 38, in Special Proceedings No. 97-81893, dated April 11, 1997. Even assuming the absence of a valid birth certificate, there is sufficient and ample proof of the complainant's age in the records. Rosilyn's Baptismal Certificate can likewise serve as proofof her age. In People v. Liban, we ruled that the birth certificate, or in lieu thereof. 5. WHEN CONSUMMATED; SUFFICIENTLYESTABLISHED IN CASE AT BAR. — True, in People v. Campuhan, we explained that thephrase, "the mere touching of the external genitalia by the penis capable of consummating thesexual act is sufficient to constitute carnal knowledge.Theinevitable contact between accusedappellant's penis, and at the very least, the labia of the pudendum of Rosilyn, was confirmed when she felt pain inside her vagina when the "idiniin" part ofaccusedappellant's sex ritual was performed. 6. STATUTORY RAPE; ELEMENTS; ESTABLISHED IN CASE AT BAR. — At the time of commission of the crimes complained of herein in 1996, statutory rape was penalizedunder Section 11 of R.A. 7659, which amended Article 335 of the Revised Penal Code, to wit:When and how rape is committed. — Rape is CONSTITUTIONAL LAW 2 CASE DIGESTS

committed by having carnal knowledge of a womanunder any of the following circumstances: 1. By using force or intimidation; 2. When the woman isdeprived of reason or otherwise unconscious; and 3. When the woman is under twelve years of age or is demented. GOVERNMENT OF THE UNITED STATES OF AMERICA, represented by the Philippine Department of Justice, petitioner, Hon. GUILLERMO G. PURGANAN, Morales, and Presiding Judge, Regional Trial Court of Manila, Branch 42; and MARK B. JIMENEZ a.k.a. MARIO BATACAN CRESPO, respondents. G.R. No. 148571. September 24, 2002 Facts: The petition at bar seeking to void and set aside the Orders issued by the Regional Trial Court (RTC) of Manila, Branch 42. The first assailed Order set for hearing petitioner’s application for the issuance of a warrant for the arrest of Respondent Mark B. Jimenez. Pursuant to the existing RP-US Extradition Treaty, the US Government requested the extradition of Mark Jimenez. A hearing was held to determine whether a warrant of arrest should be issued. Afterwards, such warrant was issued but the trial court allowed Jimenez to post bail for his provisional liberty. Issue/s: Whether or not the right to bail is available in extradition proceedings Discussions: The constitutional right to bail “flows from the presumption of innocence in favor of every accused who should not be subjected to the loss of freedom as thereafter he would be entitled to acquittal, unless his guilt be proved beyond reasonable doubt.” It follows that the constitutional provision on bail will not apply to a case like extradition, where the presumption of innocence is not at issue. Ruling/s: CONSTITUTIONAL LAW 2 CASE DIGESTS

No. The court agree with petitioner. As suggested by the use of the word “conviction,” the constitutional provision on bail quoted above, as well as Section 4 of Rule 114 of the Rules of Court, applies only when a person has been arrested and detained for violation of Philippine criminal laws. It does not apply to extradition proceedings, because extradition courts do not render judgments of conviction or acquittal. It is also worth noting that before the US government requested the extradition of respondent, proceedings had already been conducted in that country. But because he left the jurisdiction of the requesting state before those proceedings could be completed, it was hindered from continuing with the due processes prescribed under its laws. His invocation of due process now has thus become hollow. He already had that opportunity in the requesting state; yet, instead of taking it, he ran away. RODOLFO FARINAS VS EXECUTIVE SECRETARY [G.R. No. 147387. December 10, 2003] NATURE OF THE CASE: Petitions under Rule 65 of the Rules of Court, as amended, seeking to declare as unconstitutional Section 14 of Republic Act No. 9006 (The Fair Election Act), insofar as it expressly repeals Section 67 of Batas Pambansa Blg. 881 (The Omnibus Election Code) which provides: SEC. 67. Candidates holding elective office. – Any elective official, whether national or local, running for any office other than the one which he is holding in a permanent capacity, except for President and Vice-President, shall be considered ipso facto resigned from his office upon the filing of his certificate of candidacy. FACTS: The petitioners now come to the Court alleging in the main that Section 14 of Rep. Act No. 9006, insofar as it repeals Section 67 of the Omnibus Election Code, is unconstitutional for being in violation of Section 26(1), Article VI of the CONSTITUTIONAL LAW 2 CASE DIGESTS

Constitution, requiring every law to have only one subject which should be expressed in its title. According to the petitioners, the inclusion of Section 14 repealing Section 67 of the Omnibus Election Code in Rep. Act No. 9006 constitutes a proscribed rider. They point out the dissimilarity in the subject matter of Rep. Act No. 9006, on the one hand, and Section 67 of the Omnibus Election Code, on the other. Rep. Act No. 9006 primarily deals with the lifting of the ban on the use of media for election propaganda and the elimination of unfair election practices, while Section 67 of the Omnibus Election Code imposes a limitation on elective officials who run for an office other than the one they are holding in a permanent capacity by considering them as ipso facto resigned therefrom upon filing of the certificate of candidacy. The repeal of Section 67 of the Omnibus Election Code is thus not embraced in the title, nor germane to the subject matter of Rep. Act No. 9006. The petitioners also assert that Section 14 of Rep. Act No. 9006 violates the equal protection clause of the Constitution because it repeals Section 67 only of the Omnibus Election Code, leaving intact Section 66 thereof which imposes a similar limitation to appointive officials, thus: SEC. 66. Candidates holding appointive office or position. – Any person holding a public appointive office or position, including active members of the Armed Forces of the Philippines, and officers and employees in government-owned or controlled corporations, shall be considered ipso facto resigned from his office upon the filing of his certificate of candidacy. They contend that Section 14 of Rep. Act No. 9006 discriminates against appointive officials. By the repeal of Section 67, an elective official who runs for office other than the one which he is holding is no longer considered ipso facto resigned therefrom upon filing his certificate of candidacy. Elective officials continue in public office even as they campaign for reelection or election for another elective position. On the other hand, Section 66 has been retained; thus, the limitation on appointive officials remains - they are still considered ipso facto resigned from their offices upon the filing of their certificates of candidacy.

CONSTITUTIONAL LAW 2 CASE DIGESTS

The petitioners assert that Rep. Act No. 9006 is null and void in its entirety as irregularities attended its enactment into law. The law, not only Section 14 thereof, should be declared null and void. Even Section 16 of the law which provides that “[t]his Act shall take effect upon its approval” is a violation of the due process clause of the Constitution, as well as jurisprudence, which require publication of the law before it becomes effective. Finally, the petitioners maintain that Section 67 of the Omnibus Election Code is a good law; hence, should not have been repealed. The petitioners cited the ruling of the Court in Dimaporo v. Mitra, Jr.,[13] that Section 67 of the Omnibus Election Code is based on the constitutional mandate on the “Accountability of Public Officers:” Sec. 1. Public office is a public trust. Public officers and employees must at all times be accountable to the people, serve them with utmost responsibility, integrity, loyalty and efficiency, act with patriotism and justice, and lead modest lives. Consequently, the respondents Speaker and Secretary General of the House of Representatives acted with grave abuse of discretion amounting to excess or lack of jurisdiction for not considering those members of the House who ran for a seat in the Senate during the May 14, 2001 elections as ipso facto resigned therefrom, upon the filing of their respective certificates of candidacy. ISSUES: W/N Section 14 of Rep. Act No. 9006 Is a Rider. W/N Section 14 of Rep. Act No. 9006 Is Violative of the Equal Protection Clause of the Constitution. W/N Section 16 of the law which provides that “[t]his Act shall take effect upon its approval” is a violation of the due process clause of the Constitution, as well as jurisprudence, which require publication of the law before it becomes effective.

CONSTITUTIONAL LAW 2 CASE DIGESTS

HELD: To determine whether there has been compliance with the constitutional requirement that the subject of an act shall be expressed in its title, the Court laid down the rule that – Constitutional provisions relating to the subject matter and titles of statutes should not be so narrowly construed as to cripple or impede the power of legislation. The requirement that the subject of an act shall be expressed in its title should receive a reasonable and not a technical construction. It is sufficient if the title be comprehensive enough reasonably to include the general object which a statute seeks to effect, without expressing each and every end and means necessary or convenient for the accomplishing of that object. Mere details need not be set forth. The title need not be an abstract or index of the Act. The title of Rep. Act No. 9006 reads: “An Act to Enhance the Holding of Free, Orderly, Honest, Peaceful and Credible Elections through Fair Election Practices.” The Court is convinced that the title and the objectives of Rep. Act No. 9006 are comprehensive enough to include the repeal of Section 67 of the Omnibus Election Code within its contemplation. To require that the said repeal of Section 67 of the Code be expressed in the title is to insist that the title be a complete index of its content. The purported dissimilarity of Section 67 of the Omnibus Election Code, which imposes a limitation on elective officials who run for an office other than the one they are holding, to the other provisions of Rep. Act No. 9006, which deal with the lifting of the ban on the use of media for election propaganda, does not violate the “one subject-one title” rule. This Court has held that an act having a single general subject, indicated in the title, may contain any number of provisions, no matter how diverse they may be, so long as they are not inconsistent with or foreign to the general subject, and may be considered in furtherance of such subject by providing for the method and means of carrying out the general subject. The legislators considered Section 67 of the Omnibus Election Code as a form of harassment or discrimination that had to be done away with and repealed. The executive department found cause with Congress when the President of the Philippines signed the measure into law. For sure, some sectors of society and in government may believe that the repeal of Section 67 is bad policy as it would CONSTITUTIONAL LAW 2 CASE DIGESTS

encourage political adventurism. But policy matters are not the concern of the Court. Government policy is within the exclusive dominion of the political branches of the government. It is not for this Court to look into the wisdom or propriety of legislative determination. Indeed, whether an enactment is wise or unwise, whether it is based on sound economic theory, whether it is the best means to achieve the desired results, whether, in short, the legislative discretion within its prescribed limits should be exercised in a particular manner are matters for the judgment of the legislature, and the serious conflict of opinions does not suffice to bring them within the range of judicial cognizance. Congress is not precluded from repealing Section 67 by the ruling of the Court in Dimaporo v. Mitra upholding the validity of the provision and by its pronouncement in the same case that the provision has a laudable purpose. Over time, Congress may find it imperative to repeal the law on its belief that the election process is thereby enhanced and the paramount objective of election laws – the fair, honest and orderly election of truly deserving members of Congress – is achieved. Substantial distinctions clearly exist between elective officials and appointive officials. The former occupy their office by virtue of the mandate of the electorate. They are elected to an office for a definite term and may be removed therefrom only upon stringent conditions. On the other hand, appointive officials hold their office by virtue of their designation thereto by an appointing authority. Some appointive officials hold their office in a permanent capacity and are entitled to security of tenure while others serve at the pleasure of the appointing authority. Finally, the “Effectivity” clause (Section 16) of Rep. Act No. 9006 which provides that it “shall take effect immediately upon its approval,” is defective. However, the same does not render the entire law invalid. In Tañada v. Tuvera, this Court laid down the rule: ... the clause “unless it is otherwise provided” refers to the date of effectivity and not to the requirement of publication itself, which cannot in any event be omitted. This clause does not mean that the legislator may make the law effective immediately upon approval, or on any other date without its previous publication. Publication is indispensable in every case, but the legislature may in its discretion provide that the usual fifteen-period shall be shortened or extended…. CONSTITUTIONAL LAW 2 CASE DIGESTS

Following Article 2 of the Civil Code and the doctrine enunciated in Tañada, Rep. Act No. 9006, notwithstanding its express statement, took effect fifteen days after its publication in the Official Gazette or a newspaper of general circulation. In conclusion, it bears reiterating that one of the firmly entrenched principles in constitutional law is that the courts do not involve themselves with nor delve into the policy or wisdom of a statute. That is the exclusive concern of the legislative branch of the government. When the validity of a statute is challenged on constitutional grounds, the sole function of the court is to determine whether it transcends constitutional limitations or the limits of legislative power. No such transgression has been shown in this case. PHILIPPINE AMUSEMENT v. BIR, GR No. 172087, 2011-03-15 Facts: Petitioner further seeks to prohibit the implementation of Bureau of Internal Revenue (BIR) Revenue Regulations No. 16-2005 for being contrary to law. With the enactment of R.A. No. 9337[10] on May 24, 2005, certain sections of the National Internal Revenue Code of 1997 were amended. Different groups came to this Court via petitions for certiorari and prohibition[11] assailing the validity and constitutionality of R.A. No. 9337 10% Value Added Tax (VAT) on sale of goods and properties 10% VAT on importation of goods 10% VAT on sale of services and use or lease of properties... the Court dismissed all the petitions and upheld the constitutionality of R.A. No. 9337. On the same date, respondent BIR issued Revenue Regulations (RR) No. 16-2005, [13] specifically identifying PAGCOR as one of the franchisees subject to 10% VAT imposed under Section 108 of the National Internal Revenue Code of 1997, as amended by R.A.No. 9337. CONSTITUTIONAL LAW 2 CASE DIGESTS

Furthermore, according to the OSG,... public respondent BIR exceeded its statutory authority when it enacted RR No. 16-2005, because the latter's provisions are contrary to the mandates of P.D. No. 1869 in relation to R.A. No. 9337. Issues: whether or not PAGCOR is still exempt... hether or not PAGCOR is still exempt... t from VAT with the enactment of R.A. No. 9337. Ruling: Anent the validity of RR No. 16-2005, the Court holds that the provision subjecting PAGCOR to 10% VAT is invalid for being contrary to R.A. No. 9337. Nowhere in R.A. No. 9337 is it provided that petitioner can be subjected to VAT. R.A. No. 9337 is clear only as to... the removal of petitioner's exemption from the payment of corporate income tax, which was already addressed above by this Court. As pointed out by the OSG, R.A. No. 9337 itself exempts petitioner from VAT pursuant to Section 7 (k) thereof... the following transactions shall be exempt from the value-added tax: Transactions which are exempt under international agreements to which the Philippines is a signatory or under special laws Petitioner is exempt from the payment of VAT, because PAGCOR's charter, P.D. No. 1869, is a special law that grants petitioner exemption from taxes. Moreover, the exemption of PAGCOR from VAT is supported by Section 6 of R.A. No. 9337 The following services performed in the Philippines by VAT-registered persons shall be subject to zero percent (0%) rate; Services rendered to persons or entities whose exemption under special laws... subjects the supply of such services to zero percent (0%) rate... although R.A. No. 9337 introduced amendments to Section 108 of R.A. No. 8424 by imposing VAT on CONSTITUTIONAL LAW 2 CASE DIGESTS

other services not previously covered, it did not amend the portion of Section 108 (B) (3) that subjects to zero percent rate services performed by VAT-registered persons to persons or entities whose exemption under special laws or international agreements to which the Philippines is a signatory effectively subjects the supply of such services to 0% rate. PUBLIC POLICY CENTRAL BANK EMPLOYEES ASSOCIATION V. BSP (2004) Facts: The new Central Bank Act took effect and gave way for the creation of Bangko Sentral ng Pilipinas. Other Governmental Financial Institutions (GFIs) also amended their charters. After almost 8 years following the amendment of the GFIs’ charters, BSP’s employees, through petitioner, filed a petition for prohibition against the BSP and the Executive Secretary to restrain the respondents from further implementing the last proviso in Sec. 15, Art. II of the New Central Bank Act (i.e., the exemption from the Salary Standardization Law (SSL) of all employees with salary grade of 19 and the non-exemption of those having a salary grade under 19). They alleged its constitutionality for being an invalid “class legislation”. Petitioner’s Contentions: The said proviso violates equal protection clause because only the officers of the BSP (those holding the salary grade of 19 and up) are exempted from the SSL. Those belonging from 19 and up and those 19 below do not really differ from one other in terms of the nature of work and expertise. Other GFIs, which are the same as the BSP, exempt all their rank-and-file personnel from SSL without any distinction. BSP’s contention: The proviso is not unconstitutional as it can stand the constitutional test, provided it is construed in harmony with other provisions of the same law, such as the mandate of the Monetary Board to “establish professionalism and excellence at all levels in accordance with sound principles of management.” CONSTITUTIONAL LAW 2 CASE DIGESTS

Solicitor General, on behalf of respondent Executive Secretary: The proviso is not unconstitutional as the classification is based on actual and real differentiation, even as it adheres to the enunciated policy of the new SB Act to establish professionalism and excellence within the BSP subject to prevailing laws and policies of the national government. Issue: WON the proviso is unconstitutional for being violative of equal protection clause. Held: YES, the proviso is unconstitutional for being violative of the equal protection clause. Equal protection clause does not prevent the Legislature from establishing classes of individuals or objects upon which different rules shall operate – so long as the classification is not unreasonable. Equality of operation of statutes does not mean indiscriminate operation on persons themselves, but on persons according to the circumstances surrounding them. It guarantees equality, not identity of rights. In the case at bar, it is clear in the legislative deliberations that the exemption of officers (SG 20 and above) from the SSL was intended to address the BSP’s lack of competitiveness in terms of attracting competent officers and executives. It was not intended to discriminate against the rank-and-file and the resulting discrimination or distinction has a rational basis and is not palpably, purely, and entirely arbitrary in the legislative sense. However, in the subsequent passages of the amendment on the charters of other GFI, the surrounding circumstances of the case changed. The subsequent amendments of the other GFIs’ charter (i.e., express authorization to determine and institute its own compensation and wage structure, and explicit exemption – without distinction as to salary grade or position – all employees of the GFI from the SSL) resulted to the oppressive results of Congress’ inconsistent and unequal policy towards the BSP rank-and-file and those of the seven other GFI. In the case at bar, it is precisely the fact that as regards the exemption from the SSL, there are no characteristics peculiar only to the seven GFIs or their rank-and-file so as to justify the exemption which BSP rankand-file employees were denied (not to mention the anomaly of the SEC getting one). The distinction made by the law is not only superficial, but also CONSTITUTIONAL LAW 2 CASE DIGESTS

arbitrary. It is not based on substantial distinctions that make real differences between the BSP rank-and-file and the seven other GFIs. The subsequent grant to the rank-and-file of the seven other GFIs and continued denial to the BSP rank-and-file employees of the exemption from SSL breached the latter’s right to equal protection. The equal protection clause does not demand absolute equality but it requires that all persons shall be treated alike, under like circumstances and conditions both as to privileges conferred and liabilities enforced. Philippine National Bank vs Palma GR 157279 09 August 2005 Facts: Salary Standardization Law took effect on 01 July 1989. The Department of Budget and Management (DBM) issued Corporate Compensation Circular No. 10 (DBM-CCC No. 10) to implement R.A. 6758. It enumerated the other allowances/fringe benefits which are not integrated into the basic salary rates prescribed under R.A. 6758, but were allowed to be continued only for incumbents as of 30 June 1989. The same was ruled ineffective due to nonpublication. Respondents filed petition for mandamus alleging that they were deprived of the said allowances enjoyed by other employees. They contend that the withholding of their entitlement to the same benefits is an unfair discrimination and a violation of their equal protection clause of the Constitution. Issue: Whether or not unconstitutionality of RA 6758 on the ground of violation of equal protection clause be attacked collaterally? Decision: Respondents further argue that upholding the distinction among the employees on the basis of the date of their hiring is violative of the equal protection clause of the Constitution. For reasons of public policy, the constitutionality of a law cannot be attacked in a collateral way. A law is deemed valid unless declared null and void by a competent court; more so when the issue has not been duly pleaded in the trial court. The question of constitutionality must be raised at the earliest opportunity. Respondents not only failed to challenge the constitutionality of RA CONSTITUTIONAL LAW 2 CASE DIGESTS

6758; worse, they used it in seeking compensation from petitioner. The settled rule is that courts will not anticipate a question of constitutional law in advance of the necessity of deciding it. A valid classification was made by the law in segregating other employees from the incumbents who were already receiving the benefits on July 1, 1989. Olivarez v. Sandiganbayan G.R. No. 118533 October 4, 1995 248 SCRA 700 (1995) Fact: Baclaran Credit Cooperative, Inc. (BCCI), through its board member charged petitioner Parañaque Mayor Olivarez with Violation of the Anti-Graft and Corrupt Practices Act for unreasonably refusing to issue a mayor’s permit despite request and follow-ups to implement Parañaque Sangguniang Bayan Resolution which petitioner himself approved. the resolution authorized BCCI to set up a night manufacturer’s fair during the Christmas fiesta celebration of and at Baclaran for 60 days for which they will use a portion of the service road of Roxas Boulevard from the corner of Opena to Rivera Streets. BCCI requesting assistance for the issuance of a mayor’s permit, BCCI counsel Atty. Renato Dilag to petitioner formally demanding implementation of Resolution. petitioner replied letter to Atty. Dilag stating among others that the non-implementation of Resolution was due to BCCI’s failure to apply for appropriate permit and license to operate the Night Manufacturer’s Fair which was one of the conditions in the authorization. Issue: Whether the petitioner disregarded right to the Equal Protection of BCCI? Held: Yes, petitioner failed to show, in apparent disregard of BCCI’s right to equal protection, that BCCI and the unidentified Baclaran-based vendors’ associations were not similarly situated as to give at least a semblance of legality to the apparent haste with which said executive order was issued. It would seem that if there was any interest served by such executive order, it was that of herein petitioner. As the mayor of the municipality, the officials referred to were definitely under his authority and he was not without recourse to take appropriate action on the letter-application of BCCI although the same was not CONSTITUTIONAL LAW 2 CASE DIGESTS

strictly in accordance with normal procedure. There was nothing to prevent him from referring said letter-application to the licensing department, but which paradoxically he refused to do. Whether petitioner was impelled by any material interest or ulterior motive may be beyond us for the moment since this is a matter of evidence, but the environmental facts and circumstances are sufficient to create a belief in the mind of a reasonable man that this would not be completely improbable, absent countervailing clarification. Lastly, it may not be amiss to add that petitioner, as a municipal mayor, is expressly authorized and has the power to issue permits and licenses for the holding of activities for any charitable or welfare purpose, pursuant to the Local Government Code of. Hence, he cannot really feign total lack of authority to act on the letter-application of BCCI.. Coconut Oil Refiners vs Torres GR 132527 29 July 2005 Facts: This is a Petition to enjoin and prohibit the public respondent Ruben Torres in his capacity as Executive Secretary from allowing other private respondents to continue with the operation of tax and duty-free shops located at the Subic Special Economic Zone (SSEZ) and the Clark Special Economic Zone (CSEZ). The petitioner seeks to declare Republic Act No. 7227 as unconstitutional on the ground that it allowed only tax-free (and duty-free) importation of raw materials, capital and equipment. It reads: The Subic Special Economic Zone shall be operated and managed as a separate customs territory ensuring free flow or movement of goods and capital within, into and exported out of the Subic Special Economic Zone, as well as provide incentives such as tax and duty-free importations of raw materials, capital and equipment. However, exportation or removal of goods from the territory of the Subic Special Economic Zone to the other parts of the Philippine territory shall be subject to customs duties and taxes under the Customs and Tariff Code and other relevant tax laws of thePhilippines [RA 7227, Sec 12 (b)]. Petitioners contend that the wording of Republic Act No. 7227 clearly limits the grant of tax incentives to the importation of raw materials, capital and equipment only thereby violating the equal protection clause of the Constitution. CONSTITUTIONAL LAW 2 CASE DIGESTS

He also assailed the constitutionality of Executive Order No. 97-A for being violative of their right to equal protection. They asserted that private respondents operating inside the SSEZ are not different from the retail establishments located outside. Issue: Whether or not Republic Act No. 7227 is valid on the ground that it violates the equal protection clause. Decision: The SC ruled in the negative. The phrase ‘tax and duty-free importations of raw materials, capital and equipment was merely cited as an example of incentives that may be given to entities operating within the zone. Public respondent SBMA correctly argued that the maxim expressio unius est exclusio alterius, on which petitioners impliedly rely to support their restrictive interpretation, does not apply when words are mentioned by way of example. The petition with respect to declaration of unconstitutionality of Executive Order No. 97-A cannot be, likewise, sustained. The guaranty of the equal protection of the laws is not violated by a legislation based which was based on reasonable classification. A classification, to be valid, must (1) rest on substantial distinction, (2) be germane to the purpose of the law, (3) not be limited to existing conditions only, and (4) apply equally to all members of the same class. Applying the foregoing test to the present case, this Court finds no violation of the right to equal protection of the laws. There is a substantial distinctions lying between the establishments inside and outside the zone. There are substantial differences in a sense that, investors will be lured to establish and operate their industries in the so-called ‘secured area and the present business operators outside the area. There is, then, hardly any reasonable basis to extend to them the benefits and incentives accorded in R.A. 7227.

Beltran vs Secretary of Health GR 133640 25 November 2005 CONSTITUTIONAL LAW 2 CASE DIGESTS

Facts: The promotion of public health is a fundamental obligation of the State. The health of the people is a primordial governmental concern. The National Blood Services Act was enacted in the exercise of the State’s police power in order to promote and preserve public health and safety. What may be regarded as a denial of the equal protection of the laws is a question not always easily determined. No rule that will cover every case can be formulated. Class legislation, discriminating against some and favoring others is prohibited but classification on a reasonable basis and not made arbitrarily or capriciously is permitted. Republic Act No. 7719 or the National Blood Services Act of 1994 was enacted into law on April 2, 1994. The Act seeks to provide an adequate supply of safe blood by promoting voluntary blood donation and by regulating blood banks in the country. It was approved by then President Fidel V. Ramos on May 15, 1994 and was subsequently published in the Official Gazette on August 18, 1994. The law took effect on August 23, 1994. On April 28, 1995, Administrative Order No. 9, Series of 1995, constituting the Implementing Rules and Regulations of said law was promulgated by respondent Secretary of the Department of Health (DOH). Section 7 of R.A. 7719 provides, Phase-out of Commercial Blood Banks – All commercial blood banks shall be phased-out over a period of two (2) years after the effectivity of this Act, extendable to a maximum period of two (2) years by the Secretary. ” Section 23. Process of Phasing Out. — The Department shall effect the phasing-out of all commercial blood banks over a period of two (2) years, extendible for a maximum period of two (2) years after the effectivity of R.A. 7719. The decision to extend shall be based on the result of a careful study and review of the blood supply and demand and public safety.” Years prior to the passage of the National Blood Services Act of 1994, petitioners have already been operating commercial blood banks under Republic Act No. 1517, entitled “An Act Regulating the Collection, Processing and Sale of Human Blood, and the Establishment and Operation of Blood Banks and Blood Processing Laboratories.” CONSTITUTIONAL LAW 2 CASE DIGESTS

The law, which was enacted on June 16, 1956, allowed the establishment and operation by licensed physicians of blood banks and blood processing laboratories. On May 20, 1998, prior to the expiration of the licenses granted to petitioners, they filed a petition for certiorari with application for the issuance of a writ of preliminary injunction or temporary restraining order under Rule 65 of the Rules of Court assailing the constitutionality and validity of the aforementioned Act and its Implementing Rules and Regulations. Issue: Whether or not Section 7 of RA 7719 and its implementing rules is valid on the ground that it violates the equal protection clause. Decision: Petition granted. The assailed law and its implementing rules are constitutional and valid. What may be regarded as a denial of the equal protection of the laws is a question not always easily determined. No rule that will cover every case can be formulated. Class legislation, discriminating against some and favoring others is prohibited but classification on a reasonable basis and not made arbitrarily or capriciously is permitted. The classification, however, to be reasonable: (a) must be based on substantial distinctions which make real differences; (b) must be germane to the purpose of the law; (c) must not be limited to existing conditions only; and, (d) must apply equally to each member of the class. Republic Act No. 7719 or The National Blood Services Act of 1994, was enacted for the promotion of public health and welfare. Based on the foregoing, the Legislature never intended for the law to create a situation in which unjustifiable discrimination and inequality shall be allowed. To effectuate its policy, a classification was made between nonprofit blood banks/centers and commercial blood banks. We deem the classification to be valid and reasonable for the following reasons: First, it was based on substantial distinctions. The former operates for purely humanitarian reasons and as a CONSTITUTIONAL LAW 2 CASE DIGESTS

medical service while the latter is motivated by profit. Also, while the former wholly encourages voluntary blood donation, the latter treats blood as a sale of commodity. Second, the classification, and the consequent phase out of commercial blood banks is germane to the purpose of the law, that is, to provide the nation with an adequate supply of safe blood by promoting voluntary blood donation and treating blood transfusion as a humanitarian or medical service rather than a commodity. This necessarily involves the phase out of commercial blood banks based on the fact that they operate as a business enterprise, and they source their blood supply from paid blood donors who are considered unsafe compared to voluntary blood donors as shown by the USAID-sponsored study on the Philippine blood banking system. Third, the Legislature intended for the general application of the law. Its enactment was not solely to address the peculiar circumstances of the situation nor was it intended to apply only to the existing conditions. Lastly, the law applies equally to all commercial blood banks without exception.The promotion of public health is a fundamental obligation of the State. The health of the people is a primordial governmental concern. Basically, the National Blood Services Act was enacted in the exercise of the State’s police power in order to promote and preserve public health and safety. Based on the grounds raised by petitioners to challenge the constitutionality of the National Blood Services Act of 1994 and its Implementing Rules and Regulations, the Court finds that petitioners have failed to over overcome the presumption of constitutionality of the law. As to whether the Act constitutes a wise legislation, considering the issues being raised by petitioners, is for Congress to determine. MIRASOL VS DPWH GR. NO. 158793, June 8, 2006 FACTS: Petitioners filed before the court a petition for declaratory judgment with application for temporary restraining order and injunction. It seeks the declaration of nullification of administrative issuances for being inconsistent with the provisions of Republic Act 2000 (Limited Access Highway Act) which was enacted in 1957.

CONSTITUTIONAL LAW 2 CASE DIGESTS

Previously, pursuant to its mandate under RA 2000, DPWH issued on June 25, 1998 Dept. Order no. 215 declaring the Manila Cavite (Coastal Road) Toll Expressway as limited access facilities. Petitioners filed an Amended Petition on February 8, 2001 wherein petitioners sought the declaration of nullity of the aforesaid administrative issuances. The petitioners prayed for the issuance of a temporary restraining order to prevent the enforcement of the total ban on motorcycles along NLEX, SLEX, Manila-Cavite (Coastal Road) toll Expressway under DO 215. RTC, after due hearing, granted the petitioner’s application for preliminary injunction conditioned upon petitioner’s filing of cash bond in the amount of P100, 000 which petitioners complied. DPWH issued an order (DO 123) allowing motorcycles with engine displacement of 400 cubic centimeters inside limited access facilities (toll ways). Upon assumption of Hon. Presiding Judge Cornejo, both the petitioners and respondents were required to file their Memoranda. The court issued an order dismissing the petition but declaring invalid DO 123. The petitioners moved for reconsideration but it was denied. RTC ruled that DO 74 is valid but DO 123 is invalid being violative of the equal protection clause of the Constitution ISSUE: Whether RTC’s decision is barred by res judicata? Whether DO 74, DO 215 and the TRB regulation contravene RA 2000. Whether AO 1 is unconstitutional. HELD: CONSTITUTIONAL LAW 2 CASE DIGESTS

1. NO. The petitioners are mistaken because they rely on the RTC’s Order granting their prayer for a writ of preliminary injunction. Since petitioners did not appeal from that order, the petitioners presumed that the order became a final judgment on the issues. The order granting the prayer is not an adjudication on the merits of the case that would trigger res judicata. A preliminary injunction does not serve as a final determination of the issues, it being a provisional remedy. 2. YES. The petitioners claimed that DO 74, DO 215 and TRB’s rules and regulation issued under them unduly expanded the power of the DPWH in sec. 4 of RA 2000 to regulate toll ways. They contend that DPWH’s regulatory authority is limited to acts like redesigning curbings or central dividing sections. They claim that DPWH is only allowed to redesign the physical structure of toll ways and not to determine “who or what can be qualifies as toll ways user”. The court ruled that DO 74 and DO 215 are void because the DPWH has no authority to declare certain expressways as limited access facilities. Under the law, it is the DOTC which is authorized to administer and enforce all laws, rules and regulations in the field of transportation and to regulate related activities. Since the DPWH has no authority to regulate activities relative to transportation, the Toll Regulatory Board (TRB) cannot derive its power from the DPWH to issue regulations governing limited access facilities. The DPWH cannot delegate a power or function which it does not possess in the first place. 3. NO. The Court emphasized that the secretary of the then Department of Public Works and Communications had issued AO 1 in February 1968, as authorized under Section 3 of Republic Act 2000, prior to the splitting of the department and the eventual devolution of its powers to the DOTC. CONSTITUTIONAL LAW 2 CASE DIGESTS

Because administrative issuances had the force and effect of law, AO 1 enjoyed the presumption of validity and constitutionality. The burden to prove its unconstitutionality rested on the party assailing it, more so when police power was at issue and passed the test of reasonableness. The Administrative Order was not oppressive, as it did not impose unreasonable restrictions or deprive petitioners of their right to use the facilities. It merely set rules to ensure public safety and the uninhibited flow of traffic within those limited-access facilities. The right to travel did not mean the right to choose any vehicle in traversing a tollway. Petitioners were free to access the tollway as much as the rest of the public. However, the mode in which they wished to travel, pertaining to their manner of using the tollway, was a subject that could validly be limited by regulation. There was no absolute right to drive; on the contrary, this privilege was heavily regulated.

INTERNATIONAL SCHOOL ALLIANCE OF EDUCATORS v. QUISUMBING 333 SCRA 13 G.R. No. 128845 June 1, 2000 FACTS: International School Alliance of Educators (the School) hires both foreign and local teachers as members of its faculty, classifying the same into two: (1) foreignhires and (2) local-hires. In which, the School grants foreign-hires certain benefits not accorded local-hires including housing, transportation, shipping costs, taxes, home leave travel allowance and a salary rate 25% more than local hires based on “significant economic disadvantages” The labor union and the collective bargaining representative of all faculty members of the School, contested the difference in salary rates between foreign and local-hires. CONSTITUTIONAL LAW 2 CASE DIGESTS

The Union claims that the point-of-hire classification employed by the School is discriminatory to Filipinos and that the grant of higher salaries to foreign-hires constitutes racial discrimination. ISSUE: Whether or not the Union can invoke the equal protection clause to justify its claim of parity. RULING: Yes. The Labor Code’s and the Constitution’s provisions impregnably institutionalize in this jurisdiction the long honored legal truism of "equal pay for equal work." Persons who work with substantially equal qualifications, skill, effort and responsibility, under similar conditions, should be paid similar salaries. If an employer accords employees the same position and rank, the presumption is that these employees perform equal work. If the employer pays one employee less than the rest, it is not for that employee to explain why he receives less or why the others receive more. That would be adding insult to injury. The employer in this case has failed to discharge this burden. There is no evidence here that foreign-hires perform 25% more efficiently or effectively than the localhires. Both groups have similar functions and responsibilities, which they perform under similar working conditions. Hence, the Court finds the point-of-hire classification employed by respondent School to justify the distinction in the salary rates of foreign-hires and local hires to be an invalid classification. There is no reasonable distinction between the services rendered by foreign-hires and local-hires.

ARRESTS, SEARCHES AND SEIZURES ART. III, SEC 2 & 3 PURPOSE AND IMPORTANCE OF THE GUARANTY CONSTITUTIONAL LAW 2 CASE DIGESTS

AURELIO S. ALVERO vs ARSENIO P. DIZON, et al., G.R. No. L-342 May 4, 1946 FACTS: The petitioner has been accused of treason; that at the hearing on his petition for bail, the prosecution presented, as part of its evidence, certain documents which had been allegedly seized by soldiers of the United States Army, accompanied by Filipino Guerrillas in the petitioner’s house. The Petitioner further contends that the seized documents should be returned as it obtained by means of force and intimidation or through coercion, those are not his personal papers but part of the files of the New Leader’s Association, which was proven to be an organization created for the purpose of collaborating with the enemy. Lastly, the presentation of the seized documents in the trial is tantamount to compelling him to testify against himself, in violation of his constitutional rights. ISSUES: Whether or not the “seized” documents are legal? Whether or not the documents seized should be admitted as evidence in the trial court? HELD. No. The petition for Certiorari with Injunction is absolutely no merit. RATIONALE: The right of the officer and men of the United States Army to arrest the petitioner as a collaborationist suspect, and to seize his personal papers is unquestionable. Also, proclamation of General Douglas McArthur, as Commander in Chief of the United States of Army, declaring his purpose to remove certain citizens of the Philippines, who had voluntarily given aid and comfort to the enemy, in violation of the allegiance. EXCEPTION: Important exception to the necessity for a Search Warrant is the right of search and seizure as an incident to a lawful arrest. A lawful arrest may be made either while a crime is being committed or after its commission. The right to search includes in both instances that of searching the person of him who is arrested, in CONSTITUTIONAL LAW 2 CASE DIGESTS

order to find and seize things arrested with the crime as its fruits as the means by which it was committed. The Petitioner consented to the presentation of the seized documents, as part of the evidence for the prosecution, at the hearing in his petition for bail and at the trial of the case on the merits, without having insisted that the question of the alleged illegality of the search and seizure of said papers and documents should first have been directly litigated and established by a motion. COMPULSORY SELF-INCRIMINATION Not violated by the use of evidence of articles obtained by an unconstitutional search and seizure. Thus, the petitioner is estopped from questioning their admission. PURPOSE: (Adam vs New York) The purpose of the constitutional provisions against unlawful searched and seizures is to prevent violations of private security in person and property, and unlawful invasions of the sanctity of the home, by officers of the law acting under legislative and judicial sanction, and to give remedy against such usurpations when attempted. TO WHOM DIRECTED People vs. Andre Marti G.R. No. 81561, January 18, 1991 193 SCRA 57 (1991) Fact: In 1987, the appellant informed Anita Reyes that he was sending the packages to a friend in Zurich, Switzerland. Appellant filled up the contract necessary for the transaction, writing therein his name, passport number, the date of shipment and the name and address of the consignee, namely, “WALTER FIERZ, Mattacketr II, 8052 Zurich, Switzerland” Anita Reyes then asked the appellant if she could examine and inspect the packages. Appellant, however, refused, assuring her that the packages simply contained books, cigars, and gloves and were gifts to his friend in Zurich. In view of appellant’s representation, Anita Reyes no longer insisted on inspecting the packages. Before delivery of appellant’s box to the Bureau of Customs and/or Bureau of Posts, Mr. Job Reyes (proprietor) and husband of Anita (Reyes), following standard operating procedure, opened the CONSTITUTIONAL LAW 2 CASE DIGESTS

boxes for final inspection. When he opened appellant’s box, a peculiar odor emitted therefrom. His curiousity aroused, He made an opening on one of the cellophane wrappers and took several grams of the contents thereof. Job Reyes forthwith prepared a letter reporting the shipment to the NBI and requesting a laboratory examination of the samples he extracted from the cellophane wrapper. He brought the letter and a sample of appellant’s shipment to the Narcotics Section of the NBI and informed the them that the rest of the shipment was still in his office. Therefore, Job Reyes and three NBI agents, and a photographer, went to the Reyes’ office at Ermita. The package which allegedly contained books was likewise opened by Job Reyes. He discovered that the package contained bricks or cake-like dried marijuana leaves. The package which allegedly contained tabacalera cigars was also opened. It turned out that dried marijuana leaves were neatly stocked underneath the cigars. The NBI agents made an inventory and took charge of the box and of the contents thereof, after signing a “Receipt” acknowledging custody of the said effects . Thereafter, an Information was filed against appellant for violation of RA 6425, otherwise known as the Dangerous Drugs Act. Issue: Whether the search and seizure committed by the private individual inviolate the constitutional right of the accused against unlawful searches and seizures? Held: No, The constitutional proscription against unlawful searches and seizures therefore applies as a restraint directed only against the government and its agencies tasked with the enforcement of the law. Thus, it could only be invoked against the State to whom the restraint against arbitrary and unreasonable exercise of power is imposed. Corolarilly, alleged violations against unreasonable search and seizure may only be invoked against the State by an individual unjustly traduced by the exercise of sovereign authority. To agree with appellant that an act of a private individual in violation of the Bill of Rights should also be construed as an act of the State would result in serious legal complications and an absurd interpretation of the constitution. That the Bill of Rights embodied in the Constitution is not meant to be invoked against acts of private individuals finds support in the deliberations of the Constitutional Commission. True, the liberties guaranteed by the fundamental law of the land must always be subject to protection. CONSTITUTIONAL LAW 2 CASE DIGESTS

DOES THE PROHIBITION ON UNLAWFUL SEARCH INCLUDES BARANGAY TANODS Ruben Del Castillo vs. People of the Philippines G.R. No. 185125 January 30, 2012 Facts: Pursuant to a confidential information that petitioner Del Castillo was engaged in selling shabu, police officers headed by SPO3 Bienvenido Masnayon, after conducting surveillance and test-buy operation at the house of petitioner, secured a search warrant from the RTC. Upon arrival to the residence of Del Castillo to implement the search warrant, SPO3 Masnayon claimed that he saw petitioner run towards a small structure, a nipa hut, in front of his house. Masnayon chased him but to no avail, because he and his men were not familiar with the entrances and exits of the place. They all went back to the residence of Del Castillo and requested his men to get a barangay tanod and a few minutes thereafter, his men returned with two barangay tanods who searched the house of petitioner including the nipa hut where the petitioner allegedly ran for cover. His men who searched the residence of the petitioner found nothing, but one of the barangay tanods was able to confiscate from the nipa hut several articles, including four (4) plastic packs containing white crystalline substance. Thus, an information was filed against Del Castillo for violation of Section 16, Article III of R.A. 6425 and was found guilty by the RTC and affirmed by the Court of Appeals. Petitioner filed with the Supreme Court the petition for certiorari contending among others that CA erred in finding him guilty beyond reasonable doubt of illegal possession of prohibited drugs, because he could not be presumed to be in possession of the same just because they were found inside the nipa hut. Issue: Can petitioner Del Castillo be held liable for violation of Section 16, Article III of R.A. 6425 by mere presumption that the petitioner has dominion and control over the place where the shabu was found? CONSTITUTIONAL LAW 2 CASE DIGESTS

Held: No. While it is not necessary that the property to be searched or seized should be owned by the person against whom the search warrant is issued, there must be sufficient showing that the property is under petitioner’s control or possession. The records are void of any evidence to show that petitioner owns the nipa hut in question nor was it established that he used the said structure as a shop. The RTC, as well as the CA, merely presumed that petitioner used the said structure due to the presence of electrical materials, the petitioner being an electrician by profession. The prosecution must prove that the petitioner had knowledge of the existence and presence of the drugs in the place under his control and dominion and the character of the drugs. With the prosecution’s failure to prove that the nipa hut was under petitioner’s control and dominion, there casts a reasonable doubt as to his guilt. In considering a criminal case, it is critical to start with the law’s own starting perspective on the status of the accused — in all criminal prosecutions, he is presumed innocent of the charge laid unless the contrary is proven beyond reasonable doubt. Proof beyond reasonable doubt, or that quantum of proof sufficient to produce a moral certainty that would convince and satisfy the conscience of those who act in judgment, is indispensable to overcome the constitutional presumption of innocence. WHO MAY INVOKE THE RIGHT? Bache & Co Inc vs. Ruiz GR L-32409, 27 February 1971 Facts: On 24 February 1970, Misael P. Vera, Commissioner of Internal Revenue, wrote a letter addressed to Judge Vivencio M. Ruiz requesting the issuance of a search warrant against Bache & Co. (Phil.), Inc. and Frederick E. Seggerman for violation of Section 46(a) of the National Internal Revenue Code (NIRC), in relation to all other pertinent provisions thereof, particularly Sections 53, 72, 73, 208 and 209, and authorizing Revenue Examiner Rodolfo de Leon to make and file the application for search warrant which was attached to the letter. In the afternoon of the following day, De Leon and his witness, Arturo Logronio, went to the Court of First Instance (CFI) of Rizal. They brought with them the CONSTITUTIONAL LAW 2 CASE DIGESTS

following papers: Vera’s letter-request; an application for search warrant already filled up but still unsigned by De Leon; an affidavit of Logronio subscribed before De Leon; a deposition in printed form of Logronio already accomplished and signed by him but not yet subscribed; and a search warrant already accomplished but still unsigned by Judge. At that time the Judge was hearing a certain case; so, by means of a note, he instructed his Deputy Clerk of Court to take the depositions of De Leon and Logronio. After the session had adjourned, the Judge was informed that the depositions had already been taken. The stenographer, upon request of the Judge, read to him her stenographic notes; and thereafter, the Judge asked Logronio to take the oath and warned him that if his deposition was found to be false and without legal basis, he could be charged for perjury. The Judge signed de Leon’s application for search warrant and Logronio’s deposition. Search Warrant 2-M-70 was then signed by Judge and accordingly issued. 3 days later (a Saturday), the BIR agents served the search warrant to the corporation and Seggerman at the offices of the corporation on Ayala Avenue, Makati, Rizal. The corporation’s lawyers protested the search on the ground that no formal complaint or transcript of testimony was attached to the warrant. The agents nevertheless proceeded with their search which yielded 6 boxes of documents. On 3 March 1970, the corporation and Seggerman filed a petition with the Court of First Instance (CFI) of Rizal praying that the search warrant be quashed, dissolved or recalled, that preliminary prohibitory and mandatory writs of injunction be issued, that the search warrant be declared null and void, and that Vera, Logronio, de Leon, et. al., be ordered to pay the corporation and Seggerman, jointly and severally, damages and attorney’s fees. After hearing and on 29 July 1970, the court issued an order dismissing the petition for dissolution of the search warrant. In the meantime, or on 16 April 1970, the Bureau of Internal Revenue made tax assessments on the corporation in the total sum of P2,594,729.97, partly, if not entirely, based on the documents thus seized. The corporation and Seggerman filed an action for certiorari, prohibition, and mandamus. Issue: Whether the corporation has the right to contest the legality of the seizure of documents from its office. CONSTITUTIONAL LAW 2 CASE DIGESTS

Held: The legality of a seizure can be contested only by the party whose rights have been impaired thereby, and that the objection to an unlawful search and seizure is purely personal and cannot be availed of by third parties. In Stonehill, et al. vs. Diokno, et al. (GR L-19550, 19 June 1967; 20 SCRA 383) the Supreme Court impliedly recognized the right of a corporation to object against unreasonable searches and seizures; holding that the corporations have their respective personalities, separate and distinct from the personality of the corporate officers, regardless of the amount of shares of stock or the interest of each of them in said corporations, whatever, the offices they hold therein may be; and that the corporate officers therefore may not validly object to the use in evidence against them of the documents, papers and things seized from the offices and premises of the corporations, since the right to object to the admission of said papers in evidence belongs exclusively to the corporations, to whom the seized effects belong, and may not be invoked by the corporate officers in proceedings against them in their individual capacity. The distinction between the Stonehill case and the present case is that: in the former case, only the officers of the various corporations in whose offices documents, papers and effects were searched and seized were the petitioners; while in the latter, the corporation to whom the seized documents belong, and whose rights have thereby been impaired, is itself a petitioner. On that score, the corporation herein stands on a different footing from the corporations in Stonehill. Moreover, herein, the search warrant was void inasmuch as First, there was no personal examination conducted by the Judge of the complainant (De Leon) and his witness (Logronio). The Judge did not ask either of the two any question the answer to which could possibly be the basis for determining whether or not there was probable cause against Bache & Co. and Seggerman. The participation of the Judge in the proceedings which led to the issuance of Search Warrant 2-M-70 was thus limited to listening to the stenographer’s readings of her notes, to a few words of warning against the commission of perjury, and to administering the oath to the complainant and his witness. This cannot be consider a personal examination. Second, the search warrant was issued for more than one specific offense. The search warrant was issued for at least 4 distinct offenses under the Tax Code. The first is the violation of Section 46(a), Section 72 and Section 73 (the filing of CONSTITUTIONAL LAW 2 CASE DIGESTS

income tax returns), which are interrelated. The second is the violation of Section 53 (withholding of income taxes at source). The third is the violation of Section 208 (unlawful pursuit of business or occupation); and the fourth is the violation of Section 209 (failure to make a return of receipts, sales, business or gross value of output actually removed or to pay the tax due thereon). Even in their classification the 6 provisions are embraced in 2 different titles: Sections 46(a), 53, 72 and 73 are under Title II (Income Tax); while Sections 208 and 209 are under Title V (Privilege Tax on Business and Occupation). Lastly, the search warrant does not particularly describe the things to be seized. Search Warrant No. 2-M-70 tends to defeat the major objective of the Bill of Rights, i.e., the elimination of general warrants, for the language used therein is so all-embracing as to include all conceivable records of the corporation, which, if seized, could possibly render its business inoperative. Thus, Search Warrant 2-M70 is null and void. Stonehill vs Diokno 20 SCRA 383 Facts: Respondents herein secured a total of 42 search warrants against petitioners herein and/or the corporations of which they were officers, to search “books of accounts, financial records, vouchers, correspondence, receipts, ledgers, journals, portfolios, credit journals, typewriters, and other documents and/or papers showing all business transactions including disbursements receipts, balance sheets and profit and loss statements and Bobbins (cigarette wrappers),” as “the subject of the offense; stolen or embezzled and proceeds or fruits of the offense,” or “used or intended to be used as the means of committing the offense,” which is described in the applications adverted to above as “violation of Central Bank Laws, Tariff and Customs Laws, Internal Revenue (Code) and the Revised Penal Code.” The petitioner contended that the search warrants are null and void as their issuance violated the Constitution and the Rules of Court for being general warrants. The documents, papers, and things seized under the alleged authority of the warrants in question may be split into two (2) major groups, namely: (a) those CONSTITUTIONAL LAW 2 CASE DIGESTS

found and seized in the offices of the aforementioned corporations, and (b) those found and seized in the residences of petitioners herein. Issue: Whether petitioners can validly assail the search warrant against the corporation. Held: No. As regards the first group, we hold that petitioners herein have no cause of action to assail the legality of the contested warrants and of the seizures made in pursuance thereof, for the simple reason that said corporations have their respective personalities, separate and distinct from the personality of herein petitioners, regardless of the amount of shares of stock or of the interest of each of them in said corporations, and whatever the offices they hold therein may be. Indeed, it is well settled that the legality of a seizure can be contested only by the party whose rights have been impaired thereby, and that the objection to an unlawful search and seizure is purely personal and cannot be availed of by third parties. Consequently, petitioners herein may not validly object to the use in evidence against them of the documents, papers and things seized from the offices and premises of the corporations adverted to above, since the right to object to the admission of said papers in evidence belongs exclusively to the corporations, to whom the seized effects belong, and may not be invoked by the corporate officers in proceedings against them in their individual capacity. Zurcher v. The Stanford Daily, 436 U.S. 547, 98 S.Ct. 1970 (1978) FACTS: On April 9, 1971, officers of the Palo Alto Police Department and the Santa Clara County Sheriff’s Department (California) responded to a call from Stanford University Hospital, to remove a large group of demonstrators who had barricaded themselves inside the administrative offices. The officers tried to convince the demonstrators to persuade the demonstrators to leave, with no success. The officers then forced their way into the area, and a group of demonstrators ran from the opposite side. Armed with “sticks and clubs,” the assaulted the officers in that area, all nine of the officers suffered injury. The officers were only able to identify two of their assailants. There were no police photographers in the area, but there was another individual present there, taking photographs. The next day, CONSTITUTIONAL LAW 2 CASE DIGESTS

the Stanford Daily (the student newspaper) published a story and photographs, and it was apparent that the individual the officers knew had been at the site of the assault took the photos. The prosecutor sought and received a search warrant for the Daily’s offices, for any negatives or film that might show the incident. The search only revealed the pictures that had already been published, and nothing was taken from the offices. The Daily and various staff members filed a Sec. 1983 lawsuit. The District Court found that because the subject of the search was not a suspect in a crime, and that because the subject of the search was a representative of the press, that such a search was permitted “only in the rare circumstance where there is a clear showing that (1) important materials will be destroyed or removed from the jurisdiction; and (2) a restraining order would be futile.” ISSUE: May a search warrant be issued on an innocent third party believed to be in possession of relevant evidence? HOLDING: Yes DISCUSSION: The Court discussed how the Fourth Amendment should be applied to “third party” searches, when officers believe that evidence of a crime is in the possession of someone who is not implicated in the crime. The Court stated that “nothing on the face of the Amendment” prohibited third party searches, and that “valid warrants may be issued to search any property, whether or not occupied by a third party, at which there is probable cause to believe that fruits, instrumentalities, or evidence of a crime will be found.” Finally, the Court concluded that “[T]he critical element in a reasonable search is not that the owner of the property is suspected of crime but that there is reasonable cause to believe that the specific ‘things’ to be searched for and seized are located on the property to which entry is sought.” Wilson v. Layne (2002) CONSTITUTIONAL LAW 2 CASE DIGESTS

Facts: When police executed an arrest warrant for Plaintiff Wilson’s son at his home, they brought along a newspaper reporter and photographer who observed and photographed the event. The son was not in the house. P filed a Section:1983 suit against police for a Fourth Amendment violation. Issue: Does a media “ride along” violate the Fourth Amendment when police bring a reporter to the house where they plan to execute an arrest warrant? Rule: (Rehnquist, C. J.) The Fourth Amendment requires that police actions in execution of a warrant must be related to the objectives of the authorized intrusion, which is to take a person into custody; the police conduct of bringing media parties to a home violates the Fourth Amendment because the presence of these parties was not “in aid of the warrant’s execution.” Government interests such as publicizing the crime control efforts of police, or minimizing police abuses, or facilitating accurate reporting, do not justify the intrusion of media “ride alongs”on the Fourth Amendment privacy interest of the homeowner. CONDITIONS FOR A VALID WARRANT *EXISTENCE OF PROBABLE CAUSE Jose Burgos vs. Chief of Staff G.R. No L-64261 December 26, 1984

Facts: Two warrants were issued against petitioners for the search on the premises of “Metropolitan Mail” and “We Forum” newspapers and the seizure of items alleged to have been used in subversive activities. Petitioners prayed that a writ of preliminary mandatory and prohibitory injunction be issued for the return of the CONSTITUTIONAL LAW 2 CASE DIGESTS

seized articles, and that respondents be enjoined from using the articles thus seized as evidence against petitioner. Petitioners questioned the warrants for the lack of probable cause and that the two warrants issued indicated only one and the same address. In addition, the items seized subject to the warrant were real properties. Issue: Whether or not the two warrants were valid to justify seizure of the items. Held: The defect in the indication of the same address in the two warrants was held by the court as a typographical error and immaterial in view of the correct determination of the place sought to be searched set forth in the application. The purpose and intent to search two distinct premises was evident in the issuance of the two warrant. As to the issue that the items seized were real properties, the court applied the principle in the case of Davao Sawmill Co. v. Castillo, ruling “that machinery which is movable by nature becomes immobilized when placed by the owner of the tenement, property or plant, but not so when placed by a tenant, usufructuary, or any other person having only a temporary right, unless such person acted as the agent of the owner.” In the case at bar, petitioners did not claim to be the owners of the land and/or building on which the machineries were placed. This being the case, the machineries in question, while in fact bolted to the ground remain movable property susceptible to seizure under a search warrant. However, the Court declared the two warrants null and void. Probable cause for a search is defined as such facts and circumstances which would lead a reasonably discreet and prudent man to believe that an offense has been committed and that the objects sought in connection with the offense are in the place sought to be searched. The Court ruled that the affidavits submitted for the application of the warrant did not satisfy the requirement of probable cause, the statements of the witnesses having been mere generalizations. CONSTITUTIONAL LAW 2 CASE DIGESTS

Furthermore, jurisprudence tells of the prohibition on the issuance of general warrants. (Stanford vs. State of Texas). The description and enumeration in the warrant of the items to be searched and seized did not indicate with specification the subversive nature of the said items. Chandler vs Miller April 15, 1997, d-96-126 Facts of the case Under a Georgia statute, all candidates for elected state office must pass a urinalysis drug test within 30 days prior to their qualifying for nomination or election. Chandler, on behalf of several state office nominees from the Libertarian Party, challenged the statute's constitutionality, naming Georgia's governor and two other regulatory officials as defendants. On appeal from an adverse District Court ruling, the Eleventh Circuit affirmed and the Supreme Court granted certiorari.

Question Did Georgia's drug testing statute violate the Fourth Amendment's guarantee against illegal search and seizures? Conclusion Yes. In an 8-to-1 opinion, the Court noted that while the Fourth Amendment generally prohibits officials from conducting search and seizures without individualized suspicion, there does exist a narrowly defined category of permissible suspicionless searches and seizures. The Court held, however, that Georgia's statute did not fall in this exceptional category, since it failed to show why its desire to avoid drug users in its high political offices should outweigh candidates' privacy interests. In addition to Georgia's failure to provide evidence of a drug problem among its state officials, the Court concluded that even if such a problem did exist, the affected officials would most likely not perform the kind of CONSTITUTIONAL LAW 2 CASE DIGESTS

high-risk, safety sensitive tasks, which might justify the statute's proposed incursion on their individual privacy rights. People v. Chua Ho San 308 SCRA 432 (1999) G.R. No. 128222, June 17, 1999 Fact: In response to reports of rampant smuggling of firearms and other contraband, CID began patrolling the Bacnotan coastline with his officers. While monitoring the coastal area he intercepted a radio call from ALMOITE requesting police assistance regarding an unfamiliar speedboat. CID and six of his men. When the speedboat landed, the male passenger alighted, and using both hands, carried what appeared a multicolored strawbag. He then walked towards the road. By this time, ALMOITE, CID and BADUA, the latter two conspicuous in their uniform and issued side-arms, became suspicious of the man as he suddenly changed direction and broke into a run upon seeing the approaching officers. BADUA, however, prevented the man from fleeing by holding on to his right arm. Although CID introduced themselves as police officers, the man appeared impassive. Speaking in English, CID then requested the man to open his bag, but he seem not to understand. CID thus tried speaking Tagalog, then Ilocano, but still to no avail. CID then resorted to what he termed “sign language;” he motioned with his hands for the man to open the bag. This time, the man apparently understood and acceded to the request. A search of the bag yielded several transparent plastic packets containing yellowish crystalline substances. which was later found out that it was Shabu. CID then gestured to the man to close the bag, which he did. As CID wished to proceed to the police station, he signaled the man to follow, but the latter did not to comprehend. Hence, CID placed his arm around the shoulders of the man and escorted the latter to the police headquarters. CHUA was initially charged with illegal possession of methaphetamine hydrochloride before the RTC. The RTC convicted Chua Ho San guilty beyond reasonable doubt. Chua Ho San prays for his acquitttal and the reversal of the judgment of the RTC. Issue: Whether the accused who was acting suspiciously constitute Probable Cause impelling the police officers from effecting an in flagrante delicto arrest. Held: CONSTITUTIONAL LAW 2 CASE DIGESTS

No, the Court, finds that these do not constitute “probable cause.” None of the telltale clues, e.g., bag or package emanating the pungent odor of marijuana or other prohibited drug, confidential report and/or positive identification by informers of courier(s) of prohibited drug and/or the time and place where they will transport/deliver the same, suspicious demeanor or behavior and suspicious bulge in the waist — accepted by this Court as sufficient to justify a warrantless arrest exists in this case. The term probable cause had been understood to mean a reasonable ground of suspicion supported by circumstances sufficiently strong in themselves to warrant a cautious man’s belief that the person accused is guilty of the offense with which he is charged. Specifically with respect to arrests, it is such facts and circumstances which would lead a reasonably discreet and prudent man to believe that an offense has been committed by the person sought to be arrested. In cases of in fragrante delicto, arrests, a peace officer or a private person may without a warrant, arrest a person, when, in his presence, the person to be arrested has committed, is actually committing, or is attempting to commit an offense. The arresting officer, therefore, must have personal knowledge of such facts or as recent case law adverts to, personal knowledge of facts or circumstances convincingly indicative or constitutive of probable cause. PEOPLE OF THE PHILIPPINES vs. DELIA C. MOLINA ET. AL G.R. No. 229712, February 28, 2018 Facts: Appellant was charged for the crime of Illegal Recruitment in Large Scale under Section[s] 6 and 7 of Republic Act No. 8042 in an Information which alleges that the accused, mutually helping and aiding one another, feloniously recruit for a fee, promise employment/job placement abroad to five (5) persons, hence, committed in large scale, and received payments from complainants in connection with the documentation and processing of their papers for purposes of their deployment, but said accused failed or refused to deploy herein complainants abroad without the fault of the latter and to reimburse the amounts to said complainants, to the damage and prejudice of the latter. Appellant claimed that she has not met personally all the private complainants in this case. On cross-examination, accused Molina admitted that there were about 100 cases of illegal recruitment filed against her in different courts and that she CONSTITUTIONAL LAW 2 CASE DIGESTS

was convicted of illegal recruitment in the RTC of Makati City, Branch 148 and Branch 150. Issue: Whether or not appellant is guilty of the crime of illegal recruitment in large scale. Ruling: Yes, the appellant is guilty as charged. Illegal recruitment when committed by a syndicate or in large scale shall be considered an offense involving economic sabotage. Illegal recruitment is deemed committed by a syndicate if carried out by a group of three (3) or more persons conspiring or confederating with one another. It is deemed committed in large scale if committed against three (3) or more persons individually or as a group. In this case, appellant cannot escape from liability for large scale illegal recruitment as the recruitment was made in the recruitment agency of which accused-appellant is the President. Moreover, private complainants testified that they saw accused-appellant at the agency and she was introduced to them by Pacon as the owner of the agency, and she even assured them that they would be deployed for employment soon. Appellant, as President of the recruitment agency, is therefore liable for failure to reimburse the expenses incurred by private complainants in connection with their documentation and processing for purposes of deployment to South Korea, which did not actually take place without their fault. Ratio Decidendi: In case of juridical persons, the officers having control, management or direction of their business shall be liable. Gist: This is an appeal from the Decision, affirming the Decision of the RTC, finding accused-appellant Delia C. Molina guilty beyond reasonable doubt of the crime of illegal recruitment in large scale. Columbia Pictures v. Flores, G.R. No. 78631, June 29, 1993 FACTS: As a consequence of a complaint filed by the Motion Picture Association of America, Inc., NBI agents conducted surveillance operations on certain video establishments, among them respondent FGT Video Network, Inc. (FGT), for CONSTITUTIONAL LAW 2 CASE DIGESTS

“unauthorized sale, rental, reproduction and/or disposition of copyrighted film," a violation of PD 49 (the old Intellectual Property Law). After an NBI agent was able to have copyrighted motion pictures “Cleopatra” (owned by 20th Century Fox) and “The Ten Commandments” (owned by Paramount) reproduced in video format in FGT, the NBI applied for and was able to obtain from the respondent judge the subject Search Warrant No. 45 which reads: TO ANY PEACE OFFICER: GREETINGS: It appearing to the satisfaction of the Undersigned after examining under oath NBI Senior Agent Lauro C. Reyes and his witnesses Mr. Danilo Manalang and Ms. Rebecca Benitez-Cruz, that there is a probable cause to believe that Violation of Section 56 P.D. No. 49 as amended by P.D. No. 1988 (otherwise known as the Decree on Protection of Intellectual Property) has been committed and that there are good and sufficient reasons to believe that FGT Video Network, Inc., Manuel Mendoza, Alfredo C. Ongyanco, Eric Apolonio, Susan Yang and Eduardo Yotoko are responsible and have in control/possession at No. 4 Epifanio de los Santos corner Connecticut, Greenhills, San Juan, Metro Manila (per attached sketch and list of MPAA member Company Titles) the following properties to wit: (a) Pirated video tapes of the copyrighted motion pictures/films the titles of which are mentioned in the attached list; (b) Posters, advertising leaflets, flyers, brochures, invoices, lists of titles being reproduced or retaped, journals, ledgers, jon (sic) order slips, delivery slips and books of accounts bearing and/or mentioning the pirated films with titles (as per attached list), or otherwise used in the reproduction/retaping business of the defendants; (c) Television sets, video cassette recorders, rewinders, tape head cleaners, accessories, equipment and other machines and paraphernalia or materials used or intended to be used in the unlawful sale, lease, distribution, or possession for purpose of sale, lease, distribution, circulation or public exhibition of the abovementioned pirated video tapes which they are keeping and concealing in the premises above-described, which should be seized and brought to the Undersigned. CONSTITUTIONAL LAW 2 CASE DIGESTS

You are hereby commanded to make an immediate search at any time in the day between 8:00 A.M. to 5:00 P.M. of the premises above-described and forthwith seize and take possession of the above-enumerated personal properties, and bring said properties to the undersigned immediately upon implementation to be dealt with as the law directs. In the course of the implementation of the search warrant in the premises of FGT, the NBI agents found and seized various video tapes of copyrighted films owned and exclusively distributed by petitioners. Also seized were machines and equipment, television sets, paraphernalia, materials, accessories, rewinders, tape head cleaners, statements of order, return slips, video prints, flyers, production orders, and posters. FGT moved for the release of the seized television sets, video cassette recorders, rewinders, tape head cleaners, accessories, equipment and other machines or paraphernalia seized by virtue of the subject warrant. It argued that as a licensed video reproducer, it had the right possess the seized reproduction equipment, which are not illegal per se, but are rather exclusively used and intended to be used for reproduction and not in the “sale, lease, distribution or possession for purposes of sale, lease distribution, circulation or public exhibition of pirated video tapes.” Finding that FGT was a registered and duly licensed distributor and in certain instances and under special instructions and conditions reproducer of videograms and that, therefore, its right to possess and use the seized equipment had been placed in serious doubt, the lower court ordered the return of the “television sets, video cassette recorders, rewinders, tape head cleaners, accessories, equipment and other machines or paraphernalia” to FGT. ISSUE Did the respondent judge act with grave abuse of discretion amounting to lack of jurisdiction in ordering the immediate return of some of the items seized by virtue of the search warrant? RULING CONSTITUTIONAL LAW 2 CASE DIGESTS

[The High Tribunal DISMISSED the petition and AFFIRMED the order of the respondent Judge Flores.] NO, the respondent judge DID NOT act with grave abuse of discretion amounting to lack of jurisdiction in ordering the immediate return of some of the items seized by virtue of the search warrant. Search Warrant No. 45 fails to satisfy the test of legality. This is more so because the Court has previously decided a case dealing with virtually the same kind of search warrant. In 20th Century Fox vs. CA, the Court upheld the legality of the order of the lower court lifting the search warrant issued under circumstances similar to those obtaining in the case at bar. A striking similarity between this case and 20th Century Fox is the fact that Search Warrant No. 45, specifically paragraph (c) thereof describing the articles to be seized, contains an almost identical description as the warrant issued in the 20th Century Fox case, to wit: (c) Television sets, Video Cassettes Recorders, rewinders, tape head cleaners, accessories, equipments and other machines used or intended to be used in the unlawful reproduction, sale, rental/lease, distribution of the above-mentioned video tapes which she is keeping and concealing in the premises above-described. On the propriety of the seizure of the articles above-described, the Court held in 20th Century Fox: Television sets, video cassette recorders, rewinders and tape cleaners are articles which can be found in a video tape store engaged in the legitimate business of lending or renting out betamax tapes. In short, these articles and appliances are generally connected with, or related to a legitimate business not necessarily involving piracy of intellectual property or infringement of copyright laws. Hence, including these articles without specification and/or particularity that they were really instruments in violating an Anti-Piracy law makes the search warrant too general which could result in the confiscation of all items found in any video store. The language used in paragraph (c) of Search Warrant No. 45 is thus too allembracing as to include all the paraphernalia of FGT in the operation of its business. As the search warrant is in the nature of a general one, it is constitutionally objectionable. CONSTITUTIONAL LAW 2 CASE DIGESTS

The Court concluded that the respondent judge did not gravely abuse his discretion in ordering the immediate release of the enumerated items, but that he was merely correcting his own erroneous conclusions in issuing Search Warrant No. 45. This can be gleaned from his statement that “. . . the machines and equipment could have been used or intended to be used in the illegal reproduction of tapes of the copyrighted motion pictures/films, yet, it cannot be said with moral certainty that the machines or equipment(s) were used in violating the law by the mere fact that pirated video tapes of the copyrighted motion pictures/films were reproduced. As already stated, FGT Video Network, Inc. is a registered and duly licensed distributor and in certain instances and under special instructions . . . reproducer of videograms, and as such, it has the right to keep in its possession, maintain and operate reproduction equipment(s) and paraphernalia(s).”

HO vs PEOPLE (1997) FACTS On August 8, 1991, the Anti-Graft League of the Philippines, represented by its chief prosecutor Atty. Reynaldo L. Bagatsing, filed with the Office of the Ombudsman a complaint against Doris Teresa Ho, Rolando S. Narciso (petitioners in G.R. Nos. 106632 and 106678, respectively), Anthony Marden, Arsenio Benjamin Santos and Leonardo Odoño. The complaint was for alleged violation of Section 3 (g) of Republic Act 3019 prohibiting a public officer from entering into any contract or transaction on behalf of the government if it is manifestly and grossly disadvantageous to the latter, whether or not the public officer profited or will profit thereby. According to the information, Rolando Narciso, being then the Vice-President of the National Steel Corporation (NSC), a government-owned or controlled corporation organized and operating under the Philippine laws, and Doris Ho, the President of National Marine Corporation (NMC), a private corporation organized and operating under our Corporation law, was said to have entered without legal justification into a negotiated contract of affreightment disadvantageous to the NSC for the haulage of its products at the rate of P129.50/MT, from Iligan City to Manila. Such contract was entered into despite their full knowledge that the rate they have agreed upon was much higher than those offered by the Loadstar CONSTITUTIONAL LAW 2 CASE DIGESTS

Shipping Company, Inc. (LSCI) and Premier Shipping Lines, Inc. (PSLI), in the amounts of P109.56 and P123.00 per Metric Ton, respectively, in the public bidding, thereby giving unwarranted benefits to the National Marine Corporation. Ho and Narciso alleged that the Sandiganbayan, in determining probable cause for the issuance of the warrant for their arrest, merely relied on the information and the resolution attached thereto, filed by the Ombudsman without other supporting evidence, in violation of the requirements of Section 2, Article III of the Constitution, and settled jurisprudence. They contend that a judge, in personally determining the existence of probable cause, must have before him sufficient evidence submitted by the parties, other than the information filed by the investigating prosecutor, to support his conclusion and justify the issuance of an arrest warrant. Such evidence should not be “merely described in a prosecutor’s resolution.” Sandiganbayan’s Denial: “Considering, therefore, that this Court did not rely solely on the certification appearing in the information in this case in the determination of whether probable cause exists to justify the issuance of the warrant of arrest but also on the basis predominantly shown by the facts and evidence appearing in the resolution/memorandum of responsible investigators/ prosecutors, then the recall of the warrant of arrest, or the reconsideration sought for, cannot be granted. More so, when the information, as filed, clearly shows that it is sufficient in form and substance based on the facts and evidence adduced by both parties during the preliminary investigation. To require this Court to have the entire record of the preliminary investigation to be produced before it, including the evidence submitted by the complainant and the accusedrespondents, would appear to be an exercise in futility.” ISSUE May a judge determine probable cause and issue a warrant of arrest solely on the basis of the resolution of the prosecutor (in the instant case, the Office of the Special Prosecutor of the Ombudsman) who conducted the preliminary investigation, without having before him any of the evidence (such as complainant’s affidavit, respondent’s counter-affidavit, exhibits, etc.) which may have been submitted at the preliminary investigation? DECISION & RATIO NO. • Art III Section 2, 1987 Constitution: The right of the people to be secure in their persons, houses, papers, and effects against unreasonable searches and CONSTITUTIONAL LAW 2 CASE DIGESTS

seizures of whatever nature and for any purpose shall be inviolable, and no search warrant or warrant of arrest shall issue except upon probable cause to be determined personally by the judge after examination under oath or affirmation of the complainant and the witnesses he may produce and particularly describing the place to be searched and the persons or things to be seized. (Art III Section 2, 1987 Constitution) The word “personally” does not appear in the corresponding provisions of our previous Constitutions. This emphasis shows the present Constitution’s intent to place a greater degree of responsibility upon trial judges than that imposed under the previous Charters. • Soliven vs. Makasiar: “In satisfying himself of the existence of probable cause for the issuance of a warrant of arrest, the judge is not required to personally examine the complainant and his witnesses. Following established doctrine and procedure, he shall: (1) personally evaluate the report and the supporting documents submitted by the fiscal regarding the existence of probable cause and, on the basis thereof, issue a warrant of arrest; or (2) if on the basis thereof he finds no probable cause, he may disregard the fiscal’s report and require the submission of supporting affidavits of witnesses to aid him in arriving at a conclusion as to the existence of probable cause.” • People vs. Inting: There is a difference between the judge’s goal from that of the prosecutor’s. First, the determination of probable cause is a function of the Judge. It is not for the Provincial Fiscal or Prosecutor or for the Election Supervisor to ascertain. Second, the preliminary inquiry made by a Prosecutor does not bind the Judge. It merely assists him to make the determination of probable cause. Third, Judges and Prosecutors alike should distinguish the preliminary inquiry which determines probable cause for the issuance of a warrant of arrest from the preliminary investigation proper which ascertains whether the offender should be held for trial or released. Even if the two inquiries are conducted in the course of one and the same proceeding, there should be no confusion about the objectives. • The Court, in this case, reiterated and elaborated on the doctrine laid down in People vs. Inting and ruled that: “First, as held in Inting, the determination of probable cause by the prosecutor is for a purpose different from that which is to be made by the judge. Whether CONSTITUTIONAL LAW 2 CASE DIGESTS

there is reasonable ground to believe that the accused is guilty of the offense charged and should be held for trial is what the prosecutor passes upon. The judge, on the other hand, determines whether a warrant of arrest should be issued against the accused, i.e. whether there is a necessity for placing him under immediate custody in order not to frustrate the ends of justice. Thus, even if both should base their findings on one and the same proceeding or evidence, there should be no confusion as to their distinct objectives. Second, since their objectives are different, the judge cannot rely solely on the report of the prosecutor in finding probable cause to justify the issuance of a warrant of arrest. Obviously and understandably, the contents of the prosecutor’s report will support his own conclusion that there is reason to charge the accused of an offense and hold him for trial. However, the judge must decide independently. Hence, he must have supporting evidence, other than the prosecutor’s bare report, upon which to legally sustain his own findings on the existence (or nonexistence) of probable cause to issue an arrest order. This responsibility of determining personally and independently the existence or nonexistence of probable cause is lodged in him by no less than the most basic law of the land. Parenthetically, the prosecutor could ease the burden of the judge and speed up the litigation process by forwarding to the latter not only the information and his bare resolution finding probable cause, but also so much of the records and the evidence on hand as to enable His Honor to make his personal and separate judicial finding on whether to issue a warrant of arrest Lastly, it is not required that the complete or entire records of the case during the preliminary investigation be submitted to and examined by the judge. We do not intend to unduly burden trial courts by obliging them to examine the complete records of every case all the time simply for the purpose of ordering the arrest of an accused. What is required, rather, is that the judge must have sufficient supporting documents (such as the complaint, affidavits, counteraffidavits, sworn statements of witnesses or transcripts of stenographic notes, if any) upon which to make his independent judgment or, at the very least, upon which to verify the findings of the prosecutor as to the existence of probable cause. The point is: he cannot rely solely and entirely on the prosecutor’s recommendation, as Respondent Court did in this case. Although the prosecutor enjoys the legal presumption of regularity in the performance of his official duties and functions, which in turn gives his report the presumption of accuracy, the Constitution, we repeat, commands the judge to personally determine probable cause in the issuance of warrants of arrest. This Court has consistently held that a CONSTITUTIONAL LAW 2 CASE DIGESTS

judge fails in his bounden duty if he relies merely on the certification or the report of the investigating officer.” IN THE INSTANT CASE, the public respondent relied fully and completely upon the resolution of the graft investigation officer and the memorandum of the reviewing prosecutor, attached to the information filed before it, and its conjecture that the Ombudsman would not have approved their recommendation without supporting evidence. It had no other documents from either the complainant (the Anti-Graft League of the Philippines) or the People from which to sustain its own conclusion that probable cause exists. Respondent Court palpably committed grave abuse of discretion in ipso facto issuing the challenged warrant of arrest on the sole basis of the prosecutor’s findings and recommendation, and without determining on its own the issue of probable cause based on evidence other than such bare findings and recommendation.

PARTIALLY VALID WARRANT PEOPLE v. SALANGUIT FACTS: Two criminal cases were filed against Salanguit, the first for possession/use of shabu, and the second, for possession/use of marijuana. Sr. Insp. Aguilar applied in the RTC of Cavite a warrant to search the premises of Robert Salanguit for shabu and shabu paraphernalias. He presented as a witness Edmund Badua, an undercover officer, which transacted with Salanguit for the purchase of shabu. The application was granted and the team of Aguilar proceeded to the premises of Salanguit in QC to serve the warrant. The operatives proceeded to knock on Salanguit’s door but the same was left unanswered. The operatives heard people panicking inside the house and they began to force their way inside the house. They indicated their authority to conduct the search and began which yielded to the finding of clear plastic bags with shabu and 2 bricks of dried marijuana leaves covered in newspaper. Salanguit refused to sign the receipt for the confiscated drugs. During his arraignment, he pleaded not guilty and in the trial court, he gave stated that he never got the chance to review the purported warrant that Aguilar and his team CONSTITUTIONAL LAW 2 CASE DIGESTS

has. He further stated that the operatives ate their food and took his cash and valuable, as well as canned goods. The RTC found him guilty for possession/use of shabu and marijuana. Salanguit appealed the said decision and argues that the shabu allegedly recovered from his residence is inadmissible as evidence against him on the ground that the warrant used to obtain it was invalid and that the marijuana seized from him was also inadmissible as evidence against him pursuant to the plain view doctrine, and that the operatives employed unnecessary force in executing the warrant. ISSUES: 1. W/N the warrant used to seize the shabu was valid and the said shabu was inadmissible in evidence against him. 2. W/N the marijuana seized was admissible in evidence against Salanguit pursuant to plain view doctrine. HELD: 1. Yes, all the requisites for the issuance of a search warrant were satisfied. 2. No, the marijuana was not one of the drugs indicated in the warrant and it was not in plain view when it was seized. RATIO: 1. The warrant authorized the seizure of undetermined quantity of shabu and drug paraphernalia. Salanguit contends that it should be void as it did not indicate the existence of drug paraphernalias. The warrant was valid as to the seizure of shabu and void as to the seizure of drug paraphernalia. It is to be noted that no drug paraphernalia was seized. Salanguit further contends that the warrant was issued for more than one specific offense because possession or uses are punished under two different provisions in the Dangerous Drugs Act. This Court has decided in the case of People v Dichoso that a warrant that does not specify what provisions of the law were violated, is valid as to the authority to search and seize marijuana, shabu and drug paraphernalias. Lastly, Salanguit argues that the search warrant failed to indicate the place to be searched with sufficient particularity. The rule is that a description of the place to be searched is sufficient if the officer with the warrant can, with reasonable effort, ascertain and identify the place to be searched. The location of Salanguit’s house being indicated by the evidence on record, there can be no doubt that the warrant described the place to be searched with sufficient particularity. CONSTITUTIONAL LAW 2 CASE DIGESTS

2. Because the location of the shabu was indicated in the warrant and thus known to the police operatives, it is reasonable to assume that the police found the packets and shabu first. Once the valid portion of the search warrant has been executed, the plain view doctrine can no longer provide basis for admitting the other items subsequently found. The marijuana bricks were wrapped in newsprint. There was no apparent illegality to justify their seizure. Not being in a transparent container, the contents wrapped in newsprint could not have been readily discernible as marijuana. That being said, we hold that the marijuana is inadmissible in evidence against Salanguit. MICROSOFT CORPORATION petitioners, vs. MAXICORP, INC., respondent.

and

LOTUS

DEVELOPMENT

CORPORATION,

FACTS: This case involves the issuance of search warrant to the respondent MAXICORP Inc for alleged violation of Section 29 of Intellectual Property and Article 189 of the RPC (unfair competition). Armed with the search warrants, NBI agents conducted a search of Maxicorp’s premises and seized property fitting the description stated in the search warrants. Maxicorp filed a motion to quash the search warrants alleging that there was no probable cause for their issuance and that the warrants are in the form of "general warrants." Where the RTC denied the motion and at the same time denied their motion for reconsideration. According to RTC they’ve found a probable cause to issue such warrant after examining the NBI agent and the computer technician who visited Maxicorp. Maxicorp filed a petition for certiorari with the Court of Appeals seeking to set aside the RTC’s order. The Court of Appeals reversed the RTC’s order denying Maxicorp’s motion to quash the search warrants. Petitioners moved for reconsideration. The Court of Appeals denied petitioners’ motion on 29 November 1999. The Court of Appeals held that NBI Agent Samiano failed to present during the preliminary examination conclusive evidence that Maxicorp produced or sold the counterfeit products. The Court of Appeals pointed out that the sales receipt NBI Agent Samiano presented as evidence that he bought the products from Maxicorp was in the name of a certain "Joel Diaz." CONSTITUTIONAL LAW 2 CASE DIGESTS

Hence, this petition. ISSUE/S: 1. Whether or not there’s a probable cause on the part of CA to quash the search warrants issued by RTC 2. Whether or not respondent violated the intellectual property right of the petitioner. RULING: According to the SC the offense charged against Maxicorp is copyright infringement under Section 29 of PD 49 and unfair competition under Article 189 of the RPC. To support these charges, petitioners presented the testimonies of NBI Agent Samiano, computer technician Pante, and Sacriz, a civilian. The offenses that petitioners charged Maxicorp contemplate several overt acts. The sale of counterfeit products is but one of these acts. Both NBI Agent Samiano and Sacriz related to the RTC how they personally saw Maxicorp commit acts of infringement and unfair competition. Probable cause means "such reasons, supported by facts and circumstances as will warrant a cautious man in the belief that his action and the means taken in prosecuting it are legally just and proper."Thus, probable cause for a search warrant requires such facts and circumstances that would lead a reasonably prudent man to believe that an offense has been committed and the objects sought in connection with that offense are in the place to be searched. The testimonies of these two witnesses, coupled with the object and documentary evidence they presented, are sufficient to establish the existence of probable cause. From what they have witnessed, there is reason to believe that Maxicorp engaged in copyright infringement and unfair competition to the prejudice of petitioners. Both NBI Agent Samiano and Sacriz were clear and insistent that the counterfeit software were not only displayed and sold within Maxicorp’s premises, they were also produced, packaged and in some cases, installed there. The determination of probable cause does not call for the application of rules and standards of proof that a judgment of conviction requires after trial on the merits. As implied by the words themselves, "probable cause" is concerned with probability, not absolute or even moral certainty. The prosecution need not present at this stage proof beyond reasonable doubt. The standards of judgment CONSTITUTIONAL LAW 2 CASE DIGESTS

are those of a reasonably prudent man,24 not the exacting calibrations of a judge after a full-blown trial. No law or rule states that probable cause requires a specific kind of evidence. No formula or fixed rule for its determination exists.25 Probable cause is determined in the light of conditions obtaining in a given situation.26 Thus, it was improper for the Court of Appeals to reverse the RTC’s findings simply because the sales receipt evidencing NBI Agent Samiano’s purchase of counterfeit goods is not in his name. For purposes of determining probable cause, the sales receipt is not the only proof that the sale of petitioners’ software occurred. During the search warrant application proceedings, NBI Agent Samiano presented to the judge the computer unit that he purchased from Maxicorp, in which computer unit Maxicorp had preinstalled petitioners’ software. The Supreme Court held: “xxx No provision of the law exists which requires that a warrant, partially defective in specifying some items sought to be seized yet particular with respect to the other items, should be nullified as a whole. A partially defective warrant remains valid as to the items specifically described in the warrant. A search warrant is severable, the items not sufficiently described may be cut off without destroying the whole warrant.” In addition the Highest Court stated: “xxx The exclusionary rule found in Section 3(2) of Article III of the constitution renders inadmissible in any proceeding all evidence obtained through unreasonable searches and seizures. Thus, all items seized under paragraph © after search warrants, not falling under paragraphs a, b, c, d, e, f, should be returned to Maxico

Can a police just seize porno materials without warrant? PITA VS COURT OF APPEALS Facts: CONSTITUTIONAL LAW 2 CASE DIGESTS

On December 1 and 3, 1983, pursuing an Anti-Smut Campaign initiated by the Mayor of the City of Manila, Ramon D. Bagatsing, elements of the Special AntiNarcotics Group, Auxilliary Services Bureau, Western Police District, INP of the Metropolitan Police Force of Manila, seized and confiscated from dealers, distributors, newsstand owners and peddlers along Manila sidewalks, magazines, publications and other reading materials believed to be obscene, pornographic and indecent and later burned the seized materials in public at the University belt along C.M. Recto Avenue, Manila, in the presence of Mayor Bagatsing and several officers and members of various student organizations. Among the publications seized, and later burned, was "Pinoy Playboy" magazines published and co-edited by plaintiff Leo Pita. Plaintiff filed a case for injunction with prayer for issuance of the writ of preliminary injunction against Mayor Bagatsing and Narcisco Cabrera, as superintendent of Western Police District of the City of Manila, seeking to enjoin said defendants and their agents from confiscating plaintiff’s magazines or from preventing the sale or circulation thereof claiming that the magazine is a decent, artistic and educational magazine which is not per se obscene, and that the publication is protected by the Constitutional guarantees of freedom of speech and of the press. Plaintiff also filed an Urgent Motion for issuance of a temporary restraining order against indiscriminate seizure, confiscation and burning of plaintiff's "Pinoy Playboy" Magazines, pending hearing on the petition for preliminary injunction. The Court granted the temporary restraining order. The case was set for trial upon the lapse of the TRO. RTC ruled that the seizure was valid. This was affirmed by the CA. Issue: Whether or Not the seizure violative of the freedom of expression of the petitioner. Held: Freedom of the press is not without restraint as the state has the right to protect society from pornographic literature that is offensive to public morals, as indeed we have laws punishing the author, publishers and sellers of obscene publications. However, It is easier said than done to say, that if the pictures here in question were used not exactly for art's sake but rather for commercial purposes, the pictures are not entitled to any constitutional protection. Using the Kottinger rule: the test of obscenity is "whether the tendency of the matter charged as obscene, CONSTITUTIONAL LAW 2 CASE DIGESTS

is to deprave or corrupt those whose minds are open to such immoral influences and into whose hands a publication or other article charged as being obscene may fall." Another is whether it shocks the ordinary and common sense of men as an indecency. Ultimately "whether a picture is obscene or indecent must depend upon the circumstances of the case and that the question is to be decided by the "judgment of the aggregate sense of the community reached by it." The government authorities in the instant case have not shown the required proof to justify a ban and to warrant confiscation of the literature First of all, they were not possessed of a lawful court order: (1) finding the said materials to be pornography, and (2) authorizing them to carry out a search and seizure, by way of a search warrant. The court provides that the authorities must apply for the issuance of a search warrant from a judge, if in their opinion an obscenity seizure is in order and that; 1. The authorities must convince the court that the materials sought to be seized are obscene and pose a clear and present danger of an evil substantive enough to warrant State interference and action; 2. The judge must determine whether or not the same are indeed obscene. The question is to be resolved on a case-to-case basis and on the judge’s sound discretion;

OTHER CASES STA. ROSA MINING COMPANY vs. ASSISTANT PROV FISCAL AUGUSTO ZABALA Facts: Mandamus to compel respondent Fiscal to prosecute Criminal Case. In March 1974, petitioner filed a complaint for attempted theft against Romeo Garrido and Gil Alapan with the Office of the Provincial Fiscal of Camarines Norte. The case was assigned to Assistant Fiscal Esteban P. Panotes for preliminary investigation who filed the information for Attempted Theft on a prima facie case which was approved by Prov Fiscal Joaquin Ilustre. Fiscal Ilustre filed with the Court of First Instance of Camarines Norte the Information. On March 6, 1975, the Secretary of Justice reversed the findings of prima facie case and directed said prosecuting officer to dismiss the case. CONSTITUTIONAL LAW 2 CASE DIGESTS

On April 19, 1976, respondent Fiscal filed a Motion to Dismiss the case which was denied. The fiscal manifested that he would not prosecute the case. Issue: Whether or not the fiscal be compelled to prosecute the case, after motion to dismiss has been denied by the trial court? Held: Notwithstanding his personal convictions, the fiscal must proceed with his duty of presenting evidence to the court to enable the court to arrive at its own independent judgment. Accordingly, if the fiscal is not at all convinced that a prima facie case exists, he simply cannot move for the dismissal of the case and, when denied, refuse to prosecute the same. He is obliged by law to proceed and prosecute the criminal action. He cannot impose his opinion on the trial court. At least what he can do is to continue appearing for the prosecution and then turn over the presentation of evidence to another fiscal. The rule therefore in this jurisdiction is that once a complaint or information is filed in Court any disposition of the case as its dismissal or the conviction or acquittal of the accused rests in the sound discretion of the Court.

Paderanga v Drilon (1991) 1. On 16 October 1986, an information for multiple murder was filed in the Regional Trial Court, Gingoog City, against Felipe Galarion, Manuel Sabit,Cesar Sabit, Julito Ampo, Eddie Torion, John Doe, Peter Doe and Richard Doe, for the deaths on 1 May 1984 of Renato Bucag, his wife MelchoraBucag, and theirson Renato Bucag II. Venue was, however, transferred to Cagayan de Oro City per Administrative Matter 87-2-244. 2. Only Felipe Galarion was tried and found guilty as charged. The rest of the accused remained at large. Felipe Galarion, however, escaped from detention and has not been apprehended since then. In an amended CONSTITUTIONAL LAW 2 CASE DIGESTS

information filed on 6 October 1988, Felizardo Roxas, alias "Ely Roxas,""Fely Roxas" and "Lolong Roxas," was included as a co-accused. Roxasretained Atty. Miguel P. Paderanga as his counsel. 3. As counsel for Roxas, Paderanga filed an Omnibus Motion to dismiss, to Quash the Warrant of Arrest and to Nullify the Arraignment on 14 October1988. The trial court denied the omnibus motion but directed the City Prosecutor "to conduct another preliminary investigation or reinvestigationin order to grant the accused all the opportunity to adduce whateverevidence he has in support of his defense." 4. In the course of the preliminary investigation, through a signed affidavit,Felizardo Roxas implicated Atty. Paderanga in the commission of the crime charged. The City Prosecutor of Cagayan de Oro City inhibited himself from further conducting the preliminary investigation against Paderanga atthe instance of the latter's counsel, per his resolution dated 7 July 1989. 5. In his first indorsement to the Department of Justice, dated 24July 1989,said city prosecutor requested the Department of Justice to designate a state prosecutor to continue the preliminary investigation against Paderanga. In a resolution dated 6 September 1989, the State Prosecutor Henrick F.Gingoyon, who was designated to continue with the conduct of the preliminary investigation against Paderanga, directed the amendment of the previously amended information to include and implead Paderanga as one of the accused therein. Paderanga moved for reconsideration, contending that the preliminary investigation was not yet completed when said resolution was promulgated, and that he was deprived of his right to presenta corresponding counter-affidavit and additional evidence crucial to the determination of his alleged "linkage" to the crime charged. 6. The motion was, however, denied by Gingoyon in his order dated 29January 1990. From the aforesaid resolution and order, Paderanga filed a Petition for Review with the Department of Justice. Thereafter, he submitted a Supplemental Petition with Memorandum, and then aSupplemental Memorandum with Additional Exculpatory/ExoneratingEvidence Annexed, attaching thereto an affidavit of Roxas dated 20 June1990 and purporting to be a retraction of his affidavit of 30 March 1990wherein he implicated Paderanga. On CONSTITUTIONAL LAW 2 CASE DIGESTS

10 August 1990, the Department ofJustice, through Undersecretary Silvestre H. Bello III, issued Resolution648 dismissing the said petition for review. His motion for reconsideration having been likewise denied, Paderanga then filed the petition for mandamus and prohibition before the Supreme Court ISSUE: What is the quantum of evidence needed for probable in preliminary investigation? (I think #2 under “held” is the one relevant in this case) HELD: 1. Petitioner avers that he was deprived of full preliminary investigation because when the resolution was issued there were still incidents pending such as the validity of testimonies and affidavits of Roxas, Hanpol as bases for preliminary investigation, the polygraph test of Roxas which he failed, the clarifactory question that were supposed to be propounded by petitioner’s counsel to Roxas and Hanapol. He also claims he was deprived of the opportunity to file his counter-affidavit to the subpoena of April 25 - BUT THESE CONTENTIONS ARE WITHOUT MERI Ta. He already filed his counter-affidavit pursuant to the subpoena issued tohim where he controverted the charge against him and dismissed it saying it was malicious design of his political opponents. He also failed To show the subpoena issued involved a separate complaint charging an offense different from that charged in the complaint attached in the 1st subpoena b. The credibility of witness and their testimonies are matters of defense best addressed trial court for evaluation c. Right to ask clarifactory question is not absolute. Fiscal has discretion if hewill propound these questions to the parties or witnesses concerned. d. Proper forum before which absence of preliminary investigation should be ventilated is the Court of First Instance, not this Court.. Absence of a preliminary investigation does not go to the jurisdiction of the court but merely to the regularity of the proceedings. It could even be waived. Indeed, it is frequently waived. These are matters to be inquired into by the trial court 2. Petitioner also alleged that there is no prima facie evidence, or probable cause, or sufficient justification to hold him to a tedious and prolonged public trial, on the basis of the following grounds: a. the questioned resolution of respondent Gingoyon is full of factual misrepresentations or misapprehensions CONSTITUTIONAL LAW 2 CASE DIGESTS

b. respondent’s reliance on the decision of the Regional Trial Court against Felipe Galarion suffers from constitutional and procedural infirmities considering that petitioner was not a party thereto, much less was he given any opportunity to comment on or rebut the prosecution evidence; c. reliance on Rogelio Hanopol’s testimony is likewise “contemptible,” it being merely hearsay in addition to the fact that petitioner was never given the opportunity to cross-examine Hanopol at the time he testified in court; d. the affidavit of Roxas dated March 30, 1989, which is the only evidence against petitioner, has been rendered nugatory by his affidavit of retraction dated June 20, 1990. Preliminary investigation is generally inquisitorial, and it is often the only means of discovering the persons who may be reasonably charged with a crime, to enable the fiscal to prepare his complaint or information. It is not a trial of the case on the merits and has no purpose except that of determining whether a crime has been committed and whether there is probable cause to believe that the accused is guilty thereof, and it does not place the person against whom it is taken in jeopardy.  The quantum of evidence now required in preliminary investigation is such evidence sufficient to “engender a well-founded belief” as to the fact of the commission of a crime and the respondent’s probable guilt thereof. A preliminary investigation is not the occasion for the full and exhaustive display of the parties’ evidence; it is for the presentation of such evidence only as may engender a wellgrounded belief that an offense has been committed and that the accused is probably guilty thereof.  we are in accord with the state prosecutor’s findings in the case at bar that there exists prima facie evidence of petitioner’s involvement in the commission of the crime, it being sufficiently supported by the evidence presented and the facts obtaining therein.GR: The institution of a criminal action depends upon the sound discretion of the fiscal. He has the quasi-judicial discretion to determine whether or not a criminal case should be filed in court. Hence, the general rule is that an injunction will notbe granted to restrain a criminal prosecution XPN: Citing the case of Brocka et al vs Enrile a. To afford adequate protection to the constitutional rights of the accused; b. When necessary for the orderly administration of justice or to avoidoppression or multiplicity of actions; c. When there is a pre-judicial question which is sub judice; d. When the acts of the officer are without or in excess of authority; e. Where the prosecution is under an invalid law, ordinance or regulation; f. When double jeopardy is clearly apparent; CONSTITUTIONAL LAW 2 CASE DIGESTS

g. Where the court has no jurisdiction over the offense; h. Where it is a case of persecution rather than prosecution; i. Where the charges are manifestly false and motivated by the lust for vengeance; and j. When there is clearly no prima facie case against the accused and a motion to quash on that ground has been denied. - In this case, the circumstances of the case do not fall in any of the exceptions. 3. As to petitioner’s contention that he was not granted the opportunity of crossexamination:  It is a fundamental principle that the accused in a preliminary investigation has no right to cross-examine the witnesses which the complainant may present.  Section 3, Rule 112 of the Rules of Court expressly provides that the respondent shall only have the right to submit a counter-affidavit, to examine all other evidence submitted by the complainant and, where the fiscal sets a hearing to propound clarificatory questions to the parties or their witnesses, to be afforded an opportunity to be present but without the right to examine or crossexamine.  Thus, even if petitioner was not given the opportunity to cross-examine Galarion and Hanopol at the time they were presented to testify during these parate trial of the case against Galarion and Roxas, he cannot assert any legal right to cross-examine them at the preliminary investigation precisely because such right was never available to him.  The admissibility or inadmissibility of said testimonies should be ventilated before the trial court during the trial proper and not in the preliminary investigation.

Abdula vs Guiani Facts: A petition for certiorari and prohibition to set aside the warrant of arrest issued by herein respondent Japal guiani, then presiding judge of Branch 14 of RTC of Cotabato City, was filed before the Supreme Court. A complaint for murder was filed but was dismissed by the provincial prosecutor on the gorund that there was no prima facie case for murder again a number of accused (6). However, he recommended the filing of an information for murder against one of the respondents (accused) only before the sala of the respondent CONSTITUTIONAL LAW 2 CASE DIGESTS

judge Guiani. Guiani returned the case to the provincial prosecutor for further investigation since there was no necessary resolution required under the Rules of Court to show how the investigating prosecutor arrived at such a conclusion (charging only one of the 8 respondent-accused). Upon the return of the records of the case, it was assigned for reinvestigation to another prosecutor who then recommended the filing of charges against 5 accused, 2 of whom are herein petitioners. On January 2, 1995, an information was filed against petitioner-spouses and 3 others. The following day, January 3, respondent Judge issued a warrant for the arrest of petitioners. On January 4, petitioners filed an urgent Ex-Parte motion for the setting aside of saide warrant of arrest. On January 11, a petition for review was filed with the DOJ. Despite said filing, respondent judge did not act upon petitioner’s pending Motion to Set Aside the Warrant of Arrest. Hence, this Petition for Certiorari and Prohibition praying the warrant of Arrest be set aside and declared void ab initio. Issue: WON the Warrant of Arrest should be set aside and declared void ab initio. Held: Section 2, Art. III, 1987 Constititution: “The right of the people to be secure in their persons, houses, papers, and effects against unreasonable searches and seizures of whatever nature and for any purpose shall be inviolable, and NO SEARCH WARRANT OR WARRANT OF ARREST SHALL ISSUE EXCEPT UPON PROBABLE CAUSE TO BE DETERMINED PERSONALLY BY THE JUDGE AFTER EXAMINATION UNDER OATH OR AFFIRMATION OF THE COMPLAINANT AND THE WITNESSES HE MAY PRODUCE and particularly describing the place to be searched and the persons or things to be seized.” It must be stressed that the 1987 Constitution requires the judge to determine probable cause "personally," a requirement which does not appear in the corresponding provisions of our previous constitutions. This emphasis evinces the intent of the framers to place a greater degree of responsibility upon trial judges than that imposed under previous Constitutions. CONSTITUTIONAL LAW 2 CASE DIGESTS

What the Constitution underscores is the exclusive and personal responsibility of the issuing judge to satisfy himself of the existence of probable cause. In satisfying himself of the existence of probable cause for the issuance of a warrant of arrest, the judge is not required to personally examine the complainant and his witnesses. Following established doctrine and procedure, he shall: (1) personally evaluate the report and the supporting documents submitted by the fiscal regarding the existence of probable cause and, on the basis thereof, issue a warrant of arrest; or (2) if on the basis thereof he finds no probable cause, he may disregard the fiscal's report and require the submission of supporting affidavits of witnesses to aid him in arriving at a conclusion as to the existence of probable cause. Ho vs. People 41 summarizes existing jurisprudence on the matter as follows: Lest we be too repetitive, we only wish to emphasize three vital matters once more: First, as held in Inting, the determination of probable cause by the prosecutor is for a purpose different from that which is to be made by the judge. Whether there is reasonable ground to believe that the accused is guilty of the offense charged and should be held for trial is what the prosecutor passes upon. The judge, on the other hand, determines whether a warrant of arrest should be issued against the accused, i.e., whether there is a necessity for placing him under immediate custody in order not to frustrate the ends of justice. Second, since their objectives are different, the judge cannot rely solely on the report of the prosecutor in finding probable cause to justify the issuance of a warrant of arrest. The judge must decide independently. Hence, he must have supporting evidence, other than the prosecutor's bare report, upon which to legally sustain his own findings on the existence (or nonexistence) of probable cause to issue an arrest order. This responsibility of determining personally and independently the existence or nonexistence of probable cause is lodged in him by no less than the most basic law of the land. Lastly, it is not required that the complete or entire records of the case during the preliminary investigation be submitted to and examined by the judge.What is required, rather, is that the judge must have sufficient supporting documents (such as the complaint, affidavits, counter-affidavits, sworn statements of witnesses or transcript of stenographic notes, if any) upon which to make his independent judgment or, at the very least, upon which to verify the findings of the prosecutor as to the existence of probable cause. The point is: he cannot rely solely and entirely on the prosecutor's recommendation, as Respondent Court did in this case. CONSTITUTIONAL LAW 2 CASE DIGESTS

In the case at bench, respondent admits that he issued the questioned warrant as there was "no reason for (him) to doubt the validity of the certification made by the Assistant Prosecutor that a preliminary investigation was conducted and that probable cause was found to exist as against those charged in the information filed." The statement is an admission that respondent relied solely and completely on the certification made by the fiscal that probable cause exists as against those charged in the information and issued the challenged warrant of arrest on the sole basis of the prosecutor's findings and recommendations. He adopted the judgment of the prosecutor regarding the existence of probable cause as his own. CONSEQUENTLY, THE WARRANT OF ARREST SHOULD BE DECLARED NULL AND VOID.

People of the Philippines, plaintiff-appellee Vs. Olive Rubio Mamaril, Accused-appellant FACTS: On 25 March 2003, at 9:30 o’clock in the evening, SPO4 Alexis Gotidoc, along with the members of Intel Operatives of Tarlac City Police Station and Philippine Drug Enforcement Agency (PDEA), implemented Search Warrant No. 144C dated 18 March 2003 issued by Judge Alipio Yumul of Branch 66, Regional Trial Court, Capas, Tarlac against the appellant in her residence at Zone 1, Barangay Maliwalo, Tarlac City, Province of Tarlac. Prior to the search, the police team invited Barangay Kagawad Oscar Tabamo of Barangay Maliwalo to witness the conduct of the search and seizure operation in the appellant’s house. With Barangay Kagawad Tabamo, the police team presented the search warrant to appellant and informed her of the purpose of the search and her constitutional rights. Afterwards, SPO4 Gotidoc, the designated searcher, started searching the appellant’s house, in the presence of the appellant and Kagawad Tabamo. During his search, he found on the top cover of the refrigerator one (1) plastic sachet containing white crystalline substance. Thereafter he prepared a Certificate of Good Search and Confiscation Receipt which the appellant refused to sign. The plastic sachet was brought to the Tarlac Provincial Crime Laboratory located at Tarlac Provincial Hospital for qualitative examination. The examination conducted CONSTITUTIONAL LAW 2 CASE DIGESTS

by Engr. Marcene G. Agala, the Forensic Chemist who tested the white crystalline substance, yielded positive results for 0.055 gram of Methamphetamine Hydrochloride, commonly known as shabu, a dangerous drug. The factual version presented by the defense is: On 25 March 2003, at 9:30 o’ clock in the evening the police officers arrived at appellant’s house and showed her a search warrant. Thereafter, the policemen searched her house but found nothing. Then a certain Police Officer Pangilinan asked her where she was sleeping. When she replied that she was inside the hut, the police officers proceeded to and searched the place and found the plastic sachet containing the shabu. Thereafter, she was brought to the sub-station at Maliwalo and was told, particularly by SPO4 Gotidoc and a certain Ma’am Dulay that in exchange of P20,000.00, no case would be filed against her. When she told them that she did not have money, she was detained. However, on cross-examination, the appellant admitted that the alleged extortion of P20,000.00 was not reported to the higher ranking police officers. Appellant claims that the police officers framed her up and planted the shabu inside her house because of her refusal to give them money. ISSUE: Whether or not the accused-appellant is innocent of violating Section 11, Article II, of RA 9165. RULING: No. The Court of Appeals ruled that the evidence for the prosecution fully proved beyond reasonable doubt the elements necessary to successfully prosecute a case for illegal possession of a regulated drug, namely, (a) the accused is in possession of an item or an object identified to be a prohibited or a regulated drug, (b) such possession is not authorized by law and (c) the accused freely and consciously possessed said drug. Centered on the conduct of the search of appellant’s house that yielded the prohibited substance, the Court of Appeals upheld the trial court on the finding that “after a careful evaluation and analysis of the arguments presented by the prosecution and the defense, we hold that the search conducted by the INTEL Operatives of Tarlac City Police Station, in coordination with the PDEA, on the residence of the accused-appellant on 25 March 2003 at Zone 1, Barangay Maliwalo, Tarlac City and the seizure therein of one (1) plastic pack of white CONSTITUTIONAL LAW 2 CASE DIGESTS

crystalline substance of methamphetamine hydrochloride or “shabu” weighing 0.055 gram are legal. As a consequence of the legal search, the said methamphetamine hydrochloride or “shabu” seized on the occasion thereof, is admissible in evidence against the accused-appellant.” The accused-appellant, through her new counsel from the Public Attorney’s Office, goes further back, presenting new arguments, that (1) the search warrant was not based on probable cause, hence, the evidence allegedly obtained through it may not be admitted to support the accused-appellant’s conviction and (2) the presumption of regularity in the performance of official functions by public officers cannot prevail over the presumption of innocence. The original position of the accused which, in this petition, begins with the contention of non-compliance with all the requisites of illegal possession of dangerous drugs. We agree with the rulings of the trial court and the Court of Appeals that there was indeed full satisfaction of the requisites for the conviction of the accused. The trial court found that the evidence presented by the prosecution was not adequately defeated. Re-stating that in illegal possession of prohibited drugs, there are only three (3) elements to secure conviction: (1) accused is in possession of the prohibited drugs; (2) such possession is not authorized by law; and (3) accused consciously and freely possessed the prohibited drugs, the trial court held that all these were established beyond doubt. It determined that appellant failed to proffer evidence enough to discredit the prosecution and render doubtful his guilt. The argument is without merit. In the case at hand, the so-called frame-up was virtually pure allegation bereft of credible proof. The narration of the police officer who implemented the search warrant, was found after trial and appellate review as the true story. It is on firmer ground than the self-serving statement of the accused-appellant of frame-up.The defense cannot solely rely upon the constitutional presumption of innocence for, while it is constitutional, the presumption is not conclusive. Notably, the accusedappellant herself stated in her brief that “no proof was proffered by the accusedappellant of the police officers’ alleged ill motive.” HUBERT J. P. WEBB, petitioner VS. CONSTITUTIONAL LAW 2 CASE DIGESTS

HONORABLE RAUL E. DE LEON, the Presiding Judge of the Regional Trial Court of Parañaque, Branch 258, HONORABLE ZOSIMO V. ESCANO, the Presiding Judge of the Regional Trial Court of Parañaque, Branch 259, PEOPLE OF THE PHILIPPINES, ZENON L. DE GUIYAB, JOVENCITO ZUÑO, LEONARDO GUIYAB, JR., ROBERTO LAO, PABLO FORMARAN, and NATIONAL BUREAU OF INVESTIGATION, and HONORABLE AMELITA G. TOLENTINO, the Presiding Judge of the Regional Trial Court of Parañaque, Branch 274, respondents LAURO VIZCONDE, intervenor FACTS: On June 19, 1994, the National Bureau of Investigation (NBI) filed with the Department of Justice a letter-complaint charging petitioners Hubert Webb, Michael Gatchalian, Antonio J. Lejano and six (6) other persons with the crime of Rape and Homicide of Carmela N. Vizconde, her mother Estrellita NicolasVizconde, and her sister Anne Marie Jennifer in their home at Number 80 W. Vinzons, St., BF Homes Paranaque, Metro Manila on June 30, 1991. Forthwith, the Department of Justice formed a panel of prosecutors headed by Assistant Chief State Prosecutor Jovencio R. Zuno to conduct the preliminary investigation. ARGUMENTS: Petitioners fault the DOJ Panel for its finding of probable cause. They assail the credibility of Jessica Alfaro as inherently weak and uncorroborated due to the inconsistencies between her April 28, 1995 and May 22, 1995 sworn statements. They criticize the procedure followed by the DOJ Panel when it did not examine witnesses to clarify the alleged inconsistencies. Petitioners charge that respondent Judge Raul de Leon and, later, respondent Judge Amelita Tolentino issued warrants of arrest against them without conducting the required preliminary examination. Petitioners complain about the denial of their constitutional right to due process and violation of their right to an impartial investigation. They also assail the prejudicial publicity that attended their preliminary investigation. ISSUES: CONSTITUTIONAL LAW 2 CASE DIGESTS

Whether or not the DOJ Panel likewise gravely abused its discretion in holding that there is probable cause to charge them with the crime of rape and homicide Whether or not respondent Judges de Leon and Tolentino gravely abused their discretion when they failed to conduct a preliminary examination before issuing warrants of arrest against them Whether or not the DOJ Panel denied them their constitutional right to due process during their preliminary investigation Whether or not the DOJ Panel unlawfully intruded into judicial prerogative when it failed to charge Jessica Alfaro in the information as an accused. HELD: NO. NO. NO. There is no merit in this contention because petitioners were given all the opportunities to be heard. NO.

REASONS: The Court ruled that the DOJ Panel did not gravely abuse its discretion when it found probable cause against the petitioners. A probable cause needs only to rest on evidence showing that more likely than not, a crime has been committed and was committed by the suspects. Probable cause need not be based on clear and convincing evidence of guilt, neither on evidence establishing guilt beyond reasonable doubt and definitely, not on evidence establishing absolute certainty of guilt. The Court ruled that respondent judges did not gravely abuse their discretion. In arrest cases, there must be a probable cause that a crime has been committed and that the person to be arrested committed it. Section 6 of Rule 112 simply provides that “upon filing of an information, the Regional Trial Court may issue a warrant for the accused. Clearly the, our laws repudiate the submission of petitioners that respondent judges should have conducted “searching examination of witnesses” before issuing warrants of arrest against them. The DOJ Panel precisely allowed the parties to adduce more evidence in their behalf and for the panel to study the evidence submitted more fully. CONSTITUTIONAL LAW 2 CASE DIGESTS

Petitioner’s argument lacks appeal for it lies on the faulty assumption that the decision whom to prosecute is a judicial function, the sole prerogative of the courts and beyond executive and legislative interference. In truth, the prosecution of crimes appertains to the executive department of government whose principal power and responsibility is to see that our laws are faithfully executed. A necessary component of this power is the right to prosecute their violators (See R.A. No. 6981 and section 9 of Rule 119 for legal basis). With regard to the inconsistencies of the sworn statements of Jessica Alfaro, the Court believes that these have been sufficiently explained and there is no showing that the inconsistencies were deliberately made to distort the truth. With regard to the petitioners’ complaint about the prejudicial publicity that attended their preliminary investigation, the Court finds nothing in the records that will prove that the tone and content of the publicity that attended the investigation of petitioners fatally infected the fairness and impartiality of the DOJ Panel. Petitioners cannot just rely on the subliminal effects of publicity on the sense of fairness of the DOJ Panel, for these are basically unbeknown and beyond knowing.

EXAMINATION OF WITNESSES MATA VS BAYONA FACTS: Soriano Mata was accused under Presidential Decree (PD) 810, as amended by PD 1306, the information against him alleging that Soriano Mata offered, took and arranged bets on the Jai Alai game by “selling illegal tickets known as ‘Masiao tickets’ without any authority from the Philippine Jai Alai & Amusement Corporation or from the government authorities concerned.” Mata claimed that during the hearing of the case, he discovered that nowhere from the records of the said case could be found the search warrant and other pertinent papers connected to the issuance of the same, so that he had to inquire from the City Fiscal its whereabouts, and to which inquiry Judge Josephine K. Bayona, presiding Judge of the City Court of Ormoc replied, “it is with the court”. The Judge then handed the records to the Fiscal who attached them to the records. This led Mata to file a motion to quash and annul the search warrant and for the return of the CONSTITUTIONAL LAW 2 CASE DIGESTS

articles seized, citing and invoking, among others, Section 4 of Rule 126 of the Revised Rules of Court. The motion was denied by the Judge on 1 March 1979, stating that the court has made a thorough investigation and examination under oath of Bernardo U. Goles and Reynaldo T. Mayote, members of the Intelligence Section of 352nd PC Co./Police District II INP; that in fact the court made a certification to that effect; and that the fact that documents relating to the search warrant were not attached immediately to the record of the criminal case is of no moment, considering that the rule does not specify when these documents are to be attached to the records. Mata’s motion for reconsideration of the aforesaid order having been denied, he came to the Supreme Court, with the petition for certiorari, praying, among others, that the Court declare the search warrant to be invalid for its alleged failure to comply with the requisites of the Constitution and the Rules of Court, and that all the articles confiscated under such warrant as inadmissible as evidence in the case, or in any proceedings on the matter. ISSUE: WON the judge must before issuing the warrant personally examine on oath or affirmation the complainant and any witnesses he may produce and take their depositions in writing, and attach them to the record, in addition to any affidavits presented to him? HELD: YES. Under the Constitution “no search warrant shall issue but upon probable cause to be determined by the Judge or such other responsible officer as may be authorized by law after examination under oath or affirmation of the complainant and the witnesses he may produce”. More emphatic and detailed is the implementing rule of the constitutional injunction, The Rules provide that the judge must before issuing the warrant personally examine on oath or affirmation the complainant and any witnesses he may produce and take their depositions in writing, and attach them to the record, in addition to any affidavits presented to him. Mere affidavits of the complainant and his witnesses are thus not sufficient. The examining Judge has to take depositions in writing of the complainant and the witnesses he may produce and to attach them to the record. Such written deposition is necessary in order that the Judge may be able to properly determine the existence or nonexistence of the probable cause, to hold liable for perjury the person giving it if it will be found later that his declarations are false. We, therefore, hold that the search warrant is tainted with illegality by the failure of CONSTITUTIONAL LAW 2 CASE DIGESTS

the Judge to conform with the essential requisites of taking the depositions in writing and attaching them to the record, rendering the search warrant invalid. ONSICA VS IGNALAGA G.R. No. 72301 July 31, 1987 FACTS: The chief issue raised by the petitioners in this case is whether or not Section 143 of the Local Government Code1granting power to the municipal mayor to conduct preliminary investigations and order the arrest of the accused, was repealed by the 1985 Rules on Criminal Procedure promulgated by this Court; and is, in addition, unconstitutional as vesting the power to conduct preliminary investigations in an official who cannot be deemed a "neutral and detached magistrate" within the contemplation of Section 3, Article IV of the 1973 Constitution. The issue is hereby resolved adversely to the petitioners, with the stressed qualification that the mayor's power to order arrest ceased to exist as of February 2, 1987 when the new Constitution was ratified by the Filipino people, and that, in any event, the investigation actually conducted by respondent mayor in the case at bar was fatally defective. RULING: While it is true that the mayors do "exercise general supervision over units and elements of the INP stationed or assigned in their respective jurisdictions," they are not themselves directly involved in police work and cannot in any sense be described, as the petitioners do, as being deeply involved in law enforcement functions. And even if that "deep involvement" be conceded, it does not follow that this would necessarily preclude their assuming "the cold neutrality of an impartial judge" in conducting preliminary investigations of persons suspected of crimes. As the law now stands, the mayor may no longer conduct preliminary investigation, the authority to do so being limited under Section 2, Rule 1 1 2 of the Rules of Court to (1) provincial or city fiscals and their assistants; (2) judges of the Municipal Trial Courts and Municipal Circuit Trial Courts; (3) national and regional state prosecutors; and (d) such other officers as may be authorized by law. But only "the judge" may issue search and arrest warrants after due determination of probable cause. CONSTITUTIONAL LAW 2 CASE DIGESTS

SOLIVEN, petitioner VS. JUDGE MAKASIAR, respondent 167 SCRA 393 FACTS: This case is a PETITION for certiorari and prohibition to review the decision of the Regional Trial Court of Manila ISSUES: Whether or not the petitioners were denied due process when information for libel were filed against them although the finding of the existence of a prima facie case was still under review by the Secretary of Justice and, subsequently by the President Whether or not the constitutional rights of Beltran (petitioner) were violated when respondent RTC judge issued a warrant for his arrest without personally examining the complainant and the witnesses, if any, to determine probable clause Whether or not the President of the Philippines, under the Constitution, may initiate criminal proceedings against the petitioners through filing of a complaintaffidavit DECISION: Finding no grave abuse of discretion amounting to excess or lack of jurisdiction on the part of the public respondents, the Court Resolved to DISMISS the petitions. The Order to maintain the status quo contained in the Resolution of the Court en banc is LIFTED. RATIO: Background of the first issue MARCH 30, 1988: Secretary of Justice denied petitioner’s motion for reconsideration APRIL 7, 1988: A second motion for reconsideration filed by petitioner Beltran was denied by the Secretary of Justice MAY 2, 1988: On appeal, the President, through Executive Secretary, affirmed the resolution of the Secretary of Justice CONSTITUTIONAL LAW 2 CASE DIGESTS

MAY 16, 1988: Motion for reconsideration was denied by the Executive Secretary Petitioner Beltran alleges that he has been denied due process of law. -This is negated by the fact that instead of submitting his counter-affidavits, he filed a “Motion to Declare Proceedings Closed”, in effect, waiving his right to refute the complaint by filing counter-affidavits. Due process of law does not require that the respondent in a criminal case actually file his counter-affidavits before the preliminary investigation is deemed completed. All that is required is that the respondent be given the opportunity to submit counter-affidavits if he is so minded. Second issue This calls for an interpretation of the constitutional provision on the issuance of warrants of arrest: Art. III, Sec.2. The right of the people to be secure in their persons, houses, papers and effects against unreasonable searches and seizures of whatever nature and for any purpose shall be inviolable, and no search warrant or warrant of arrest shall issue except upon probable cause to be determined personally by the judge after examination under oath or affirmation of the complainant and the witnesses he may produce, and particularly describing the place to be searched and the persons or things to be seized. Petitioner Beltran is convinced that the Constitution requires the judge to personally examine the complainant and his witness in his determination of probable cause for the issuance of warrants of arrests. -However, what the Constitution underscores is the exclusive and personal responsibility of the issuing judge to satisfy himself of the existence of probable cause. In doing so, the judge is not required to personally examine the complainant and his witness.

Following the established doctrine of procedure, the judge shall: (1) Personally evaluate the report and supporting documents submitted by the fiscal regarding CONSTITUTIONAL LAW 2 CASE DIGESTS

the existence of probable cause (and on the basis, thereof, issue a warrant of arrest); or (2) If on the basis thereof he finds no probable cause, he may disregard the fiscal’s report and require the submission of supporting affidavits of witnesses to aid him in arriving at a conclusion as to the evidence of probable cause. Third issue Petitioner Beltran contends that proceedings ensue by virtue of the President’s filing of her complaint-affidavit, she may subsequently have to be a witness for the prosecution, bringing her under the trial court’s jurisdiction. àThis would in an indirect way defeat her privilege of immunity from suit, as by testifying on the witness stand, she would be exposing herself to possible contempt of court or perjury. -This privilege of immunity from suit, pertains to the President by virtue of the office and may be invoked only by the holder of the office; not by any other person in the President’s behalf. -The choice of whether to exercise the privilege or to waive is solely the President’s prerogative. It is a decision that cannot be assumed and imposed by any other person (And there is nothing in our laws that would prevent the President from waiving the privilege). Additional Issue: Beltran contends that he could not be held liable for libel because of the privileged character of the publication. He also says that to allow the libel case to proceed would produce a “chilling effect” on press freedom. -Court reiterates that it is not a trier of facts And Court finds no basis at this stage to rule on the “chilling effect” point. CID VS JUDGE DE LA ROSA GR. NO. 95122-23, MAY 31, 1991 FACTS: The then Secretary of Justice issued a memorandum directing the Board of Commissioners to review all cases where entry was allowed on the ground that CONSTITUTIONAL LAW 2 CASE DIGESTS

the entrant was a Philippine citizen, including that of respondents Gatchalian. Petitioner Board reversed the decision of the Board of Special Inquiry admitting respondents Gatchalian as Filipino citizens. Petitioner Commissioner of Immigration issued a mission order commanding the arrest of respondent William Gatchalian. ISSUE(S): Whether or not the warrant of arrest issued by Commissioner of Immigration was valid. HELD: NO. A warrant of arrest issued by the Commissioner of Immigration, to be valid, must be for the sole purpose of executing a final order of deportation. A warrant of arrest issued by the Commissioner of Immigration for purposes of investigation only is null and void for being unconstitutional. A reading of the mission order/warrant of arrest issued by the Commissioner of Immigration clearly indicates that the same was issued only for purposes of investigation of the suspects, respondent Gatchalian included. Petition is DISMISSED for lack of merit. PARTICULARITY of Description PAPER INDUSTRIES CORPORATION OF THE PHILIPPINES vs JUDGE MAXIMO ASUNCION FACTS: Police Chief Inspector Napoleon B. Pascua applied for a search warrant before Regional Trial Court (RTC) of Quezon City, that the Paper Industries Corporation of the Philippines located at Bislig, Surigao De Sur is in possession or has in its control high powered firearms, ammunitions, explosives, which are the subject of the offense. ISSUE: Whether the Search Warrant is Valid? CONSTITUTIONAL LAW 2 CASE DIGESTS

HELD: No, petition for Certiorari and prohibition is hereby granted and Search Warrant No. 799 accordingly declared null and void, TRO permanent. RATIONALE: There is a question of fact when the doubt arises as to the truth or falsity of the alleged facts. Main Issue: Validity of Search Warrrant (Article III, Section 2, 1987, 1987 Constitution & Rule 126, Sections 3&4 of Rules of Court). Requisites of Valid Search Warrant are: 1. Probable Cause is present 2. Such presence is determined personally by the Judge 3. The complainant and the Witnesses he or she may produce are personally examined by the Judge, in writing and under oath or affirmation 4. The applicant and the Witnesses testify on facts personally known to them 5. The Warrant specifically describes the place to be searched and things to be seized a. Present case, the Search Warrant is INVALID because i. The Trial Court failed to examine personally the complainant and the other dependents ii. SP03 Bacolod had no personal knowledge that petitioners were not licensed to possess the subject firearms iii. The place to be searched was not described with particularity Mere affidavits of the complainant and his witnesses does are thus not sufficient. The examining Judge has to take depositions in writing of the complainant and the witnesses he may produce and attached them to the record. Particularity of the Place to be searched: The belief to value privacy of home and person and to afford it constitutional protection against the long reach of government is no less than to value human dignity, and that his privacy must not be disturbed except in case of overriding social need and then only under stringent procedural safeguards. CONSTITUTIONAL LAW 2 CASE DIGESTS

The aforementioned premises, did not specify such premises. The warrant identifies only one place and that is Paper Industries Corporation of the Philippines, located at PICOP compound. However; it was made of 200 offices / building, 15 plants, 84 staffs houses, airstrip, 3 piers / wharves, and 23 warehouses.

OTHER CASES: PRUDENTE VS JUDGE DAYRIT 180 SCRA 69 (1989) FACTS: The Chief of the Intelligence Special Action Division (ISAD) filed with the Regional Trial Court (RTC) Manila, Judge Abelardo Dayrit, for the issuance of Search Warrant for violation of PD No. 1866 (Illegal Possession of Firearm, etc). In the deposition of witness (P/Lt. Florencio C. Angeles), it was made mentioned of “result of our continuous surveillance conducted for several days. We gathered information from verified sources that the holders of said firearms and explosives as well as ammunitions aren’t licensed to possess said firearms and ammunition. Further, the premises is a school and the holders of these firearms are not student who were not supposed to possess firearms, explosives and ammunitions. Person to be searched in Nemesio Prudente at the Polytechnic University of the Philippines, Sta. Mesa, Sampaloc, Manila, has in his control or possession firearms, explosives hand grenades and ammunitions which are illegally possesses at the office of Department of Military Science and Tactics and at the office of the President. Petitioner moved to quash the Search Warrant. He claimed that: Petitioners, had no personal knowledge of the facts The examination of the said witness was not in form of searching questions and answers CONSTITUTIONAL LAW 2 CASE DIGESTS

Search warrant was a general warrant Violation of Circular No. 19 of the Supreme Court in that the complainant failed to allege under oath that the issuance of the search warrant on a Saturday, urgent. ISSUE: Whether or not the search and seizure was valid? HELD: Search Warrant annulled and set aside. RATIONALE: Valid search warrant to issue, there must be probable cause, which is to be determined personally by the Judge, after examination under oath and affirmation of the complainant, and that witnesses he may produce and particularly describing the place to be searched and the persons and things to be seized. The probable cause must be in connection with one specific offense and the Judge must, before issuing Search Warrant, personally examine in the form of searching questions and answers, In writing and under oath, the complainant and any witnesses he may produce, on facts personally known to them and attach to the record their sworn statements together with any affidavits submitted.

“Probable Cause” for a valid search warrant, has been defined as such facts and circumstances which would lead a reasonably discreet and prudent man to believe that an offense has been committed, and that objects sought in connection which the offense are in the place sought to be searched. – This probable case must be shown to be personal knowledge and of the complainant and witnesses he may produce and not based on mere hearsay.

20th Century Fox Film v. Court of Appeals, G.R. Nos. 76649-51, August 19, 1988 CONSTITUTIONAL LAW 2 CASE DIGESTS

THE FACTS Petitioner 20th Century Fox Film Corporation sought the assistance of the NBI in conducting searches and seizures in connection with the NBI’s anti-film piracy campaign. Petitioner alleged that certain videotape outlets all over Metro Manila are engaged in the unauthorized sale and renting out of copyrighted films in violation of PD No. 49 (the old Intellectual Property Law). The NBI conducted surveillance and investigation of the outlets pinpointed by the petitioner and subsequently filed three (3) applications for search warrants against the video outlets owned by the private respondents. The lower court issued the desired search warrants. The NBI, accompanied by the petitioner's agents, raided the video outlets and seized the items described in the three warrants. Private respondents later filed a motion to lift the search warrants and release the seized properties, which was granted by the lower court. Petitioner’s motion for reconsideration was denied by the lower court. The CA affirmed the trial court. THE ISSUE Did the judge properly lift the search warrants he issued earlier? THE RULING [The Court DISMISSED the petition and AFFIRMED the questioned decision and resolution of the CA.] YES, the judge properly lifted the search warrants he issued earlier. The lower court lifted the three (3) questioned search warrants in the absence of probable cause that the private respondents violated P.D. 49. NBI agents who acted as witnesses during the application for search warrant did not have personal knowledge of the subject matter of their testimony, which was the alleged commission of the offense of piracy by the private respondents. Only the petitioner’s counsel who was also a witness during the application stated that he CONSTITUTIONAL LAW 2 CASE DIGESTS

had personal knowledge that the confiscated tapes owned by the private respondents were pirated tapes taken from master tapes belonging to the petitioner. The lower court lifted the warrants, declaring that the testimony of petitioner’s counsel did not have much credence because the master tapes of the allegedly pirated tapes were not shown to the court during the application. The presentation of the master tapes of the copyrighted films, from which the pirated films were allegedly copied, was necessary for the validity of search warrants against those who have in their possession the pirated films. The petitioner's argument to the effect that the presentation of the master tapes at the time of application may not be necessary as these would be merely evidentiary in nature and not determinative of whether or not a probable cause exists to justify the issuance of the search warrants is not meritorious. The court cannot presume that duplicate or copied tapes were necessarily reproduced from master tapes that it owns. The essence of a copyright infringement is the similarity or at least substantial similarity of the purported pirated works to the copyrighted work. Hence, the applicant must present to the court the copyrighted films to compare them with the purchased evidence of the video tapes allegedly pirated to determine whether the latter is an unauthorized reproduction of the former. This linkage of the copyrighted films to the pirated films must be established to satisfy the requirements of probable cause. Mere allegations as to the existence of the copyrighted films cannot serve as basis for the issuance of a search warrant.

Nolasco vs. Cruz Pano, 132 SCRA 152 (1985) FACTS: Milagros Aguilar-Roque was arrested together with Cynthia Nolasco by the Constabulary Security Group (CSG). Milagros had been wanted as a high ranking officer of the CPP. The arrest took place at 11:30 a.m. of August 6, 1984. At noon of the same day, her premises were searched and 428 documents, a portable typewriter and 2 boxes were seized.

CONSTITUTIONAL LAW 2 CASE DIGESTS

Earlier that day, Judge Cruz Paño issued a search warrant to be served at AguilarRoque’s leased residence allegedly an underground house of the CPP/NPA. On the basis of the documents seized, charges of subversion and rebellion by the CSG were filed by but the fiscal’s office merely charged her and Nolasco with illegal possession of subversive materials. Aguilar-Roque asked for suppression of the evidence on the ground that it was illegally obtained and that the search warrant is void because it is a general warrant since it does not sufficiently describe with particularity the things subject of the search and seizure, and that probable cause has not been properly established for lack of searching questions propounded to the applicant’s witness. ISSUE: WON the search warrant was valid? HELD: NO. Section 3, Article IV of the Constitution, guarantees the right of the people to be secure in their persons, houses, papers and effects against unreasonable searches and seizures of whatever nature and for any purpose. It also specifically provides that no Search Warrant shall issue except upon probable cause to be determined by the Judge or such other responsible officer as may be authorized by law, after examination under oath or affirmation of the complainant and the witnesses he may produce, and particularly describing the place to be searched and the things to be seized.

VALLEJO VS CA FACTS: A SW was applied for and subsequently issued by respondents to be served in the Registry of Deeds, provincial capitol of Isabela in which it enumerated the things to be seized: 1. Undetermined number of Fake Land Titles, Official Receipts in the Cashier's Office, Judicial Form No. 39 known as Primary Entry Book under No. 496 and other pertinent documents related therewith; CONSTITUTIONAL LAW 2 CASE DIGESTS

2. Blank Forms of Land Titles kept inside the drawers of every table of employees of the Registry (sic) of Deeds; 3. Undetermined number of land Transfer transactions without the corresponding payment of Capital Gains Tax and payment of documentary Stamps. A motion to quash the SW was filed by the respondent contending that the things to be seized were not described with particularity and was in a nature of a general warrant, therefore, is a violation of the constitutional prohibition against unreasonable searches and seizures but was denied by the RTC and the CA. Hence, the present petition for certiorari. (The OSG in its comment agreed with petitioners.) ISSUE: WON the warrant issued by the RTC was valid. HELD: Sec. 4. Requisites for issuing search warrant. – A search warrant shall not issue except upon probable cause in connection with one specific offense to be determined personally by the judge after examination under oath or affirmation of the complainant and the witnesses he may produce, and particularly describing the place to be searched and the things to be seized which may be anywhere in the Philippines. Sec. 5. Examination of complainant; record. – The judge must, before issuing the warrant, personally examine in the form of searching questions and answers, in writing and under oath, the complainant and the witnesses he may produce on facts personally known to them and attach to the record their sworn statements, together with the affidavits submitted. The things to be seized must be described with particularity. Technical precision of description is not required. It is only necessary that there be reasonable particularity and certainty as to the identity of the property to be searched for and seized, so that the warrant shall not be a mere roving commission. Any description of the place or thing to be searched that will enable the officer making the search with reasonable certainty to locate such place or thing is sufficient. Thus, the specific property to be searched for should be so particularly described as to preclude any possibility of seizing any other property(test of particularity). As correctly pointed out by the petitioner and the OSG, the terms expressly used in the warrant were too all-embracing, with the obvious intent of subjecting all CONSTITUTIONAL LAW 2 CASE DIGESTS

the records pertaining to all the transactions of the petitioner's office at the Register of Deeds to search and seizure. Such tenor of a seizure warrant contravenes the explicit command of the Constitution that there be a particular description of the things to be seized.

Bache & Co Inc vs. Ruiz GR L-32409, 27 February 1971 Facts: On 24 February 1970, Misael P. Vera, Commissioner of Internal Revenue, wrote a letter addressed to Judge Vivencio M. Ruiz requesting the issuance of a search warrant against Bache & Co. (Phil.), Inc. and Frederick E. Seggerman for violation of Section 46(a) of the National Internal Revenue Code (NIRC), in relation to all other pertinent provisions thereof, particularly Sections 53, 72, 73, 208 and 209, and authorizing Revenue Examiner Rodolfo de Leon to make and file the application for search warrant which was attached to the letter. In the afternoon of the following day, De Leon and his witness, Arturo Logronio, went to the Court of First Instance (CFI) of Rizal. They brought with them the following papers: Vera’s letter-request; an application for search warrant already filled up but still unsigned by De Leon; an affidavit of Logronio subscribed before De Leon; a deposition in printed form of Logronio already accomplished and signed by him but not yet subscribed; and a search warrant already accomplished but still unsigned by Judge. At that time the Judge was hearing a certain case; so, by means of a note, he instructed his Deputy Clerk of Court to take the depositions of De Leon and Logronio. After the session had adjourned, the Judge was informed that the depositions had already been taken. The stenographer, upon request of the Judge, read to him her stenographic notes; and thereafter, the Judge asked Logronio to take the oath and warned him that if his deposition was found to be false and without legal basis, he could be charged for perjury.

CONSTITUTIONAL LAW 2 CASE DIGESTS

The Judge signed de Leon’s application for search warrant and Logronio’s deposition. Search Warrant 2-M-70 was then signed by Judge and accordingly issued. 3 days later (a Saturday), the BIR agents served the search warrant to the corporation and Seggerman at the offices of the corporation on Ayala Avenue, Makati, Rizal. The corporation’s lawyers protested the search on the ground that no formal complaint or transcript of testimony was attached to the warrant. The agents nevertheless proceeded with their search which yielded 6 boxes of documents. On 3 March 1970, the corporation and Seggerman filed a petition with the Court of First Instance (CFI) of Rizal praying that the search warrant be quashed, dissolved or recalled, that preliminary prohibitory and mandatory writs of injunction be issued, that the search warrant be declared null and void, and that Vera, Logronio, de Leon, et. al., be ordered to pay the corporation and Seggerman, jointly and severally, damages and attorney’s fees. After hearing and on 29 July 1970, the court issued an order dismissing the petition for dissolution of the search warrant. In the meantime, or on 16 April 1970, the Bureau of Internal Revenue made tax assessments on the corporation in the total sum of P2,594,729.97, partly, if not entirely, based on the documents thus seized. The corporation and Seggerman filed an action for certiorari, prohibition, and mandamus. Issue: Whether the corporation has the right to contest the legality of the seizure of documents from its office. Held: The legality of a seizure can be contested only by the party whose rights have been impaired thereby, and that the objection to an unlawful search and seizure is purely personal and cannot be availed of by third parties. In Stonehill, et al. vs. Diokno, et al. (GR L-19550, 19 June 1967; 20 SCRA 383) the Supreme Court CONSTITUTIONAL LAW 2 CASE DIGESTS

impliedly recognized the right of a corporation to object against unreasonable searches and seizures; holding that the corporations have their respective personalities, separate and distinct from the personality of the corporate officers, regardless of the amount of shares of stock or the interest of each of them in said corporations, whatever, the offices they hold therein may be; and that the corporate officers therefore may not validly object to the use in evidence against them of the documents, papers and things seized from the offices and premises of the corporations, since the right to object to the admission of said papers in evidence belongs exclusively to the corporations, to whom the seized effects belong, and may not be invoked by the corporate officers in proceedings against them in their individual capacity. The distinction between the Stonehill case and the present case is that: in the former case, only the officers of the various corporations in whose offices documents, papers and effects were searched and seized were the petitioners; while in the latter, the corporation to whom the seized documents belong, and whose rights have thereby been impaired, is itself a petitioner. On that score, the corporation herein stands on a different footing from the corporations in Stonehill. Moreover, herein, the search warrant was void inasmuch as First, there was no personal examination conducted by the Judge of the complainant (De Leon) and his witness (Logronio). The Judge did not ask either of the two any question the answer to which could possibly be the basis for determining whether or not there was probable cause against Bache & Co. and Seggerman. The participation of the Judge in the proceedings which led to the issuance of Search Warrant 2-M-70 was thus limited to listening to the stenographer’s readings of her notes, to a few words of warning against the commission of perjury, and to administering the oath to the complainant and his witness. This cannot be consider a personal examination. Second, the search warrant was issued for more than one specific offense. The search warrant was issued for at least 4 distinct offenses under the Tax Code. The first is the violation of Section 46(a), Section 72 and Section 73 (the filing of income tax returns), which are interrelated. The second is the violation of Section 53 (withholding of income taxes at source). CONSTITUTIONAL LAW 2 CASE DIGESTS

The third is the violation of Section 208 (unlawful pursuit of business or occupation); and the fourth is the violation of Section 209 (failure to make a return of receipts, sales, business or gross value of output actually removed or to pay the tax due thereon). Even in their classification the 6 provisions are embraced in 2 different titles: Sections 46(a), 53, 72 and 73 are under Title II (Income Tax); while Sections 208 and 209 are under Title V (Privilege Tax on Business and Occupation). Lastly, the search warrant does not particularly describe the things to be seized. Search Warrant No. 2-M-70 tends to defeat the major objective of the Bill of Rights, i.e., the elimination of general warrants, for the language used therein is so all-embracing as to include all conceivable records of the corporation, which, if seized, could possibly render its business inoperative. Thus, Search Warrant 2-M70 is null and void.

Marquez v. Desierto (G.R. No. 135882) Facts: Petitioner Lourdes Marquez received an Order from respondent Ombudsman Aniano Desierto to produce several bank documents for purposes of inspection in camera relative to various accounts maintained at the bank where petitioner is the branch manager. The accounts to be inspected are involved in a case pending with the Ombudsman entitled, Fact-Finding and Intelligence Bureau (FFIB) v. Amado Lagdameo. It appears that a certain George Trivinio purchased trail managers check and deposited some of it to an account maintained at petitioner’s branch. Petitioner after meeting with the FFIB Panel to ensure the veracity of the checks agreed to the in camera inspection. Petitioner being unable to readily identify the accounts in question, the Ombudsman issued an order directing petitioner to produce the bank documents. Thus, petitioner sought a declaration of her rights from the court due to the clear conflict between RA 6770 and RA 1405. Meanwhile, FFIB moved to cite petitioner in contempt before the Ombudsman. CONSTITUTIONAL LAW 2 CASE DIGESTS

Issue: Whether or not the order of Ombudsman to have an in camera inspection of the accounts is an allowable exception of R.A. No. 1405. Ruling: NO. The order of the Ombudsman to produce for in camera inspection the subject accounts with the Union Bank of the Philippines, Julia Vargas Branch, is based on a pending investigation at the Office of the Ombudsman against Amado Lagdameo, et. al. for violation of R.A. No. 3019, Sec. 3 (e) and (g) relative to the Joint Venture Agreement between the Public Estates Authority and AMARI. We rule that before an in camera inspection may be allowed, there must be a pending case before a court of competent jurisdiction. Further, the account must be clearly identified, the inspection limited to the subject matter of the pending case before the court of competent jurisdiction. The bank personnel and the account holder must be notified to be present during the inspection, and such inspection may cover only the account identified in the pending case. In the case at bar, there is yet no pending litigation before any court of competent authority. What is existing is an investigation by the Office of the Ombudsman. In short, what the office of the ombudsman would wish to do is to fish for additional evidence to formally charge Amado Lagdameo, et. al., with the Sandiganbayan. Clearly, there was no pending case in court which would warrant the opening of the bank account for inspection. *In contrast to Ejercito v. Sandiganbayan. Interestingly, time is of the essence. A different ruling in Ejercito was enunciated because there was already a pending investigation months before the ruling made in this case as to the exemption in the power of the Ombudsman. Republic v Judge Eugenio G.R. No. 174629, February 14, 2008 Sec. 2 of the Bank Secrecy Act itself prescribes exceptions whereby these bank accounts may be examined by any person, government official, bureau or offial; namely when: (1) upon written permission of the depositor; (2) in cases of CONSTITUTIONAL LAW 2 CASE DIGESTS

impeachment; (3) the examination of bank accounts is upon order of a competent court in cases of bribery or dereliction of duty of public officials; and (4) the money deposited or invested is the subject matter of the litigation. Section 8 of R.A. Act No. 3019, the Anti-Graft and Corrupt Practices Act, has been recognized by this Court as constituting an additional exception to the rule of absolute confidentiality, and there have been other similar recognitions as well.[ Facts: Under the authority granted by the Resolution, the AMLC filed an application to inquire into or examine the deposits or investments of Alvarez, Trinidad, Liongson and Cheng Yong before the RTC of Makati, Branch 138, presided by Judge (now Court of Appeals Justice) Sixto Marella, Jr. The application was docketed as AMLC No. 05-005. The Makati RTC heard the testimony of the Deputy Director of the AMLC, Richard David C. Funk II, and received the documentary evidence of the AMLC.[14] Thereafter, on 4 July 2005, the Makati RTC rendered an Order (Makati RTC bank inquiry order) granting the AMLC the authority to inquire and examine the subject bank accounts of Alvarez, Trinidad, Liongson and Cheng Yong, the trial court being satisfied that there existed p]robable cause [to] believe that the deposits in various bank accounts, details of which appear in paragraph 1 of the Application, are related to the offense of violation of Anti-Graft and Corrupt Practices Act now the subject of criminal prosecution before the Sandiganbayan as attested to by the Informations, Exhibits C, D, E, F, and G Pursuant to the Makati RTC bank inquiry order, the CIS proceeded to inquire and examine the deposits, investments and related web accounts of the four.[16] Meanwhile, the Special Prosecutor of the Office of the Ombudsman, Dennis VillaIgnacio, wrote a letter dated 2 November 2005, requesting the AMLC to investigate the accounts of Alvarez, PIATCO, and several other entities involved in the nullified contract. The letter adverted to probable cause to believe that the bank accounts were used in the commission of unlawful activities that were committed a in relation to the criminal cases then pending before the Sandiganbayan. Attached to the letter was a memorandum on why the investigation of the [accounts] is necessary in the prosecution of the above criminal cases before the Sandiganbayan. In response to the letter of the Special Prosecutor, the AMLC promulgated on 9 December 2005 Resolution No. 121 Series of 2005,[19] which authorized the executive director of the AMLC to inquire into and examine the accounts named in the letter, including one maintained by CONSTITUTIONAL LAW 2 CASE DIGESTS

Alvarez with DBS Bank and two other accounts in the name of Cheng Yong with Metrobank. The Resolution characterized the memorandum attached to the Special Prosecutors letter as extensively justif[ying] the existence of probable cause that the bank accounts of the persons and entities mentioned in the letter are related to the unlawful activity of violation of Sections 3(g) and 3(e) of Rep. Act No. 3019, as amended. Issue: Whether or not the bank accounts of respondents can be examined. Held: Any exception to the rule of absolute confidentiality must be specifically legislated. Section 2 of the Bank Secrecy Act itself prescribes exceptions whereby these bank accounts may be examined by any person, government official, bureau or offial; namely when: (1) upon written permission of the depositor; (2) in cases of impeachment; (3) the examination of bank accounts is upon order of a competent court in cases of bribery or dereliction of duty of public officials; and (4) the money deposited or invested is the subject matter of the litigation. Section 8 of R.A. Act No. 3019, the Anti-Graft and Corrupt Practices Act, has been recognized by this Court as constituting an additional exception to the rule of absolute confidentiality, and there have been other similar recognitions as well. The AMLA also provides exceptions to the Bank Secrecy Act. Under Section 11, the AMLC may inquire into a bank account upon order of any competent court in cases of violation of the AMLA, it having been established that there is probable cause that the deposits or investments are related to unlawful activities as defined in Section 3(i) of the law, or a money laundering offense under Section 4 thereof. Further, in instances where there is probable cause that the deposits or investments are related to kidnapping for ransom,[certain violations of the Comprehensive Dangerous Drugs Act of 2002,hijacking and other violations under R.A. No. 6235, destructive arson and murder, then there is no need for the AMLC to obtain a court order before it could inquire into such accounts. It cannot be successfully argued the proceedings relating to the bank inquiry order under Section 11 of the AMLA is a litigation encompassed in one of the exceptions to the Bank Secrecy Act which is when money deposited or invested is the subject matter of the litigation. The orientation of the bank inquiry order is simply to serve as a provisional relief or remedy. As earlier stated, the application for such does not entail a full-blown trial. Nevertheless, just because the AMLA establishes CONSTITUTIONAL LAW 2 CASE DIGESTS

additional exceptions to the Bank Secrecy Act it does not mean that the later law has dispensed with the general principle established in the older law that all deposits of whatever nature with banks or banking institutions in the Philippines x x x are hereby considered as of an absolutely confidential nature. Indeed, by force of statute, all bank deposits are absolutely confidential, and that nature is unaltered even by the legislated exceptions referred to above. OBJECTS OF SEIZURE UNILAB, INC. vs. ERNESTO ISIP and/or SHALIMAR PHILIPPINES G.R. No. 163858. June 28, 2005 Facts: UNILAB hired a private investigator to investigate a place purported to be manufacturing fake UNILAB products, especially Revicon multivitamins. The agent took some photographs where the clandestine manufacturing operation was taking place. UNILAB then sought the help of the NBI, which thereafter filed an application for the issuance of search warrant in the RTC of Manila. After finding probable cause, the court issued a search warrant directing the police to seize “finished or unfinished products of UNILAB, particularly REVICON multivitamins.” No fake Revicon was however found; instead, sealed boxes where seized, which, when opened contained 60 ml bottles of Disudrin and 200mg tablets of Inoflox, both were brands used by UNILAB. NBI prayed that some of the sized items be turned over to the custody of the Bureau of Food and Drugs (BFAD) for examination. The court granted the motion. The respondents then filed a motion to quash the search warrant or to suppress evidence, alleging that the seized items are considered to be fruit of a poisonous tree, and therefore inadmissible for any purpose in any proceeding, which the petitioners opposed alleging that the boxes of Disudrin and Inoflox were seized under the plain view doctrine. The court, however, granted the motion of the respondents. Issue: CONSTITUTIONAL LAW 2 CASE DIGESTS

Whether or not the seizure of the sealed boxes which, when opened, contained Disudrin syrup and Inoflox, were valid under the plain view doctrine. Held: It is true that things not described in the warrant may be seized under the plain view doctrine. However, seized things not described in the warrant cannot be presumed as plain view. The State must adduce evidence to prove that the elements for the doctrine to apply are present, namely: (a) the executing law enforcement officer has a prior justification for an initial intrusion or otherwise properly in a position from which he can view a particular order; (b) the officer must discover incriminating evidence inadvertently; and (c) it must be immediately apparent to the police that the items they observe may be evidence of a crime, contraband, or otherwise subject to seizure It was thus incumbent on the NBI and the petitioner to prove that the items were seized on plain view. It is not enough that the sealed boxes were in the plain view of the NBI agents. However, the NBI failed to present any of officers who were present when the warrant was enforced to prove that the the sealed boxes was discovered inadvertently, and that such boxes and their contents were incriminating and immediately apparent. It must be stressed that only the enforcing officers had personal knowledge whether the sealed boxes and their contents thereof were incriminating and that they were immediately apparent. There is even no showing that the NBI agents knew the contents of the sealed boxes before they were opened. In sum then, the petitioner and the NBI failed to prove that the plain view doctrine applies to the seized items.

People v. Chua Ho San 308 SCRA 432 (1999) G.R. No. 128222, June 17, 1999 Fact: In response to reports of rampant smuggling of firearms and other contraband, CID began patrolling the Bacnotan coastline with his officers. While monitoring the coastal area he intercepted a radio call from ALMOITE requesting police assistance regarding an unfamiliar speedboat. CID and six of his men. When the CONSTITUTIONAL LAW 2 CASE DIGESTS

speedboat landed, the male passenger alighted, and using both hands, carried what appeared a multicolored strawbag. He then walked towards the road. By this time, ALMOITE, CID and BADUA, the latter two conspicuous in their uniform and issued side-arms, became suspicious of the man as he suddenly changed direction and broke into a run upon seeing the approaching officers. BADUA, however, prevented the man from fleeing by holding on to his right arm. Although CID introduced themselves as police officers, the man appeared impassive. Speaking in English, CID then requested the man to open his bag, but he seem not to understand. CID thus tried speaking Tagalog, then Ilocano, but still to no avail. CID then resorted to what he termed “sign language;” he motioned with his hands for the man to open the bag. This time, the man apparently understood and acceded to the request. A search of the bag yielded several transparent plastic packets containing yellowish crystalline substances. which was later found out that it was Shabu. CID then gestured to the man to close the bag, which he did. As CID wished to proceed to the police station, he signaled the man to follow, but the latter did not to comprehend. Hence, CID placed his arm around the shoulders of the man and escorted the latter to the police headquarters. CHUA was initially charged with illegal possession of methaphetamine hydrochloride before the RTC. The RTC convicted Chua Ho San guilty beyond reasonable doubt. Chua Ho San prays for his acquitttal and the reversal of the judgment of the RTC. Issue: Whether the accused who was acting suspiciously constitute Probable Cause impelling the police officers from effecting an in flagrante delicto arrest. Held: No, the Court, finds that these do not constitute “probable cause.” None of the telltale clues, e.g., bag or package emanating the pungent odor of marijuana or other prohibited drug, confidential report and/or positive identification by informers of courier(s) of prohibited drug and/or the time and place where they will transport/deliver the same, suspicious demeanor or behavior and suspicious bulge in the waist — accepted by this Court as sufficient to justify a warrantless arrest exists in this case. The term probable cause had been understood to mean a reasonable ground of suspicion supported by circumstances sufficiently strong in themselves to warrant a cautious man’s belief that the person accused is guilty of the offense with which he is charged. Specifically with respect to arrests, it is such facts and circumstances which would lead a reasonably discreet and prudent man CONSTITUTIONAL LAW 2 CASE DIGESTS

to believe that an offense has been committed by the person sought to be arrested. In cases of in fragrante delicto, arrests, a peace officer or a private person may without a warrant, arrest a person, when, in his presence, the person to be arrested has committed, is actually committing, or is attempting to commit an offense. The arresting officer, therefore, must have personal knowledge of such facts or as recent case law adverts to, personal knowledge of facts or circumstances convincingly indicative or constitutive of probable cause. The search cannot therefore be denominated as incidental to an arrest. While a contemporaneous search of a person arrested may be effected to deliver dangerous weapons or proofs or implements used in the commission of the crime and which search may extend to the area within his immediate control where he might gain possession of a weapon or evidence he can destroy, a valid arrest must precede the search. The process cannot be reversed. In a search incidental to a lawful arrest, as the precedent arrest determines the validity of the incidental search, the legality of the arrest is questioned in a large majority of these cases, e.g., whether an arrest was merely used as a pretext for conducting a search. In this instance, the law requires that there be first a lawful arrest before a search can be made — the process cannot be reversed. WARRANTLESS SEARCHERS G.R. No. 197788, February 29, 2012 RODEL LUZ y ONG, Petitioner, vs PEOPLE OF THE PHILIPPINES, Respondent. Facts: PO3 Emmanuel L. Alteza testified that he saw the accused driving a motorcycle without a helmet and so he flagged him down. He invited the accused to come inside their sub-station since the place where he flagged down the accused is almost in front of the sub-station to where he is assigned as a traffic enforcer. The accused violated a municipal ordinance which requires all motorcycle drivers to wear helmet while driving said motor vehicle. While the officers were issuing a citation ticket for violation of municipal ordinance, PO3 Alteza noticed that the accused was uneasy and kept on reaching something from his jacket. He was CONSTITUTIONAL LAW 2 CASE DIGESTS

alerted and told the accused to take out the contents of his jacket’s pocket as the latter may have a weapon inside it. The accused obliged, slowly put out the contents of his jacket’s pocket which included two plastic sachets of suspected shabu. The RTC convicted petitioner of illegal possession of dangerous drugs as the substances are positive of methampethamine hydrochloride. Upon appeal, the CA affirmed the RTCs Decision. Upon a petition for reiew on certiorari, petitioner claims that there was no lawful search and seizure, because there was no lawful arrest. He claims that the finding that there was a lawful arrest was erroneous, since he was not even issued a citation ticket or charged with violation of the city ordinance. Even assuming there was a valid arrest, he claims that he had never consented to the search conducted upon him. Issue: Whether or not the arrest, searches and seizure were invalid. Held: Yes, there was no valid arrest. When he was flagged down for committing a traffic violation, he was not, ipso facto and solely for this reason, arrested. There being no valid arrest, the warrantless search that resulted from it was likewise illegal. Under R.A. 4136, or the Land Transportation and Traffic Code, the general procedure for dealing with a traffic violation is not the arrest of the offender, but the confiscation of the drivers license of the latter. At the time that he was waiting for PO3 Alteza to write his citation ticket, petitioner could not be said to have been under arrest. rior to the issuance of the ticket, the period during which petitioner was at the police station may be characterized merely as waiting time. The subject items seized during the illegal arrest are inadmissible. The drugs are the very corpus delicti of the crime of illegal possession of dangerous drugs. Thus, their inadmissibility precludes conviction and calls for the acquittal of the accused.

CONSTITUTIONAL LAW 2 CASE DIGESTS

G.R. No. 180661 December 11, 2013 GEORGE ANTIQUERA y CODES, Petitioner, vs. PEOPLE OF THE PHILIPPINES, Respondent. Facts: Police officers were conducting a police visibility patrol in Pasay City when they saw two unidentified men rush out of a house and boarded a jeep. Believing that there was a crime, the police officers approached the house. When they peeked through the partially opened door, they saw Antiquera and Cruz engaged in a pot session. The police officers entered the house, introduced themselves and arrested Antiquera and Cruz. While inspecting the vicinity, PO1 Cabutihan saw a jewellery box which contained shabu and unused paraphernalia. The RTC found them guilty of illegal possession of paraphernalia for dangerous drugs. The court affirmed the decision of RTC. Issue: Whether or not the arrest was invalid. Held: Yes, there was unlawful arrest because the circumstances here do not make out a case of arrest made in flagrante delicto. Admittedly, the police officers did not notice anything amiss going on in the house from the street where they stood. Indeed, even as they peeked through its partially opened door, they saw no activity that warranted their entering it. Clearly, no crime was plainly exposed to the view of the arresting officers that authorized the arrest of accused Antiquera without warrant under the above-mentioned rule. Considering that his arrest was illegal, the search and seizure that resulted from it was likewise illegal. Anonymous Letter-Complaint against Atty. Miguel Morales, Clerk of Court, MeTC Manila Facts: Atty. Morales, Branch Clerk of Court of MeTC, Branch 67, Manila was investigated on the basis of an anonymous letter alleging that he was consuming his working CONSTITUTIONAL LAW 2 CASE DIGESTS

hours filing and attending to personal cases, using office supplies, equipment and utilities. The OCA conducted a spot investigation aided by NBI agents. The team was able to access Atty. Morales personal computer and print two documents stored in its hard drive, which turned out to be two pleadings, one filed in the CA and another in the RTC of Manila, both in the name of another lawyer. Atty. Morales computer was seized and taken in custody of the OCA but was later ordered released on his motion, but with order to the MISO to first retrieve the files stored therein. Atty. Morales, in defense, argues that since the pleadings were acquired from his personal computer which was confiscated without any valid search and seizure order, such evidence should be considered as the fruits of a poisonous tree as it violated his right to privacy. The OCA disagreed with the report of the Investigating Judge that there was no evidence to support the charge against Atty. Morales as no one from the OCC personnel who were interviewed would give a categorical and positive statement affirming the charges against Atty. Morales, along with other court personnel also charged in the same case. The OCA recommended that Atty. Morales should be found guilty of gross misconduct. Issues: 1. Are the pleadings found in Atty. Morales's personal computer admissible in the present administrative case against him? 2. May the right against unreasonable searches and seizures be invoked in an administrative case? 3. Was there consented warrantless search in this case? 4. Is there a ground to hold Atty. Morales liable of the charge? Held: 1. While Atty. Morales may have fallen short of the exacting standards required of every court employee, the Court cannot use the evidence obtained from his CONSTITUTIONAL LAW 2 CASE DIGESTS

personal computer against him for it violated his constitutional right against unreasonable searches and seizures. 2. As expounded in Zulueta v. Court of Appeals, any violation of the aforestated constitutional right renders the evidence obtained inadmissible for any purpose in any proceeding. 3. Consent to a search is not to be lightly inferred and must be shown by clear and convincing evidence. It must be voluntary in order to validate an otherwise illegal search; that is, the consent must be unequivocal, specific, intelligently given and uncontaminated by any duress or coercion. The burden of proving, by clear and positive testimony, that the necessary consent was obtained and that it was freely and voluntarily given lies with the State. Acquiescence in the loss of fundamental rights is not to be presumed and courts indulge every reasonable presumption against waiver of fundamental constitutional rights. To constitute a valid consent or waiver of the constitutional guarantee against obtrusive searches, it must be shown that (1) the right exists; (2) that the person involved had knowledge, either actual or constructive, of the existence of such right; and (3) the said person had an actual intention to relinquish the right. In this case, what is missing is a showing that Atty. Morales had an actual intention to relinquish his right. While he may have agreed to the opening of his personal computer and the printing of files therefrom during the spot investigation, it is also of record that Atty. Morales immediately filed an administrative case against said persons questioning the validity of the investigation, specifically invoking his constitutional right against unreasonable search and seizure. 4. And as there is no other evidence, apart from the pleadings, retrieved from the unduly confiscated personal computer of Atty. Morales, to hold him administratively liable, the Court had no choice but to dismiss the charges against him for insufficiency of evidence

WAIVERS CONSTITUTIONAL LAW 2 CASE DIGESTS

PEOPLE VS. SUZUKI Facts: Appellant was found guilty of illeal possession of prohibited drugs and was sentenced to death. Hence, the instant review. The facts of the case are: Appellant was in the airport for his flight to Manila. PASCOM and NARCOM agents were in the airport to follow on reports on drug trafficking. When he walked through the metal detector, the alarm sounded. He was bodily frisked and nothing was found on his person so they proceeded to check his luggage but appellant refused then consented eventually and opened it. There they found packs of aluminum foil and when opened, it was found to be marijuana. Issue: WON the PASCOM agents were authorized to conduct the search. WON accused consented to the search. WON the search was conducted was incidental to a lawful arrest. WON the confiscated items were in plain view. Held: In PPvs.Canton and PPvs.Johnson we validated the search conducted on the departing passengers and the consequent seizure of the shabu found in their persons, thus: "Persons may lose the protection of the search and seizure clause by exposure of their persons or property to the public in a manner reflecting a lack of subjective expectation of privacy, which expectation society is prepared to recognize as reasonable.xxx" It should be stressed, however, that whenever the right against unreasonable search and seizure is challenged, an individual may choose between invoking the constitutional protection or waiving his right by giving consent to the search or seizure.9 Here, appellant voluntarily gave his consent to the search conducted by the PASCOM agents. It is axiomatic that a reasonable search is not to be determined by any fixed formula but is to be resolved according to the facts of each case.12 Given the circumstances obtaining here, we find the search conducted by the airport authorities reasonable and, therefore, not violative of his constitutional rights. CONSTITUTIONAL LAW 2 CASE DIGESTS

Hence, when the search of the box of piaya revealed several marijuana fruiting tops, appellant is deemed to have been caught in flagrante delicto, justifying his arrest even without a warrant under Section 5(a), Rule 113 of the Rules of Criminal Procedure.13 The packs of marijuana obtained in the course of such valid search are thus admissible as evidence against appellant.14 Nonetheless, we find the trial court’s reliance on the plain view doctrine misplaced. Such doctrine finds application only when the incriminating nature of the object is in the "plain view" of the police officer.15 Here, it is beyond cavil that the marijuana seized from appellant is contained in the box of piaya, wrapped in aluminum foil and not immediately apparent to the airport authorities. Neither was the search incidental to a lawful arrest since appellant was not yet arrested at the time of the search. To be considered a search incidental to a lawful arrest, the law requires that there must be a lawful arrest before the search can be made. VEROY VS LAYAGUE FACTS: Capt. Reynaldo Obrero, acting upon a directive, raided the Davao City residence of petitioner spouses who were already residing in Quezon City on information that the said residence was being used as a safehouse of rebel soldiers. The team did not enter the house since the owner was not present and they did not have a search warrant. When contacted by Obrero to ask permission to search the house, Ma. Luisa relented if the search would be conducted in the presence of longtime family friend, a ranking military officer. The team gained entry into the yard and the kitchen. With the help of a locksmith, the team entered the children’s room and conducted the search with resulted in the recovery of several articles which include a firearm and jute sacks containing printed materials of RAM-SFP. ISSUE(S): Whether or not the officers violated the petitioners’ constitutional right against unreasonable search and seizure. HELD: YES. The reason for searching the house of herein petitioners is that it was CONSTITUTIONAL LAW 2 CASE DIGESTS

reportedly being used as a hideout and recruitment center for rebel soldiers. While Capt. Obrero was able to enter the compound, he did not enter the house because he did not have a search warrant and the owners were not present. This shows that he himself recognized the need for a search warrant, hence, he did not persist in entering the house but rather contacted the petitioners to seek permission to enter the same. Permission was indeed granted by petitioner to enter the house but only to ascertain the presence of rebel soldiers. Under the circumstances, it is undeniable that the police officers had ample time to procure a search warrant but did not. Petition is GRANTED and the criminal case against the petitioners for illegal possession of firearms is DISMISSED. People vs. Tudtud GR 144037 September 23, 2003 FACTS: Sometime during the months of July and August 1999, the Toril Police Station, Davao City received a report from a “civilian asset” named Bobong Solier about a certain Noel Tudtud. Solier related that his neighbours have been complaining about Tudtud, who was allegedly responsible for the proliferation of marijuana in their area. Relating to the report, the police conducted surveillance in Solier’s neighbourhood in Sapa, Toril, Davao City. For 5 days, they gathered information and leared that Tudtud was involved in illegal drugs. According to his neighbours, Tudtud was engaged in selling marijuana. Solier informed the police that Tudtud had headed to Cotabato and would be back later that day with new stocks of marijuana. Solier described Tudtud as big bodied and short, and usually wore a hat. At around 4:00 pm that same day, a team of policemen posted themselves at the corner of Saipon and McArthur Highway to await. Tudtud’s arrival. All wore civilian clothes. About 8:00 pm, 2 men disembarked from a bus and helped each other carry a carton marked “King Flakes.” Standing some 5 feet away from the men, PO1 Desierto and PO1 Floreta observed that one of the men fit Tudtud’s description. The same man also toted a plastic bag. PO1Floreta and PO1 Desierto then approached the suspects and identified themselves as police officers. PO1 Desierto informed them that the police had received information that stocks of illegal drugs would be arriving that CONSTITUTIONAL LAW 2 CASE DIGESTS

night. The man who resembled Tudtud’s description denied that he was carrying any drugs. PO1 Desierto asked if he could see the contents of the box. Tudtud then said “it was alright” and let them see the box which contained bundles of dried fish, one wrapped in a plastic bag and another in newspapers. When the bundles were unwrapped, there contained marijuana leaves. The police arrested Tudtud and his comapanion. They were charged with illegal possession of prohibited drugs before the RTC of Davao City which convicted the accused. ISSUE: Whether or not Tudtud’s implied acquiescence (Tudtud’s statement of “it’s alright”) is considered a waiver.

RULING: NO. The right against unreasonable searched and seizures is secured by Sec. 2, Art. 3 of the Constitution. Appellants implied acquiescence, if at all, could not have been more than mere passive conformity given under coercive or intimidating circumstances and is, thus, considered no consent at all within the purview of the constitutional guarantee. Consequently, appellants lack of objection to the search and seizure is not tantamount to a waiver of his constitutional right or a voluntary submission to the warrantless search and seizure. As the search of appellant’s box does not come under the recognized exceptions to a valid warrantless search, the marijuana leaves obtained thereby are inadmissible in evidence. And as there is no evidence other than the hearsay testimony of the arresting officers and their informant, the conviction of appellants cannot be sustained. Finally, there is an effective waiver of rights against unreasonable searches and seizures if the following requisites are present: It must appear that the rights exist; CONSTITUTIONAL LAW 2 CASE DIGESTS

The person involved had knowledge, actual or constructive, of the existence of such right; Said person had an actual intention to relinquish the right. Here, the prosecution failed to establish the second and third requisites. Records disclose that when the police officers introduced themselves as such and requested appellant that they see the contents of the carton box supposedly containing the marijuana, appellant Tudtud said it was alright. He did not resist and opened the box himself. Acquiescence in the loss of fundamental rights is not to be presumed. The fact that a person failed to object to a search does not amount to permission thereto. Decision of the Regional Trial Court of Davao City is REVERSED. Appellants Noel Tudtud y Paypa and Dindo Bolong y Naret are ACQUITTED for insufficiency of evidence.

SILAHIS INTERNATIONAL HOTEL, INC. and JOSE MARCEL PANLILIO, Petitioners, vs. ROGELIO S. SOLUTA, JOSELITO SANTOS, EDNA BERNATE, VICENTA DELOLA, FLORENTINO MATILLA, and GLOWHRAIN-SILAHIS UNION CHAPTER, Respondents. G.R. No. 163087 February 20, 2006 FACTS: Loida Somacera (Loida), a laundrywoman of the hotel, stayed overnight at the female locker room at the basement of the hotel. At dawn, she heard pounding sounds outside, she saw five men in barong tagalog whom she failed to recognize but she was sure were not employees of the hotel, forcibly opening the door of the union office. In the morning, as union officer Soluta was trying in vain to open CONSTITUTIONAL LAW 2 CASE DIGESTS

the door of the union office, Loida narrated to him what she had witnessed at dawn. Soluta immediately lodged a complaint before the Security Officer. And he fetched a locksmith. At that instant, men in barong tagalog armed with clubs arrived and started hitting Soluta and his companions. Panlilio thereupon instructed Villanueva to force open the door, and the latter did. Once inside, Panlilio and his companions began searching the office, over the objection of Babay who even asked them if they had a search warrant. A plastic bag was found containing marijuana flowering tops. As a result of the discovery of the presence of marijuana in the union office and after the police conducted an investigation of the incident, a complaint against the 13 union officers was filed before the Fiscal’s Office of Manila. RTC acquitted the accused. On appeal, the CA affirmed with modification the decision of the trial court. ISSUE: Whether respondent individual can recover damages for violation of constitutional rights. RULING: Article 32, in relation to Article 2219(6) and (10) of the Civil Code, allows so. ART. 32. Any public officer or employee, or any private individual, who directly or indirectly obstructs, defeats, violates or in any manner impedes or impairs any of the following rights and liberties of another person shall be liable to the latter for damages: x x x x In the present case, petitioners had, by their own claim, already received reports in late 1987 of illegal activities and Maniego conducted surveillance. Yet, in the morning of January 11, 1988, petitioners and their companions barged into and searched the union office without a search warrant, despite ample time for them to obtain one. The course taken by petitioners and company stinks in illegality. Petitioners’ violation of individual respondents’ constitutional right against unreasonable search thus furnishes the basis for the award of damages under Article 32 of the Civil Code. For respondents, being the lawful occupants of the office had the right to raise the question of validity of the search and seizure. Article 32 speaks of an officer or employee or person "directly or indirectly" responsible for the violation of the constitutional rights and liberties of another. Hence, it is not the actor alone who must answer for damages under Article 32; the person indirectly responsible has also to answer for the damages or injury CONSTITUTIONAL LAW 2 CASE DIGESTS

caused to the aggrieved party. Such being the case, petitioners, together with Maniego and Villanueva, the ones who orchestrated the illegal search, are jointly and severally liable for actual, moral and exemplary damages to herein individual respondents in accordance with the earlier-quoted pertinent provision of Article 32, in relation to Article 2219(6) and (10) of the Civil Code which provides: Art. 2219. Moral damages may be recovered in the following and analogous cases, among others, (6) Illegal search and (10) Acts and action referred to in Articles 21, 26, 27, 28, 29, 30, 32, 34 and 35. DECISION: Denied. RODOLFO ESPANO vs. COURT OF APPEALS and PEOPLE OF THE PHILIPPINES G.R. No. 120431 April 1, 1998 Facts: Pat. Pagilagan together with other police officers went to Zamora and Pandacan Streets, Manila to confirm reports of drug pushing in the area. They saw petitioner selling something to another person. After the alleged buyer left, they approached petitioner, identified themselves as policemen, and frisked him. The search yielded two plastic cellophane tea bags of marijuana. When asked if he had more marijuana, he replied that there was more in his house. The policemen went to his residence where they found ten more cellophane tea bags of marijuana. Petitioner was brought to the police headquarters where he was charged of possession of prohibited drugs. Issue: Whether or not the pieces of evidence were inadmissible Ruling: The Supreme Court held that Section 5 Rule 113 of the Rules of Court provides: “Arrest without warrant; when lawful – a peace officer or a private person may, without a warrant, arrest a person: CONSTITUTIONAL LAW 2 CASE DIGESTS

When, in the presence, the person to be arrested has committed, is actually committing, or is attempting to commit an offense . . . “ Petitioner’s arrest falls squarely under the aforecited rule. He was caught in flagrante as a result of a buy bust operation conducted by police officers on the basis of information received regarding the illegal trade of drugs within the area. The police officer saw petitioner handling over something to an alleged buyer. After the buyer left, they searched him and discovered two cellophane of marijuana. His arrest was, therefore, lawful and the two cellophane bag of marijuana seized were admissible in evidence, being fruits of the crime. Roldan v Arca; G.R. No. L-25434; 25 Jul 1975; 65 SCRA 336 FACTS: Petitioner Fisheries Commissioner through the Philippine Navy seized and impounded two fishing vessels owned by respondent company for illegal fishing with dynamite. ISSUE(S): Whether or not search and seizure of the fishing vessels without a warrant were unlawful. HELD: NO. Search and seizure without search warrant of vessels and air crafts for violations of the customs laws have been the traditional exception to the constitutional requirement of a search warrant. The same exception should apply to seizures of fishing vessels breaching our fishery laws. They are usually equipped with powerful motors that enable them to elude pursuing ships of the Philippine Navy or Coast Guard. Writ of preliminary mandatory injunction SET ASIDE as NULL and VOID. INCIDENT TO LAWFUL ARREST PEOPLE VS. DORIA CONSTITUTIONAL LAW 2 CASE DIGESTS

FACTS: A buy-bust operation was conducted by the police which caught accused Doria red-handed of selling prohibited drugs and during the operation the police officers searched for the marked bills that they used in buying said drugs which happened to be in the house of Gaddao, according to Doria. When they reached her house, the police officers came upon a box. He saw that one of the box's flaps was open and inside the box was something wrapped in plastic. The plastic wrapper and its contents appeared similar to the marijuana earlier "sold" to him by "Jun." His suspicion aroused, PO3 Manlangit entered "Neneth's" house and took hold of the box. He peeked inside the box and found that it contained ten (10) bricks of what appeared to be dried marijuana leaves. Both accused were convicted of the crime chared. Hence, this present petition. ISSUE: WON the warrantless arrest of Gaddao, the search of her person and house, and the admissibility of the pieces of evidence obtained therefrom was valid. HELD: We hold that the warrantless arrest of accused-appellant Doria is not unlawful. Warrantless arrests are allowed in three instances as provided by Section 5 of Rule 113. Under Section 5 (a), a person may be arrested without a warrant if he "has committed, is actually committing, or is attempting to commit an offense." Appellant Doria was caught in the act of committing an offense. When an accused is apprehended in flagrante delicto as a result of a buy-bust operation, the police are not only authorized but duty-bound to arrest him even without a warrant. The warrantless arrest of appellant Gaddao, the search of her person and residence, and the seizure of the box of marijuana and marked bills are different matters. Our Constitution proscribes search and seizure without a judicial warrant and any evidence obtained without such warrant is inadmissible for any purpose in any proceeding. 105 The rule is, however, not absolute. Search and seizure may be made without a warrant and the evidence obtained there from may be admissible in the following instances: (1) search incident to a lawful arrest; 2) search of a moving motor vehicle; (3) search in violation of customs laws; (4) seizure of evidence in plain view; (5) when the accused himself waives his right against unreasonable searches and seizures. Accused-appellant Gaddao was not caught red-handed during the buy-bust operation to give ground for her arrest under CONSTITUTIONAL LAW 2 CASE DIGESTS

Section 5 (a) of Rule 113. She was not committing any crime. Contrary to the finding of the trial court, there was no occasion at all for appellant Gaddao to flee from the policemen to justify her arrest in "hot pursuit."114 In fact, she was going about her daily chores when the policemen pounced on her. This brings us to the question of whether the trial court correctly found that the box of marijuana was in plain view, making its warrantless seizure valid. The "plain view" doctrine applies when the following requisites concur: (a) the law enforcement officer in search of the evidence has a prior justification for an intrusion or is in a position from which he can view a particular area; (b) the discovery of the evidence in plain view is inadvertent; (c) it is immediately apparent to the officer that the item he observes may be evidence of a crime, contraband or otherwise subject to seizure. The law enforcement officer must lawfully make an initial intrusion or properly be in a position from which he can particularly view the area. In the course of such lawful intrusion, he came inadvertently across a piece of evidence incriminating the accused. The object must be open to eye and hand and its discovery inadvertent. It is clear that an object is in plain view if the object itself is plainly exposed to sight. The difficulty arises when the object is inside a closed container. Where the object seized was inside a closed package, the object itself is not in plain view and therefore cannot be seized without a warrant. However, if the package proclaims its contents, whether by its distinctive configuration, its transparency, or if its contents are obvious to an observer, then the contents are in plain view and may be seized. In other words, if the package is such that an experienced observer could infer from its appearance that it contains the prohibited article, then the article is deemed in plain view. It must be immediately apparent to the police that the items that they observe may be evidence of a crime, contraband or otherwise subject to seizure. The marijuana was not in plain view and its seizure without the requisite search warrant was in violation of the law and the Constitution. 135 It was fruit of the poisonous tree and should have been excluded and never considered by the trial court.

OTHER CASES: THE PEOPLE OF THE PHILIPPINES vs. MIKAEL MALMSTEDT G.R. No. 91107 June 19, 1991 CONSTITUTIONAL LAW 2 CASE DIGESTS

Facts: Captain Alen Vasco, the commanding officer of the first regional command (NARCOM) stationed at camp Dangwa, ordered his men to set up a temporary checkpoint for the purpose of checking all vehicles coming from the Cordillera Region. The order to establish a checkpoint was prompted by persistent reports that vehicles coming from Sagada were transporting marijuana and other prohibited drugs. And an information also was received about a Caucasian coming from Sagada had in his possession prohibited drugs. In the afternoon the bus where accused was riding stopped. Sgt. Fider and CIC Galutan boarded the bus and announced that they were members of the NARCOM and that they would conduct an inspection. During the inspection CIC Galutan noticed a bulge on accused waist. Suspecting the bulge on accused waist to be a gun, the officer asked for accused’s passport and other identification papers. When accused failed to comply, the officer required him to bring out whatever it was that was bulging o his waist. And it turned out to be a pouched bag and when accused opened the same bag the officer noticed four suspicious looking objects wrapped in brown packing tape. It contained hashish, a derivative of marijuana. Thereafter, the accused was invited outside the bus for questioning. But before he alighted from the bus accused stopped to get two travelling bags. The officer inspects the bag. It was only after the officers had opened the bags that the accused finally presented his passport. The two bags contained a stuffed toy each, upon inspection the stuff toy contained also hashish. Issue: Whether or not there is a violation of the constitutional right against unreasonable search and seizure Ruling: The Supreme Court held that under Section 5 Rule 113 of the Rules of Court provides: CONSTITUTIONAL LAW 2 CASE DIGESTS

“Arrest without warrant; when lawful – a peace officer or a private person may, without a warrant, arrest a person: a) When, in the presence, the person to be arrested has committed, is actually committing, or is attempting to commit an offense; b) When an offense has in fact just been committed, and he has personal knowledge of facts indicating that the person to be arrested has committed it; and c) When the person to be arrested is a prisoner who has escaped from a penal establishment or place where he is serving final judgment or temporary confined while his case is pending, or has escaped while being transferred from one confinement to another” Accused was searched and arrested while transporting prohibited drugs. A crime was actually being committed by the accused and he was caught in flagrante delicto, thus the search made upon his personal effects falls squarely under paragraph 1 of the foregoing provision of law, which allows a warrantless search incident to a lawful arrest. Probable cause has been defined as such facts and circumstances which could lead a reasonable, discreet and prudent man to believe that an offense has been committed, and that the object sought in connection with the offense are in the placed sought to be searched. When NARCOM received the information that a Caucasian travelling from Sagada to Baguio City was carrying with him a prohibited drug, there was no time to obtain a search warrant.

People v Aminnudin; G.R. No. L-74869; 06 Jul 1988; 163 SCRA 402 Published on 29 September 2017 in Legal Chyme by Claudine CONSTITUTIONAL LAW 2 CASE DIGESTS

FACTS: Having earlier received a tip from an informer identifying the accused by name who was on board a vessel bound for Iloilo City and was carrying marijuana, the PC officers simply accosted him, inspected his bag and finding what looked like marijuana leaves took him to their headquarters for investigation. The two bundles of suspect articles were confiscated from him and later taken to the NBI laboratory for examination. When they were verified as marijuana leaves, an information for violation of the Dangerous Drugs Act was filed against him and he was eventually convicted. ISSUE(S): Whether or not Aminnudin’s arrest and search were lawful. HELD: NO. It is clear that the PC had at least two days within which they could have obtained a warrant to arrest and search Aminnudin who was coming to Iloilo on the M/V Wilcon 9. His name was known. The vehicle was identified. The date of its arrival was certain. And from the information they had received, they could have persuaded a judge that there was probable cause, indeed, to justify the issuance of a warrant. Yet they did nothing. No effort was made to comply with the law. The Bill of Rights was ignored altogether because the PC lieutenant who was the head of the arresting team, had determined on his own authority that a “search warrant was not necessary.” PEOPLE vs. LIBNAO | G.R. No. 136860 | Jan 20, 2003 FACTS: This is a case finding appellant Agpanga Libnao and her co-accused Rosita Nunga guilty of violating Art. II, Sec. 4 of R.A. No. 6425 (The Dangerous Drugs Act of 1972). The intelligence operatives of the PNP stationed in Tarlac, Tarlac began conducting surveillance operation on suspected drug dealers in the area. They learned from CONSTITUTIONAL LAW 2 CASE DIGESTS

their asset that a certain woman from Tajiri, Tarlac and a companion from Baguio City were transporting illegal drugs once a month in big bulks. On Oct. 20, 1996, at about 1AM, SPO1 Gamotea and PO3 Ferrer flagged down a passing tricycle. It had two female passengers seated inside, who were later identified as the herein appellant and her co-accused. In front of them was a black bag. Suspicious of the black bag and the twos uneasy behavior when asked about its ownership and content, the officers invited them to Kabayan Center No.2. Upon reaching the center, P03 Ferrer fetched Brgy. Captain Pascual to witness the opening of the black bag. As soon as the brgy. Captain arrived, the black bag was opened in the presence of the appellant, her co-accused and personnel of the center. Found inside were eight bricks of leaves sealed in plastic bags and covered with newspaper. The seized articles were later brought to the PNP Crime Lab in Pampanga. Forensic Chemist Babu conducted a lab exam on them and concluded that the articles were marijuana. For their part, both accused denied the accusation against them. Libnao argued that her arrest was unlawful, capitalizes on the absence of a warrant for her arrest. She also takes the issue of the fact that she was not assisted by a lawyer when police officers interrogated her. She claimed that she was not duly informed of her right to remain silent and to have competent counsel of her choice. Hence, she argues that the confession or admission obtained therein should be considered inadmissible in evidence against her. ISSUE: Whether both the accused can be convicted based on the prosecutions evidence. HELD: YES. The above contentions deserve scant attention. The warrantless search in the case at bench is not bereft of a probable cause. The Tarlac Police Intelligence Division had been conducting surveillance operation for 3 months in the area. The surveillance yielded information that once a month, appellant and her co-accused Nunga transport drugs in big bulks. It is also clear that at the time Libnao was apprehended, she was committing offense. She was making a delivery or transporting prohibited drugs in violation of Art. II, Sec. 4 of R.A. No. 6425. Under the ROC, one of the instances a police officer is permitted to carry out a CONSTITUTIONAL LAW 2 CASE DIGESTS

warrantless arrest is when the person to be arrested is caught committing a crime in flagrante delicto. Appellant also faults the trial court for appreciating and taking into account the object and documentary evidence of the prosecution despite the latter’s failure to formally offer them. She argues that absent any formal offer, they must be deemed inadmissible. The contention is untenable. Evidence not formally offered can be considered by the court as long as they have been properly identified by testimony duly recorded and they have themselves been incorporated in the records of the case. All the documentary and object evidence in this case were properly identified, presented and marked as exhibits in court, including the bricks of marijuana. Even without their formal offer; therefore, the prosecution can still establish the case because witnesses properly identified those exhibits, and their testimonies are recorded. Furthermore, appellant’s counsel had cross-examined the prosecution witnesses who testified on the exhibits. Against the credible positive testimonies of the prosecution witnesses, appellants defense of denial and alibi cannot stand. The defense of denial and alibi has been invariably viewed by the courts with disfavor for it can just as easily be concocted and is a common and standard defense ploy in most cases involving violation of the Dangerous Drugs Act. It has to be substantiated by clear and convincing evidence. The appeal is DENIED.

PEOPLE VS NUEVAS Facts: Two policemen (Cabling and Fami) conducted morning surveillance and a monitoring of illegal drug trafficking along a certain street in Olongapo City. They had received information that a certain male person, more or less 5’4" in height, 25 to 30 years old, with a tattoo mark on the upper right hand, and usually CONSTITUTIONAL LAW 2 CASE DIGESTS

wearing a sando and maong pants, would make a delivery of marijuana dried leaves. They saw a man (Nuevas) that matched the description exactly alight a motorcycle. They then approached Nuevas informed him of who they were and asked Nuevas where he was going. Nuevas then informed Fami that there were other stuff in the possession of a certain Vangie, an associate, and two other male persons. Later on, Nuevas voluntarily pointed to the officers a plastic bag which, when opened, contained marijuana dried leaves and bricks wrapped in a blue cloth. Shortly, in his bid to escape charges, Nuevas disclosed where the two (2) other male persons would make the delivery of marijuana. They then went to the place where Nuevas said his companions, Din and Inocencio, were. Din was carrying a blue plastic bag and when asked, disclosed that it was Nuevas’. Upon inspection, Fami found inside it "marijuana packed in newspaper and wrapped therein." After confiscating the items, Fami and Cabling brought Nuevas, Din and Inocencio to the police office. The police officer testified that a receipt for the property seized was issued by the other officer and that a field test was duly conducted on the confiscated items. All three accused were likewise physically examined on the basis of which corresponding medical certificates were issued. The corresponding booking sheets and arrest report were also accomplished. On cross-examination, Cabling testified that the arrest of Nuevas was the result of a tip from Fami’s informant, conceding though that the name of Nuevas was not included in the list of persons under surveillance. Cabling restated that Nuevas had voluntarily submitted the plastic bag he was holding and that after Nuevas had been informed of the violation of law attributed to him, he admitted his willingness to cooperate and point to his other cohorts. When Fami and Cabling proceeded to the identified location of Nuevas’s cohorts, they chanced upon Din and Inocencio along the road. Din was holding a bag while Inocencio was looking into its contents. Cabling averred that Din voluntarily handed the plastic bag he was holding to the police officers. Din and Inocencio are now before the Court submitting for resolution. Issue: Whether the warrantless search and seizure was valid and whether the evidence obtained is admissible in court. CONSTITUTIONAL LAW 2 CASE DIGESTS

Held: The courts below anchor appellants’ conviction on the ground that the searches and seizure conducted in the instant case based on a tip from an informant fall under one of the exceptions as Nuevas, Din and Inocencio all allegedly voluntarily surrendered the plastic bags containing marijuana to the police officers. We differ. First, the Court holds that the searches and seizures conducted do not fall under the first exception, warrantless searches incidental to lawful arrests. A search incidental to a lawful arrest is sanctioned by the Rules of Court. Recent jurisprudence holds that the arrest must precede the search; the process cannot be reversed as in this case where the search preceded the arrest. Nevertheless, a search substantially contemporaneous with an arrest can precede the arrest if the police have probable cause to make the arrest at the outset of the search. In this case, Nuevas, Din and Inocencio were not committing a crime in the presence of the police officers. Moreover, police officers Fami and Cabling did not have personal knowledge of the facts indicating that the persons to be arrested had committed an offense. The searches conducted on the plastic bag then cannot be said to be merely incidental to a lawful arrest. Reliable information alone is not sufficient to justify a warrantless arrest under Section 5(a), Rule 113. The rule requires, in addition, that the accused perform some overt act that would indicate that he "has committed, is actually committing, or is attempting to commit an offense." Secondly, neither could the searches be justified under the plain view doctrine. An object is in plain view if it is plainly exposed to sight. Where the object seized was inside a closed package, the object itself is not in plain view and therefore cannot be seized without a warrant. However, if the package proclaims its contents, whether by its distinctive configuration, its transparency, or if its contents are obvious to an observer, then the contents are in plain view and may be seized. In other words, if the package is such that an experienced observer could infer from its appearance that it contains the prohibited article, then the article is deemed in plain view. It must be immediately apparent to the police that the items that they observe may be evidence of a crime, contraband or otherwise subject to seizure. Records show that the dried marijuana leaves were inside the plastic bags that Nuevas and Din were carrying and were not readily apparent or transparent to the police officers. In Nuevas’s case, the dried marijuana leaves found inside the CONSTITUTIONAL LAW 2 CASE DIGESTS

plastic bag were wrapped inside a blue cloth. In Din’s case, the marijuana found upon inspection of the plastic bag was "packed in newspaper and wrapped therein."44 It cannot be therefore said the items were in plain view which could have justified mere seizure of the articles without further search. On the other hand, the Court finds that the search conducted in Nuevas’s case was made with his consent. In Din’s case, there was none. Indeed, the constitutional immunity against unreasonable searches and seizures is a personal right which may be waived. However, it must be seen that the consent to the search was voluntary in order to validate an otherwise illegal detention and search, i.e., the consent was unequivocal, specific, and intelligently given, uncontaminated by any duress or coercion. The consent to a search is not to be lightly inferred, but must be shown by clear and convincing evidence. The question whether a consent to a search was in fact voluntary is a question of fact to be determined from the totality of all the circumstances. Relevant to this determination are the following characteristics of the person giving consent and the environment in which consent is given: (1) the age of the defendant; (2) whether he was in a public or secluded location; (3) whether he objected to the search or passively looked on; (4) the education and intelligence of the defendant; (5) the presence of coercive police procedures; (6) the defendant's belief that no incriminating evidence will be found; (7) the nature of the police questioning; (8) the environment in which the questioning took place; and (9) the possibly vulnerable subjective state of the person consenting. It is the State which has the burden of proving, by clear and positive testimony, that the necessary consent was obtained and that it was freely and voluntarily given. In Nuevas’s case, the Court is convinced that he indeed voluntarily surrendered the incriminating bag to the police officers. Fami testified in this wise: FISCAL BELTRAN: Q Now, when you saw this accused carrying this Exhibit "D," for your part, what did you do? A I just talked to him and asked him where he was going and according to him, he acted arrogantly, sir. Q This arrogant action of the accused Jesus Nuevas, when you confronted him did he resist? A How did he show his elements, [sic] he said, "So what if you are policeman[?]" Q And being confronted with that arrogance, what did you do next? A Later on he kept calm by saying [sic] in Waray dialect, sir. Q What, exactly, did he tell you in Waray dialect? CONSTITUTIONAL LAW 2 CASE DIGESTS

A "Sir Famir[sic], don’t charge me, sir[.] I am planning to go home to Leyte. I was just earning enough money for my fare, sir." Q So when the accused speak [sic] to you in Waray, what else did you do if you did anything? A I pretended that I agree in his [sic] offer but I also asked him where are the other staffs[sic] sir. Q With respect to the bag that you confiscated from him, what did you do? A He voluntarily pointed it to me and I checked it, the bag, for verification, sir. Cabling likewise testified as follows: Q When Fami got this from the accused, he opened this thing that he got? A The subject voluntarily submitted the same, sir. Q Upon the order of Fami to open it? A Nobody ordered it, sir. There is reason to believe that Nuevas indeed willingly submitted the plastic bag with the incriminating contents to the police officers. It can be seen that in his desperate attempt to exculpate himself from any criminal liability, Nuevas cooperated with the police, gave them the plastic bag and even revealed his ‘associates,’ offering himself as an informant. His actuations were consistent with the lamentable human inclination to find excuses, blame others and save oneself even at the cost of others’ lives. Thus, the Court would have affirmed Nuevas’s conviction had he not withdrawn his appeal. However, with respect to the search conducted in the case of Din, the Court finds that no such consent had actually been given. Fami testified as follows: FISCAL BELTRAN Q Now, what did you do when you saw Din with that Exhibit "C," the plastic bag? A Din said that "Oo, Sir, that is owned by Nuevas" [sic] and I took the said plastic bag. Q When you took this plastic bag from Din…. Was the accused Jesus Nueva [sic] present when Din told you that? A Yes, sir. Nuevas alighted also [from] the vehicle with Cabling. Q And what was the reaction of Nuevas when Din told you that the bag belongs to him? A I did not react, sir. Q After getting that plastic bag from Reynaldo Din, what did you do with it? A I inspected the bag and I found out that there is still marijuana packed in newspaper and wrapped therein, sir. [Emphasis supplied.] Cabling, however, gave a different testimony, viz.: CONSTITUTIONAL LAW 2 CASE DIGESTS

FISCAL BELTRAN Q And upon siting [sic] the two subject persons you have just indicated in your earlier testimony, what did you do? A We approached them and introduced ourselves as police officers, and pinpointed by Nuevas as the ones who kept suspected prohibited drugs, sir. Q After you approached these two people, what happened? A These two people, upon introducing ourselves, [sic] voluntarily surrendered to Fami those marijuana dry leaves, sir. The police officers gave inconsistent, dissimilar testimonies regarding the manner by which they got hold of the bag. This already raises serious doubts on the voluntariness of Din’s submission of the plastic bag. Jurisprudence requires that in case of consented searches or waiver of the constitutional guarantee against obtrusive searches, it is fundamental that to constitute a waiver, it must first appear that (1) the right exists; (2) the person involved had knowledge, either actual or constructive, of the existence of such right; and (3) the said person had an actual intention to relinquish the right. The prosecution failed to clearly show that Din intentionally surrendered his right against unreasonable searches. While it may not be contrary to human nature for one to be jolted into surrendering something incriminating to authorities, Fami’s and Cabling’s testimonies do not show that Din was in such a state of mind or condition. Fami and Cabling did not testify on Din’s composure—whether he felt surprised or frightened at the time—which fact we find necessary to provide basis for the surrender of the bag. There was no mention of any permission made by the police officers to get or search the bag or of any consent given by Din for the officers to search it. It is worthy to note that in cases where the Court upheld the validity of consented search, the police authorities expressly asked, in no uncertain terms, for the consent of the accused to be searched. And the consent of the accused was established by clear and positive proof. Neither can Din’s silence at the time be construed as an implied acquiescence to the warrantless search. In People v. Burgos, the Court aptly ruled: x x x As the constitutional guaranty is not dependent upon any affirmative act of the citizen, the courts do not place the citizen in the position of either contesting an officer’s authority by force, or waiving his constitutional rights; but instead they hold that a peaceful submission to a search or seizure is not a consent or an invitation thereto, but is merely a demonstration of regard for the supremacy of the law. CONSTITUTIONAL LAW 2 CASE DIGESTS

Without the dried marijuana leaves as evidence, Din’s conviction cannot be sustained based on the remaining evidence. The Court has repeatedly declared that the conviction of the accused must rest not on the weakness of the defense but on the strength of the prosecution.1awphi1.net As such, Din deserves an acquittal. In this case, an acquittal is warranted despite the prosecution’s insistence that the appellants have effectively waived any defect in their arrest by entering their plea and by their active participation in the trial of the case. Be it stressed that the legality of an arrest affects only the jurisdiction of the court over the person of the accused. Inspite of any alleged waiver, the dried marijuana leaves cannot be admitted in evidence against the appellants, Din more specifically, as they were seized during a warrantless search which was not lawful. A waiver of an illegal warrantless arrest does not also mean a waiver of the inadmissibility of evidence seized during an illegal warrantless arrest. Turning to Inocencio’s case, the Court likewise finds that he was wrongly convicted of the crime charged. Inocencio’s supposed possession of the dried marijuana leaves was sought to be shown through his act of looking into the plastic bag that Din was carrying.58 Taking a look at an object, more so in this case peeping into a bag while held by another, is not the same as taking possession thereof. To behold is not to hold. Indeed, the act attributed to Inocencio is insufficient to establish illegal possession of the drugs or even conspiracy to illegally possess the same. The prosecution failed to show by convincing proof that Inocencio knew of the contents of the bag and that he conspired with Din to possess the illegal items. Inocencio was firm and unshakeable in his testimony that he had no part in any delivery of marijuana dried leaves. Finally, the law enforcers should be reminded of the Court’s dated but nevertheless current exhortation: x x x In the final analysis, we in the administration of justice would have no right to expect ordinary people to be law-abiding if we do not insist on the full protection of their rights. Some lawmen, prosecutors and judges may still tend to gloss over an illegal search and seizure as long as the law enforcers show the alleged evidence of the crime regardless of the methods by which they were obtained. This kind of attitude condones law-breaking in the name of law enforcement. Ironically, it only fosters the more rapid breakdown of our system of justice, and the eventual denigration of society. While this Court appreciates and encourages the efforts of law enforcers to uphold the law and to preserve the peace and security of society, we nevertheless admonish them to act with deliberate care CONSTITUTIONAL LAW 2 CASE DIGESTS

and within the parameters set by the Constitution and the law. Truly, the end never justifies the means. WHEREFORE, the Decision dated 4 April 2002 of the Regional Trial Court of Olongapo City, Branch 75, in Criminal Case No. 458-97 and No. 459-97 is reversed and modified. Appellants Reynaldo Din y Gonzaga and Fernando Inocencio y Abadeos are hereby ACQUITTED. The Director of the Bureau of Prisons is ordered to cause the immediate release of appellants from confinement, unless they are being held for some other lawful cause, and to report to this Court compliance herewith within five (5) days from receipt hereof. ROBIN PADILLA VS. Court of Appeals Facts: Petitioner was involved in a hit and run accident and was later apprehended by the police after he was chased by them. During the arrest, petitioner was found to have in his possession two different firearms and 2 other firearms were found inside his vehicle after the policemen saw the first two firearms he was carrying. Petitioner was then convicted of illegal possession of firearms. Hence the present petition. Issue: WON the warrantless search and arrest conducted on petitioner was valid Held: Warrantless arrests are sanctioned in the following instances: Sec. 5. Arrest without warrant; when lawful. — A peace officer or a private person may, without a warrant, arrest a person: (a) When, in his presence, the person to be arrested has committed, is actually committing, or is attempting to commit an offense; (b) When an offense has in fact just been committed, and he has personal knowledge of facts indicating that the person to be arrested has committed it. (c) When the person to be arrested is a prisoner who has escaped from a penal establishment or place where he is serving final judgment or temporarily confined while his case is pending, or has escaped while being transferred from one confinement to another. CONSTITUTIONAL LAW 2 CASE DIGESTS

Paragraph (a) requires that the person be arrested (i) after he has committed or while he is actually committing or is at least attempting to commit an offense, (ii) in the presence of the arresting officer or private person. 29 Both elements concurred here, as it has been established that petitioner's vehicle figured in a hit and run — an offense committed in the "presence" of Manarang, a private person, who then sought to arrest petitioner. It must be stressed at this point that "presence" does not only require that the arresting person sees the offense, but also when he "hears the disturbance created thereby AND proceeds at once to the scene." 30 As testified to by Manarang, he heard the screeching of tires followed by a thud, saw the sideswiped victim (balut vendor), reported the incident to the police and thereafter gave chase to the erring Pajero vehicle using his motorcycle in order to apprehend its driver. After having sent a radio report to the PNP for assistance, Manarang proceeded to the Abacan bridge where he found responding policemen SPO2 Borja and SPO2 Miranda already positioned near the bridge who effected the actual arrest of petitioner. The five (5) well-settled instances when a warrantless search and seizure of property is valid, 44 are as follows: 1. warrantless search incidental to a lawful arrest recognized under Section 12, Rule 126 of the Rules of Court45 and by prevailing jurisprudence 46, 2. Seizure of evidence in "plain view", the elements of which are: (a). a prior valid intrusion based on the valid warrantless arrest in which the police are legally present in the pursuit of their official duties; (b). the evidence was inadvertently discovered by the police who had the right to be where they are; (c). the evidence must be immediately apparent, and (d). "plain view" justified mere seizure of evidence without further search. 3. search of a moving vehicle. 49 Highly regulated by the government, the vehicle's inherent mobility reduces expectation of privacy especially when its transit in public thoroughfares furnishes a highly reasonable suspicion amounting to probable cause that the occupant committed a criminal activity. 4. consented warrantless search, and 5. customs search. In conformity with respondent court's observation, it indeed appears that the authorities stumbled upon petitioner's firearms and ammunitions without even undertaking any active search which, as it is commonly understood, is a prying into hidden places for that which is concealed. 51 The seizure of the Smith & Wesson revolver and an M-16 rifle magazine was justified for they came within CONSTITUTIONAL LAW 2 CASE DIGESTS

"plain view" of the policemen who inadvertently discovered the revolver and magazine tucked in petitioner's waist and back pocket respectively, when he raised his hands after alighting from his Pajero. The same justification applies to the confiscation of the M-16 armalite rifle which wasimmediately apparent to the policemen as they took a casual glance at the Pajero and saw said rifle lying horizontally near the driver's seat. 52 Thus it has been held that: (W)hen in pursuing an illegal action or in the commission of a criminal offense, the . . . police officers should happen to discover a criminal offense being committed by any person, they are not precluded from performing their duties as police officers for the apprehension of the guilty person and the taking of the, corpus delicti. 53 Objects whose possession are prohibited by law inadvertently found in plain view are subject to seizure even without a warrant. With respect to the Berreta pistol and a black bag containing assorted magazines, petitioner voluntarily surrendered them to the police. This latter gesture of petitioner indicated a waiver of his right against the alleged search and seizure 56, and that his failure to quash the information estopped him from assailing any purported defect. Even assuming that the firearms and ammunitions were products of an active search done by the authorities on the person and vehicle of petitioner, their seizure without a search warrant nonetheless can still be justified under a search incidental to a lawful arrest (first instance). Once the lawful arrest was effected, the police may undertake a protective search 58 of the passenger compartment and containers in the vehicle 59 which are within petitioner's grabbing distance regardless of the nature of the offense. 60 This satisfied the two-tiered test of an incidental search: (i) the item to be searched (vehicle) was within the arrestee's custody or area of immediate control 61 and (ii) the search was contemporaneous with the arrest. 62 The products of that search are admissible evidence not excluded by the exclusionary rule. Another justification is a search of a moving vehicle (third instance). In connection therewith, a warrantless search is constitutionally permissible when, as in this case, the officers conducting the search have reasonable or probable cause to believe, before the search, that either the motorist is a law-offender (like herein petitioner with respect to the hit and run) or the contents or cargo of the vehicle are or have been instruments or the subject matter or the proceeds of some criminal offense.

CONSTITUTIONAL LAW 2 CASE DIGESTS

PEOPLE V. DE GRACIA (1994) |SEARCHES AND SEIZURES G.R. No. 102009, 233 SCRA 716, July 6, 1994 Doctrine: Where the military operatives had reasonable grounds to believe that a crime was being committed, and had no opportunity to apply for and secure a search warrant from the courts, the same constituted an exception to the prohibition against warrantless searches. Facts: 1. Reform the Armed Forces Movement-Soldiers of the Filipino People (RAMSFP) staged coup d’état in December 1989 against the Government. 2. Efren Soria of Intelligence Division, NCR Defense Command, together with his team, conducted a surveillance of the Eurocar Sales Office in EDSA, QC on early morning of December 1, 1989, which surveillance actually started November 30, 1989 at around 10:00 PM. Such surveillance was conducted pursuant to an intelligence report that the said establishment was being occupied by the elements of the RAM-SFP as communication command post. 3. Near the Eurocar office, there were crowd watching the on-going bombardment near Camp Aguinaldo when a group of 5 men disengaged themselves and walked towards their surveillance car. Maj. Soria ordered the driver to start the car and leave the area. However, as they passed the area, then 5 men drew their guns and fired at them, which resulted to the wounding of the driver. Nobody in the surveillance team retaliated for they were afraid that civilians might be caught in the crossfire. 4. Thereafter, on the morning of December 5, 1989, a search team raided the Eurocar Sales Office and confiscated 6 cartons of M-16 ammunition, 5 bundles of C-4 dynamites, M-shells of different calibers, and molotov. 5. Obenia, who first entered the establishment, found De Gracia in the office of a certain Col. Matillano, holding a C-4 and suspiciously peeping though door. 6. No search warrant was secured by the raiding team because, according to them, there was so much disorder considering that the nearby Camp Aguinaldo was being mopped up by the rebel forces and there was simultaneous firing within the vicinity of the Eurocar office, aside from the fact that the courts were consequently closed. Issue:  Whether there was a valid search and seizure in this case. CONSTITUTIONAL LAW 2 CASE DIGESTS

Held:  







YES, there was a valid search and seizure in this case. It is admitted that the raiding team was not armed with a search warrant at that time. It was actually precipitated by intelligence reports that said office was being used as headquarters by the RAM. Prior to the raid, there was a surveillance conducted on the premises wherein the surveillance team was fired at by a group of men coming from the Eurocar building. When the military operatives raided the place, the occupants thereof refused to open the door despite requests for them to do so, thereby compelling the former to break into the office. The Eurocar Sales Office is obviously not a gun store and it is definitely not an armory or arsenal which are the usual depositories for explosives and ammunition. It is primarily and solely engaged in the sale of automobiles. The presence of an unusual quantity of high-powered firearms and explosives could not be justifiably or even colorably explained. In addition, there was general chaos and disorder at that time because of simultaneous and intense firing within the vicinity of the office and in the nearby Camp Aguinaldo which was under attack by rebel forces. The courts in the surrounding areas were obviously closed and, for that matter, the building and houses therein were deserted. Under circumstances, SC considered that the instant case falls under one of the exceptions to the prohibition against a warrantless search. In the first place, the military operatives, taking into account the facts obtaining in this case, had reasonable ground to believe that a crime was being committed. There was consequently more than sufficient probable cause to warrant their action. Furthermore, in the prevailing situation, the raiding team had no opportunity to apply for and secure a search warrant from the courts. The trial judge himself manifested that on December 5, 1989 when the raid was conducted, his court was closed. Under such urgency and exigency of the moment, a search warrant could lawfully be dispensed with.

PLAIN VIEW DOCTRINE PEOPLE VS. MUSA [217 SCRA 597; G.,R. NO. 96177; 27 JAN 1993] CONSTITUTIONAL LAW 2 CASE DIGESTS

Facts: A civilian informer gave the information that Mari Musa was engaged in selling marijuana in Suterville, Zamboanga City. Sgt. Ani was ordered by NARCOM leader T/Sgt. Belarga, to conduct a surveillance and test buy on Musa. The civilian informer guided Ani to Musa’s house and gave the description of Musa. Ani was able to buy one newspaper-wrapped dried marijuana for P10.00. The next day, a buy-bust was planned. Ani was to raise his right hand if he successfully buys marijuana from Musa. As Ani proceeded to the house, the NARCOM team positioned themselves about 90 to 100 meters away. From his position, Belarga could see what was going on. Musa came out of the house and asked Ani what he wanted. Ani said he wanted more marijuana and gave Musa the P20.00 marked money. Musa went into the house and came back, giving Ani two newspaper wrappers containing dried marijuana. Ani opened and inspected it. He raised his right hand as a signal to the other NARCOM agents, and the latter moved in and arrested Musa inside the house. Belarga frisked Musa in the living room but did not find the marked money (gave it to his wife who slipped away). T/Sgt. Belarga and Sgt. Lego went to the kitchen and found a ‘cellophane colored white and stripe hanging at the corner of the kitchen.’ They asked Musa about its contents but failed to get a response. So they opened it and found dried marijuana leaves inside. Musa was then placed under arrest. Issue: Whether or Not the seizure of the plastic bag and the marijuana inside it is unreasonable, hence, inadmissible as evidence. Held: Yes. It constituted unreasonable search and seizure thus it may not be admitted as evidence. The warrantless search and seizure, as an incident to a suspect’s lawful arrest, may extend beyond the person of the one arrested to include the premises or surroundings under his immediate control. Objects in the ‘plain view’ of an officer who has the right to be in the position to have that view are subject to seizure and may be presented as evidence. The ‘plain view’ doctrine is usually applied where a police officer is not searching for evidence against the accused, but nonetheless inadvertently comes across an incriminating object. It will not CONSTITUTIONAL LAW 2 CASE DIGESTS

justify the seizure of the object where the incriminating nature of the object is not apparent from the ‘plain view’ of the object. In the case at bar, the plastic bag was not in the ‘plain view’ of the police. They arrested the accused in the living room and moved into the kitchen in search for other evidences where they found the plastic bag. Furthermore, the marijuana inside the plastic bag was not immediately apparent from the ‘plain view’ of said object. Therefore, the ‘plain view’ does not apply. The plastic bag was seized illegally and cannot be presented in evidence pursuant to Article III Section 3 (2) of the Constitution PEOPLE OF THE PHILIPPINES vs HUANG ZHEN HUA G.R. No. 139301, September 29, 2004 Facts: Police operatives received word from their confidential informant that Peter Chan and Henry Lao, and appellants Jogy Lee and Huang Zhen Hua were engaged in illegal drug trafficking. The policemen also learned that Lee was handling the payments and accounting of the proceeds of the illegal drug trafficking activities of Lao and Chan. Officer Anciro, Jr. and other police operatives conducted surveillance operations and were able to verify that Lao and appellant Lee were living together as husband and wife. They were able to secure search warrants, one for violation of Presidential Decree (P.D.) No. 1866 (illegal possession of firearms and explosives) and two for violation of R.A No. 6425, as amended otherwise known as the Dangerous Drug Act. The implementation of the first Search Warrant, no persons were found in the are, however the policemen found two kilos shabu, paraphernalia for its production, and machines and tools apparently used for the production of fake credit cards. Thereafter, the police operatives received information that Lao and Chan would be delivering shabu. The policemen rushed to the area and saw Chan and Lao on board the latter’s car. Thereafter, the shoot-out resulted to death of the two suspect during the encounter. The policemen found two plastic bags, each containing one kilo of shabu, in Lao’s car. CONSTITUTIONAL LAW 2 CASE DIGESTS

The policemen then proceeded to the area where to enforce the other search warrant. The policemen coordinated with Antonio Pangan, the officer in charge of security in the building. The policemen, Pangan and two security guards proceeded to the condominium unit. Anciro, Jr. knocked repeatedly on the front door, but no one responded. Pangan, likewise, knocked on the door.9 until Lee peeped through the window beside the front door. The policemen allowed Pangan to communicate with appellant Lee by sign language and pointed their uniforms to her to show that they were policemen. The Lee then opened the door and allowed the team into the condominium unit. The policemen conducted the search in all the rooms within the unit. The team proceeded with the search and found other articles not described on the the search warrant. Huang Zhen Hua was found sleeping in one of the rooms during the search and was surprised to see police officers. Anciro, Jr. found two transparent plastic bags each containing one kilo of shabu, a feeding bottle, a plastic canister and assorted paraphernalia. Anciro, Jr. also found assorted documents, pictures, bank passbooks issued by the Allied Banking Corporation, credit cards, passports and identification cards of Lao and Lee. Anciro, Jr. told Lee to bring some of her clothes because they were bringing her to the PARAC headquarters. Lee did as she was told and took some clothes from the cabinet in the master’s bedroom where Anciro, Jr. had earlier found the shabu. Issue: Whether or not the articles not specified in the search warrant, are inadmissible evidence. Held: No, Admittedly, Anciro, Jr. seized and took custody of certain articles belonging to the appellant and Lao which were not described in the search warrants. However, the seizure of articles not listed in a search warrant does not render the seizure of the articles described and listed therein illegal; nor does it render inadmissible in evidence. Such articles were in plain view of Anciro, Jr. as he implemented the search warrants and was authorized to seize the said articles because of their close connection to the crime charged. An example of the applicability of the ‘plain view’ doctrine is the situation in which the police have a warrant to search a given area for specified objects, and in the course of the search come across some other article of incriminating character. An object that comes into view during a CONSTITUTIONAL LAW 2 CASE DIGESTS

search incident to arrest that is appropriately limited in scope under existing law may be seized without a warrant. Finally, the ‘plain view’ doctrine has been applied where a police officer is not searching for evidence against the accused, but nonetheless inadvertently comes across an incriminating object. It cannot be denied that the cards, passbook, passport and other documents and papers seen by the policemen have an intimate connection with the crime charged. The passport of the appellant would show when and how often she had been in and out of the country. Her credit cards and bank book would indicate how much money she had amassed while in the country and how she acquired or earned the same. The pictures and those of the other persons shown therein are relevant to show her relationship to Lao and Chan. The Supreme Court ruled that Huang Zhen Hua should be acquitted on the ground of reasonable doubt, but that the conviction of Lee should be affirmed.

PEOPLE OF THE PHILIPPINES vs. ZENAIDA BOLASA Y NAKOBOAN and ROBERTO DELOS REYES G.R. No. 125754 December 22, 1999 FACTS: An anonymous caller tipped off PO3 Dante Salonga and PO3 Albert Carizon in the early evening of 11 September 1995 that a man and a woman were repacking prohibited drugs at a certain house in Sta. Brigida St., Karuhatan, Valenzuela, Metro Manila. PO3 Salonga and PO3 Carizon together with SPO1 Fernando Arenas immediately proceeded to the house of the suspects and parked their car some three hundred (300) meters away. They walked towards their quarry's lair accompanied this time by their unnamed informer. When they reached the house they "peeped (inside) through a small window and x x x saw one man and a woman repacking suspected marijuana." They entered the house and introduced themselves as police officers to the occupants and thereupon confiscated the tea bags and some drug paraphernalia. They arrested the two (2) who turned out to be the accused Zenaida Bolasa y Nakoboan and Roberto delos Reyes. Subsequent examination of the tea bags by NBI Forensic Chemist Rubie Calalo confirmed the suspicion that the tea bags contained marijuana. . Both the accused however CONSTITUTIONAL LAW 2 CASE DIGESTS

denied on the witness stand ownership over the confiscated tea bags and drug implements. ISSUE: WoN the seizure and subsequent arrest were valid? HELD Yes. The Court held that the tea bags containing marijuana “were not seized in plain view or inadvertently discovered”. There was “no valid intrusion” and the accused were “illegally arrested”. The police officers “intentionally peeped first through the window before they saw and ascertained the activities of accused inside the room”. Further, the Court contended that the apprehending officers “should have conducted first a surveillance” considering that the identities and address of the suspected culprits were already ascertained. After conducting the surveillance and “determining the existence of probable cause” for arresting accused, they should have “secured a search warrant prior to effecting a valid arrest and seizure”. The Court stated that “the arrest being illegal ab initio, the accompanying search was likewise illegal”. Every evidence thus obtained during the illegal search cannot be used against accused. The Court held that the State “cannot in a cavalier fashion intrude into the persons of its citizens as well as into their houses, papers and effects”. The constitutional provision “protects the privacy and sanctity of the person himself against unlawful arrests and other forms of restraint”. Obiter Dictum The Court enumerated the exceptions as follows: 1. Warrantless search incidental to a lawful arrest; 2. Search of evidence in “plain view.” The elements of the plain view doctrine are: (a) a prior valid intrusion based on the valid warrantless arrest in which the police are legally present in the pursuit of their official duties; (b) the evidence was inadvertently discovered by the police who have the right to be where they are; (c) the evidence must be immediately apparent; and, (d) "plain view" justified mere seizure of evidence without further search. CONSTITUTIONAL LAW 2 CASE DIGESTS

3. Search of a moving vehicle. Highly regulated by the government, the vehicle’s inherent mobility reduces expectation of privacy especially when its transit in public thoroughfares furnishes a highly reasonable suspicion amounting to probable cause that the occupant committed a criminal activity; 4. Consented warrantless search; 5. Customs search; 6. Stop and Frisk; and 7. Exigent and emergency circumstances. Citing the Rules of Criminal Procedure on lawful warrantless arrest, the Court stated that an arrest is lawful even “in the absence of a warrant”: (a) when the person to be arrested has committed, is actually committing, or is about to commit an offense in his presence; (b) when an offense has in fact been committed and he has reasonable ground to believe that the person to be arrested has committed it; and, (c) when the person to be arrested is a prisoner who has escaped from a penal establishment or place where he is serving final judgment or temporarily confined while his case is pending, or has escaped while being transferred from one confinement to another. (A person charged with an offense may be searched for dangerous weapons or anything which may be used as proof of the commission of the offense).

EFFECTS OF NON-OBSERVANCE, ACQUITTAL? PEOPLE OF THE PHILIPPINES, G.R. No. 189806 vs FRANCISCO MANLANGIT y TRESBALLES, CONSTITUTIONAL LAW 2 CASE DIGESTS

Accused-Appellant. The Case This is an appeal from the August 28, 2009 Decision[1] of the Court of Appeals (CA) in CA-G.R. CR-H.C. No. 03273, which affirmed in toto the Decision dated July 12, 2007[2]in Criminal Case Nos. 03-4735 and 03-4961 of the Regional Trial Court (RTC), Branch 64 in Makati City. The RTC found accused-appellant Francisco Manlangit y Tresballes guilty of drug-sale and drug-use penalized by Republic Act No. (RA) 9165 or the Comprehensive Dangerous Drugs Act of 2002. The Facts On November 25, 2003, an information was filed charging Manlangit with violating Section 5, Article II of RA 9165, as follows: That on or about the 24th day of November 2003, in the City of Makati, Philippines, and within the jurisdiction of this Honorable Court, the above-named accused, not being lawfully authorized by law, did then and there willfully and feloniously sell, give away, distribute and deliver zero point zero four (0.04) gram of Methylamphetamine Hydrochloride (shabu), which is a dangerous drug.[3] On December 11, 2003, another information was filed against Manlangit for breach of Sec. 15, Art. II of RA 9165, to wit: That sometime on or before or about the 24th day of November 2003, in the City of Makati, Philippines, and within the jurisdiction of this Honorable Court, the above-named accused, not being authorized by law to use dangerous drugs, and having been arrested and found positive for use of Methylamphetamine, after a confirmatory test, did then and there willfully, unlawfully and feloniously use Methylamphetamine, a dangerous drug in violation of the said law.[4] During the arraignment for both cases, Manlangit pleaded not guilty. Afterwards, the cases were tried jointly. At the trial of the case, the prosecution adduced evidence as follows: CONSTITUTIONAL LAW 2 CASE DIGESTS

On November 24, 2003, the Makati Anti-Drug Abuse Council (MADAC) Cluster 4 office received information from an informant that a certain Negro was selling prohibited drugs along Col. Santos Street at Brgy. South Cembo, Makati City. The MADAC thereafter coordinated with the Anti-Illegal Drugs Special Operations Task Force (AIDSTOF) and the Philippine Drug Enforcement Agency to conduct a joint MADAC-police buy-bust operation. A team was assembled composed of several members of the different offices, among which Police Officer 2 Virginio Costa was designated as the team leader, with MADAC operative Wilfredo Serrano as the poseur-buyer and Roberto Bayona as his back-up. The team prepared buy-bust money for the operation, marking two (2) one hundred peso (PhP 100) bills with the initials AAM. Upon arrival on Col. Santos Street, Brgy. Cembo, Makati City, the team spotted Manlangit standing in front of his house. The informant approached Manlangit and convinced the latter that Serrano wanted to purchase shabu from him. Manlangit asked Serrano how much shabu he wanted, to which Serrano replied that he wanted two hundred pesos (PhP 200) worth of shabu. Manlangit went inside his house and later reappeared with a plastic sachet containing a white crystalline substance. Manlangit handed over the plastic sachet to Serrano who, in turn, gave Manlangit the marked money. Then Serrano gave the pre-arranged signal of lighting a cigarette to indicate to the rest of the team that the buy-bust operation had been consummated. Thus, the rest of the team approached Manlangit and proceeded to arrest him while informing him of constitutional rights and the reason for his arrest. The marked money was recovered from Manlangits pocket. The plastic sachet was then marked with the initials FTM and sent to the Philippine National Police (PNP) crime laboratory in Camp Crame, Quezon City for analysis. The PNP crime laboratory identified the white crystalline substance as Methylamphetamine Hydrochloride in Chemistry Report No. D-119003. Manlangit was also brought to the PNP crime laboratory for a drug test, which yielded a positive result for use of Methylamphetamine Hydrochloride.[5] Manlangit denied that such buy-bust operation was conducted and claimed that the recovered shabu was not from him. He claimed that he was pointed out by a certain Eli Ballesteros to Serrano and Bayona. Thereafter, he was allegedly detained at the Barangay Hall of Brgy. Pitogo. There, he was allegedly interrogated by Serrano as to the location of the shabu and its proceeds, as well as the identity CONSTITUTIONAL LAW 2 CASE DIGESTS

of the drug pushers in the area. He also claimed that whenever he answered that he did not know what Serrano was talking about, he was boxed in the chest. Later on, he said that he was brought to Camp Crame for drug testing.[6] On July 12, 2007, the RTC rendered a Decision, the dispositive portion of which reads:

WHEREFORE, premises considered, judgment is hereby rendered as follows: 1) In Criminal Case No. 03-4735, finding accused Francisco Manlangit y Tresballes GUILTY BEYOND REASONABLE DOUBT of Violation of Section 5, Art II, RA 9165 (drug-sale) and sentencing him to suffer the penalty of life imprisonment and to pay a fine in the amount of P500,000.00. Said accused shall be given credit for the period of his preventive detention. 2) In Criminal Case No. 03-4735,[7] finding accused Francisco Manlangit y Tresballes GUILTY BEYOND REASONABLE DOUBT of Violation of Section 15, Art II, RA 9165 (drug-use), and sentencing him to undergo rehabilitation for at least six (6) months in a government rehabilitation Center under the auspices of the Bureau of Correction subject to the provisions of Article VIII, RA 9165. It is further ordered that the plastic sachet containing shabu, subject of Criminal Case No. 03-4735, be transmitted to the Philippine Drug Enforcement Agency (PDEA) for the latters appropriate action. SO ORDERED.[8] From such Decision, Manlangit interposed an appeal with the CA. In his Brief, accused-appellant Manlangit claimed that the prosecution failed to prove his guilt beyond reasonable doubt. To support such contention, accusedappellant claimed that there was no buy-bust operation conducted. He pointed out that he was not in the list of suspected drug pushers of MADAC or of the AIDSTOF. He further emphasized that the buy-bust operation was conducted CONSTITUTIONAL LAW 2 CASE DIGESTS

without first conducting a surveillance or test buy to determine the veracity of the report made by the informant. He assailed the fact that despite knowledge of his identity and location, the buy-bust team failed to secure even a search warrant. Accused-appellant also raised the issue that the buy-bust team failed to comply with the procedure for the custody and control of seized prohibited drugs under Sec. 21 of RA 9165. He argued that the presumption of regularity in the performance of official function was overturned by the officers failure to follow the required procedure in the conduct of a buy-bust operation, as well as the procedure in the proper disposition, custody, and control of the subject specimen. On August 28, 2009, the CA rendered the decision which affirmed the RTCs Decision dated July 12, 2007. It ruled that contrary to accused-appellants contention, prior surveillance is not a prerequisite for the validity of a buy-bust operation. The case was a valid example of a warrantless arrest, accused-appellant having been caught in flagrante delicto. The CA further stated that accusedappellants unsubstantiated allegations are insufficient to show that the witnesses for the prosecution were actuated by improper motive, in this case the members of the buy-bust team; thus, their testimonies are entitled to full faith and credit. After examining the testimonies of the witnesses, the CA found them credible and found no reason to disturb the RTCs findings. Finally, the CA found that chain of evidence was not broken. Hence, the instant appeal. In a Manifestation (In lieu of Supplemental Brief) dated February 22, 2010, accused-appellant expressed his desire not to file a supplemental brief and reiterated the same arguments already presented before the trial and appellate courts. The Issues The issues, as raised in the Brief for the Accused-Appellant dated September 29, 2008, are: 1. The Court a quo gravely erred in convicting the accused-appellant despite the prosecutions failure to prove his built beyond reasonable doubt.[9] CONSTITUTIONAL LAW 2 CASE DIGESTS

2. The Court a quo gravely erred in finding that the procedure for the custody and control of prohibited drugs was complied with.[10] The Ruling of the Court The appeal is bereft of merit. First Issue: Accused-appellants guilt was proved beyond reasonable doubt The first paragraph of Sec. 5 of RA 9165 punishes the act of selling dangerous drugs. It provides: Section 5. Sale, Trading, Administration, Dispensation, Delivery, Distribution and Transportation of Dangerous Drugs and/or Controlled Precursors and Essential Chemicals.The penalty of life imprisonment to death and a fine ranging from Five hundred thousand pesos (P500,000.00) to Ten million pesos (P10,000,000.00) shall be imposed upon any person, who, unless authorized by law, shall sell, trade, administer, dispense, deliver, give away to another, distribute, dispatch in transit or transport any dangerous drug, including any and all species of opium poppy regardless of the quantity and purity involved, or shall act as a broker in any of such transactions. (Emphasis supplied.) While Sec. 15, RA 9165 states: Section 15. Use of Dangerous Drugs.A person apprehended or arrested, who is found to be positive for use of any dangerous drug, after a confirmatory test, shall be imposed a penalty of a minimum of six (6) months rehabilitation in a government center for the first offense, subject to the provisions of Article VIII of this Act. If apprehended using any dangerous drug for the second time, he/she shall suffer the penalty of imprisonment ranging from six (6) years and one (1) day to twelve (12) years and a fine ranging from Fifty thousand pesos (P50,000.00) to Two hundred thousand pesos (P200,000.00): Provided, That this Section shall not be applicable where the person tested is also found to have in his/her possession CONSTITUTIONAL LAW 2 CASE DIGESTS

such quantity of any dangerous drug provided for under Section 11 of this Act, in which case the provisions stated therein shall apply. (Emphasis supplied.) People v. Macatingag[11] prescribed the requirements for the successful prosecution of the crime of illegal sale of dangerous drugs, as follows. The elements necessary for the prosecution of illegal sale of drugs are (1) the identity of the buyer and the seller, the object, and consideration; and (2) the delivery of the thing sold and the payment therefor. What is material to the prosecution for illegal sale of dangerous drugs is the proof that the transaction or sale actually took place, coupled with the presentation in court of evidence of corpus delicti. The pieces of evidence found in the records amply demonstrate that all the elements of the crimes charged were satisfied. The lower courts gave credence to the prosecution witnesses testimonies, which established the guilt of accusedappellant for the crimes charged beyond reasonable doubt. The testimoniesparticularly those of the police officers involved, which both the RTC and the CA found credibleare now beyond question. As the Court ruled in Aparis v. People:[12] As to the question of credibility of the police officers who served as principal witnesses for the prosecution, settled is the rule that prosecutions involving illegal drugs depend largely on the credibility of the police officers who conducted the buy-bust operation. It is a fundamental rule that findings of the trial courts which are factual in nature and which involve credibility are accorded respect when no glaring errors; gross misapprehension of facts; or speculative, arbitrary, and unsupported conclusions can be gathered from such findings. The reason for this is that the trial court is in a better position to decide the credibility of witnesses, having heard their testimonies and observed their deportment and manner of testifying during the trial. The rule finds an even more stringent application where said findings are sustained by the Court of Appeals, as in the present case.

CONSTITUTIONAL LAW 2 CASE DIGESTS

Moreover, accused-appellants defense of denial, without substantial evidence to support it, cannot overcome the presumption of regularity of the police officers performance of official functions. Thus, the Court ruled in People v. Llamado:[13] In cases involving violations of Dangerous Drugs Act, credence should be given to the narration of the incident by the prosecution witnesses especially when they are police officers who are presumed to have performed their duties in a regular manner, unless there be evidence to the contrary. Moreover, in the absence of proof of motive to falsely impute such a serious crime against the appellant, the presumption of regularity in the performance of official duty, as well as the findings of the trial court on the credibility of witnesses, shall prevail over appellants self-serving and uncorroborated denial. (Emphasis supplied.) Contrary to accused-appellants challenge to the validity of the buy-bust operation, the Court categorically stated in Quinicot v. People that a prior surveillance or test buy is not required for a valid buy-bust operation, as long as the operatives are accompanied by their informant, thus: Settled is the rule that the absence of a prior surveillance or test buy does not affect the legality of the buy-bust operation. There is no textbook method of conducting buy-bust operations. The Court has left to the discretion of police authorities the selection of effective means to apprehend drug dealers. A prior surveillance, much less a lengthy one, is not necessary, especially where the police operatives are accompanied by their informant during the entrapment. Flexibility is a trait of good police work. We have held that when time is of the essence, the police may dispense with the need for prior surveillance. In the instant case, having been accompanied by the informant to the person who was peddling the dangerous drugs, the policemen need not have conducted any prior surveillance before they undertook the buy-bust operation.[14] (Emphasis supplied.) Furthermore, accused-appellants contention that the buy-bust team should have procured a search warrant for the validity of the buy-bust operation is misplaced. The Court had the occasion to address this issue in People v. Doria:[15]

CONSTITUTIONAL LAW 2 CASE DIGESTS

We also hold that the warrantless arrest of accused-appellant Doria is not unlawful. Warrantless arrests are allowed in three instances as provided by Section 5 of Rule 113 of the 1985 Rules on Criminal Procedure, to wit: Sec. 5. Arrest without warrant; when lawful.A peace officer or a private person may, without a warrant, arrest a person: (a) When, in his presence, the person to be arrested has committed, is actually committing, or is attempting to commit an offense; (b) When an offense has in fact just been committed, and he has personal knowledge of facts indicating that the person to be arrested has committed it; and (c) When the person to be arrested is a prisoner who escaped from a penal establishment or place where he is serving final judgment or temporarily confined while his case is pending, or has escaped while being transferred from one confinement to another. Under Section 5 (a), as above-quoted, a person may be arrested without a warrant if he has committed, is actually committing, or is attempting to commit an offense. Appellant Doria was caught in the act of committing an offense. When an accused is apprehended in flagrante delicto as a result of a buy-bust operation, the police are not only authorized but duty-bound to arrest him even without a warrant. The Court reiterated such ruling in People v. Agulay:[16] Accused-appellant contends his arrest was illegal, making the sachets of shabu allegedly recovered from him inadmissible in evidence. Accused-appellants claim is devoid of merit for it is a well-established rule that an arrest made after an entrapment operation does not require a warrant inasmuch as it is considered a valid warrantless arrest, in line with the provisions of Rule 113, Section 5(a) of the Revised Rules of Court, to wit: Section 5. Arrest without warrant; when lawful.A peace officer or a private person may, without a warrant, arrest a person: CONSTITUTIONAL LAW 2 CASE DIGESTS

(a) When, in his presence, the person to be arrested has committed, is actually committing, or is attempting to commit an offense. A buy-bust operation is a form of entrapment which in recent years has been accepted as a valid and effective mode of apprehending drug pushers. In a buybust operation, the idea to commit a crime originates from the offender, without anybody inducing or prodding him to commit the offense. If carried out with due regard for constitutional and legal safeguards, a buy-bust operation deserves judicial sanction. Second Issue: The chain of custody of the seized drug was unbroken Accused-appellant contends that the arresting officers did not comply with the requirements for the handling of seized dangerous drugs as provided for under Sec. 21(1) of RA 9165: Section 21. Custody and Disposition of Confiscated, Seized, and/or Surrendered Dangerous Drugs, Plant Sources of Dangerous Drugs, Controlled Precursors and Essential Chemicals, Instruments/Paraphernalia and/or Laboratory Equipment.The PDEA shall take charge and have custody of all dangerous drugs, plant sources of dangerous drugs, controlled precursors and essential chemicals, as well as instruments/paraphernalia and/or laboratory equipment so confiscated, seized and/or surrendered, for proper disposition in the following manner: (1) The apprehending team having initial custody and control of the drugs shall, immediately after seizure and confiscation, physically inventory and photograph the same in the presence of the accused or the person/s from whom such items were confiscated and/or seized, or his/her representative or counsel, a representative from the media and the Department of Justice (DOJ), and any elected public official who shall be required to sign the copies of the inventory and be given a copy thereof; (Emphasis supplied.) In particular, accused-appellant argues that:

CONSTITUTIONAL LAW 2 CASE DIGESTS

While the marking of the specimen was done in the place of incident by MADAC operative Soriano, the inventory of the item was done at Cluster 4. There was no photograph made of the plastic sachet in the presence of the accused, media, any elected local official, or the DOJ representatives, in clear violation of Section 21, R.A. No. 9165.[17] Based on such alleged failure of the buy-bust team to comply with the procedural requirements of Sec. 21, RA 9165, accused-appellant posits that he should, therefore, be acquitted. Such reasoning is flawed. In People v. Rosialda,[18] the Court addressed the issue of chain of custody of dangerous drugs, citing People v. Rivera, as follows: Anent the second element, Rosialda raises the issue that there is a violation of Sec. 21, Art. II of RA 9165, particularly the requirement that the alleged dangerous drugs seized by the apprehending officers be photographed in the presence of the accused or the person/s from whom such items were confiscated and/or seized, or his/her representative or counsel. Rosialda argues that such failure to comply with the provision of the law is fatal to his conviction. This contention is untenable. The Court made the following enlightening disquisition on this matter in People v. Rivera: The procedure to be followed in the custody and handling of seized dangerous drugs is outlined in Section 21, paragraph 1, Article II of Republic Act No. 9165 which stipulates: (1) The apprehending team having initial custody and control of the drugs shall, immediately after seizure and confiscation, physically inventory and photograph the same in the presence of the accused or the person/s from whom such items were confiscated and/or seized, or his/her representative or counsel, a representative from the media and the Department of Justice (DOJ), and any elected public official who shall be required to sign the copies of the inventory and be given a copy thereof.

CONSTITUTIONAL LAW 2 CASE DIGESTS

The same is implemented by Section 21(a), Article II of the Implementing Rules and Regulations of Republic Act No. 9165, viz.: (a) The apprehending team having initial custody and control of the drugs shall, immediately after seizure and confiscation, physically inventory and photograph the same in the presence of the accused or the person/s from whom such items were confiscated and/or seized, or his/her representative or counsel, a representative from the media and the Department of Justice (DOJ), and any elected public official who shall be required to sign the copies of the inventory and be given a copy thereof: Provided, further, that non-compliance with these requirements under justifiable grounds, as long as the integrity and the evidentiary value of the seized items are properly preserved by the apprehending officer/team, shall not render void and invalid such seizures of and custody over said items. The failure of the prosecution to show that the police officers conducted the required physical inventory and photograph of the evidence confiscated pursuant to said guidelines, is not fatal and does not automatically render accusedappellants arrest illegal or the items seized/confiscated from him inadmissible. Indeed, the implementing rules offer some flexibility when a proviso added that non-compliance with these requirements under justifiable grounds, as long as the integrity and the evidentiary value of the seized items are properly preserved by the apprehending officer/team, shall not render void and invalid such seizures of and custody over said items. The same provision clearly states as well, that it must still be shown that there exists justifiable grounds and proof that the integrity and evidentiary value of the evidence have been preserved. This Court can no longer find out what justifiable reasons existed, if any, since the defense did not raise this issue during trial. Be that as it may, this Court has explained in People v. Del Monte that what is of utmost importance is the preservation of the integrity and evidentiary value of the seized items, as the same would be utilized in the determination of the guilt or innocence of the accused. The existence of the dangerous drug is a condition sine qua non for conviction for the illegal sale of dangerous drugs. The dangerous drug itself constitutes the very corpus delicti of the crime and the fact of its existence is vital to a judgment of conviction. Thus, it is essential that the identity of the prohibited drug be established beyond doubt. The chain of custody requirement performs CONSTITUTIONAL LAW 2 CASE DIGESTS

the function of ensuring that the integrity and evidentiary value of the seized items are preserved, so much so that unnecessary doubts as to the identity of the evidence are removed. To be admissible, the prosecution must show by records or testimony, the continuous whereabouts of the exhibit at least between the time it came into possession of the police officers and until it was tested in the laboratory to determine its composition up to the time it was offered in evidence. (Emphasis supplied.) Here, accused-appellant does not question the unbroken chain of evidence. His only contention is that the buy-bust team did not inventory and photograph the specimen on site and in the presence of accused-appellant or his counsel, a representative from the media and the Department of Justice, and any elected public official. However, as ruled by the Court in Rosialda, as long as the chain of custody remains unbroken, even though the procedural requirements provided for in Sec. 21 of RA 9165 was not faithfully observed, the guilt of the accused will not be affected. And as aptly ruled by the CA, the chain of custody in the instant case was not broken as established by the facts proved during trial, thus: Lastly, the contention of appellant, that the police officers failed to comply with the provisions of paragraph 1, Section 21 of R.A. No. 9165 for the proper procedure in the custody and disposition of the seized drugs, is untenable. Record shows that Serrano marked the confiscated sachet of shabu in the presence of appellant at the place of incident and was turned over properly to the investigating officer together with the marked buy-bust money. Afterwards, the confiscated plastic sachet suspected to be containing shabu was brought to the forensic chemist for examination. Likewise, the members of the buy-bust team executed their Pinagsanib na Salaysay sa Pag-aresto immediately after the arrest and at the trial, Serrano positively identified the seized drugs. Indeed, the prosecution evidence had established the unbroken chain of custody of the seized drugs from the buy-bust team, to the investigating officer and to the forensic chemist. Thus, there is no doubt that the prohibited drug presented before the court a quo was the one seized from appellant and that indeed, he committed the crimes imputed against him. CONSTITUTIONAL LAW 2 CASE DIGESTS

WHEREFORE, the appeal is DENIED. The CAs August 28, 2009 Decision in CA-G.R. CR-H.C. No. 03273 is hereby AFFIRMED IN TOTO. ENFORCEMENT OF FISHING, CUSTOMS & IMMIGRATION PEOPLE VS. LEILA JOHNSON Facts: Leila Johnson was arrested at the airport after she was found to have in her possession more than 500 grams of shabu when she was initially frisked by a security personnel at a gate in the airport. The security personnel felt something hard in respondent’s abdominal area and when asked she said that she had to wear 2 girdles because of an operation. Unconvinced, the security personnel went to her supervisor. Subsequently, after a thorough search on respondent, packets of shabu were seized from her. Accused (respondent) was subsequently convicted and sentenced to reclusion perpetua. In the present appeal, respondent contended that the search made upon her was not valid and that her constitutional rights were infringed when such search was conducted. Issue: WON a valid search was made. Held: The constitutional right of the accused was not violated as she was never placed under custodial investigation but was validly arrested without warrant pursuant to the provisions of Section 5, Rule 113 of the 1985 Rules of Criminal Procedure which provides: Sec. 5. Arrest without warrant; when lawful. A peace officer or a private person may, without a warrant, arrest a person: (a) when in his presence, the person to be arrested has committed, is actually committing, or is attempting to commit an offense; CONSTITUTIONAL LAW 2 CASE DIGESTS

(b) when an offense has in fact just been committed, and he has personal knowledge of facts indicating that the person to be arrested has committed it; and… The circumstances surrounding the arrest of the accused above falls in either paragraph (a) or (b) of the Rule above cited, hence the allegation that she has been subjected to custodial investigation is far from being accurate.[18] The methamphetamine hydrochloride seized from her during the routine frisk at the airport was acquired legitimately pursuant to airport security procedures. Persons may lose the protection of the search and seizure clause by exposure of their persons or property to the public in a manner reflecting a lack of subjective expectation of privacy, which expectation society is prepared to recognize as reasonable. Such recognition is implicit in airport security procedures. With increased concern over airplane hijacking and terrorism has come increased security at the nation’s airports. Passengers attempting to board an aircraft routinely pass through metal detectors; their carry-on baggages as well as checked luggage are routinely subjected to x-ray scans. Should these procedures suggest the presence of suspicious objects, physical searches are conducted to determine what the objects are. There is little question that such searches are reasonable, given their minimal intrusiveness, the gravity of the safety interests involved, and the reduced privacy expectations associated with airline travel. Indeed, travelers are often notified through airport public address systems, signs, and notices in their airline tickets that they are subject to search and, if any prohibited materials or substances are found, such would be subject to seizure. These announcements place passengers on notice that ordinary constitutional protections against warrantless searches and seizures do not apply to routine airport procedures. The packs of methamphetamine hydrochloride having thus been obtained through a valid warrantless search, they are admissible in evidence against the accused-appellant herein. Corollarily, her subsequent arrest, although likewise without warrant, was justified since it was effected upon the discovery and recovery of “shabu” in her person in flagrante delicto.

HOW ABOUT INSPECTION AT AIRPORTS? People vs Canton CONSTITUTIONAL LAW 2 CASE DIGESTS

G.R. No. 148825

December 27, 2002

FACTS: Appellant Susan Canton was charged before the Regional Trial Court of Pasay City with the violation of Section 16 of Article III of the Dangerous Drugs Act of 1972 (Republic Act No. 6425), as amended, under an Information whose accusatory portion reads as follows: That on February 12, 1998 at the Ninoy Aquino International Airport, and within the jurisdiction of this Honorable Court, the above named accused did then and there willfully, unlawfully and feloniously has in her possession NINE HUNDRED NINETY EIGHT POINT TWO EIGHT HUNDRED ZERO NINE (998.2809) GRAMS of methamphetamine hydrochloride, a regulated drug, without the corresponding prescription or license. Unsatisfied with the decision of the trial court, SUSAN imputing to the trial court the following errors: (1) in justifying the warrantless search against her based on the alleged existence of probable cause; (2) in holding that she was caught flagrante delicto and that the warrantless search was incidental to a lawful arrest; (3) in not ruling that the frisker went beyond the limits of the “Terry search” doctrine; (4) in not ruling that SUSAN was under custodial investigation without counsel; (5) in admitting to the records of the case the report of Dr. Ma. Bernadette Arcena, which was not testified on or offered in evidence, and using the same in determining her guilt; (6) in justifying under the rule on judicial notice its cognizance of the medical report that has not been offered in evidence; and (7) in applying the ruling in People v. Johnson. ISSUE: Whether or not the warrantless search and subsequent seizure of the regulated drugs, as well as the arrest of Susan were violative of her constitutional rights. RULING: No, the search was made pursuant to routine airport security procedure, which is allowed under Section 9 of Republic Act No. 6235, “ Every ticket issued to a CONSTITUTIONAL LAW 2 CASE DIGESTS

passenger by the airline or air carrier concerned shall contain among others the following condition printed thereon: “Holder hereof and his hand-carried luggage(s) are subject to search for , and seizure of, prohibited materials or substances. Holder refusing to be searched shall not be allowed to board the aircraft,” which shall constitute a part of the contract between the passenger and the air carrier STOP AND FRISKS People v Solayao; G.R. No. 119220; 20 Sep 1996; 262 SCRA 255 FACTS: While conducting an intelligence patrol, the arresting officers met appellant who was drunk and wearing a camouflage uniform or a jungle suit. After introducing himself as PC, the police officer seized the dried coconut leaves which appellant was carrying and found wrapped in it a 49-inch long homemade firearm locally known as “latong.” The latter had no permission to possess the same. The firearm was confiscated and he was turned over to the custody of the Caibiran police who subsequently investigated him and charged him with illegal possession of firearm. ISSUE(S): Whether or not the trial court erred in admitting in evidence the homemade firearm which is a product of an unlawful warrantless search. HELD: NO. Solayao and his companion’s drunken actuations as well as the fact that Solayao was attired in camouflage uniform or a jungle suit and the flight of his companions aroused the suspicion of the police officer’s group. MALACAT vs. CA Facts: Petitioner was arrested for having in his possession a hand grenade after he was searched by a group of policemen when he was said to be acting suspiciously when he was hanging around Plaza Miranda with his eyes moving fast together with other Muslim-looking men. When the policemen approached the group of CONSTITUTIONAL LAW 2 CASE DIGESTS

men, they scattered in all directions which prompted the police to give chase and petitioner was then apprehended and a search was made on his person. He was then convicted under PD 1866 in the lower court. Hence, the present petition wherein petitioner contended that the lower court erred in holding that the search made on him and the seizure of the hand grenade from him was an appropriate incident to his arrest and that it erred in admitting the hand grenade as evidence since it was admissible because it was a product of an unreasonable and illegal search. Issue: WON the search and seizure conducted by the police was valid. Held: The general rule as regards arrests, searches and seizures is that a warrant is needed in order to validly effect the same. 31 The Constitutional prohibition against unreasonable arrests, searches and seizures refers to those effected without a validly issued warrant, 32 subject to certain exceptions. As regards valid warrantless arrests, these are found in Section 5, Rule 113 of the Rules of Court, which reads, in part: Sec. 5. — Arrest, without warrant; when lawful — A peace officer or a private person may, without a warrant, arrest a person: (a) When, in his presence, the person to be arrested has committed, is actually committing, or is attempting to commit an offense; (b) When an offense has in fact just been committed, and he has personal knowledge of facts indicating that the person to be arrested has committed it; and (c) When the person to be arrested is a prisoner who has escaped . . . A warrantless arrest under the circumstances contemplated under Section 5(a) has been denominated as one "in flagrante delicto," while that under Section 5(b) has been described as a "hot pursuit" arrest. Turning to valid warrantless searches, they are limited to the following: (1) customs searches; (2) search of moving vehicles; (3) seizure of evidence in plain view; (4) consent searches; 33 (5) a search incidental to a lawful arrest;34 and (6) a "stop and frisk.’ At the outset, we note that the trial court confused the concepts of a "stop-andfrisk" and of a search incidental to a lawful arrest. These two types of warrantless searches differ in terms of the requisite quantum of proof before they may be validly effected and in their allowable scope. CONSTITUTIONAL LAW 2 CASE DIGESTS

In a search incidental to a lawful arrest, as the precedent arrest determines the validity of the incidental search, the legality of the arrest is questioned in a large majority of these cases, e.g., whether an arrest was merely used as a pretext for conducting a search. 36 In this instance, the law requires that there first be a lawful arrest before a search can be made — the process cannot be reversed. 37 At bottom, assuming a valid arrest, the arresting officer may search the person of the arrestee and the area within which the latter may reach for a weapon or for evidence to destroy, and seize any money or property found which was used in the commission of the crime, or the fruit of the crime, or that which may be used as evidence, or which might furnish the arrestee with the means of escaping or committing violence. We now proceed to the justification for and allowable scope of a "stop-and-frisk" as a "limited protective search of outer clothing for weapons," as laid down in Terry, thus: We merely hold today that where a police officer observes unusual conduct which leads him reasonably to conclude in light of his experience that criminal activity may be afoot and that the persons with whom he is dealing may be armed and presently dangerous, where in the course of investigating this behavior he identifies himself as a policeman and makes reasonable inquiries, and where nothing in the initial stages of the encounter serves to dispel his reasonable fear for his own or others' safety, he is entitled for the protection of himself and others in the area to conduct a carefully limited search of the outer clothing of such persons in an attempt to discover weapons which might be used to assault him. Such a search is a reasonable search under the Fourth Amendment . . Other notable points of Terry are that while probable cause is not required to conduct a "stop and frisk," it nevertheless holds that mere suspicion or a hunch will not validate a "stop and frisk." A genuine reason must exist, in light of the police officer's experience and surrounding conditions, to warrant the belief that the person detained has weapons concealed about him. Finally, a "stop-and-frisk" serves a two-fold interest: (1) the general interest of effective crime prevention and detection, which underlies the recognition that a police officer may, under appropriate circumstances and in an appropriate manner, approach a person for purposes of investigating possible criminal behavior even without probable cause; and (2) the more pressing interest of safety and self-preservation which permit the police officer to take steps to assure himself that the person with whom he deals is not armed with a deadly weapon that could unexpectedly and fatally be used against the police officer. CONSTITUTIONAL LAW 2 CASE DIGESTS

PEOPLE V. MENGOTE [210 SCRA 174; G.R. NO. 87059; 22 JUN 1992] Facts: The Western Police District received a telephone call from an informer that there were three suspicious looking persons at the corner of Juan Luna and North Bay Boulevard in Tondo, Manila. A surveillance team of plainclothesmen was forthwith dispatched to the place. The patrolmen saw two men looking from side to side, one of whom holding his abdomen. They approached the persons and identified themselves as policemen, whereupon the two tried to run but unable to escape because the other lawmen surrounded them. The suspects were then searched. One of them the accused-appellant was found with a .38 caliber with live ammunitions in it, while his companion had a fan knife. The weapons were taken from them and they were turned over to the police headquarters for investigation. An information was filed before the RTC convicting the accused of illegal possession of firearm arm. A witness testified that the weapon was among the articles stolen at his shop, which he reported to the police including the revolver. For his part, Mengote made no effort to prove that he owned the fire arm or that he was licensed to possess it but instead, he claimed that the weapon was planted on him at the time of his arrest. He was convicted for violation of P.D.1866 and was sentenced to reclusion perpetua. In his appeal he pleads that the weapon was not admissible as evidence against him because it had been illegally seized and therefore the fruit of a poisonous tree. Issue: Whether or not the warrantless search and arrest was illegal. Held: An evidence obtained as a result of an illegal search and seizure inadmissible in any proceeding for any purpose as provided by Art. III sec 32 of the Constitution. Rule 113 sec.5 of the Rules of Court, provides arrest without warrant lawful when: (a) the person to be arrested has committed, is actually committing, or is attempting to commit an offense, (b) when the offense in fact has just been committed, and he has personal knowledge of the facts indicating the person arrested has committed it and (c) the person to be arrested has escaped from a CONSTITUTIONAL LAW 2 CASE DIGESTS

penal establishment or a place where he is serving final judgment or temporarily confined while his case is pending, or has escaped while being transferred from one confinement to another. These requirements have not been established in the case at bar. At the time of the arrest in question, the accused appellant was merely looking from side to side and holding his abdomen, according to the arresting officers themselves. There was apparently no offense that has just been committed or was being actually committed or at least being attempt by Mengote in their presence. Moreover a person may not be stopped and frisked in a broad daylight or on a busy street on unexplained suspicion. Judgment is reversed and set aside. Accused-appellant is acquitted. SEARCHING OF MOVING VEHICLE PAPA VS. MAGO Facts: Mago, the owner of the goods that were seized, when the truck transporting the goods was intercepted by the BOC, questioned the validity of the search conducted by them since it was made without any search warrant and whether the BOC has jurisdiction over the forfeited goods. Issue: Was the search conducted by the BOC valid? Held: Petitioner Martin Alagao and his companion policemen had authority to effect the seizure without any search warrant issued by a competent court. The Tariff and Customs Code does not require said warrant in the instant case. The Code authorizes persons having police authority under Section 2203 of the Tariff and Customs Code to enter, pass through or search any land, inclosure, warehouse, store or building, not being a dwelling house; and also to inspect, search and examine any vessel or aircraft and any trunk, package, or envelope or any person on board, or to stop and search and examine any vehicle, beast or person suspected of holding or conveying any dutiable or prohibited article introduced into the Philippines contrary to law, without mentioning the need of a search warrant in said cases. 16 But in the search of a dwelling house, the Code provides CONSTITUTIONAL LAW 2 CASE DIGESTS

that said "dwelling house may be entered and searched only upon warrant issued by a judge or justice of the peace. . . ." 17 It is our considered view, therefor, that except in the case of the search of a dwelling house, persons exercising police authority under the customs law may effect search and seizure without a search warrant in the enforcement of customs laws. In, Carroll vs US, it was made lawful for customs officers not only to board and search vessels within their own and adjoining districts, but also to stop, search and examine any vehicle, beast or person on which or whom they should suspect there was merchandise which was subject to duty, or had been introduced into the United States in any manner contrary to law, whether by the person in charge of the vehicle or beast or otherwise, and if they should find any goods, wares, or merchandise thereon, which they had probably cause to believe had been so unlawfully brought into the country, to seize and secure the same, and the vehicle or beast as well, for trial and forfeiture. SALVADOR VS. PEOPLE Facts: Petitioners, PAL ground crew employees were allegedly caught with dutiable goods (branded watches, etc) after PAF officers were observing their conduct and found it to be suspicious during a special mission given to them to make a routine surveillance to check on reports of alleged trafficking and smuggling being facilitated by PAL employees. Petitioners were convicted. Hence, the present petition. Issue: WON the seized items are admissible in evidence. Held: Here, it should be noted that during the incident in question, the special mission of the PAF operatives was to conduct a surveillance operation to verify reports of drug trafficking and smuggling by certain PAL personnel in the vicinity of the airport. In other words, the search made by the PAF team on petitioner and his coaccused was in the nature of a customs search. As such, the team properly effected the search and seizure without a search warrant since it exercised police authority under the customs law. CONSTITUTIONAL LAW 2 CASE DIGESTS

In Papa vs. Mago, involving a customs search, we held that law enforcers who are tasked to effect the enforcement of the customs and tariff laws are authorized to search and seize, without a search warrant, any article, cargo or other movable property when there is reasonable cause to suspect that the said items have been introduced into the Philippines in violation of the tariff and customs law. They may likewise conduct a warrantless search of any vehicle or person suspected of holding or conveying the said articles, as in the case at bar. In short, Mago clearly recognizes the power of the State to foil any fraudulent schemes resorted to by importers who evade payment of customs duties. The Government’s policy to combat the serious malady of smuggling cannot be reduced to futility and impotence on the ground that dutiable articles on which the duty has not been paid are entitled to the same Constitutional protection as an individual’s private papers and effects. Here, we see no reason not to apply this State policy which we have continued to affirm.

PEOPLE VS. ARUTA [288 SCRA 626; G.R. NO. 120515; 13 APR 1998] Facts: On Dec. 13, 1988, P/Lt. Abello was tipped off by his informant that a certain “Aling Rosa” will be arriving from Baguio City with a large volume of marijuana and assembled a team. The next day, at the Victory Liner Bus terminal they waited for the bus coming from Baguio, when the informer pointed out who “Aling Rosa” was, the team approached her and introduced themselves as NARCOM agents. When Abello asked “aling Rosa” about the contents of her bag, the latter handed it out to the police. They found dried marijuana leaves packed in a plastic bag marked “cash katutak”. Instead of presenting its evidence, the defense filed a demurrer to evidence alleging the illegality of the search and seizure of the items. In her testimony, the accused claimed that she had just come from Choice theatre where she watched a movie “Balweg”. While about to cross the road an old woman asked her for help in carrying a shoulder bag, when she was later on arrested by the police. She has no knowledge of the identity of the old woman and the woman was nowhere to be found. Also, no search warrant was presented. CONSTITUTIONAL LAW 2 CASE DIGESTS

The trial court convicted the accused in violation of the dangerous drugs of 1972 Issue: Whether or Not the police correctly searched and seized the drugs from the accused. Held: The following cases are specifically provided or allowed by law: 1. Warrantless search incidental to a lawful arrest recognized under Section 12, Rule 126 of the Rules of Court 8 and by prevailing jurisprudence 2. Seizure of evidence in "plain view," the elements of which are: (a) a prior valid intrusion based on the valid warrantless arrest in which the police are legally present in the pursuit of their official duties; (b) the evidence was inadvertently discovered by the police who had the right to be where they are; (c) the evidence must be immediately apparent, and (d) "plain view" justified mere seizure of evidence without further search; 3. Search of a moving vehicle. Highly regulated by the government, the vehicle's inherent mobility reduces expectation of privacy especially when its transit in public thoroughfares furnishes a highly reasonable suspicion amounting to probable cause that the occupant committed a criminal activity; 4. Consented warrantless search; 5. Customs search; 6. Stop and Frisk; 7. Exigent and Emergency Circumstances. The essential requisite of probable cause must still be satisfied before a warrantless search and seizure can be lawfully conducted.

CONSTITUTIONAL LAW 2 CASE DIGESTS

The accused cannot be said to be committing a crime, she was merely crossing the street and was not acting suspiciously for the Narcom agents to conclude that she was committing a crime. There was no legal basis to effect a warrantless arrest of the accused’s bag, there was no probable cause and the accused was not lawfully arrested. The police had more than 24 hours to procure a search warrant and they did not do so. The seized marijuana was illegal and inadmissible evidence. RULE 113, RULES OF COURT Section 5. Arrest without warrant; when lawful. — A peace officer or a private person may, without a warrant, arrest a person: (a) When, in his presence, the person to be arrested has committed, is actually committing, or is attempting to commit an offense; (b) When an offense has just been committed, and he has probable cause to believe based on personal knowledge of facts or circumstances that the person to be arrested has committed it; and (c) When the person to be arrested is a prisoner who has escaped from a penal establishment or place where he is serving final judgment or is temporarily confined while his case is pending, or has escaped while being transferred from one confinement to another. In cases falling under paragraph (a) and (b) above, the person arrested without a warrant shall be forthwith delivered to the nearest police station or jail and shall be proceeded against in accordance with section 7 of Rule 112. RULE 126, RULES OF COURT Section 2. Court where application for search warrant shall be filed. — An application for search warrant shall be filed with the following: a) Any court within whose territorial jurisdiction a crime was committed. CONSTITUTIONAL LAW 2 CASE DIGESTS

b) For compelling reasons stated in the application, any court within the judicial region where the crime was committed if the place of the commission of the crime is known, or any court within the judicial region where the warrant shall be enforced. However, if the criminal action has already been filed, the application shall only be made in the court where the criminal action is pending. Section 7. Right to break door or window to effect search. — The officer, if refused admittance to the place of directed search after giving notice of his purpose and authority, may break open any outer or inner door or window of a house or any part of a house or anything therein to execute the warrant or liberate himself or any person lawfully aiding him when unlawfully detained therein. Section 12. Delivery of property and inventory thereof to court; return and proceedings thereon. (a) The officer must forthwith deliver the property seized to the judge who issued the warrant, together with a true inventory thereof duly verified under oath. (b) Ten (10) days after issuance of the search warrant, the issuing judge shall ascertain if the return has been made, and if none, shall summon the person to whom the warrant was issued and require him to explain why no return was made. If the return has been made, the judge shall ascertain whether section 11 of this Rule has been complained with and shall require that the property seized be delivered to him. The judge shall see to it that subsection (a) hereof has been complied with. (c) The return on the search warrant shall be filed and kept by the custodian of the log book on search warrants who shall enter therein the date of the return, the result, and other actions of the judge. A violation of this section shall constitute contempt of court.

CHECKPOINTS RICARDO C. VALMONTE v. GEN. RENATO DE VILLA, GR No. 83988, 1989-09-29 CONSTITUTIONAL LAW 2 CASE DIGESTS

Facts: Petitioner Ricardo C. Valmonte sues in his capacity as citizen of the Republic, taxpayer, member of the Integrated Bar of the Philippines (IBP), and resident of Valenzuela, Metro Manila: while petitioner Union of Lawyers and Advocates for People's Rights (ULAP) sues in its capacity as an association whose members are all members of the IBP. On... n 20 January 1987, the National Capital Region District Command (NCRDC) was activated pursuant to Letter of Instruction 02787 of the Philippine General Headquarters, AFP, with the mission of conducting security operations within its area of responsibility and peripheral... areas, for the purpose of establishing an effective territorial defense, maintaining peace and order, and providing an atmosphere conducive to the social, economic and political development of the National Capital Region.[1] As... part of its duty to maintain peace and order, the NCRDC installed checkpoints in various parts of Valenzuela, Metro Manila. Petitioners aver that, because of the installation of said checkpoints, the residents of Valenzuela are worried of being harassed and of their safety being placed at the arbitrary, capricious and whimsical disposition of the military manning the checkpoints, considering that... their cars and vehicles are being subjected to regular searches and check-ups, especially at night or at dawn, without the benefit of a search warrant and/or court order. Their alleged fear for their safety increased when, at dawn of 9 July 1988. Benjamin Parpon, a supply officer of the Municipality of Valenzuela, Bulacan, was gunned down allegedly in cold blood by the members of the NCRDC manning the checkpoint along McArthur Highway at Malinta, Valenzuela, for ignoring and/or refusing to submit himself to the checkpoint and for continuing to speed off inspite of warning shots fired in the air. Petitioner Valmonte also claims that, on several occasions,... he had gone thru these checkpoints where he was stopped and his car subjected to search/checkup without a court order or search warrant. CONSTITUTIONAL LAW 2 CASE DIGESTS

Petitioners further contend that the said checkpoints give the respondents a blanket authority to make searches and/or seizures without search warrant or court order in violation of the Constitution;[2] and instances have... occurred where a citizen, while not killed, had been harassed. Issues: Petitioners' concern for their safety and apprehension at being harassed by the military manning the checkpoints are not sufficient grounds to declare the checkpoints as per se illegal. Ruling: The constitutional right against unreasonable searches and seizures is a personal right invocable only by those whose rights have been infringed,[4] or threatened to be... infringed. What constitutes a reasonable or unreasonable search and seizure in any particular case is purely a judicial question, determinable from a consideration of the circumstances... involved. Petitioner Valmonte's general allegation to the effect that he had been stopped and searched without a search warrant by the military manning the checkpoints, without more, i.e., without stating the details of the... incidents which amount to a violation of his right against unlawful search and seizure, is not sufficient to enable the Court to determine whether there was a violation of Valmonte's right against unlawful search and... seizure. Not all searches and seizures are prohibited. Those which are reasonable are not forbidden. A reasonable search is not to be determined by any fixed formula but is to be resolved according to the facts of... each case. Where, for example, the officer merely draws aside the curtain of a vacant vehicle which is parked on the public fair grounds,[7] or simply looks into a... vehicle,[8] or flashes a light therein,[9] these do not constitute unreasonable search. The setting up of the questioned checkpoints in Valenzuela (and probably in other areas) may be considered as a security measure to enable the NCRDC to pursue its mission or establishing effective territorial defense and maintaining peace and order for the... benefit of the public. Checkpoints may also be regarded as CONSTITUTIONAL LAW 2 CASE DIGESTS

measures to thwart plots to destabilize the government, in the interest of public security. In this connection, the Court may take judicial... notice of the shirt to urban centers and their suburbs of the insurgency movement, so clearly reflected in the increased killings in cities of police and military men by NPA "sparrow units," not to mention the abundance of unlicensed firearms and the alarming rise... in lawlessness and violence in such urban centers, not all of which are reported in media, most likely brought about by deteriorating economic conditions - which all sum up to what one can rightly consider, at the very least, as abnormal... times. Between the inherent right of the state to protect its existence and promote public welfare and an individual's right against a warrantless search which is however reasonably conducted, the former should... prevail. True, the manning of checkpoints by the military is susceptible of abuse by the men in uniform, in the same manner that all governmental power is susceptible of abuse. But, at the cost of occasional... inconvenience, discomfort and even irritation to the citizen, the checkpoints during these abnormal times, when conducted within reasonable limits, are part of the price we pay for an orderly society... and a peaceful community. ANIAG VS. COMELEC [237 SCRA 194; G.R. NO. 104961; 7 OCT 1994] Facts: In preparation for the synchronized national and local elections, the COMELEC issued Resolution No. 2323, “Gun Ban”, promulgating rules and regulations on bearing, carrying and transporting of firearm or other deadly weapons on security personnel or bodyguards, on bearing arms by members of security agencies or police organizations, and organization or maintenance of reaction forces during the election period. COMELEC also issued Resolution No. 2327 providing for the summary disqualification of candidates engaged in gunrunning, using and transporting of firearms, organizing special strike forces, and establishing spot checkpoints. Pursuant to the “Gun Ban”, Mr. Serrapio Taccad, Sergeant at Arms of the House of Representatives, wrote petitioner for the return of the two firearms issued to him by the House of Representatives. Petitioner then instructed his driver, Arellano, to pick up the firearms from petitioner’s house and return them to Congress. The PNP set up a checkpoint. When the car driven by Arellano approached the checkpoint, the PNP searched the car and found the firearms. CONSTITUTIONAL LAW 2 CASE DIGESTS

Arellano was apprehended and detained. He then explained the order of petitioner. Petitioner also explained that Arellano was only complying with the firearms ban, and that he was not a security officer or a bodyguard. Later, COMELEC issued Resolution No.92-0829 directing the filing of information against petitioner and Arellano for violation of the Omnibus Election Code, and for petitioner to show cause why he should not be disqualified from running for an elective position. Petitioner then questions the constitutionality of Resolution No. 2327. He argues that “gunrunning, using or transporting firearms or similar weapons” and other acts mentioned in the resolution are not within the provisions of the Omnibus Election Code. Thus, according to petitioner, Resolution No. 2327 is unconstitutional. The issue on the disqualification of petitioner from running in the elections was rendered moot when he lost his bid for a seat in Congress in the elections. Issue: Whether or Not petitioner can be validly prosecuted for instructing his driver to return the firearms issued to him on the basis of the evidence gathered from the warrant less search of his car Held: A valid search must be authorized by a search warrant issued by an appropriate authority. However, a warrantless search is not violative of the Constitution for as long as the vehicle is neither searched nor its occupants subjected to a body search, and the inspection of the vehicle is merely limited to a visual search. In the case at bar, the guns were not tucked in Arellano’s waist nor placed within his reach, as they were neatly packed in gun cases and placed inside a bag at the back of the car. Given these circumstances, the PNP could not have thoroughly searched the car lawfully as well as the package without violating the constitutional injunction. Absent any justifying circumstance specifically pointing to the culpability of petitioner and Arellano, the search could not have been valid. Consequently, the firearms obtained from the warrantless search cannot be admitted for any purpose in any proceeding. It was also shown in the facts that the PNP had not informed the public of the purpose of setting up the checkpoint. Petitioner was also not among those charged by the PNP with violation of the Omnibus Election Code. He was not informed by the City Prosecutor that he was a CONSTITUTIONAL LAW 2 CASE DIGESTS

respondent in the preliminary investigation. Such constituted a violation of his right to due process. Hence, it cannot be contended that petitioner was fully given the opportunity to meet the accusation against him as he was not informed that he was himself a respondent in the case. Thus, the warrantless search conducted by the PNP is declared illegal and the firearms seized during the search cannot be used as evidence in any proceeding against the petitioner. Resolution No. 92-0829 is unconstitutional, and therefore, set aside. PEOPLE OF THE PHILIPPINES v. VICTOR DIAZ VINECARIO, et al. 420 SCRA 280 (2004) Facts: On the night of April 10, 1995, as about fifteen police officers were manning a checkpoint at Ulas, Davao City pursuant to COMELEC Resolution No. 2735, otherwise known as the COMELEC gun ban, a motorcycle with three men on board namely appellant Victor Vinecario (Vinecario), Arnold Roble (Roble) Gerlyn Wates (Wates) sped past of the police officers. When they were ordered to return to the checkpoint, a police officer asked what the backpack contains which the appellants answered that it was only a mat. The police officers suspected that it was a bomb and when appellant opened the bag it turns out that its contents were marijuana. The three were then brought to the police station and later to Camp Catitipan and there they were investigated by police officials without the assistance of counsel, following which they were made to sign some documents which they were not allowed to read. The Regional Trial Court rendered them guilty for transporting, possessing and delivering prohibited drugs under Article IV of Republic Act No. 6425 (Dangerous Drugs Act of 1972, as amended by Republic Act No. 7659), and imposing upon them the penalty of reclusion perpetua. ISSUE: Whether or not the search upon the appellants and the seizure of the alleged 1,700 grams of marijuana violated there constitutional right against unreasonable search and seizure. HELD: CONSTITUTIONAL LAW 2 CASE DIGESTS

Although the general rule is that motorists and their vehicles as well as pedestrians passing through checkpoints may only be subjected to a routine inspection, vehicles may be stopped and extensively searched when there is probable cause which justifies a reasonable belief of the men at the checkpoints that either the motorist is a law offender or the contents of the vehicle are or have been instruments of some offense. Warrantless search of the personal effects of an accused has been declared by the Court as valid, because of existence of probable cause, where the smell of marijuana emanated from a plastic bag owned by the accused, or where the accused was acting suspiciously, and attempted to flee. In light then of Vinecario et al.‘s speeding away after noticing the checkpoint and even after having been flagged down by police officers, their suspicious and nervous gestures when interrogated on the contents of the backpack which they passed to one another, and the reply of Vinecario, when asked why he and his co-appellants sped away from the checkpoint, that he was a member of the Philippine Army, apparently in an attempt to dissuade the policemen from proceeding with their inspection, there existed probable cause to justify a reasonable belief on the part of the law enforcers that appellants were offenders of the law or that the contents of the backpack were instruments of some offense. People v dela Cruz; G.R. No. 83260; 18 Apr 1990; 184 SCRA 416 FACTS: After receiving a confidential report from their informant, a buy-bust operation was conducted by a team from the 13th Narcotics Regional Unit to catch the pusher/s. At the scene, it was the accused-appellant who first negotiated with the poseur-buyer. Appellant instructed his co-accused to give one aluminum foil of marijuana which the latter got from his pants’ pocket and delivered it to the buyer. After ascertaining that the authenticity of the marijuana, the agent gave signal. The two accused were arrested. ISSUE(S): Whether or not the arrest was valid. HELD: CONSTITUTIONAL LAW 2 CASE DIGESTS

YES. While it is conceded that in a buy-bust operation, there is seizure of evidence from one’s person without a search warrant, needless to state a search warrant is not necessary, the search being incident to a lawful arrest. A peace officer may, without a warrant, arrest a person when, in his presence, the person to be arrested was committed, is actually committing or is attempting to commit an offense. It is a matter of judicial experience that in the arrest of violators of the Dangerous Drugs Act in a buy-bust operation, the malefactors were invariably caught red-handed. Judgment of the lower court is AFFIRMED. INSPECTION OF BUILDING Camara v. Municipal Court, 387 U.S. 523, 87 S. Ct. 1727 (1967) FACTS: On November 6, 1963, a Housing inspector (Health Department) entered an apartment building for a routine annual inspection. The building manager told him that Camara, who leased the ground floor, was living in part of the space, which was not authorized for residential usage. The inspector confronted Camara and was refused entry to the space. Two days later, the inspector returned, and was again denied entry. A citation was mailed to Camara, and he failed to appear at the district attorney’s office, as ordered. Two weeks later, two more inspectors again visited Camara and informed him that he was in violation of the law. Camara was charged with violating a California law requiring him to permit warrantless inspections of his residence by housing inspectors. He was arrested and filed a writ of prohibition on the charge. The lower courts, basing their opinion on earlier Supreme Court rulings, upheld the charge against Camara. ISSUE: May the law require warrantless inspections of property? HOLDING: No DISCUSSION: While the Court held that allowing such warrantless inspections to be a violation of the Fourth Amendment, the Court agreed that the needs of the CONSTITUTIONAL LAW 2 CASE DIGESTS

community for safety might outweigh the blanket prohibition on such searches. The Court agreed that “area inspections” might be appropriate, and defined that search as designating an area in need of inspection services and requesting a blanket warrant for that area. The appropriate standard may be based upon the passage of time, the nature of the building or the condition of the entire area. The Court stated that: “The warrant procedure is designed to guarantee that a decision to search private property is justified by a reasonable governmental interest. But reasonableness is still the ultimate standard. If a valid public interest justifies the intrusion contemplated, then there is probable cause to issue a suitably restricted search warrant. Such an approach neither endangers time-honored doctrines applicable to criminal investigations nor makes a nullity of the probable cause requirement in this area. It merely gives full recognition to the competing public and private interests here at stake and, in so doing, best fulfills the historic purposes behind the consituational right to be free from unreasonable government invasions of privacy.”

WARRANTLESS ARREST Rebellion as Continuing Offense UMIL VS. RAMOS [187 SCRA 311; G.R. NO. 81567; 3 OCT 1991] Facts: On 1 February 1988, military agents were dispatched to the St. Agnes Hospital, Roosevelt Avenue, Quezon City, to verify a confidential information which was received by their office, about a "sparrow man" (NPA member) who had been admitted to the said hospital with a gunshot wound. That the wounded man in the said hospital was among the five (5) male "sparrows" who murdered two (2) Capcom mobile patrols the day before, or on 31 January 1988 at about 12:00 o'clock noon, before a road hump along Macanining St., Bagong Barrio, Caloocan City. The wounded man's name was listed by the hospital management as "Ronnie Javellon," twenty-two (22) years old of Block 10, Lot 4, South City Homes, Biñan, Laguna however it was disclosed later that the true name of the CONSTITUTIONAL LAW 2 CASE DIGESTS

wounded man was Rolando Dural. In view of this verification, Rolando Dural was transferred to the Regional Medical Servicesof the CAPCOM, for security reasons. While confined thereat, he was positively identified by the eyewitnesses as the one who murdered the 2 CAPCOM mobile patrols. Issue: Whether or Not Rolando was lawfully arrested. Held: Rolando Dural was arrested for being a member of the NPA, an outlawed subversive organization. Subversion being a continuing offense, the arrest without warrant is justified as it can be said that he was committing as offense when arrested. The crimes rebellion, subversion, conspiracy or proposal to commit such crimes, and crimes or offenses committed in furtherance therefore in connection therewith constitute direct assaults against the state and are in the nature of continuing crimes.

COMMITED IN THE PRESENT OF POLICE OFFICERS G.R. No. 185125 January 30, 2012 Ruben Del Castillo vs. People of the Philippines Facts: CONSTITUTIONAL LAW 2 CASE DIGESTS

Pursuant to a confidential information that petitioner Del Castillo was engaged in selling shabu, police officers headed by SPO3 Bienvenido Masnayon, after conducting surveillance and test-buy operation at the house of petitioner, secured a search warrant from the RTC. Upon arrival to the residence of Del Castillo to implement the search warrant, SPO3 Masnayon claimed that he saw petitioner run towards a small structure, a nipa hut, in front of his house. Masnayon chased him but to no avail, because he and his men were not familiar with the entrances and exits of the place. They all went back to the residence of Del Castillo and requested his men to get a barangay tanod and a few minutes thereafter, his men returned with two barangay tanods who searched the house of petitioner including the nipa hut where the petitioner allegedly ran for cover. His men who searched the residence of the petitioner found nothing, but one of the barangay tanods was able to confiscate from the nipa hut several articles, including four (4) plastic packs containing white crystalline substance. Thus, an information was filed against Del Castillo for violation of Section 16, Article III of R.A. 6425 and was found guilty by the RTC and affirmed by the Court of Appeals. Petitioner filed with the Supreme Court the petition for certiorari contending among others that CA erred in finding him guilty beyond reasonable doubt of illegal possession of prohibited drugs, because he could not be presumed to be in possession of the same just because they were found inside the nipa hut. Issue: Can petitioner Del Castillo be held liable for violation of Section 16, Article III of R.A. 6425 by mere presumption that the petitioner has dominion and control over the place where the shabu was found? Held: No. While it is not necessary that the property to be searched or seized should be owned by the person against whom the search warrant is issued, there must be sufficient showing that the property is under petitioner’s control or possession. The records are void of any evidence to show that petitioner owns the nipa hut in question nor was it established that he used the said structure as a shop. The RTC, as well as the CA, merely presumed that petitioner used the said structure due to the presence of electrical materials, the petitioner being an electrician by profession. CONSTITUTIONAL LAW 2 CASE DIGESTS

The prosecution must prove that the petitioner had knowledge of the existence and presence of the drugs in the place under his control and dominion and the character of the drugs. With the prosecution’s failure to prove that the nipa hut was under petitioner’s control and dominion, there casts a reasonable doubt as to his guilt. In considering a criminal case, it is critical to start with the law’s own starting perspective on the status of the accused — in all criminal prosecutions, he is presumed innocent of the charge laid unless the contrary is proven beyond reasonable doubt. Proof beyond reasonable doubt, or that quantum of proof sufficient to produce a moral certainty that would convince and satisfy the conscience of those who act in judgment, is indispensable to overcome the constitutional presumption of innocence. PEOPLE VS. SUCRO [195 SCRA 388; G.R. No. 93239; 18 Mar 1991] Facts: Pat. Fulgencio went to Arlie Regalado’s house at C. Quimpo to monitor activities of Edison SUCRO (accused). Sucro was reported to be selling marijuana at a chapel 2 meters away from Regalado’s house. Sucro was monitored to have talked and exchanged things three times. These activities are reported through radio to P/Lt. Seraspi. A third buyer was transacting with appellant and was reported and later identified as Ronnie Macabante. From that moment, P/Lt.Seraspi proceeded to the area. While the police officers were at the Youth Hostel in Maagama St. Fulgencio told Lt. Seraspi to intercept. Macabante was intercepted at Mabini and Maagama crossing in front of Aklan Medical center. Macabante saw the police and threw a tea bag of marijuana on the ground. Macabante admitted buying the marijuana from Sucro in front of the chapel. The police team intercepted and arrested SUCRO at the corner of C. Quimpo and Veterans. Recovered were 19 sticks and 4 teabags of marijuana from a cart inside the chapel and another teabag from Macabante. Issues: (1) Whether or Not arrest without warrant is lawful. CONSTITUTIONAL LAW 2 CASE DIGESTS

(2) Whether or Not evidence from such arrest is admissible. Held: Search and seizures supported by a valid warrant of arrest is not an absolute rule. Rule 126, Sec 12 of Rules of Criminal Procedure provides that a person lawfully arrested may be searched for dangerous weapons or anything, which may be used as proff of the commission of an offense, without a search warrant.(People v. Castiller) The failure of the police officers to secure a warrant stems from the fact that their knowledge required from the surveillance was insufficient to fulfill requirements for its issuance. However, warantless search and seizures are legal as long as PROBABLE CAUSE existed. The police officers have personal knowledge of the actual commission of the crime from the surveillance of the activities of the accused. As police officers were the ones conducting the surveillance, it is presumed that they are regularly in performance of their duties. MARKED MONEY People v Enrile; G.R. No. 74189; 26 May 1993; 222 SCRA 586 FACTS: A buy-bust team was dispatched to entrap appellant Rogelio Abugatal, a plan made on the strength of a tip given by a police informer. After witnessing the exchange, two policemen approached appellant and placed him under arrest, at the same time confiscating the wrapped object he gave the poseur-buyer. Upon prodding, appellant Abugatal led the police to the house of his co-accused Enrile where he identified the latter as the source of the marijuana. Appellant Enrile was frisked and the marked money was found inside his front pocket. ISSUE(S): Whether or not appellant Enrile’s warrantless arrest and search was justified. HELD: CONSTITUTIONAL LAW 2 CASE DIGESTS

NO. The policemen who later arrested Enrile at his house had no personal knowledge that he was the source of the marijuana. The discovery of the marked money on him did not mean he was caught in the act of selling marijuana. The marked money was not prohibited per se. Even if it were, that fact alone would not retroactively validate the warrantless search and seizure LACK OF URGENCY People v. Pasudag G.R. No. 128822, May 4, 2001 Facts: Around 1:30 in the afternoon, SPO2 Pepito Calip urinated at a bushy bamboo fence behind the public school. About five (5) meters away, he saw a garden of about 70 square meters. There were marijuana plants in between corn plants and camote tops. He inquired from a storekeeper nearby as to who owned the house with the garden. The storeowner told him that Alberto Pasudag owned it. He went to the Police Station and reported to Chief of Police. The latter dispatched team to conduct an investigation. At around 2:30 in that same afternoon, the team arrived and went straight to the house of accused Pasudag. SPO3 Fajarito looked for accused Pasudag and asked him to bring the team to his backyard garden which was about five 5 meters away. Upon seeing the marijuana plants, the policemen called for a photographer, who took pictures of accused Pasudag standing besides one of the marijuana plants. They uprooted 7 marijuana plants. The team brought accused Pasudag and the marijuana plants to the police station. At the police station, accused Pasudag admitted, in the presence of Chief of Police that he owned the marijuana plants. On March 18, 1997, the trial court rendered a decision finding the accused guilty as charged and, taking into consideration his educational attainment (he reached only grade IV), Hence, this appeal. In his brief, accused-appellant contended that the trial court erred in finding that the marijuana plant submitted for laboratory examination was one of the seven (7) marijuana plants confiscated from his garden; that the trial court erred in concluding that the confiscation report was not an extrajudicial admission which required the intervention of his counsel; and in convicting him on the basis of inference that he planted, cultivated and cultured the seven (7) plants, owned the same or that he permitted others to cultivate the same. The Solicitor General contended that accused-appellant admitted before the lower court that tile CONSTITUTIONAL LAW 2 CASE DIGESTS

specimen was one of the plants confiscated in his backyard; that appellant was not under custodial investigation when he signed the confiscation report; and that the inferences deduced by the lower court strengthened the conviction of accused-appellant. Issue: Whether the arrest of the accused requires urgency or necessity for the warrantless search Held: No, the Court ruled that search and seizure conducted without the requisite judicial warrant is illegal and void ab initio. The prosecution’s evidence clearly established that the police conducted a search of accused’s backyard garden without a warrant; they had sufficient time to obtain a search warrant; they failed to secure one. There was no showing of urgency or necessity for the warrantless search, or the immediate seizure of the marijuana plants. The Court is not unmindful of the difficulties of law enforcement agencies in suppressing the illegal traffic of dangerous drugs. However, quick solutions of crimes and apprehension of malefactors do not justify a callous disregard of the Bill of Rights.”We need not underscore that the protection against illegal search and seizure is constitutionally mandated and only under specific instances are searches allowed without warrants.” “The mantle of protection extended by the Bill of Rights covers both innocent and guilty alike against any form of high handedness of law enforcers, regardless of the praise worthiness of their intentions.” With the illegal seizure of the marijuana plants subject of this case, the seized plants are inadmissible in evidence against accused-appellant. EFFECT OF ENTRY OF PLEA People v. Plana (G.R. No. 128285) Facts: Helen Pirote had been missing for 3 days, and when she was found, she was already lifeless and decomposing. A witness (Lagud) came forward and recounted that 3 days before Helen’s body was found, on that same place, he saw 4 persons ganging up on a girl; the girl was being raped and later stabbed. Frightened that the assailants would see him, he ran away. He later learned that the deceased was CONSTITUTIONAL LAW 2 CASE DIGESTS

Helen. In his affidavit and in his testimony in court, Lagud identified the 4 men he saw as the defendant appellants herein Plana, Perayra, Saldevea, and Banday. After due trial, the court found the 4 accused guilty beyond reasonable doubt of the crime of Rape with Homicide and ordered them to pay jointly and severally the heirs of the victim P50,000 as civil liability. Issue: Whether or not the amount of the award of civil liability is proper. Ruling: NO. However, there is need to modify the damages awarded to the heirs of Helen by the trial court. In addition to the sum of P25,000.00 as actual damages, the trial court awarded to the heirs of Helen the sum of P50,000.00 as civil indemnity. This amount should be increased in consonance with prevailing jurisprudence fixing the civil indemnity in cases of rape with homicide at P100,000.00. The Court, likewise, finds it proper to award the sum of P50,000.00 as moral damages. The award of moral damages may be made to the heirs of the victim in a criminal proceeding without need of proof. The fact that they suffered the trauma of mental or physical and psychological sufferings which constitute the basis for moral damages under the Civil Code are too obvious to still require recital thereof at trial. VALIDITY OF CONVICTION PEOPLE vs. CONDE FACTS: The RTC ruled that Conde, Atis and Perez, Jr. were guilty beyond reasonable doubt of the special complex crime of Robbery with Homicide. Oscar Conde claims that he was illegally arrested by the authorities. He adds that the Indian Embassy was pressuring the police to solve the murder. He also wants the Court to disregard as evidence the stolen items and weapons illegally seized by the police. CONSTITUTIONAL LAW 2 CASE DIGESTS

ISSUES: 1. Whether or not the appellants were illegally arrested. 2. Whether or not the items seized were admissible as evidence. RULING: 1. YES The arrests of the appellants came after the lapse of 5 days from the time they were seen committing the crime. At the time they were arrested, the police were not armed with any warrants for their arrests. Section 5 of Rule 113, of the Revised Rules of Criminal Procedure enumerates the instances when an arrest can be made without warrant, namely: (a) When, in his presence the person to be arrested has committed, is actually committing, or is attempting to commit an offense; (b) When an offense has in fact just been committed, and he has probable cause to believe based on personal knowledge of facts or circumstances that the person to be arrested has committed it; and (c) When the person to be arrested is a prisoner who has escaped from a penal establishment or place where he is serving final judgment or temporarily confined while his case is pending, or has escaped while being transferred from one confinement to another. None of the above circumstances is present in this case. Appellants were merely walking along Tandang Sora Avenue and were not committing any crime. Neither can it be said that the crime had just been committed. It cannot also be said that the arresting officers had probable cause based on personal knowledge. PO3 Sevillano admitted that they learned about the suspects from Apollo Romero and certain unnamed informants. The third circumstance is patently not present. The lapse of five days gave the police more than enough time to conduct surveillance of the appellants and apply for a warrant of arrest. Clearly, appellants rights provided in Sec. 2, Art. III of the Constitution were violated. Unfortunately, appellants did not assert their constitutional rights prior to their arraignment. An accused is estopped from assailing the legality of his arrest if he failed to move for the quashing of the Information against him before his arraignment. When the appellants entered their pleas on arraignment without invoking their rights to question any irregularity, which might have accompanied their arrests, they voluntarily submitted themselves to the jurisdiction of the court and the judicial process. It is much too late for appellants to raise the question of their warrantless arrests. CONSTITUTIONAL LAW 2 CASE DIGESTS

Furthermore, the illegal arrest of an accused is not sufficient cause for setting aside a valid judgment rendered upon a sufficient complaint after trial free from error. The warrantless arrest, even if illegal, cannot render void all other proceedings including those leading to the conviction of the appellants and his coaccused, nor can the state be deprived of its right to convict the guilty when all the facts on record point to their culpability. 2. NO The warrantless search in the house of a certain Jimmy, based on the confession of accused Perez, Jr., is definitely questionable. PO3 Sevillano categorically stated that they were able to recover the stolen items, i.e., the beach towel and the umbrella, because of the confession of Perez, Jr. who was not assisted by counsel when he confessed and eventually led the police to the whereabouts of the said items. The use of evidence against the accused obtained by virtue of his testimony or admission without the assistance of counsel while under custodial investigation is proscribed under Sections 12 and 17, Article III of the Constitution. Moreover, the Court find in order the search of the bag of Felicidad Macabare, at the time she was visiting her husband who was a detainee. PO3 Sevillano testified, this search is part of police standard operating procedure and is recognized as part of precautionary measures by the police to safeguard the safety of the detainees as well as the over-all security of the jail premises. However, the weapons confiscated from Felicidad Macabare, were not formally offered as evidence by the prosecution, hence probatively valueless. With regards to the crime committed, appellants are guilty of two counts of homicide and not robbery with homicide because only the facts and causes of deaths were established with moral certainty. Demaisip v Court of Appeals; G.R. No. 89393; 25 Jan 1991; 193 SCRA 373 FACTS: Petitioner was tried for and convicted of illegal possession of marijuana. The Court of Appeals admitted in evidence the marijuana seized from him and affirmed his conviction. ISSUE(S): CONSTITUTIONAL LAW 2 CASE DIGESTS

Whether or not petitioner may object to the admissibility of the evidence against him during this appeal. HELD: NO. Objections to the legality of the search warrant and to the admissibility of the evidence obtained thereby were deemed waived when no objection to the legality of the search warrant was raised during the trial of the case nor to the admissibility of the evidence obtained through said warrant. Decision appealed from is AFFIRMED. EXCLUSIONARY RULE Roan v. Gonzales, 145 SCRA 687 (1986) FACTS: The challenged search warrant was issued by the respondent judge on May 10, 1984. The petitioner's house was searched two days later but none of the articles listed in the warrant was discovered. However, the officers conducting the search found in the premises one Colt Magnum revolver and eighteen live bullets which they confiscated. They are now the bases of the charge against the petitioner. Respondent Judge said that when PC Capt. Mauro P. Quinosa personally filed his application for a search warrant on May 10, 1984, he appeared before him in the company of his two (2) witnesses, Esmael Morada and Jesus Tohilida, both of whom likewise presented to him their respective affidavits taken by Pat. Josue V. Lining, a police investigator. As the application was not yet subscribed and sworn to, he proceeded to examine Captain Quillosa on the contents thereof to ascertain, among others, if he knew and understood the same. Afterwards, he subscribed and swore to the same before him. ISSUE: Whether the Respondent Judge failed to comply with the proper procedure in issuing the Search Warrant. HELD: CONSTITUTIONAL LAW 2 CASE DIGESTS

Yes, mere affidavits of the complainant and his witnesses are thus not sufficient. The examining Judge has to take depositions in writing of the complainant and the witnesses he may produce and attach them to the record. Such written deposition is necessary in order that the Judge may be able to properly determine the existence or non-existence of the probable cause, to hold liable for perjury the person giving it if it will be found later that his declarations are false. We, therefore, hold that the search warrant is tainted with illegality by the failure of the Judge to conform with the essential requisites of taking the depositions in writing and attaching them to the record, rendering the search warrant invalid. (See Rule 126, Sec 4) The respondent judge also declared that he "saw no need to have applicant Quillosa's deposition taken considering that he was applying for a search warrant on the basis of the information provided by the witnesses whose depositions had already been taken by the undersigned. In other words, the applicant was asking for the issuance of the search warrant on the basis of mere hearsay and not of information personally known to him, as required by settled jurisprudence.

PRIVACY OF COMMUNICATION & CORRESPONDENCE Vivares vs St. Theresa’s College GR No 202666 29 September 2014 Facts: This case involves graduating students of the STC-Cebu City; wherein, the students involved posted pictures on their Facebook account of them wearing wearing no shirt, but only brassieres from waist up. Said photos were taken while they were changing into their swimsuits for a beach party. The said photos were reported to the STC’s computer teacher, named Mylene Rheza Escudero. Escudero asked several of her students to show her other photos of Julia and Julianne, abovementioned graduating students, they saw photos of: them along the streets of Cebu wearing clothing which shows their black brassieres (duh, Sinulog? Hello?); them drinking hard liquor and smoking cigarettes inside a bar (Private property OUTSIDE school premises); and that their Facebook accounts were accessible to any Facebook user. CONSTITUTIONAL LAW 2 CASE DIGESTS

Upon discovery thereof, Escudero reported the matter to the school authorities. The poor students involved were investigated and were barred to attend their highschool graduation rites which is experienced by a person once in their lifetime. Heartless. A case was filed against the STC and its officials for Injunction and Damages. Injunction as to the order of the school not to allow the poor children to attend their graduation rites. A petition for the issuance of the writ of habeas data was also filed. Petitioners (Parents of the students involved) assert that the privacy of the children were unlawfully invaded. Since the Facebook accounts of the children are set at “Friends Only”; That the photos were owned by the ladies, thus cannot be used and reproduced without their consent. Old hag, however, violated this by saving digital copies and subsequently showed them to the STC’s officials. RTC issued the writ and directed the respondents to file their verified written return within 5 working days from service of the writ. Respondent denied the petitioners allegation, among others, because there can be no violation of their right to privacy as there is no reasonable expectation of privacy on Facebook. RTC dismissed the petition for habeas data. Issue: Whether the writ of habeas data is a proper remedy? Decision: No. The writ of habeas data is a remedy available to any person whose right to privacy in life, liberty or security is violated or threatened by an unlawful act or omission of a public official or employee, or of a private individual or entity engaged in the gathering, collecting or storing data or information regarding the person, family, home and correspondence of the aggrieved party. PURPOSE: CONSTITUTIONAL LAW 2 CASE DIGESTS

It is an independent and summary remedy designed to protect the image, privacy, honor, information, and freedom of information of an individual, and to provide a forum to enforce one’s right to the truth and to informational privacy. THE WRIT OF HABEAS DATA IS NOT ONLY CONFINED TO CASES OF EXTRALEGAL KILLINGS AND ENFORCED DISAPPEARANCES. Section 2 of the Rule on the Writ of Habeas Data Sec. 2. Who May File. – Any aggrieved party may file a petition for the writ of habeas data. However, in cases of extralegal killings and enforced disappearances, the petition may be filed by: (a) Any member of the immediate family of the aggrieved party, namely: the spouse, children and parents; or (b) Any ascendant, descendant or collateral relative of the aggrieved party within the fourth civil degree of consanguinity or affinity, in default of those mentioned in the preceding paragraph. (emphasis supplied) Had the framers of the Rule intended to narrow the operation of the writ only to extralegal killings or enforced disappearances, the above underscored portion of Section 2, a variance of habeas data situations, would not have been made. It is designed to safeguard individual freedom from abuse in the information age. RESPONDENT CONTENDS THAT IT IS NOT AN ENTITY ENGAGED IN THE GATHERING, COLLETING OR STORING OF DATA OR INFORMATION REGARDING THE PERSON, FAMILY, HOME AND CORRESPONDENCE OF THE AGGRIEVED PARTY. -THIS IS ERRONEOUS. Such individual need not be in the business of such. To “engage” in something is different from undertaking a business endeavor. To “engage” means “to do or take part in something.” It does not necessarily mean that the activity must be done in pursuit of a business. What matters is that the person or entity must be gathering, collecting or storing said data or information about the aggrieved party or his or her family. Regularity is immaterial. THREE STRANDS OF RIGHT TO PRIVACY: CONSTITUTIONAL LAW 2 CASE DIGESTS

Locational/Situational Informational (case at bar) Decisional RIGHT TO PRIVACY WAS NOT VIOLATED because: Facebook has privacy safeguard tools. Utilization of this tools is the manifestation, in the cyber world, of the user’s invocation of his right to informational privacy. That the photos are viewable by “friends only” does not necessarily bolsters the petitioners’ contention. It is well emphasize at this point that setting a post’s or profile detail’s to “Friends” is no assurance that it can no longer be viewed by another user who is not Facebook friends with the source of the content. The user’s own Facebook friend can share said content or tag his or her own Facebook friend thereto, regardless of whether the user tagged by the latter is Facebook friends or not with the former.

Jose Jesus M. Disini, JR., Rowena S. Disini, Lianne Ivy P. Medina, Janette Toral and Ernesto Sonido, JR., Petitioners vs The Secretary of Justice, The Secretary of the Department of the Interior and Local government, The Executive Director of the Information and Communications Technology Office, The Chief of the Philippine National Police, and The Director of the National Bureau of Investigation, Respondents. GR No. 203335 11 February 2014 Facts: These consolidated petitions seek to declare several provisions of Republic Act (RA) 10175, the Cybercrime Prevention Act of 2012, unconstitutional and void. The cybercrime law aims to regulate access to and use of the cyberspace. The cyberspace is a boon to the need of a current generation for greater information CONSTITUTIONAL LAW 2 CASE DIGESTS

and facility of communication. But all is not well with the system since it could not filter out a number of persons of ill will who would want to use cyberspace technology for mischiefs and crimes. One of them can, for instance, avail himself of the system to unjustly ruin the reputation of another or bully the latter by posting defamatory statements against him that people can read. And because linking with the internet opens up a user to communication from others, the ill-motivated can use the cyberspace for committing theft by hacking into or surreptitiously accessing his bank account or credit card or defrauding him through false representations. The wicked can use the cyberspace, too, for illicit trafficking in sex or for exposing to pornography guileless children who have access to the internet. For these reasons, the government has a legitimate right to regulate the use of cyberspace and contain and punish wrongdoings. The government certainly has the duty and the right to prevent these tomfooleries from happening and punish their perpetrators, hence the Cybercrime Prevention Act. But petitioners claim that the means adopted by the cybercrime law for regulating undesirable cyberspace activities violate certain of their constitutional rights. Pending hearing and adjudication of the issues presented in these cases, on February 5, 2013, the Court extended the original 120-day temporary restraining order (TRO) that it earlier issued on October 9, 2012, enjoining respondent government agencies from implementing the cybercrime law until further orders. ISSUES: 1. The petitioners contend that Section 4(a)(1) fails to meet the strict scrutiny standard required of laws that interfere with the fundamental rights of the people. Is Section 4(a)(1) on Illegal Access unconstitutional? 2. Petitioners claim that Section 4(a)(3) suffers from overbreadth in that, while is seeks to discourage data interference, it intrudes into the area of protected speech and expression, creating a chilling and deterrent effect on these guaranteed freedoms. 3. Petitioners claim that Section 4(a)(6) or cyber-squatting violates the equal protection clause in that, not being narrowly tailored, it will cause a user CONSTITUTIONAL LAW 2 CASE DIGESTS

using his real name to suffer the same fate as those who use aliases or take the name of another in satire, parody, or any other literary device. 4. Petitioners claim that Section 4(b)(3) violates the constitutional rights to due process and to privacy and correspondence, and transgresses the freedom of the press. 5. Petitioners claim that cybersex violates the freedom of expression clause of the Constitution. 6. Petitioners are wary that a person who merely doodles on paper and imagines a sexual abuse of a 16-year old is not criminally liable for producing child pornography but one who formulates the idea on his laptop would be. 7. Is Section 4(c)(3) unconstitutional for penalizing the transmission of unsolicited commercial communications? 8. Petitioners dispute the constitutionality of both the penal code provisions on libel as well as Section4(c)(4) of the Cybercrime Prevention Act on cyberlibel. 9. Petitioners assail the constitutionality of Section 5 that renders criminally liable any person who wilfully abets or aids in the commission or attempts to commit any of the offenses enumerated as cybercrimes. It suffers from overbreadth, creating a chilling and deterrent effect on protected expression. 10.Is Section 6 on the penalty of one degree higher constitutional? 11.Is Section 7 on the prosecution under both the Revised Penal Code (RPC) and RA 10175 constitutional? 12.Is Section 8 valid and constitutional? 13.Is Section 12 on Real-Time collection of traffic data valid and constitutional? 14.Is Section 13 on preservation of computer data valid and constitutional? 15.Is Section 14 on disclosure of computer data valid and constitutional? 16.Is Section 15 on search, seizure and examination of computer data valid and constitutional? 17.Is Section 17 on destruction of computer data valid and constitutional? 18.Is Section 19 on restricting or blocking access to computer data valid and constitutional? 19.Is Section 20 on obstruction of justice valid and constitutional? 20.Is Section 24 on Cybercrime Investigation and Coordinating Center (CICC) valid and constitutional? 21.Is Section 26(a) on CICC’s power and functions valid and constitutional? CONSTITUTIONAL LAW 2 CASE DIGESTS

Ruling: 1. No. The strict scrutiny standard, an American constitutional construct, is useful in determining the constitutionality of laws that tend to target a class of things or persons. According to this standard, a legislative classification that impermissibly interferes with the exercise of fundamental right or operates to the peculiar class disadvantage of a suspect class is presumed unconstitutional. The Court finds nothing in Section 4(a)(1) that calls for the application of the strict scrutiny standard since no fundamental freedom, like speech, is involved in punishing what is essentially a condemnable act – accessing the computer system of another without right. It is a universally condemnable act. 2. Under the overbreadth doctrine, a proper governmental purpose, constitutionally subject to state regulation, may not be achieved by means that unnecessarily sweep its subject broadly, thereby invading the area of protected speech. Section 4(a)(3) does not encroach on these freedoms at all. It simply punishes what essentially is a form of vandalism, the act of wilfully destroying without right the things that belong to others, in this case their computer data, electronic document, or electronic data message. Such act has no connection to guaranteed freedoms. Ergo, there is no freedom to destroy other people’s computer systems and private documents. All penal laws, like the cybercrime law, have of course an inherent chilling effect, an in terrorem effect, or the fear of possible prosecution that hangs on the heads of citizens who are minded to step beyond the boundaries of what is proper. But to prevent the State from legislating criminal laws because they instil such kind of fear is to render the state powerless in addressing and penalizing socially harmful conduct. 3. No, the challenge to the constitutionality of Section 4(a)(6) is baseless. The law is reasonable in penalizing the act of acquiring the domain name in bad faith to profit, mislead, destroy reputation, or deprive others who are not ill-motivated of the rightful opportunity of registering the same. It is the evil purpose for which one uses the name that the law condemns. 4. No. In assessing the challenge that the State has impermissibly intruded into these zones of privacy, a court must determine whether a person has exhibited a reasonable expectation of privacy and, if so, whether that expectation has been violated by unreasonable government intrusion. The law punishes those who acquire or use identifying information without CONSTITUTIONAL LAW 2 CASE DIGESTS

5.

6.

7.

8.

right, implicitly to cause damage. Petitioners fail to show how government effort to curb computer-related identity theft violates the right to privacy and correspondence as well as the right to due process. There is no fundamental right to acquire another’s personal right. The Court has defined intent to gain as an internal act which can be established through overt acts of the offender, and it may be presumed from the furtive taking of useful property pertaining to another, unless special circumstances reveal a different intent on the part of the perpetrator. As such, the press, whether in the quest of news reporting or social investigation, has nothing to fear since a special circumstance is present to negate intent to gain which is required by this Section. The Court will not declare Section 4(c)(1) unconstitutional where it stands a construction that makes it apply only to persons engaged in the business of maintaining, controlling, or operating, directly or indirectly, the lascivious exhibition of sexual organs or sexual activity with the aid of a computer system as Congress has intended. The constitutionality of Section 4(c)(3) is not successfully challenged. The law makes the penalty higher by one degree when the crime is committed in cyberspace. But no one can complain since the intensity or duration of penalty is a legislative prerogative and there is a rational basis for such higher penalty. Yes, because to prohibit the transmission of unsolicited ads would deny a person the right to read his emails, even unsolicited commercial ads addressed to him. Commercial speech is a separate category of speech which us not accorded the same level of protection as that given to other constitutionally guaranteed forms of expression but is nonetheless entitled to protection. The State cannot rob him of this right without violating the constitutionally guaranteed freedom of expression. Thus, unsolicited advertisements are legitimate forms of expression. Since the penal code and implicitly, the cybercrime law, mainly target libel against private persons, the Court recognizes that these laws imply a stricter standard of malice to convict the author of a defamatory statement where the offended party is a public figure. The elements of libel are: (a) the allegation of a discreditable act or condition concerning another; (b) publication of the charge; (c) identity of the person defamed; and (d) existence of malice.There is actual malice or malice in fact when the offender makes the defamatory statement with the knowledge that it is

CONSTITUTIONAL LAW 2 CASE DIGESTS

false or with reckless disregard of whether it was false or not. The reckless disregard standard used here required a high degree of awareness of probable falsity. There must be sufficient evidence to permit the conclusion that the accused in fact entertained serious doubts as to the truth of the statement he published. Gross or even extreme negligence is not sufficient to establish actual malice. The defense of absence of actual malice, even when the statement turns out to be false, is available where the offended party is a public official or a public figure. But, where the offended party is a private individual, the prosecution need not prove the presence of actual malice. For his defense, the accused must show that he has a justifiable reason for the defamatory statement even if it was in fact true. 9. A governmental purpose, which seeks to regulate the use of cyberspace communication technology to protect a person’s reputation and peace of mind, cannot adopt means that will unnecessarily and broadly sweep, invading the area of protected freedoms. If such means are adopted, selfinhibition borne of fear of what sinister predicaments await internet users will suppress otherwise robust discussion of public issues. Democracy will be threatened and with it, all liberties. Penal laws should provide reasonably clear guidelines for law enforcement officials and triers of facts to prevent arbitrary and discriminatory enforcement. The terms “aiding or abetting” constitute broad sweep that generates chilling effect on those who express themselves through cyberspace posts, comments, and other messages. Hence, Section 5 of the cybercrime law that punishes “aiding or abetting” libel on the cyberspace is a nullity. 10.Yes, because there exists a substantial distinction between crimes committed through the use of information and communication technology and similar crimes committed using other means. In using the technology in question, the offender often evades identification and is able to reach far more victims or cause greater harm. 11.The Court resolves to leave the determination of the correct application of Section 7 that authorizes prosecution of the offender under both the Revised Penal Code and Republic Act 10175 to actual cases, with the exception of the crimes of: 1. 1. Online libel as to which, charging the offender under both Section 4(c)(4) of Republic Act 10175 and Article 353 of the CONSTITUTIONAL LAW 2 CASE DIGESTS

Revised Penal Code constitutes a violation of the proscription against double jeopardy; as well as 2. Child pornography committed online as to which, charging the offender under both Section 4(c)(2) of Republic Act 10175 and Republic Act 9775 or the Anti-Child Pornography Act of 2009 also constitutes a violation of the same proscription, and, in respect to these, is void and unconstitutional. 12.Valid and constitutional, because the matter of fixing penalties for the commission of crimes is as a rule a legislative prerogative. 13.Void and unconstitutional, because Section 12 does not permit law enforcement authorities to look into the contents of the messages and uncover the identities of the sender and the recipient. Thus, the authority that Section 12 gives law enforcement agencies is too sweeping and lacks restraint. 14.Valid and constitutional, because the user ought to have kept a copy of that data when it crossed his computer if he was so minded. There was no undue deprivation of property since the data that service providers preserve on orders of law enforcement authorities are not made accessible to users by reasons of the issuance of such orders. 15.Valid and constitutional, because what Section 14 envisions is merely the enforcement of a duly issued court warrant. Disclosure can be made only after judicial intervention. 16.Valid and constitutional, because Section 15 merely enumerates the duties of law enforcement authorities that would ensure proper collection, preservation, and use of computer system or data that have been seized by virtue of a court warrant. 17.Valid and constitutional, because it is unclear that the user has a demandable right to require the service provider to have that copy of data saved indefinitely for him in its storage system. 18.Void and unconstitutional, because Section 19 not only precludes any judicial intervention but it also disregards jurisprudential guidelines established to determine the validity of restrictions on speech. 19.Valid and constitutional insofar as it applies to the provisions of Chapter IV which are not struck down by the Court. 20.and 21. Valid and constitutional, because cybercrime law is complete in itself when it directed the CICC to formulate and implement a national CONSTITUTIONAL LAW 2 CASE DIGESTS

cybersecurity plan. The law gave sufficient standards for the CICC to follow when it provided a definition of cybersecurity. OTHER CASES: GANAAN V IAC G.R. No. L-69809 | October 16, 1986 | J. Gutierrez Jr. Facts: Complainant Atty. Tito Pintor and his client Manuel Montebon were in the living room of complainant’s residence discussing the terms for the withdrawal of the complaint for direct assault which they filed with the Office of the City Fiscal of Cebu against Leonardo Laconico. After they had decided on the proposed conditions, complainant made a telephone call to Laconico. That same morning, Laconico telephoned appellant, who is a lawyer, to come to his office and advise him on the settlement of the direct assault case because his regular lawyer, Atty. Leon Gonzaga, went on a business trip. When complainant called, Laconico requested appellant to secretly listen to the telephone conversation through a telephone extension so as to hear personally the proposed conditions for the settlement. Twenty minutes later, complainant called again to ask Laconico if he was agreeable to the conditions. Laconico answered ‘Yes’. Complainant then told Laconico to wait for instructions on where to deliver the money. Complainant called again and instructed Laconico to give the money to his wife at the office of the then Department of Public Highways. Laconico who earlier alerted his friend Colonel Zulueta of the Criminal Investigation Service of the Philippine Constabulary, insisted that complainant himself should receive the money. When he received the money at the Igloo Restaurant, complainant was arrested by agents of the Philippine Constabulary. Appellant executed on the following day an affidavit stating that he heard complainant demand P8,000.00 for the withdrawal of the case for direct assault. Laconico attached the affidavit of appellant to the complainant for robbery/extortion which he filed against complainant. Since appellant listened to CONSTITUTIONAL LAW 2 CASE DIGESTS

the telephone conversation without complainant’s consent, complainant charged appellant and Laconico with violation of the Anti-Wiretapping Act. The lower court found both Gaanan and Laconico guilty of violating Section 1 of Republic Act No. 4200, which prompted petitioner to appeal. The IAC affirmed with modification hence the present petition for certiorari. Issue: W/N an extension telephone is covered by the term “device or arrangement” under Rep. Act No. 4200 Held: No. The law refers to a “tap” of a wire or cable or the use of a “device or arrangement” for the purpose of secretly overhearing, intercepting, or recording the communication. There must be either a physical interruption through a wiretap or the deliberate installation of a device or arrangement in order to overhear, intercept, or record the spoken words. An extension telephone cannot be placed in the same category as a dictaphone, dictagraph or the other devices enumerated in Section 1 of RA No. 4200 as the use thereof cannot be considered as “tapping” the wire or cable of a telephone line. The telephone extension in this case was not installed for that purpose. It just happened to be there for ordinary office use.

RAMIREZ V CA G.R. No. 93833 | September 28, 1995 Facts:

CONSTITUTIONAL LAW 2 CASE DIGESTS

A civil case damages was filed by petitioner Socorro Ramirez in the Quezon City RTC alleging that the private respondent, Ester Garcia, in a confrontation in the latter’s office, allegedly vexed, insulted and humiliated her in a “hostile and furious mood” and in a manner offensive to petitioner’s dignity and personality,” contrary to morals, good customs and public policy.” In support of her claim, petitioner produced a verbatim transcript of the event and sought damages. The transcript on which the civil case was based was culled from a tape recording of the confrontation made by petitioner. As a result of petitioner’s recording of the event and alleging that the said act of secretly taping the confrontation was illegal, private respondent filed a criminal case before the Pasay RTC for violation of Republic Act 4200, entitled “An Act to prohibit and penalize wire tapping and other related violations of private communication, and other purposes.” Petitioner filed a Motion to Quash the Information, which the RTC later on granted, on the ground that the facts charged do not constitute an offense, particularly a violation of R.A. 4200. The CA declared the RTC’s decision null and void and denied the petitioner’s MR, hence the instant petition. Issue: W/N the Anti-Wiretapping Act applies in recordings by one of the parties in the conversation Held: Yes. Section 1 of R.A. 4200 entitled, ” An Act to Prohibit and Penalized Wire Tapping and Other Related Violations of Private Communication and Other Purposes,” provides: Sec. 1. It shall be unlawful for any person, not being authorized by all the parties to any private communication or spoken word, to tap any wire or cable, or by using any other device or arrangement, to secretly overhear, intercept, or record CONSTITUTIONAL LAW 2 CASE DIGESTS

such communication or spoken word by using a device commonly known as a dictaphone or dictagraph or detectaphone or walkie-talkie or tape recorder, or however otherwise described. The aforestated provision clearly and unequivocally makes it illegal for any person, not authorized by all the parties to any private communication to secretly record such communication by means of a tape recorder. The law makes no distinction as to whether the party sought to be penalized by the statute ought to be a party other than or different from those involved in the private communication. The statute’s intent to penalize all persons unauthorized to make such recording is underscored by the use of the qualifier “any”. Consequently, as respondent Court of Appeals correctly concluded, “even a (person) privy to a communication who records his private conversation with another without the knowledge of the latter (will) qualify as a violator” under this provision of R.A. 4200. A perusal of the Senate Congressional Records, moreover, supports the respondent court’s conclusion that in enacting R.A. 4200 our lawmakers indeed contemplated to make illegal, unauthorized tape recording of private conversations or communications taken either by the parties themselves or by third persons. The nature of the conversations is immaterial to a violation of the statute. The substance of the same need not be specifically alleged in the information. What R.A. 4200 penalizes are the acts of secretly overhearing, intercepting or recording private communications by means of the devices enumerated therein. The mere allegation that an individual made a secret recording of a private communication by means of a tape recorder would suffice to constitute an offense under Section 1 of R.A. 4200. As the Solicitor General pointed out in his COMMENT before the respondent court: “Nowhere (in the said law) is it required that before one can be regarded as a violator, the nature of the conversation, as well as its communication to a third person should be professed.” Petitioner’s contention that the phrase “private communication” in Section 1 of R.A. 4200 does not include “private conversations” narrows the ordinary meaning of the word “communication” to a point of absurdity. The word communicate comes from the latin word communicare, meaning “to share or to impart.” In its ordinary signification, communication connotes the act of sharing or imparting CONSTITUTIONAL LAW 2 CASE DIGESTS

signification, communication connotes the act of sharing or imparting, as in a conversation, or signifies the “process by which meanings or thoughts are shared between individuals through a common system of symbols (as language signs or gestures)” These definitions are broad enough to include verbal or non-verbal, written or expressive communications of “meanings or thoughts” which are likely to include the emotionally-charged exchange, on February 22, 1988, between petitioner and private respondent, in the privacy of the latter’s office. Any doubts about the legislative body’s meaning of the phrase “private communication” are, furthermore, put to rest by the fact that the terms “conversation” and “communication” were interchangeably used by Senator Tañada in his Explanatory Note to the Bill. Alejano v. Cabuay G.R. No. 160792, August 25, 2005 Facts: Early morning of 27 July 2003, some 321 armed soldiers, led by the now detained junior officers, entered and took control of the Oakwood Premier Luxury Apartments (“Oakwood”). The soldiers disarmed the security officers of Oakwood and planted explosive devices in its immediate surroundings. The junior officers publicly renounced their support for the administration and called for the resignation of President Gloria Macapagal-Arroyo and several cabinet members. Around 7:00 p.m. of the same date, the soldiers voluntarily surrendered to the authorities after several negotiations with government emissaries. The soldiers later defused the explosive devices they had earlier planted. The soldiers then returned to their barracks. Gen. Abaya, as the Chief of Staff of the AFP, issued a directive to all the Major Service Commanders to turn over custody of ten junior officers to the ISAFP Detention Center. The transfer took place while military and civilian authorities were investigating the soldiers’ involvement in the Oakwood incident. Government prosecutors filed an Information for coup d’etat with the RTC against the soldiers involved in the Oakwood incident. the CA rendered its decision ordered Gen. Cabuay, who was in charge of implementing the regulations in the ISAFP Detention Center, to uphold faithfully the rights of the detainees in accordance with Standing Operations Procedure No. 0263-04. The appellate court directed Gen. Cabuay to adhere to his commitment made in court regarding CONSTITUTIONAL LAW 2 CASE DIGESTS

visiting hours and the detainees’ right to exercise for two hours a day. The appellate court declared that while the opening and reading of Trillanes’ letter is an abhorrent violation of his right to privacy of communication, this does not justify the issuance of a writ of habeas corpus. The violation does not amount to illegal restraint, which is the proper subject of habeas corpus proceedings. Issue: Whether the opening, inspection and reading of the letter of the detainees is an infringement of a citizen’s privacy rights. Held: No, the SC do not agree with the CA that the opening and reading of the detainees’ letters violated the detainees’ right to privacy of communication. The letters were not in a sealed envelope. The inspection of the folded letters is a valid measure as it serves the same purpose as the opening of sealed letters for the inspection of contraband. The letters alleged to have been read by the ISAFP authorities were not confidential letters between the detainees and their lawyers. The petitioner who received the letters from detainees Trillanes and Maestrecampo was merely acting as the detainees’ personal courier and not as their counsel when he received the letters for mailing. In the present case, since the letters were not confidential communication between the detainees and their lawyers, the officials of the ISAFP Detention Center could read the letters. If the letters are marked confidential communication between the detainees and their lawyers, the detention officials should not read the letters but only open the envelopes for inspection in the presence of the detainees. The right to privacy of those detained is subject to Section 4 of RA 7438, as well as to the limitations inherent in lawful detention or imprisonment. By the very fact of their detention, pre-trial detainees and convicted prisoners have a diminished expectation of privacy rights. The detainees in the present case are junior officers accused of leading 300 soldiers in committing coup d’etat, a crime punishable with reclusion perpetua. The junior officers are not ordinary detainees but visible leaders of the Oakwood incident involving an armed takeover of a civilian building in the heart of the financial district of the country. As members of the military armed forces, the detainees are subject to the Articles of War. Moreover, the junior officers are detained with other high-risk persons from the Abu Sayyaf and the NPA. Thus, we must give the military custodian a wider range of deference in implementing the regulations in the ISAFP Detention Center. The military custodian is in a better CONSTITUTIONAL LAW 2 CASE DIGESTS

position to know the security risks involved in detaining the junior officers, together with the suspected Abu Sayyaf and NPA members. Since the appropriate regulations depend largely on the security risks involved, we should defer to the regulations adopted by the military custodian in the absence of patent arbitrariness. PRIVILEGED COMMUNICATIONS Pollo v. Constantino-David, G.R. No. 181881, 18 October 2011 Facts Respondent CSC Chair Constantino-David received an anonymous letter complaint alleging of an anomaly taking place in the Regional Office of the CSC. The respondent then formed a team and issued a memo directing the team “to back up all the files in the computers found in the Mamamayan Muna (PALD) and Legal divisions.” Several diskettes containing the back-up files sourced from the hard disk of PALD and LSD computers were turned over to Chairperson David. The contents of the diskettes were examined by the CSC’s Office for Legal Affairs (OLA). It was found that most of the files in the 17 diskettes containing files copied from the computer assigned to and being used by the petitioner, numbering about 40 to 42 documents, were draft pleadings or lettersin connection with administrative cases in the CSC and other tribunals. On the basis of this finding, Chairperson David issued the Show-Cause Order, requiring the petitioner, who had gone on extended leave, to submit his explanation or counter-affidavit within five days from notice. In his Comment, petitioner denied the accusations against him and accused the CSC Officials of “fishing expedition” when they unlawfully copied and printed personal files in his computer. He was charged of violating R.A. No. 6713 (Code of Conduct and Ethical Standards for Public Officials and Employees). He assailed the formal charge and filed an Omnibus Motion ((For Reconsideration, to Dismiss and/or to Defer) assailing the formal charge as without basis having proceeded from an illegal search which is CONSTITUTIONAL LAW 2 CASE DIGESTS

beyond the authority of the CSC Chairman, such power pertaining solely to the court. The CSC denied the omnibus motion and treated the motion as the petitioner’s answer to the charge. In view of the absence of petitioner and his counsel, and upon the motion of the prosecution, petitioner was deemed to have waived his right to the formal investigation which then proceeded ex parte. The petitioner was dismissed from service. He filed a petition to the CA which was dismissed by the latter on the ground that it found no grave abuse of discretion on the part of the respondents. He filed a motion for reconsideration which was further denied by the appellate court. Hence, this petition. Issue WON the search conducted by the CSC on the computer of the petitioner constituted an illegal search and was a violation of his constitutional right to privacy Ruling The search conducted on his office computer and the copying of his personal files was lawful and did not violate his constitutional right. Ratio Decidendi In this case, the Court had the chance to present the cases illustrative of the issue raised by the petitioner. Katz v. United States 389 U.S. 437 (1967), the US Supreme Court held that the act of FBI agents in electronically recording a conversation made by petitioner in an enclosed public telephone booth violated his right to privacy and constituted a “search and seizure”. Because the petitioner had a reasonable expectation of privacy in using the enclosed booth to make a personal telephone call, the protection of the Fourth Amendment extends to such area. Moreso, the concurring opinion of Mr. Justice Harlan noted that the existence of privacy right under prior decisions involved a two-fold requirement: first, that a person has CONSTITUTIONAL LAW 2 CASE DIGESTS

exhibited an actual (subjective) expectation of privacy; and second, that the expectation be one that society is prepared to recognize as reasonable (objective). Mancusi v. DeForte 392 U.S. 364, 88 S.Ct. 2120, 20 L.Ed2d 1154 (1968),thus “recognized that employees may have a reasonable expectation of privacy against intrusions by police.” O’Connor v. Ortega 480 U.S. 709 (1987), the Court categorically declared that “[i]ndividuals do not lose Fourth Amendment rights merely because they work for the government instead of a private employer.” In O’Connor the Court recognized that “special needs” authorize warrantless searches involving public employees for work-related reasons. The Court thus laid down a balancing test under which government interests are weighed against the employee’s reasonable expectation of privacy. This reasonableness test implicates neither probable cause nor the warrant requirement, which are related to law enforcement. Social Justice Society (SJS) v. Dangerous Drugs Board G.R. Nos. 157870, 158633 and 161658, November 3, 2008, 570 SCRA 410, 427, (citing Ople v. Torres, G.R. No. 127685, July 23, 1998, 293 SCRA 141, 169), recognized the fact that there may be such legitimate intrusion of privacy in the workplace. The Court ruled that the petitioner did not have a reasonable expectation of privacy in his office and computer files. As to the second point of inquiry, the Court answered in the affirmative. The search authorized by the CSC Chair, the copying of the contents of the hard drive on petitioner’s computer reasonable in its inception and scope. The Court noted that unlike in the case of Anonymous Letter-Complaint against Atty. Miguel Morales, Clerk of Court, Metropolitan Trial Court of Manila A.M. Nos. P-08-2519 and P-08-2520, November 19, 2008, 571 SCRA 361, the case at bar involves the computer from which the personal files of the petitioner were retrieved is a government-issued computer, hence government property the use of which the CSC has absolute right to regulate and monitor. People vs. Albofera, 152 SCRA 123 (1987) CONSTITUTIONAL LAW 2 CASE DIGESTS

FACTS: Sometime in June or July 1980, accused Albofera and 3 others killed Teodoro Carancio a forester. Rodrigo Esma was at the house of one of the accused but did not participate in the killing. The matter was later brought to the attention of the authorities by a certain Sisneros and accused Albofera was arrested. The accused Lawi-an was subsequently arrested. Albofera executed an extra-judicial confession before the Municipal Circuit Judge. He stated therein that he was forced to join the NPA movement for fear of his life; that said group had ordered the “arrest” of the victim, Carancio, and that the group “sentenced him (the victim) to die by stabbing.” Esma testified against the accused during the trial. While in prison, accused Albofera sent a letter to Esma. Said letter was thereafter introduced as evidence by prosecution. In his letter, accused Albofera was asking Esma to change his declaration in his Affidavit and testify in his favor instead. Later the accused were convicted of murder. ISSUE: Whether the Albofera’s letter to Esma should be excluded as evidence in light of alleged unwarranted intrusion or invasion of the accused’s privacy? HELD: No. The production of that letter by the prosecution was not the result of an unlawful search and seizure nor was it through unwarranted intrusion or invasion into Albofera’s privacy. Albofera admitted having sent the letter and it was its recipient, Rodrigo Esma himself, who produced and identified the same in the course of his testimony in Court. Besides, there is nothing really self-incriminatory in the letter. Albofera mainly pleaded that Esma change his declaration in his Affidavit and testify in his (Albofera’s) favor. Furthermore, nothing Alboferas tated in his letter is being taken against him in arriving at a determination of his culpability. CONSTITUTIONAL LAW 2 CASE DIGESTS

OTHER CASES: ZULUETA VS. COURT OF APPEALS G.R. No. 107383, February 20, 1996 Facts: This is a petition to review the decision of the Court of Appeals, affirming the decision of the Regional Trial Court of Manila (Branch X) which ordered petitioner to return documents and papers taken by her from private respondent's clinic without the latter's knowledge and consent. Petitioner Cecilia Zulueta is the wife of private respondent Alfredo Martin. On March 26, 1982, petitioner entered the clinic of her husband, a doctor of medicine, and in the presence of her mother, a driver and private respondent's secretary, forcibly opened the drawers and cabinet in her husband's clinic and took 157 documents consisting of private correspondence between Dr. Martin and his alleged paramours, greetings cards, cancelled checks, diaries, Dr. Martin's passport, and photographs. The documents and papers were seized for use in evidence in a case for legal separation and for disqualification from the practice of medicine which petitioner had filed against her husband. Issue: (1) Whether or not the documents and papers in question are inadmissible in evidence; Held: (1) No. Indeed the documents and papers in question are inadmissible in evidence. The constitutional injunction declaring "the privacy of communication and correspondence [to be] inviolable" is no less applicable simply because it is the wife (who thinks herself aggrieved by her husband's infidelity) who is the party against whom the constitutional provision is to be enforced. The only exception to the prohibition in the Constitution is if there is a "lawful order [from a] court or when public safety or order requires otherwise, as prescribed by law." CONSTITUTIONAL LAW 2 CASE DIGESTS

Any violation of this provision renders the evidence obtained inadmissible "for any purpose in any proceeding." The intimacies between husband and wife do not justify any one of them in breaking the drawers and cabinets of the other and in ransacking them for any telltale evidence of marital infidelity. A person, by contracting marriage, does not shed his/her integrity or his right to privacy as an individual and the constitutional protection is ever available to him or to her. The law insures absolute freedom of communication between the spouses by making it privileged. Neither husband nor wife may testify for or against the other without the consent of the affected spouse while the marriage subsists. Neither may be examined without the consent of the other as to any communication received in confidence by one from the other during the marriage, save for specified exceptions. But one thing is freedom of communication; quite another is a compulsion for each one to share what one knows with the other. And this has nothing to do with the duty of fidelity that each owes to the other. The review for petition is DENIED for lack of merit. Waterhouse Drug Corp. v NLRC Facts: Antonia Melodia Catolico was hired as a pharmacist by Waterous Drug Corp. Catolico sold to YSP Inc. 10 bottles of Voren Tablets at P384 per unit. However, the normal selling price is P320 per unit. Catolico overcharged by P64 per unit for a total of P640. YSP sent a check payable to Catolico as a “refund” for the jacked-up price. It was sent in an envelope addressed to her. Saldana, the clerk of Waterous Drug Corp. opened the envelope and saw that there was a check for P640 for Catolico. Waterous Drug Corp. ordered the termination of Catolico for acts of dishonesty. NLRC: Dismissed the Petition. Evidence of respondents (check from YSP) being rendered inadmissible, by virtue of the constitutional right invoked by complainants. CONSTITUTIONAL LAW 2 CASE DIGESTS

Petitioners: In the light of the decision in the People v. Marti, the constitutional protection against unreasonable searches and seizures refers to the immunity of one’s person from interference by government and cannot be extended to acts committed by private individuals so as to bring it within the ambit of alleged unlawful intrusion by the government. Issue: Whether or not the check is admissible as evidence. Held: Yes. The Bill of Rights does not protect citizens from unreasonable searches and seizures perpetrated by private individuals. It is not true, as counsel for Catolico claims, that the citizens have no recourse against such assaults. On the contrary, and as said counsel admits, such an invasion gives rise to both criminal and civil liabilities. Despite this, the SC ruled that there was insufficient evidence of cause for the dismissal of Catolico from employment Suspicion is not among the valid causes provided by the Labor Code for the termination of Employment. RIGHT TO PRIVACY Social Justice Society v. Dangerous Drugs Board, G.R. No. 157870 (and other consolidated petitions), November 3, 2008 DECISION I.

THE FACTS

These consolidated petitions challenge the constitutionality of Sec. 36 of R.A. 9165, the Comprehensive Dangerous Drugs Act of 2002, insofar as it requires mandatory drug testing of (1) candidates for public office; (2) students of secondary and tertiary schools; (3) officers and employees of public and private offices; and (4) persons charged before the prosecutor’s office of a crime with an imposable penalty of imprisonment of not less than 6 years and 1 day. The challenged section reads: CONSTITUTIONAL LAW 2 CASE DIGESTS

SEC. 36. Authorized Drug Testing. Authorized drug testing shall be done by any government forensic laboratories or by any of the drug testing laboratories accredited and monitored by the DOH to safeguard the quality of the test results. x x x The drug testing shall employ, among others, two (2) testing methods, the screening test which will determine the positive result as well as the type of drug used and the confirmatory test which will confirm a positive screening test. x x x The following shall be subjected to undergo drug testing: xxx

xxx

xxx

(c) Students of secondary and tertiary schools. Students of secondary and tertiary schools shall, pursuant to the related rules and regulations as contained in the school's student handbook and with notice to the parents, undergo a random drug testing x x x; (d) Officers and employees of public and private offices. Officers and employees of public and private offices, whether domestic or overseas, shall be subjected to undergo a random drug test as contained in the company's work rules and regulations, x x x for purposes of reducing the risk in the workplace. Any officer or employee found positive for use of dangerous drugs shall be dealt with administratively which shall be a ground for suspension or termination, subject to the provisions of Article 282 of the Labor Code and pertinent provisions of the Civil Service Law; xxx

xxx

xxx

(f) All persons charged before the prosecutor's office with a criminal offense having an imposable penalty of imprisonment of not less than six (6) years and one (1) day shall undergo a mandatory drug test; (g) All candidates for public office whether appointed or elected both in the national or local government shall undergo a mandatory drug test. Sec. 36(g) is implemented by COMELEC Resolution No. 6486. II. THE ISSUES CONSTITUTIONAL LAW 2 CASE DIGESTS

1. Do Sec. 36(g) of RA 9165 and COMELEC Resolution No. 6486 impose an additional qualification for candidates for senator? Corollarily, can Congress enact a law prescribing qualifications for candidates for senator in addition to those laid down by the Constitution? 2. Are paragraphs (c), (d), and (f) of Sec. 36, RA 9165 unconstitutional? III. THE RULING [The Court GRANTED the petition in G.R. No. 161658 and declared Sec. 36(g) of RA 9165 and COMELEC Resolution No. 6486 as UNCONSTITUTIONAL. It alsoPARTIALLY GRANTED the petition in G.R. Nos. 157870 and 158633 by declaring Sec. 36(c) and (d) of RA 9165 CONSTITUTIONAL, but declaring its Sec. 36(f)UNCONSTITUTIONAL. The Court thus permanently enjoined all the concerned agencies from implementing Sec. 36(f) and (g) of RA 9165.] 1. YES, Sec. 36(g) of RA 9165 and COMELEC Resolution No. 6486 impose an additional qualification for candidates for senator; NO, Congress CANNOT enact a law prescribing qualifications for candidates for senator in addition to those laid down by the Constitution. In essence, Pimentel claims that Sec. 36(g) of RA 9165 and COMELEC Resolution No. 6486 illegally impose an additional qualification on candidates for senator. He points out that, subject to the provisions on nuisance candidates, a candidate for senator needs only to meet the qualifications laid down in Sec. 3, Art. VI of the Constitution, to wit: (1) citizenship, (2) voter registration, (3) literacy, (4) age, and (5) residency. Beyond these stated qualification requirements, candidates for senator need not possess any other qualification to run for senator and be voted upon and elected as member of the Senate. The Congress cannot validly amend or otherwise modify these qualification standards, as it cannot disregard, evade, or weaken the force of a constitutional mandate, or alter or enlarge the Constitution. Pimentel’s contention is well-taken. Accordingly, Sec. 36(g) of RA 9165 should be, as it is hereby declared as, unconstitutional. Sec. 36(g) of RA 9165, as sought to be implemented by the assailed COMELEC resolution, effectively enlarges the qualification requirements enumerated in the CONSTITUTIONAL LAW 2 CASE DIGESTS

Sec. 3, Art. VI of the Constitution. As couched, said Sec. 36(g) unmistakably requires a candidate for senator to be certified illegal-drug clean, obviously as a pre-condition to the validity of a certificate of candidacy for senator or, with like effect, a condition sine qua non to be voted upon and, if proper, be proclaimed as senator-elect. The COMELEC resolution completes the chain with the proviso that “[n]o person elected to any public office shall enter upon the duties of his office until he has undergone mandatory drug test.” Viewed, therefore, in its proper context, Sec. 36(g) of RA 9165 and the implementing COMELEC Resolution add another qualification layer to what the 1987 Constitution, at the minimum, requires for membership in the Senate. Whether or not the drug-free bar set up under the challenged provision is to be hurdled before or after election is really of no moment, as getting elected would be of little value if one cannot assume office for non-compliance with the drug-testing requirement. 2. NO, paragraphs (c) and (d) of Sec. 36, RA 9165 are NOT UNCONSTITUTIONAL; YES, paragraphs (f) thereof is UNCONSTITUTIONAL. As to paragraph (c), covering students of secondary and tertiary schools Citing the U.S. cases of Vernonia School District 47J v. Acton and Board of Education of Independent School District No. 92 of Pottawatomie County, et al. v. Earls, et al., the Court deduced and applied the following principles: (1) schools and their administrators stand in loco parentis with respect to their students; (2) minor students have contextually fewer rights than an adult, and are subject to the custody and supervision of their parents, guardians, and schools; (3) schools, acting in loco parentis, have a duty to safeguard the health and well-being of their students and may adopt such measures as may reasonably be necessary to discharge such duty; and (4) schools have the right to impose conditions on applicants for admission that are fair, just, and non-discriminatory. Guided by Vernonia, supra, and Board of Education, supra, the Court is of the view and so holds that the provisions of RA 9165 requiring mandatory, random, and suspicionless drug testing of students are constitutional. Indeed, it is within the prerogative of educational institutions to require, as a condition for admission, compliance with reasonable school rules and regulations and policies. To be sure, the right to enrol is not absolute; it is subject to fair, reasonable, and equitable requirements. CONSTITUTIONAL LAW 2 CASE DIGESTS

As to paragraph (d), covering officers and employees of public and private offices As the warrantless clause of Sec. 2, Art III of the Constitution is couched and as has been held, “reasonableness” is the touchstone of the validity of a government search or intrusion. And whether a search at issue hews to the reasonableness standard is judged by the balancing of the government-mandated intrusion on the individual's privacy interest against the promotion of some compelling state interest. In the criminal context, reasonableness requires showing of probable cause to be personally determined by a judge. Given that the drug-testing policy for employees—and students for that matter—under RA 9165 is in the nature of administrative search needing what was referred to in Vernonia as “swift and informal disciplinary procedures,” the probable-cause standard is not required or even practicable. Be that as it may, the review should focus on the reasonableness of the challenged administrative search in question. The first factor to consider in the matter of reasonableness is the nature of the privacy interest upon which the drug testing, which effects a search within the meaning of Sec. 2, Art. III of the Constitution, intrudes. In this case, the office or workplace serves as the backdrop for the analysis of the privacy expectation of the employees and the reasonableness of drug testing requirement. The employees' privacy interest in an office is to a large extent circumscribed by the company's work policies, the collective bargaining agreement, if any, entered into by management and the bargaining unit, and the inherent right of the employer to maintain discipline and efficiency in the workplace. Their privacy expectation in a regulated office environment is, in fine, reduced; and a degree of impingement upon such privacy has been upheld. Just as defining as the first factor is the character of the intrusion authorized by the challenged law. Reduced to a question form, is the scope of the search or intrusion clearly set forth, or, as formulated in Ople v. Torres, is the enabling law authorizing a search "narrowly drawn" or "narrowly focused"? The poser should be answered in the affirmative. For one, Sec. 36 of RA 9165 and its implementing rules and regulations (IRR), as couched, contain provisions specifically directed towards preventing a situation that would unduly embarrass the employees or place them under a humiliating experience. While every officer CONSTITUTIONAL LAW 2 CASE DIGESTS

and employee in a private establishment is under the law deemed forewarned that he or she may be a possible subject of a drug test, nobody is really singled out in advance for drug testing. The goal is to discourage drug use by not telling in advance anyone when and who is to be tested. And as may be observed, Sec. 36(d) of RA 9165 itself prescribes what, in Ople, is a narrowing ingredient by providing that the employees concerned shall be subjected to “random drug test as contained in the company’s work rules and regulations x x x for purposes of reducing the risk in the work place.” For another, the random drug testing shall be undertaken under conditions calculated to protect as much as possible the employee's privacy and dignity. As to the mechanics of the test, the law specifies that the procedure shall employ two testing methods, i.e., the screening test and the confirmatory test, doubtless to ensure as much as possible the trustworthiness of the results. But the more important consideration lies in the fact that the test shall be conducted by trained professionals in access-controlled laboratories monitored by the Department of Health (DOH) to safeguard against results tampering and to ensure an accurate chain of custody. In addition, the IRR issued by the DOH provides that access to the drug results shall be on the “need to know” basis; that the “drug test result and the records shall be [kept] confidential subject to the usual accepted practices to protect the confidentiality of the test results.” Notably, RA 9165 does not oblige the employer concerned to report to the prosecuting agencies any information or evidence relating to the violation of the Comprehensive Dangerous Drugs Act received as a result of the operation of the drug testing. All told, therefore, the intrusion into the employees’ privacy, under RA 9165, is accompanied by proper safeguards, particularly against embarrassing leakages of test results, and is relatively minimal. Taking into account the foregoing factors, i.e., the reduced expectation of privacy on the part of the employees, the compelling state concern likely to be met by the search, and the well-defined limits set forth in the law to properly guide authorities in the conduct of the random testing, we hold that the challenged drug test requirement is, under the limited context of the case, reasonable and, ergo, constitutional. Like their counterparts in the private sector, government officials and employees also labor under reasonable supervision and restrictions imposed by the Civil CONSTITUTIONAL LAW 2 CASE DIGESTS

Service law and other laws on public officers, all enacted to promote a high standard of ethics in the public service. And if RA 9165 passes the norm of reasonableness for private employees, the more reason that it should pass the test for civil servants, who, by constitutional command, are required to be accountable at all times to the people and to serve them with utmost responsibility and efficiency. As to paragraph (f), covering persons charged before the prosecutor’s office with a crime with an imposable penalty of imprisonment of not less than 6 years and 1 day Unlike the situation covered by Sec. 36(c) and (d) of RA 9165, the Court finds no valid justification for mandatory drug testing for persons accused of crimes. In the case of students, the constitutional viability of the mandatory, random, and suspicionless drug testing for students emanates primarily from the waiver by the students of their right to privacy when they seek entry to the school, and from their voluntarily submitting their persons to the parental authority of school authorities. In the case of private and public employees, the constitutional soundness of the mandatory, random, and suspicionless drug testing proceeds from the reasonableness of the drug test policy and requirement. We find the situation entirely different in the case of persons charged before the public prosecutor's office with criminal offenses punishable with 6 years and 1 day imprisonment. The operative concepts in the mandatory drug testing are “randomness” and “suspicionless.” In the case of persons charged with a crime before the prosecutor's office, a mandatory drug testing can never be random or suspicionless. The ideas of randomness and being suspicionless are antithetical to their being made defendants in a criminal complaint. They are not randomly picked; neither are they beyond suspicion. When persons suspected of committing a crime are charged, they are singled out and are impleaded against their will. The persons thus charged, by the bare fact of being haled before the prosecutor’s office and peaceably submitting themselves to drug testing, if that be the case, do not necessarily consent to the procedure, let alone waive their right to privacy. To impose mandatory drug testing on the accused is a blatant attempt to harness a medical test as a tool for criminal prosecution, contrary to the stated objectives of RA 9165. Drug testing in this case would violate a person’s right to CONSTITUTIONAL LAW 2 CASE DIGESTS

privacy guaranteed under Sec. 2, Art. III of the Constitution. Worse still, the accused persons are veritably forced to incriminate themselves.

Silverthorne Lumber Co. v. U.S., 251 U.S. 385, 40 S.Ct. 182 (1920) FACTS: Silverthorne (the individual who owned the company) was cited for contempt for refusing to produce books and documents before the Grand Jury. Silverthorne had been indicted and arrested. While Silverthorne (and his father) were detained, a U.S. Marshal “without a shadow of authority,” went to their company and seized all books and papers held there. The papers were seized pursuant to an invalid warrant, and a new warrant was drafted based on information in the documents seized. The Court ordered the original documents returned, and then issued a subpoena for the documents. The Silverthornes refused to produce the documents, arguing that the Court was benefiting from the original unlawful seizure, as without that seizure, they would not have been able to draft a new warrant for the materials. ISSUE: Is it permissible for the government to benefit from an unlawful act? HOLDING: No DISCUSSION: The Court agreed that it “reduces the Fourth Amendment to a form of words” by allowing the government to use the knowledge obtained unlawfully. The Court agreed that “[I]f knowledge of them (the evidence) is gained from an independent source they may be proved like any others, but the knowledge gained by the Government’s own wrong cannot be used by it in the way proposed.” In other words, if the government can show it could have obtained the needed information from another source, it may be permitted to keep the evidence, but absent that proof, the evidence will be inadmissible. ABERCA v. VER CONSTITUTIONAL LAW 2 CASE DIGESTS

FACTS Task Force Makabansa (TFM) was ordered by General Fabian Ver to conduct preemptive strikes against Communist- Terrorist underground houses. TFM raided several houses, employing in most cases defectively judicial search warrants, arrested people without warrant of arrest, denied visitation rights, and interrogated them with the use of threats and tortures. A motion to dismiss was filed by defendants, stating that 1) plaintiffs may not cause a judicial inquiry about their detention because the writ of habeas corpus was suspended; 2) defendants are immune from liability for acts done in their official duties; 3) there was no cause of action. On Nov 8, 1983, Judge Fortun granted the motion to dismiss, which prompted plaintiffs to file a MR on Nov 18, 1983. He later inhibited himself and was replaced Judge Lising, who denied the MR for being filed out of time. Another MR was filed, and was only modified to include Maj. Aguinaldo and MSgt. Balaba for officers accountable in the said complaint. ISSUES 1. Whether or not immunity from suit may be invoked? 2. Whether petitioners have the right to question the alleged violation of their rights in the constitution? 3. Whether the superior officers who gave the orders are liable? HELD 1. NO, Article 32 of the Civil Code provides a sanction to rights and freedom enshrined in the constitution. These rights cannot be violated just because of an order given by a superior. The rule of law must prevail, or else liberty will perish. Even though they just followed the orders of their superior, these do not authorize them to disregard the rights of the petitioners, and therefore cannot be considered “acts done in their official duties”. Article 32 speaks of any public officer or private individual, and violation of these constitutional rights does not exempt them from responsibility. CONSTITUTIONAL LAW 2 CASE DIGESTS

2. YES, the suspension of the writ of habeas corpus does not prevent petitioners from claiming damages for the illegal arrest and detention in violation of their constitutional rights by seeking judicial authority. What the writ suspends is merely the right of an individual to seek release from detention as a speedy means of obtaining liberty. It cannot suspend their rights and cause of action for injuries suffered due to violation of their rights. 3. YES, Article 32 speaks of the liabilities of people who are in direct violation of the rights stated, as well as people who are indirectly responsible for such acts. In the case at hand, the superior officers are the ones who gave the order, and can be considered indirectly responsible. It was also stated in the complaint who were the ones who directly and indirectly participated in those acts. By filing a motion to dismiss, they admitted all the facts stated in the complaint. FREEDOM OF INFORMATION Request for copy of 2008 SALN and Personal Data Sheet or Curriculum Vitae of Justices, June 13, 2012 FACTS: Rowena Paraan, Research Director of the PCIJ, sought copies of the SALN of the Justices of the Supreme Court for the year 2008. She also requested for copies of the Personal Data Sheet of the Justices of this court for the purpose of updating their database of information on government officials. ISSUES: Issue #1 Can the SALN of justices be accessed via the right to information? Ruling: Yes. The right to information goes hand-in-hand with the constitutional policies of full public disclosure and honesty in the public service. Issue #2 CONSTITUTIONAL LAW 2 CASE DIGESTS

What are the limitations on the constitutional right to information> Ruling: The right to information is not absolute. It is further subject to such limitations as may be provided by law. Jurisprudence has provided the following limitations to that right: (1) national security matters and intelligence information; (2) trade secrets and banking transactions; (3) criminal matters; and (4) other confidential information such as confidential or classified information officially known to public officers and employees by reason of their office and not made available to the public as well as diplomatic correspondence, closed door Cabinet meetings and executive sessions of either house of Congress, and the internal deliberations of the Supreme Court.

Baldoza vs. Dimaano, 71 SCRA

FACTS: Municipal Secretary of Taal, Batangas, charges Municipal Judge Dimaano with abuse of authority in refusing to allow employees of the Municipal Mayor to examine the criminal docket records of the Municipal Court to secure data in connection with their contemplated report on peace and order conditions of the municipality.

Respondent answered that there has never been an intention to refuse access to official court records but that the same is always subject to reasonable regulation as to who, when, where and how they may be inspected. He further asserted that a court has the power to prevent an improper use or inspection of its records and furnishing copies may be refused when the motivation is not serious and legitimate interest, out of whim or fancy or mere curiosity or to gratify private site or promote public scandal. CONSTITUTIONAL LAW 2 CASE DIGESTS

In his answer respondent observed: - Restrictions are imposed by the Court for fear of an abuse in the exercise of the right. - There has been recent tampering of padlocks of the door of the Court and with this, to allow an indiscriminate and unlimited exercise of the right to free access, might do more harm than good. - Request of such magnitude cannot be immediately granted without adequate deliberation and advisement. - Authority should first be secured from the Supreme Court. Case was referred to Judge Riodique for investigation and report. At the preliminary hearing, Taal Mayor Corazon Caniza files a motion to dismiss the complaint to preserve harmony and cooperation among officers. This motion was denied by investigating Judge but he recommended the exoneration of the respondent. Investigating Judge’s report avers that complainant was aware of the motion to dismiss and he was in conformity with it. Communications between complainant and respondent reveal that the respondent allowed the complainant to open and view the docket books of the respondent under certain conditions and under his control and supervision. Under the conditions, the Court found that the respondent has not committed any abuse of authority. ISSUE: WON respondent acted arbitrarily in the premises (when he allowed the complainant to open and view the docket books of respondent) HELD: No. The respondent allowed the complainant to open and view the docket books of respondent under certain conditions and under his control and supervision. It has not been shown that the rules and condition imposed by the respondent were CONSTITUTIONAL LAW 2 CASE DIGESTS

unreasonable. The access to public records is predicated on the right of the people to acquire information on public concern. RULES/PRINCIPLES: In People ex rel. title Guarantee & T. Co vs. Railly, the Court said: “…what the law expects and requires from his is the exercise of an unbiased and impartial judgment, by which all persons resorting to the office, under legal authority, and conducting themselves in an orderly manner shall be secured their lawful rights and privileges, and that a corporation formed in the manner in which the relator has been, shall be permitted to obtain all the information either by searches, abstracts, or copies, that the law has entitled it to obtain. Except, perhaps, when it is clear that the purpose of the examination is unlawful, or sheer, idle curiosity…It is not their prerogative to see that the information which the records contain is not flaunted before public gaze or that scandal is not made of it…It is the legislature and not the officials having custody thereof which is called upon to devise a remedy.”

Tanada vs Tuvera, supra FACTS: The petitioner ask the Honorable Court, demanding the respondent to disclose the number of Presidential decrees which has not been published, invoking due process. The government argued that although publication is necessary as a rule, it was not so when it was otherwise provided. They contended that the decrees themselves have become effective immediately upon approval. The solicitor general, furthermore contended that since under Article 2 of the Civil Code, the phrase “unless otherwise provided” was not always imperative and that publication, when necessary, may not be made in an official gazette. ISSUE: CONSTITUTIONAL LAW 2 CASE DIGESTS

Whether or not laws and presidential decree has to be published and such publication must be in the official gazette? HELD: The Supreme Court ruled in affirmative. RATIO: Upon examination of the court on Article 2 of the NCC, the phrase “unless otherwise provided”, refers to the date of effectivity and not the requirement of publication, which cannot be omitted and does not make any law immediately effective upon approval without previous publication. However, legislator may provide that its effectivity may not be after 15 days after completion of its publication, it may be shorter or longer. The contention of the respondent is untenable since it would deny the public of due process and public knowledge. The legal presumption that every person has knowledge of the law presupposes that the law was published. Under Sec. 6 of the Bill of Rights, “the right of the people to information on matters of public concern” applies to legislative enactment. We hold therefore that all statutes, including those local in application and private laws, shall be published as a condition for its effectivity, which shall be given effect 15 days after its publication unless the legislators otherwise provide. Included in this rule are presidential decrees in exercise of legislative power whenever the same are validly delegated by the legislator or directly conferred by the constitution. Administrative rules and regulation should also be published if their purpose is to enforce or implement existing laws pursuant to valid delegation. Interpretative rules and regulation and those merely internal in nature, regulating only personnel in administrative agency, need not be published. Neither those letter of instruction issued by administrative superiors concerning rules and guidelines to be followed by the subordinates in the performance of their duties. Circulars issued by the Monetary Board must be published if they were not meant merely to interpret laws but to fill the details of the Central Bank Act.

CONSTITUTIONAL LAW 2 CASE DIGESTS

We also agree that publication must be full or it is no publication at all since its purpose is to inform the public of the law. Mere mention of its number or decree is not sufficient. Valmonte vs. Belmonte, 170 SCRA 256 (1989) FACTS: Petitioners in this special civil action for mandamus with preliminary injunction invoke their right to information and pray that respondent be directed: (a) ti furnish petitioners in the list of names of the Batasang Pambansa members belonging to the UNIDO and PDP-Laban who were able to secure clean loans immediately before the February 7 election thru the intercession/marginal note of the then First Lady Imelda Marcos; and/or (b) to furnish petitioners with certified true copies of the documents evidencing their respective loans; and/or (c) to allow petitioners access to the public records for the subject information on June 20, 1986, apparently not having yet received the reply of the Government Service and Insurance System (GSIS) Deputy General Counsel, petitioner Valmonte wrote respondents another letter, saying that the failure to receive a reply, “We are now considering ourselves free to do whatever action necessary within the premises to pursue our desired objective in pursuance of public interest. “ ISSUE: WON Valmonte, et.al. are entitled as citizens and taxpayers to inquire upon GSIS records on behest loans given by the former First Lady Imelda Marcos to Batasang Pambansa members belonging to the UNIDO and PDP-LABAN political parties. HELD: Respondent has failed to cite any law granting the GSIS the privilege of confidentiality as regards the documents subject of this petition. His position is apparently based merely on considerations of policy. The judiciary does not settle policy issues. The Court can only declare what the law us, and not what the law should be. Under our system of government, policy issues are within the domain of the political branches of the government, and of the people themselves as the CONSTITUTIONAL LAW 2 CASE DIGESTS

repository of all State power. The concerned borrowers themselves may not succeed if they choose their right to privacy, considering the public offices they were holding at the time the loans were alleged to have been granted. It cannot be denied that because of the interest they generate and their newsworthiness, public figures, most especially those holding responsible positions in government, enjoy a more limited right to privacy as compared to ordinary individuals, their actions being subjected to closer public scrutiny. The “transactions” used here I suppose is generic and, therefore, it can cover both steps leading to a contract, and already a consummated contract. Considering the intent of the framers of the Constitution which, though not binding upon the Court, are nevertheless persuasive, and considering further that government-owned and controlled corporations, whether performing proprietary or governmental functions are accountable to the people, the Court is convinced that transactions entered into by the GSIS, a government-controlled corporation created by special legislation are within the ambit of the people’s right to be informed pursuant to the constitutional policy of transparency in government dealings. Although citizens are afforded the right to information and, pursuant thereto, are entitled to “access to official records,” the Constitution does not accord them a right to compel custodians of official records to prepare lists, abstracts, summaries and the like in their desire to acquire information on matters of public concern.

Legaspi vs CSC, 150 SCRA 530 (1987) FACTS: The fundamental right of the people to information on matters of public concern is invoked in this special civil action for mandamus instituted by petitioner Valentin L. Legaspi against the Civil Service Commission. The respondent had earlier denied Legaspi’s request for information on the civil service eligibilities of certain persons employed as sanitarians in the Health Department of Cebu City. These government employees, Julian Sibonghanoy and Mariano Agas, had

CONSTITUTIONAL LAW 2 CASE DIGESTS

allegedly represented themselves as civil service eligible who passed the civil service examinations for sanitarians. ISSUE: WON the petitioner has legal access to government records to validate the civil service eligibilities of the Health Department employees. HELD: The constitutional guarantee to information on matters of public concern is not absolute. It does not open every door to any and all information. Under the Constitution, access to official records, papers, etc., are “subject to limitations as may be provided by law”. The law may therefore, exempt certain types of information from public scrutiny, such as those affecting national security. It follows that, in every case, the availability of access to a particular public record must be circumscribed by the nature of the information sought, i.e., (a) being of public concerm or one that involves public interest, and, (b) not being exempted by law from the operation of the constitutional guarantee. The threshold question is, therefore, whether or not the information sought is of public interest or public concern. This question if first addressed to the government agency having custody of the desired information. However, as already discussed, this does not give the agency concerned any discretion to grant or deny access. In case of denial of access, the government agency has the burden of showing that the information requested is not of public concern, or, if it is of public concern, that the same has been exempted by the law from the operation of the guarantee. To hold otherwise will serve to dilute the constitutional right. As aptly observed, “…the government is in an advantageous position to marshall and interpret arguments against release…” (87 Harvard Law Review 1511 {1974}). To safeguard the constitutional right, every denial of access by the government agency concerned is subject to review by the courts, and in the proper case, access may be compelled by a writ of Mandamus Public office being a public trust. It is the legitimate concern of citizens to ensure that government positions requiring civil service eligibility are occupied only by persons who are eligibles. Public officers are at all CONSTITUTIONAL LAW 2 CASE DIGESTS

times accountable to the people even as to their eligibilities for their respective positions. In the instant, case while refusing to confirm or deny the claims of eligibility, the respondent has failed to cite any provision in the Civil Service Law which would limit the petitioner’s right to know who are, and who are not, civil service eligibles. We take judicial notice of the fact that the names of those who pass the civil service examinations, as in bar examinations and licensure examinations for various professions, are released to the public. Hence, there is nothing secret about one’s civil service eligibility, if actually possessed. Petitioner’s request is, therefore, neither unusual nor unreasonable. And when, as in this case, the government employees concerned claim to be civil service eligibles, the public, through any citizen has a right to verify their professed eligibilities from the Civil Service Commission. The civil service eligibility of a sanitarian being of public concern, and in the absence of express limitations under the law upon access to the register of civil service eligibles for said position, the duty of the respondent Commission to confirm or deny the civil service eligibility of any person occupying the position becomes imperative. Mandamus, therefore lies.

Garcia vs BOI, 177 SCRA 374 (1989) FACTS: Former Bataan Petrochemical Corporation (BPC), now Luzon Petrochemical Corporation, formed by a group of Taiwanese investors, was granted by the BOI to have its plant site for the products “naphta cracker” and “naphta” to be based in Bataan. In February 1989, one year after the BPC began its production in Bataan, the corporation applied to the BOI to have its plant site transferred from Bataan to Batangas. Despite vigorous opposition from petitioner Cong. Enrique Garcia and others, the BOI granted private respondent BPC’s application, stating that the investors have the final choice as to where to have their plant site because they are the ones who risk capital for the project. ISSUE: CONSTITUTIONAL LAW 2 CASE DIGESTS

Whether or not the BOI committed a grave abuse of discretion in yielding to the application of the investors without considering the national interest. COURT RULING: The Supreme Court found the BOI to have committed grave abuse of discretion in this case, and ordered the original application of the BPC to have its plant site in Bataan and the product naphta as feedstock maintained. The ponente, Justice Gutierrez, Jr., first stated the Court’s judicial power to settle actual controversies as provided for by Section 1 of Article VIII in our 1987 Constitution before he wrote the reasons as to how the Court arrived to its conclusion. He mentioned that nothing is shown to justify the BOI’s action in letting the investors decide on an issue which, if handled by our own government, could have been very beneficial to the State, as he remembered the word of a great Filipino leader, to wit: “…he would not mind having a government run like hell by Fillipinos than one subservient to foreign dictation”. Justice Griño Aquino, in her dissenting opinion, argued that the petition was not well-taken because the 1987 Investment Code does not prohibit the registration of a certain project, as well as any decision of the BOI regarding the amended application. She stated that the fact that petitioner disagrees with the BOI dies not make the BOI wrong in its decision, and that petitioner should have appealed to the President of the country and not to the Court, as provided for by Section 36 of the 1987 Investment Code. Justice Melecio-Herrera, in another dissenting opinion, stated that the Constitution does not vest in the Court the power to enter the realm of policy consideration, such as in this case.

Chavez vs Public Estates Authority, July 9, 2002 FACTS:

CONSTITUTIONAL LAW 2 CASE DIGESTS

The petition seeks to compel the Public Estates Authority to disclose all facts on PEA’s then on-going renegotiations with Amari Coastal Bay and Development Corporation to reclaim portions of Manila Bay. The petition further seeks to enjoin PEA from signing a new agreement with AMARI involving such reclamation. PEA argues that petitioner has no standing to institute mandamus proceedings to enforce his constitutional right to information without a showing that PEA refused to perform an affirmative duty imposed on PEA by the Constitution. PEA also claims that petitioner has not shown that he will suffer any concrete injury because of the signing or implementation of the Amended JVA. Thus, there is no actual controversy requiring the exercise of the power of judicial review. ISSUE: Whether or not the petitioner has legal standing to compel PEA to comply with its constitutional duties

DECISION: Petition granted. The petitioner has standing to bring this taxpayer’s suit because the petition seeks to compel PEA to comply with its constitutional duties. The right of citizens to information on matters of public concern and the application of a constitutional provision intended to insure the equitable distribution of alienable lands of the public domain among Filipino citizens are two constitutional rights involved. The thrust of the first issue is to compel PEA to disclose publicly information on the sale of government lands worth billions of pesos. The second issue is to prevent PEA from alienating hundreds of hectares of alienable lands of the public domain in violation of the Constitution, compelling PEA to comply with a constitutional duty to the nation.

Echegaray vs. Sec. of Justice, GR No. 132601 FACTS: CONSTITUTIONAL LAW 2 CASE DIGESTS

The SC affirmed the conviction of petitioner Leo Echegaray y Pilo for the crime of rape of the 10-year old daughter of his common-law spouse and the imposition upon him of the death penalty for the said crime. He filed an MFR and a supplemental MFR raising for the first time the issue of the constitutionality of Republic Act No. 7659 and the death penalty for rape. The Court denied both motions. In the meantime, Congress had seen it fit to change the mide of execution of the death penalty from electrocution to lethal injection, and passed Republic Act No. 8177, AN ACT DESIGNATING DEATH BY LETHAL INJECTION AS THE METHOD OF CARRYING OUT CAPITAL PUNISHMENT, AMENDING FOR THE PURPOSE ARTICLE 81 OF THE REVISED PENAL CODE, AS AMENDED BY SECTION 24 OF REPUBLIC ACT NO. 7659. The convict filed a petition for prohibition from carrying out the lethal injection against him under the grounds that it constituted cruel, degrading, or unusual punishment, being violative of due process, a violation of the Philippines’ obligations under international covenants, an undue delegation of delegated powers by the Secretary of Justice to respondent Director.

In this motion to amend, the petitioner added equal protection as a ground. The Office of the Solicitor General stated that this Court has already upheld the constitutionality of the Death Penalty Law, and has repeatedly declared that the death penalty is not cruel. Unjust, excessive or unusual punishment; execution by lethal injection, as authorized under R.A. No. 8177 and the questioned rules, is constitutional, lethal injection being the most modern, more humane, more economical, safer and easier to apply (than electrocution or the gas chamber); the International Covenant on Civil and Political Rights does not expressly or impliedly prohibit the imposition of the death penalty; R.A. No. 8177 properly delegate legislative power to respondent Director; and that R.A. No. 8177 confers the power to promulgate the implementing rules to the Secretary of Justice, Secretary of Health and the Bureau of Corrections. The Commission on Human Rights filed a Motion for Leave of Court to intervene and/or appear as Amicus Curiae with the attached petition to intervene and/or appear as Amicus Curiae. They alleged similarly with Echegaray’s arguments. The petitioner filed a reply similar to his CONSTITUTIONAL LAW 2 CASE DIGESTS

first arguments. The court gave due course to the petition. Concisely put, petitioner argues that that R.A. No. 8177 and its implementing rules do not pass constitutional muster for: (a) violation of the constitutional proscription against cruel, degrading or inhumane punishment, (b) violation of our international treaty obligations, (c) being an undue delegation of legislative power, and (d) being discriminatory.

ISSUE: 1. Is it a violation of the constitutional proscription against cruel, degrading or inhumane punishment? 2. is it a violation of our international treaty obligations? 3. Is it an undue delegation of legislative power? 4. Is it discriminatory and contrary to law? HELD: No to first three. Yes to last. Petition denied. RATIO: 1. Petitioner contends that death by lethal injection constitutes cruek, degrading and inhumane punishment considering that (1) R.A. No. 8177 fails to provide for the drugs to be used in carrying out lethal injection, the dosage for each drug to be administered and the procedure in administering said drug/s into the accused; (2) R.A. No. 8177 and its implementing rules are uncertain as to the date of the execution, time of notification, the court which will fix he date of execution, which uncertainties cause the greatest pain and suffering for the convict; and (3) the possibility of “botched executions” or mistakes in administering the drugs renders lethal injection inherently cruel. Now it is well-settled in jurisprudence that the death penalty per se is not a cruel, degrading or inhuman punishment. Harden v. Director of Prisons- “punishments are cruel when they involve torture or a lingering death; but the punishment of death is not cruel, within the meaning of CONSTITUTIONAL LAW 2 CASE DIGESTS

that word as used in the constitution. It implies something inhuman and barbarous, something more than the mere extinguishment of life.” Would the lack in particularity then as to the details involved in the execution by lethal injection render said law “cruel, degrading or inhuman”? The Court believes not. For reasons discussed, the implementing details of R.A. No. 8177 are matters which are properly left to the competence and expertise of administrative officials.

Petitioner contends that Sec. 16 of R.A. No. 8177 is uncertain as to which “court” will fix the time and date of execution, and the date of execution and time of notification of the death of the convict. As petitioner already knows, the “court” which designates the date of execution is the trial court which convicted the accused. The procedure is that the “judgment is entered fifteen (15) days after its promulgation, and 10 days thereafter, the records are remanded to the court below including a certified copy of the judgment for execution. Neither is there any uncertainty as to the date of execution nor the time of notification. As to the date of execution, Section 15 of the implementing rules must be read in conjunction with the last sentence of Section 1 of R.A. No. 8177 which provides that the death sentence shall be carried out “not earlier than one (1) year not later than eighteen (18) months from the time the judgment imposing the death penalty become final and executory, without prejudice to the exercise by the President of his executive clemency powers at all times.” Hence, the death convict is in effect assured of eighteen (18) months from the time the judgment imposing the death penalty became final and executor wherein he can seek executive clemency and attend to all his temporal and spiritual affairs. Petitioner further contends that the infliction of “wanton pain” in case of possible complications in the intravenous injection that respondent Director is an untrained and untested person insofar as the choice and administration of lethal injection is concerned, renders lethal injection, a cruel, degrading and inhuman punishment. This is unsubstantiated.

CONSTITUTIONAL LAW 2 CASE DIGESTS

First. Petitioner has neither alleged nor presented evidence that lethal injection required the expertise only of phlebotomists and not trained personnel and that the drugs to be administered are unsafe or ineffective. Petitioner simply cites situations in the United States wherein execution by lethal injection allegedly resulted in prolonged and agonizing death for the convict, without any other evidence whatsoever. Second. Petitioner overlooked Section 1, third paragraph of R.A. No. 8177 which requires that all personnel involved in the execution proceedings should be trained prior to the performance of such task. We must presume that the public officials entrusted with the implementation of the death penalty will carefully avoid inflicting cruel punishment. Third. Any infliction of pain in lethal injection is merely incidental in carrying out the execution of death penalty and does not fall within the constitutional proscription against cruel, degrading and inhuman punishment. “In a limited sense, anything is cruel which is calculated to give pain or distress, and since punishment imports pain or suffering to the convict, it may be said that all punishments are cruel. But of course the Constitution does not mean that crime, for this reason, is to go unpunished.” The cruelty against which the Constitution protects a convicted is cruelty inherent in the method of punishment not the necessary suffering involved in any method employed to extinguish life humanely. What is cruel and unusual is “not fastened to the obsolete but may acquire meaning as public opinion becomes enlightened by a humane justice” and “must draw its meaning from the evolving standards of decency that marks the progress of a maturing society. 2. International Covenant on Civil and Political Rights states: “In countries which have not abolished the death penalty, sentence of death may be imposed only for the most serious crimes in accordance with the law in force at the time of the commission of the crime and not contrary to the provisions of the present Covenant and to the Covention on the Prevention and Punishment of the crime of Genocide. This penalty can only be carried out pursuant to a final judgment rendered by a competent court.” CONSTITUTIONAL LAW 2 CASE DIGESTS

The punishment was subject to the limitation that it be imposed for the “most serious crimes. Included with the declaration was the Second Optional Protocol to the International Covenant on Civil and Political Rights, Aiming at the Abolition of the Death Penalty was adopted by the General Assembly on December 15, 1989. The Philippines neither signed nor ratified said document. 3. R.A. No. 8177 likewise provides the standards which define the legislative policy, mark its limits, map out is boundaries, and specify the public agencies which will apply it. It indicates the circumstances under which the legislative purpose may be carried out. R.A. No. 8177 specifically requires that “the death sentence shall be executed under the authority of the Director of the Bureau of Corrections, endeavoring so far as possible to mitigate the sufferings of the person under the sentence during the lethal injection as well during the proceedings prior to the execution. “Further, “the Director of the Bureau of Corrections shall take steps to ensure that the lethal injection to be administered is sufficient to cause the instantaneous death of the convict.” The legislature also mandated that “all personnel involved in the administration of lethal injection shall be trained prior to the performance of such task.” The Court cannot see that any useful purpose would be served by requiring greater detail. The question raised is not the definition of what constitutes a criminal offense, but the mode of carrying out the penalty already imposed by the Courts. In this sense, R.A. No. 8177 is sufficiently definite and the exercise of discretion by the administrative officials concerned is, canalized within banks that keep it from overflowing. However the Rules and Regulations to implement Republic Act No. 8177 suffer serious flaws that could not be overlooked. To begin with, something basic appears missing in Section 19 of the implementing rules as a vacuum. The Secretary of Justice has practically abdicated the power to promulgate the manual on the execution procedure to the Director of the Bureau of Corrections, by not providing for a mode of review and approval. Being a mere constituent unit of the Department of Justice, the Bureau of Corrections could not promulgate a manual that would not bear the imprimatur of the administrative superior, the Secretary of Justice as the rule-making authority under R.A. No. 8177. Such apparent abdication of departmental responsibility renders the said paragraph invalid. CONSTITUTIONAL LAW 2 CASE DIGESTS

4. Petitioner contends that Section 17 of the Implementing Rules is unconstitutional for being discriminatory as well as for being an invalid exercise of the power to legislate by respondent Secretary. Petitioner insists that Section 17 amends the instances when lethal injection may be suspended, without an express amendment of Article 83 of the Revised Penal Code, as amended by section 25 of R.A. No. 7659. “SEC. 17. SUSPENSION OF THE EXECUTION OF THE DEATH SENTENCE. Execution by lethal injection shall not be inflicted upon a woman within the three years next following the date of sentence or while she is pregnant, nor upon any person over seventy (70) years of age. In this latter case, the death penalty shall be commuted to the penalty of reclusion perpetua with the accessory penalties provided in Article 40 of the Revised Penal Code.” Petitioner contends that Section 17 is unconstitutional for being discriminatory as well as for being an invalid exercise of the power to legislate by respondent Secretary. Petitioner insists that that Section 17 amends the instances when lethal injection may be suspended, without and express amendment of Article 83 of the Revised Penal Code, as amended by Section 25 of R.A. No. 7659, stating that the death sentence shall not be inflicted upon a woman while she is pregnant or within one (1) year after delivery, nor upon any person over seventy years of age. While Article 83 of the Revised Penal Code, as amended by Section 25 of Republic Act No. 7659, suspends the implementation of the death penalty while a woman is pregnant or within one (1) year after delivery, Section 17 of the implementing rules omits the one (1) year period following delivery as an instance when the death sentence is suspended, and adds a ground for suspension of sentence no longer found under Article 83 of the Revised Penal Code as amended which is the three-year reprieve after a woman is sentenced. This addition is, in petitioner’s view, tantamount to a gender-based discrimination sans statutory basis, while the omission is an impermissible contravention of the applicable law. Being merely an implementing rule, Section 17 aforecited must not override, but instead remain consistent and in harmony with the law it seeks to apply and implement. CONSTITUTIONAL LAW 2 CASE DIGESTS

FREEDOM OF EXPRESSION Occena v COMELEC; G.R. No. L-60258; 31 Jan 1984; 127 SCRA 404 FACTS: Petitioner challenges the constitutionality of some sections of the Barangay Election Act of 1982 (BP Blg. 222) which prohibits any political party, political group, political committee from intervening in the barangay election.

ISSUE(S): Whether or not the ban on the intervention of political parties in the barangay election violates the right to form associations.

HELD: NO. The right to form associations or societies for purposes not contrary to law is neither absolute nor illimitable; it is always subject to the pervasive and dominant police power of the state and may constitutionally be regulated or curtailed to serve appropriate and important public interests. Political parties may freely be formed although there is a restriction on their activities. But the ban is narrow, not total. It operates only on concerted or group action of political parties. Members of political and kindred organizations, acting individually, may intervene in the barangay election. Petition is DENIED.

In Re: Edillon, 84 SCRA 568 (AC 1928) FACTS:

CONSTITUTIONAL LAW 2 CASE DIGESTS

Atty. Marcial Edillon was dibarred due to non-payment of his IBP dues, hence the petitioner on this case. He claimed that the provisions of Sec. 10 of Rule 139-A of the Rules of Court is unconstitutional as he is being compelled, as a precondition in maintaining his good standing as a lawyer, to pay and settle his dues to the IBP. Petitioner stubbornly insisted his take and refused to admit full competence of the court in this matter. But after some time in realization, his recalcitrance and defiance were gone in his subsequent communication with the court. He appealed that his health, advanced age, and concern to his former clients’ welfare be considered in his prayer so that he can again practice law. ISSUE: Whether or not Atty. Edillon should be reinstated as member of the bar. HELD: YES. RATIO: Admission to the bar is a privilege burdened with condition. Failure to abide entails loss of such privilege. Considered in addition was the two (2) years Atty. Edillon was barred to practice law, and the dictum of Justice Malcolm in Villavicencio v. Lukban that “the power to discipline, especially if amounting to disbarment, should be exercised in a preservative and not on the vindictive principle”. After contrition on the part of the petitioner, the court finds reinstatement in order.

GOVERNMENT SERVICE INSURANCE SYSTEM (GSIS) and WINSTON F. GARCIA, in his capacity as GSIS President & General Manager, petitioners, vs. KAPISANAN NG MGA MANGGAGAWA SA GSIS, respondents.

CONSTITUTIONAL LAW 2 CASE DIGESTS

FACTS: Forming a huge part of the October 4 to October 7, 2004 mass action participants were GSIS personnel, among them members of the herein respondent Kapisanan Ng Mga Manggagawa sa GSIS (“KMG” or the “Union”), a public sector union of GSIS rank-and-file employees. On or about October 10, 2004, the manager of the GSIS Investigating Unit issued a memorandum directing 131 union and non-union members to show cause why they should not be charged administratively for their participation in said rally. In reaction, KMG’s counsel, Atty. Manuel Molina, sought reconsideration of said directive on the ground, among others, that the subject employees resumed work on October 8, 2004 in obedience to the return-to-work order thus issued. The plea for reconsideration was, however, effectively denied by the filing, on October 25, 2004, of administrative charges against some 110 KMG members for grave misconduct and conduct prejudicial to the best interest of the service. KMG filed a petition for prohibition with the CA against these charges. The CA granted the petition and enjoined the GSIS from implementing the issued formal charges and from issuing other formal charges arising from the same facts and events. CA equated the right to form associations with the right to engage in strike and similar activities available to workers in the private sector. In the concrete, the appellate court concluded that inasmuch as GSIS employees are not barred from forming, joining or assisting employees’ organization, petitioner Garcia could not validly initiate charges against GSIS employees waging or joining rallies and demonstrations notwithstanding the service-disruptive effect of such mass action.

ISSUE: WON the strike conducted by the GSIS employees were valid HELD: NO The 1987 Constitution expressly guaranteeing, for the first time, the right of government personnel to self-organization to complement the provision according workers the right to engage in “peaceful concerted activities, including the right to CONSTITUTIONAL LAW 2 CASE DIGESTS

strike in accordance with law.”. It was against the backdrop of the aforesaid provisions of the 1987 Constitution that the Court resolved Bangalisan v. Court of Appeals. In it, we held, citing MPSTA v. Laguio, Jr., that employees in the public service may not engage in strikes or in concerted and unauthorized stoppage of work; that the right of government employees to organize is limited to the formation of unions or associations, without including the right to strike.

Specifically, the right of civil servants to organize themselves was positively recognized in Association of Court of Appeals Employees vs. Ferrer-Caleja. But, as in the exercise of the rights of free expression and of assembly, there are standards for allowable limitations such as the legitimacy of the purpose of the association, [and] the overriding considerations of national security. As regards the right to strike, the Constitution itself qualifies its exercise with the provision “in accordance with law.” This is a clear manifestation that the state may, by law, regulate the use of this right, or even deny certain sectors such right. Executive Order 180 which provides guidelines for the exercise of the right of government workers to organize, for instance, implicitly endorsed an earlier CSC circular which “enjoins under pain of administrative sanctions, all government officers and employees from staging strikes, demonstrations, mass leaves, walkouts and other forms of mass action which will result in temporary stoppage or disruption of public service” by stating that the Civil Service law and rules governing concerted activities and strikes in government service shall be observed. Public employees going on disruptive unauthorized absences to join concerted mass actions may be held liable for conduct prejudicial to the best interest of the service. With the view we take of the events that transpired on October 4-7, 2004, what respondent’s members launched or participated in during that time partook of a strike or, what contextually amounts to the same thing, a prohibited concerted activity. The phrase “prohibited concerted activity” refers to any collective activity CONSTITUTIONAL LAW 2 CASE DIGESTS

undertaken by government employees, by themselves or through their employees’ organization, with the intent of effecting work stoppage or service disruption in order to realize their demands or force concessions, economic or otherwise; it includes mass leaves, walkouts, pickets and acts of similar nature. Indeed, for four straight days, participating KMG members and other GSIS employees staged a walk out and waged or participated in a mass protest or demonstration right at the very doorstep of the GSIS main office building. The record of attendance for the period material shows that, on the first day of the protest, 851 employees, or forty eight per cent (48%) of the total number of employees in the main office (1,756) took to the streets during office hours, from 6 a.m. to 2 p.m.,leaving the other employees to fend for themselves in an office where a host of transactions take place every business day. On the second day, 707 employees left their respective work stations, while 538 participated in the mass action on the third day. A smaller number, i.e., 306 employees, but by no means an insignificant few, joined the fourth day activity. In whatever name respondent desires to call the four-day mass action in October 2004, the stubborn fact remains that the erring employees, instead of exploring non-crippling activities during their free time, had taken a disruptive approach to attain whatever it was they were specifically after. As events evolved, they assembled in front of the GSIS main office building during office hours and staged rallies and protests, and even tried to convince others to join their cause, thus provoking work stoppage and service-delivery disruption, the very evil sought to be forestalled by the prohibition against strikes by government personnel. To petitioner Garcia, as President and General Manager of GSIS, rests the authority and responsibility, under Section 45 of Republic Act No. 8291, the GSIS Act of 1997, to remove, suspend or otherwise discipline GSIS personnel for cause. At bottom then, petitioner Garcia, by filing or causing the filing of administrative charges against the absenting participants of the October 4-7, 2004 mass action, merely performed a duty expected of him and enjoined by law. Regardless of the mood petitioner Garcia was in when he signed the charge sheet, his act can easily be sustained as legally correct and doubtless within his jurisdiction. CONSTITUTIONAL LAW 2 CASE DIGESTS

G.R. No. 122226 March 25, 1998 UNITED PEPSI-COLA SUPERVISORY UNION (UPSU), petitioner, vs. HON. BIENVENIDO E. LAGUESMA and PEPSI-COLA PRODUCTS, PHILIPPINES, INC. respondents. FACTS: Petitioner is a union of supervisory employees. It appears that on March 20, 1995 the union filed a petition for certification election on behalf of the route managers at Pepsi-Cola Products Philippines, Inc. However, its petition was denied by the med-arbiter and, on appeal, by the Secretary of Labor and Employment, on the ground that the route managers are managerial employees and, therefore, ineligible for union membership under the first sentence of Art. 245 of the Labor Code, which provides: Ineligibility of managerial employees to join any labor organization; right of supervisory employees. — Managerial employees are not eligible to join, assist or form any labor organization. Supervisory employees shall not be eligible for membership in a labor organization of the rank-and-file employees but may join, assist or form separate labor organizations of their own. Petitioner brought this suit challenging the validity of the order, dismissed. Hence, this petition. Pressing for resolution its contention that the first sentence of Art. 245 of the Labor Code, so far as it declares managerial employees to be ineligible to form, assist or join unions, contravenes Art. III, §8 of the Constitution which provides: The right of the people, including those employed in the public and private sectors, to form unions, associations, or societies for purposes not contrary to law shall not be abridged. ISSUES:

CONSTITUTIONAL LAW 2 CASE DIGESTS

(1) whether the route managers at Pepsi-Cola Products Philippines, Inc. are managerial employees and (2) whether Art. 245, insofar as it prohibits managerial employees from forming, joining or assisting labor unions, violates Art. III, §8 of the Constitution. HELD: YES and NO As a class, managers constitute three levels of a pyramid: (1) Top management; (2) Middle Management; and (3) First-line Management [also called supervisors]. FIRST-LINE MANAGERS — The lowest level in an organization at which individuals are responsible for the work of others is called first-line or first-level management. First-line managers direct operating employees only; they do not supervise other managers. Examples of first-line managers are the “foreman” or production supervisor in a manufacturing plant, the technical supervisor in a research department, and the clerical supervisor in a large office. First-level managers are often called supervisors. MIDDLE MANAGERS — The term middle management can refer to more than one level in an organization. Middle managers direct the activities of other managers and sometimes also those of operating employees. Middle managers’ principal responsibilities are to direct the activities that implement their organizations’ policies and to balance the demands of their superiors with the capacities of their subordinates. A plant manager in an electronics firm is an example of a middle manager. TOP MANAGERS — Composed of a comparatively small group of executives, top management is responsible for the overall management of the organization. It establishes operating policies and guides the organization’s interactions with its environment. Typical titles of top managers are “chief executive officer,” “president,” and “senior vice-president.” Actual titles vary from one organization to another and are not always a reliable guide to membership in the highest management classification.

CONSTITUTIONAL LAW 2 CASE DIGESTS

A distinction exists between those who have the authority to devise, implement and control strategic and operational policies (top and middle managers) and those whose task is simply to ensure that such policies are carried out by the rankand-file employees of an organization (first-level managers/supervisors). What distinguishes them from the rank-and-file employees is that they act in the interest of the employer in supervising such rank-and-file employees. “Managerial employees” may therefore be said to fall into two distinct categories: the “managers” per se, who compose the former group described above, and the “supervisors” who form the latter group. #1: It appears that this question was the subject of two previous determinations by the Secretary of Labor and Employment, in accordance with which this case was decided by the med-arbiter. To qualify as managerial employee, there must be a clear showing of the exercise of managerial attributes under paragraph (m), Article 212 of the Labor Code as amended. Designations or titles of positions are not controlling. As to the route managers and accounting manager, we are convinced that they are managerial employees. Their job descriptions clearly reveal so (Worker’s Alliance Trade Union (WATU) v. Pepsi-Cola Products Philippines, Inc., Nov. 13, 1991) This finding was reiterated in Case No. OS-A-3-71-92. entitled In Re: Petition for Direct Certification and/or Certification Election-Route Managers/Supervisory Employees of Pepsi-Cola Products Phils.Inc. * doctrine of res judicata certainly applies to adversary administrative proceedings Thus, we have in this case an expert’s view that the employees concerned are managerial employees within the purview of Art. 212. At the very least, the principle of finality of administrative determination compels respect for the finding of the Secretary of Labor that route managers are managerial employees as defined by law in the absence of anything to show that such determination is without substantial evidence to support it. CONSTITUTIONAL LAW 2 CASE DIGESTS

The Court now finds that the job evaluation made by the Secretary of Labor is indeed supported by substantial evidence. The nature of the job of route managers is given in a four-page pamphlet, prepared by the company, called “Route Manager Position Description,” the pertinent parts of which read: A. BASIC PURPOSE A Manager achieves objectives through others. As a Route Manager, your purpose is to meet the sales plan; and you achieve this objective through the skillful MANAGEMENT OF YOUR JOB AND THE MANAGEMENT OF YOUR PEOPLE. These then are your functions as Pepsi-Cola Route Manager. Within these functions — managing your job and managing your people — you are accountable to your District Manager for the execution and completion of various tasks and activities which will make it possible for you to achieve your sales objectives. Xxxx Distinction is evident in the work of the route managers which sets them apart from supervisors in general. Unlike supervisors who basically merely direct operating employees in line with set tasks assigned to them, route managers are responsible for the success of the company’s main line of business through management of their respective sales teams. Such management necessarily involves the planning, direction, operation and evaluation of their individual teams and areas which the work of supervisors does not entail. The route managers cannot thus possibly be classified as mere supervisors because their work does not only involve, but goes far beyond, the simple direction or supervision of operating employees to accomplish objectives set by those above them. While route managers do not appear to have the power to hire and fire people (the evidence shows that they only “recommended” or “endorsed” the taking of

CONSTITUTIONAL LAW 2 CASE DIGESTS

disciplinary action against certain employees), this is because thisis a function of the Human Resources or Personnel Department of the company. # 2: Constitutionality of Art. 245 Art.245 is the result of the amendment of the Labor Code in 1989 by R.A. No. 6715, otherwise known as the Herrera-Veloso Law. Unlike the Industrial Peace Act or the provisions of the Labor Code which it superseded, R.A. No. 6715 provides separate definitions of the terms “managerial” and “supervisory employees,” as follows: Art. 212. Definitions. . . . (m) “managerial employee” is one who is vested with powers or prerogatives to lay down and execute management policies and/or to hire transfer, suspend, lay off, recall, discharge, assign or discipline employees. Supervisory employees are those who, in the interest of the employer, effectively recommend such managerial actions if the exercise of such authority is not merely routinary or clerical in nature but requires the use of independent judgment. All employees not falling within any of the above definitions are considered rank-and-file employees for purposes of this Book. The distinction between top and middle managers, who set management policy, and front-line supervisors, who are merely responsible for ensuring that such policies are carried out by the rank and file, is articulated in the present definition. 30 When read in relation to this definition in Art. 212(m), it will be seen that Art. 245 faithfully carries out the intent of the Constitutional Commission in framing Art. III, §8 of the fundamental law. *Framer’s Intent: MR. LERUM. My amendment is on Section 7, page 2, line 19, which is to insert between the words “people” and “to” the following: WHETHER EMPLOYED BY THE STATE OR PRIVATE ESTABLISHMENTS. In other words, the section will now read as follows: “The right of the people WHETHER EMPLOYED BY THE STATE OR PRIVATE ESTABLISHMENTS to form associations, unions, or societies for purposes not contrary to law shall not be abridged.” CONSTITUTIONAL LAW 2 CASE DIGESTS

Nor is the guarantee of organizational right in Art. III, §8 infringed by a ban against managerial employees forming a union. The right guaranteed in Art. III, §8 is subject to the condition that its exercise should be for purposes “not contrary to law.” In the case of Art. 245, there is a rational basis for prohibiting managerial employees from forming or joining labor organizations. PETITION is DISMISSED

. FREEDOM OF RELIGION

Islamic Da'wah Council Of The Philippines Vs. Executive Secretary 405 SCRA 497 GR 153888 July 9, 2003 Facts: Petitioner Islamic Da'wah Council of the Philippines, Inc. (IDCP) is a corporation that operates under Department of Social Welfare and Development, a non-governmental organization that extends voluntary services to the Filipino people, especially to Muslim communities. It claims to be a federation of national Islamic organizations and an active member of international organizations such as the Regional Islamic Da'wah Council of Southeast Asia and the Pacific (RISEAP) and The World Assembly of Muslim Youth. The RISEAP accredited petitioner to issue halal certifications in the Philippines. Thus, among the functions petitioner carries out is to conduct seminars, orient manufacturers on halal food and issue halal certifications to qualified products and manufacturers. Petitioner alleges that, the actual need to certify food products as halal and also due to halal food producers' request, petitioner formulated in 1995 internal rules and procedures based on the Qur'an and the Sunnah for the analysis of food, CONSTITUTIONAL LAW 2 CASE DIGESTS

inspection thereof and issuance of halal certifications. In that same year, petitioner began to issue, for a fee, certifications to qualified products and food manufacturers. Petitioner even adopted for use on its halal certificates a distinct sign or logo registered in the Philippine Patent Office. On 2001, respondent Office of the Executive Secretary issued EO 465 creating the Philippine Halal Certification Scheme and designating respondent OMA to oversee its implementation. Under the EO, respondent OMA has the exclusive authority to issue halal certificates and perform other related regulatory activities.

Issue: Whether or Not EO violates the constitutional provision on the separation of Church and State.

Held: It is unconstitutional for the government to formulate policies and guidelines on the halal certification scheme because said scheme is a function only religious organizations, entity or scholars can lawfully and validly perform for the Muslims. According to petitioner, a food product becomes halal only after the performance of Islamic religious ritual and prayer. Thus, only practicing Muslims are qualified to slaughter animals for food. A government agency like herein respondent OMA cannot therefore perform a religious function like certifying qualified food products as halal. Without doubt, classifying a food product as halal is a religious function because the standards used are drawn from the Qur'an and Islamic beliefs. By giving OMA the exclusive power to classify food products as halal, EO 46 encroached on the religious freedom of Muslim organizations like herein petitioner to interpret for Filipino Muslims what food products are fit for Muslim consumption. Also, by arrogating to itself the task of issuing halal certifications, the State has in effect forced Muslims to accept its own interpretation of the Qur'an and Sunnah on halal food.

CONSTITUTIONAL LAW 2 CASE DIGESTS

In the case at bar, we find no compelling justification for the government to deprive Muslim organizations, like herein petitioner, of their religious right to classify a product as halal, even on the premise that the health of Muslim Filipinos can be effectively protected by assigning to OMA the exclusive power to issue halal certifications. The protection and promotion of the muslim Filipinos' right to health are already provided for in existing laws and ministered to by government agencies charged with ensuring that food products released in the market are fit for human consumption, properly labeled and safe. Unlike EO 46, these laws do not encroach on the religious freedom of Muslims

GARCES VS. ESTENZO [104 SCRA 510; G.R. L-53487; 25 MAY 1981] Facts: Two resolutions of the Barangay Council of Valencia, Ormoc City were passed: a. Resolution No. 5- Reviving the traditional socio-religious celebration every fifth of April. This provided for the acquisition of the image of San Vicente Ferrer and the construction of a waiting shed. Funds for the said projects will be obtained through the selling of tickets and cash donations. b. Resolution No. 6- The chairman or hermano mayor of the fiesta would be the caretaker of the image of San Vicente Ferrer and that the image would remain in his residence for one year and until the election of his successor. The image would be made available to the Catholic Church during the celebration of the saint’s feast day.

These resolutions have been ratified by 272 voters, and said projects were implemented. The image was temporarily placed in the altar of the Catholic Church of the barangay. However, after a mass, Father Sergio Marilao Osmeña

CONSTITUTIONAL LAW 2 CASE DIGESTS

refused to return the image to the barangay council, as it was the church’s property since church funds were used in its acquisition.

Resolution No. 10 was passed for the authorization of hiring a lawyer for the replevin case against the priest for the recovery of the image. Resolution No. 12 appointed Brgy. Captain Veloso as a representative to the case. The priest, in his answer assailed the constitutionality of the said resolutions. The priest with Andres Garces, a member of the Aglipayan Church, contends that Sec. 8 Article IV1 and Sec 18(2) Article VIII) 2 of the constitution was violated. Issue: Whether or Not any freedom of religion clause in the Constitution violated. Held: No. As said by the Court this case is a petty quarrel over the custody of the image. The image was purchased in connection with the celebration of the barrio fiesta and not for the purpose of favoring any religion nor interfering with religious matters or beliefs of the barrio residents. Any activity intended to facilitate the worship of the patron saint(such as the acquisition) is not illegal. Practically, the image was placed in a layman’s custody so that it could easily be made available to any family desiring to borrow the image in connection with prayers and novena. It was the council’s funds that were used to buy the image, therefore it is their property. Right of the determination of custody is their right, and even if they decided to give it to the Church, there is no violation of the Constitution, since private funds were used. Not every government activity which involves the expenditure of public funds and which has some religious tint is violative of the constitutional provisions regarding separation of church and state, freedom of worship and banning the use of public money or property.

Aglipay v. Ruiz, GR No. L-45459, March 13, 1937 Facts:

CONSTITUTIONAL LAW 2 CASE DIGESTS

Petitioner Aglipay, the head of Phil. Independent Church, filed a writ of prohibition against respondent Ruiz, the Director of Post, enjoining the latter from issuing and selling postage stamps commemorative of the 33rd Intl Eucharistic Congress organized by the Roman Catholic. The petitioner invokes that such issuance and selling, as authorized by Act 4052 by the Phil. Legislature, contemplates religious purpose – for the benefit of a particular sect or church. Hence, this petition. Issue: Whether or not the issuing and selling of commemorative stamps is constitutional? Held/Reason: The Court said YES, the issuing and selling of commemorative stamps by the respondent does not contemplate any favor upon a particular sect or church, but the purpose was only ‘to advertise the Philippines and attract more tourist’ and the government just took advantage of an event considered of international importance, thus, not violating the Constitution on its provision on the separation of the Church and State. Moreover, the Court stressed that ‘Religious freedom, as a constitutional mandate is not inhibition of profound reverence for religion and is not denial of its influence in human affairs’. Emphasizing that, ‘when the Filipino people ‘implored the aid of Divine Providence’, they thereby manifested reliance upon Him who guides the destinies of men and nations. The elevating influence of religion in human society is recognized here as elsewhere. In fact, certain general concessions are indiscriminately accorded to religious sects and denominations.

ANG LADLAD VS. COMELEC Facts: Petitioner is a national organization which represents the lesbians, gays, bisexuals, and trans-genders. It filed a petition for accreditation as a party-list organization to public respondent. However, due to moral grounds, the latter denied the said CONSTITUTIONAL LAW 2 CASE DIGESTS

petition. To buttress their denial, COMELEC cited certain biblical and quranic passages in their decision. It also stated that since their ways are immoral and contrary to public policy, they are considered nuissance. In fact, their acts are even punishable under the Revised Penal Code in its Article 201. A motion for reconsideration being denied, Petitioner filed this instant Petition on Certiorari under Rule 65 of the ROC. Ang Ladlad argued that the denial of accreditation, insofar as it justified the exclusion by using religious dogma, violated the constitutional guarantees against the establishment of religion. Petitioner also claimed that the Assailed Resolutions contravened its constitutional rights to privacy, freedom of speech and assembly, and equal protection of laws, as well as constituted violations of the Philippines’ international obligations against discrimination based on sexual orientation.

In its Comment, the COMELEC reiterated that petitioner does not have a concrete and genuine national political agenda to benefit the nation and that the petition was validly dismissed on moral grounds. It also argued for the first time that the LGBT sector is not among the sectors enumerated by the Constitution and RA 7941, and that petitioner made untruthful statements in its petition when it alleged its national existence contrary to actual verification reports by COMELEC’s field personnel. Issue: WON Respondent violated the Non-establishment clause of the Constitution; WON Respondent erred in denying Petitioners application on moral and legal grounds. Held: Respondent mistakenly opines that our ruling in Ang Bagong Bayani stands for the proposition that only those sectors specifically enumerated in the law or related to said sectors (labor, peasant, fisherfolk, urban poor, indigenous cultural CONSTITUTIONAL LAW 2 CASE DIGESTS

communities, elderly, handicapped, women, youth, veterans, overseas workers, and professionals) may be registered under the party-list system. As we explicitly ruled in Ang Bagong Bayani-OFW Labor Party v. Commission on Elections, “the enumeration of marginalized and under-represented sectors is not exclusive”. The crucial element is not whether a sector is specifically enumerated, but whether a particular organization complies with the requirements of the Constitution and RA 7941. Our Constitution provides in Article III, Section 5 that “[n]o law shall be made respecting an establishment of religion, or prohibiting the free exercise thereof.” At bottom, what our non-establishment clause calls for is “government neutrality in religious matters.” Clearly, “governmental reliance on religious justification is inconsistent with this policy of neutrality.” We thus find that it was grave violation of the non-establishment clause for the COMELEC to utilize the Bible and the Koran to justify the exclusion of Ang Ladlad. Be it noted that government action must have a secular purpose. Respondent has failed to explain what societal ills are sought to be prevented, or why special protection is required for the youth. Neither has the COMELEC condescended to justify its position that petitioner’s admission into the party-list system would be so harmful as to irreparably damage the moral fabric of society. We also find the COMELEC’s reference to purported violations of our penal and civil laws flimsy, at best; disingenuous, at worst. Article 694 of the Civil Code defines a nuisance as “any act, omission, establishment, condition of property, or anything else which shocks, defies, or disregards decency or morality,” the remedies for which are a prosecution under the Revised Penal Code or any local ordinance, a civil action, or abatement without judicial proceedings. A violation of Article 201 of the Revised Penal Code, on the other hand, requires proof beyond reasonable doubt to support a criminal conviction. It hardly needs to be emphasized that mere allegation of violation of laws is not proof, and a mere blanket invocation of public morals cannot replace the institution of civil or criminal proceedings and a judicial determination of liability or culpability. CONSTITUTIONAL LAW 2 CASE DIGESTS

As such, we hold that moral disapproval, without more, is not a sufficient governmental interest to justify exclusion of homosexuals from participation in the party-list system. The denial of Ang Ladlad’s registration on purely moral grounds amounts more to a statement of dislike and disapproval of homosexuals, rather than a tool to further any substantial public interest.

ANTI-EVOLUTION

EPPERSON VS ARKANSAS Facts of the case The Arkansas legislature passed a law prohibiting teachers in public or statesupported schools from teaching, or using textbooks that teach, human evolution. Epperson, a public school teacher, sued, claiming the law violated her First Amendment right to free speech as well as the Establishment Clause. The State Chancery Court ruled that it violated her free speech rights; the State Supreme Court reversed. Question Does a law forbidding the teaching of evolution violate either the free speech rights of teachers or the Establishment clause of the First Amendment? Conclusion Yes. Seven members of the Court held that the statute violated the Establishment clause. Writing for the Court, Justice Abe Fortas stated that the law had been based solely on the beliefs of fundamentalist Christians, who felt that evolutionary theories directly contradicted the biblical account of Creation. This use of state power to prohibit the teaching of material objectionable to a particular sect ammounted to an unconstitutional Establishment of religion. Justice Fortas wrote, CONSTITUTIONAL LAW 2 CASE DIGESTS

"The State's undoubted right to prescribe the curriculum for its public schools does not carry with it the right to prohibit, on pain of criminal penalty, the teaching of a scientific theory or doctrine where that prohibition is based upon reasons that violate the First Amendment." The two other members of the Court concurred in the result, writing that it violated either the Due Process clause of the 14th Amendment (because it was unconstitutionally vague) or the Free Speech clause of the First Amendment.

PRAYER & BIBLE READING PUBLIC SCHOOLS ENGEL VS VITALE Facts of the case The New York State Board of Regents authorized a short, voluntary prayer for recitation at the start of each school day. A group of organizations joined forces in challenging the prayer, claiming that it violated the Establishment Clause of the First Amendment. The New York Court of Appeals rejected their arguments. Question Does the reading of a nondenominational prayer at the start of the school day violate the "establishment of religion" clause of the First Amendment? Conclusion The state cannot hold prayers in public schools, even if participation is not required and the prayer is not tied to a particular religion. In an opinion authored by Hugo L. Black, the Court held that respondent's decision to use its school system to facilitate recitation of the official prayer violated the Establishment Clause. Specifically, the policy breached the constitutional wall of separation between church and state. The Court ruled that the constitutional prohibition of laws establishing religion meant that government had no business drafting formal prayers for any segment of its population to CONSTITUTIONAL LAW 2 CASE DIGESTS

repeat in a government-sponsored religious program. The Court held that respondent's provision of the contested daily prayer was inconsistent with the Establishment Clause. Justice Douglas concurred in the judgment on the ground that the state's financing a religious exercise violated the First Amendment. Justice Stewart dissented, arguing that no "official religion" was established by permitting those who want to say a prayer to say it.

ABINGTON SCHOOL DISTRICT VS SCHEMP Facts of the case Under Pennsylvania law, public schools were required to read from the bible at the opening of each school day. The school district sought to enjoin enforcement of the statute. The district court ruled that the statute violated the First Amendment, even after the statute had been amended to permit a student to excuse himself. The Court consolidated this case with one involving Maryland atheists who challenged a city rule that provided for opening exercises in the public schools that consisted primarily of reading a chapter from the bible and the Lord's Prayer. The state's highest court held the exercise did not violate the First Amendment. The religious character of the exercise was admitted by the state. Question Did the Pennsylvania law requiring public school students to participate in classroom religious exercises violate the religious freedom of students as protected by the First and Fourteenth Amendments?

Conclusion CONSTITUTIONAL LAW 2 CASE DIGESTS

Public schools cannot sponsor Bible readings and recitations of the Lord’s Prayer under the First Amendment’s Establishment Clause. In an opinion authored by Justice Clark, the majority concluded that, in both cases, the laws required religious exercises and such exercises directly violated the First Amendment. The Court affirmed the Pennsylvania decision, and reversed and remanded the Maryland decision because the mandatory reading from the bible before school each day was found to be unconstitutional. Justice Stewart dissented, expressing the view that on the records it could not be said that the Establishment Clause had necessarily been violated. He would remand both cases for further hearings.

STONE VS GRAHAM Facts of the case Sydell Stone and a number of other parents challenged a Kentucky state law that required the posting of a copy of the Ten Commandments in each public school classroom. They filed a claim against James Graham, the superintendent of public schools in Kentucky.

Question Did the Kentucky statute violate the Establishment Clause of the First Amendment? Conclusion In a 5-to-4 per curiam decision, the Court ruled that the Kentucky law violated the first part of the test established in Lemon v. Kurtzman, and thus violated the Establishment Clause of the Constitution. The Court found that the requirement that the Ten Commandments be posted "had no secular legislative purpose" and was "plainly religious in nature." The Court noted that the Commandments did CONSTITUTIONAL LAW 2 CASE DIGESTS

not confine themselves to arguably secular matters (such as murder, stealing, etc.), but rather concerned matters such as the worship of God and the observance of the Sabbath Day.

TAX EXEMPTION MUELLER VS ALLEN Facts of the case A Minnesota law allowed taxpayers to deduct from their state income tax expenses incurred in providing tuition, textbooks, and transportation for their children's elementary or secondary school education. Parents who sent their children to parochial school also qualified for the deductions. Question Did the law violate the Establishment Clause of the First Amendment? Conclusion No. The Court held that the law did not have "the primary effect of advancing the sectarian aims of the non-public schools," nor did it "excessively entangle" the state in religion. Most importantly, argued Justice Rehnquist, the deductions were available to all parents; in effect, Minnesota did not "confer any imprimatur of state approval" on religious schools. Thus, the law passed the Court's threepronged test announced in Lemon v. Kurtzman (1971) used to evaluate claims regarding the Establishment Clause.

LEMON VS KURTZMAN Facts of the case Both Pennsylvania and Rhode Island adopted statutes that provided for the state to pay for aspects of non-secular, non-public education. The Pennsylvania statute CONSTITUTIONAL LAW 2 CASE DIGESTS

was passed in 1968 and provided funding for non-public elementary and secondary school teachers’ salaries, textbooks, and instructional materials for secular subjects. Rhode Island’s statute was passed in 1969 and provided state financial support for non-public elementary schools in the form of supplementing 15% of teachers’ annual salaries. The appellants in the Pennsylvania case represented citizens and taxpayers in Pennsylvania who believed that the statute violated the separation of church and state described in the First Amendment. Appellant Lemon also had a child in Pennsylvania public school. The district court granted the state officials’ motion to dismiss the case. In the Rhode Island case, the appellees were citizens and tax payers of Rhode Island who sued to have the statute in question declared unconstitutional by arguing that it violated the Establishment Clause of the First Amendment. The district court found in favor of the appellees and held that the statute violated the First Amendment. Question Do statutes that provide state funding for non-public, non-secular schools violate the Establishment Clause of the First Amendment? Conclusion Yes. Chief Justice Warren E. Burger delivered the opinion for the 8-0 majority as to the Pennsylvania statute and 8-1 as to the Rhode Island statute. The Court held that a statute must pass a three-pronged test in order to avoid violating the Establishment Clause. The statute must have a secular legislative purpose, its principal or primary effect must be one that neither promotes nor inhibits religion, and it must not foster “excessive government entanglement with religion.” The Court held that both the state statutes in question had secular legislative purposes because they reflected the desire of the states to ensure minimum secular education requirements were being met in the non-public schools. The Court did not reach a holding regarding the second prong of the test, but it did find that the statutes constituted an excessive government entanglement with religion. In the Rhode Island program, the amount of oversight CONSTITUTIONAL LAW 2 CASE DIGESTS

of teachers and curricula required to ensure that there is no unnecessary injection of religion into secular topics would require the government to become excessively involved in the nuances of religious education. The same danger holds true for the Pennsylvania statute, which additionally provides state funding directly to a church-related organization. Government financial involvement in such institutions inevitably leads to “an intimate and continuing relationship” between church and state. The Court also noted the potential political implications of public funding, as there is a risk of religious issues becoming politically divisive.

In his concurring opinion, Justice William O. Douglas wrote that the intrusion of the government into the running of non-public schools through grants and other funding creates the entanglement that the Establishment Clause prohibits. He also argued that non-secular schools are so thoroughly governed by religious ideologies that any amount of public funding supports those doctrines, which the Framers of the Constitution dictated the government must not do. Justice Hugo L. Black joined in the concurrence, and Justice Thurgood Marshall joined in the parts relating to case numbers 569 and 570. Justice William J. Brennan, Jr. wrote a separate concurrence in which he argued that the danger was not only that religion would infiltrate the government, but also that the government would push secularization onto religious creeds. An analysis of the statutes in question shows that they impermissible involve the government in “essentially religious activities,” which the Establishment Clause is meant to prevent. In his opinion concurring in part and dissenting in part, Justice Byron R. White wrote that the majority opinion goes too far and, in restricting the use of state funds in nonsecular schools, creates an obstacle to the use of public funds for secular education. He argued that there was no proof that religion would invade secular education or that the government oversight of the use of public funds would be so extensive as to constitute entanglement.

CONSTITUTIONAL LAW 2 CASE DIGESTS

Justice Thurgood Marshall did not participate in the discussion or decision of case number 89.

WALLACE VS JAFFREE Facts of the case An Alabama law authorized teachers to conduct regular religious prayer services and activities in school classrooms during the school day. Three of Jaffree's children attended public schools in Mobile. Question Did Alabama law violate the First Amendment's Establishment Clause? Conclusion Yes. The Court determined the constitutionality of Alabama's prayer and meditation statute by applying the secular purpose test, which asked if the state's actual purpose was to endorse or disapprove of religion. The Court held that Alabama's passage of the prayer and meditation statute was not only a deviation from the state's duty to maintain absolute neutrality toward religion, but was an affirmative endorsement of religion. As such, the statute clearly lacked any secular purpose as it sought to establish religion in public schools, thereby violating the First Amendment's Establishment Clause.

INTRAMURAL RELIGIOUS DISPUTES

FONACIER VS. COURT OF APPEALS [96 PHIL 417; G.R. L-5917; 28 JAN 1955] Facts:

CONSTITUTIONAL LAW 2 CASE DIGESTS

Case was filed by Iglesia Filipina Independiente (IFI), represented by its supreme bishop Gerardo Bayaca, against Bishop Fonacier seeking to render an accounting of his administration of all the temporal properties and to recover the same on the ground that he ceased to be the supreme bishop of IFI. Isabelo De los Reyes Jr. had been elected as the Supreme Bishop. Petitioner claims that he was not properly removed as Supreme Bishop and his legal successor was Juan Jamias. He claims that the there was an accounting of his administration and was turned over to bishop Jamias. Also, that Isabelo De los Reyes and Bayaca have abandoned their faith and formally joined the Prostestant Episcopal Church of America. CFI rendered judgment declaring Isabelo De Los Reyes, Jr. as the sole and legitimate Supreme Bishop of IFI and ordered Fonacier to render an accounting of his admistration CA affirmed the decision of the CFI Issue: Whether or not the petitioner should still be regarded as the legitimate supreme bishop of IFI. Held: Supreme Court affirmed CA’s decision. The legitimate Supreme Bishop of IFI is Isabelo De los Reyes, Jr. The Supreme Court affirms the validity of the election of Bishop Delos Reyes as the Supreme Bishop based on their internal laws To finally dispose of the property issue, the Court, citing Watson v. Jones,368 declared that the rule in property controversies within religious congregations strictly independent of any other superior ecclesiastical association (such as the Philippine Independent Church) is that the rules for resolving such controversies should be those of any voluntary association. If the congregation adopts the majority rule then the majority should prevail; if it adopts adherence

CONSTITUTIONAL LAW 2 CASE DIGESTS

to duly constituted authorities within the congregation, then that should be followed.

Taruc vs. Bishop Dela Cruz G.R. No. 144801. March 10, 2005 Facts: Petitioners were lay members of the Philippine Independent Church (PIC). On June 28, 1993, Bishop de la Cruz declared petitioners expelled/excommunicated from the Philippine Independent Church. Because of the order of expulsion/excommunication, petitioners filed a complaint for damages with preliminary injunction against Bishop de la Cruz before the Regional Trial Court.They contended that their expulsion was illegal because it was done without trial thus violating their right to due process of law. Issue: Whether or not there was a violation of religious rights in this case? Held: No. The expulsion/excommunication of members of a religious institution/organization is a matter best left to the discretion of the officials, and the laws and canons, of said institution/organization. It is not for the courts to exercise control over church authorities in the performance of their discretionary and official functions. Rather, it is for the members of religious institutions/organizations to conform to just church regulations. “Civil Courts will not interfere in the internal affairs of a religious organization except for the protection of civil or property rights. Those rights may be the subject of litigation in a civil court, and the courts have jurisdiction to determine controverted claims to the title, use, or possession of church property.” Obviously, there was no violation of a civil right in the present case.

CONSTITUTIONAL LAW 2 CASE DIGESTS

FREE EXERCISE CLAUSE Estrada vs. Escritor AM P-02-1651, August 4, 2003 FACTS: Soledad Escritor is a court interpreter since 1999 in the RTC of Las Pinas City. Alejandro Estrada, the complainant, wrote to Judge Jose F. Caoibes, presiding judge of Branch 253, RTC of Las Pinas City, requesting for an investigation of rumors that Escritor has been living with Luciano Quilapio Jr., a man not her husband, and had eventually begotten a son. Escritor’s husband, who had lived with another woman, died a year before she entered into the judiciary. On the other hand, Quilapio is still legally married to another woman. Estrada is not related to either Escritor or Quilapio and is not a resident of Las Pinas but of Bacoor, Cavite. According to the complainant, respondent should not be allowed to remain employed in the judiciary for it will appear as if the court allows such act. Escritor is a member of the religious sect known as the Jehovah’s Witnesses and the Watch Tower and Bible Tract Society where her conjugal arrangement with Quilapio is in conformity with their religious beliefs. After ten years of living together, she executed on July 28, 1991 a “Declaration of Pledging Faithfulness” which was approved by the congregation. Such declaration is effective when legal impediments render it impossible for a couple to legalize their union. Gregorio, Salazar, a member of the Jehovah’s Witnesses since 1985 and has been a presiding minister since 1991, testified and explained the import of and procedures for executing the declaration which was completely executed by Escritor and Quilapio’s in Atimonan, Quezon and was signed by three witnesses and recorded in Watch Tower Central Office. CONSTITUTIONAL LAW 2 CASE DIGESTS

ISSUE: Whether or not respondent should be found guilty of the administrative charge of “gross and immoral conduct” and be penalized by the State for such conjugal arrangement. HELD: A distinction between public and secular morality and religious morality should be kept in mind. The jurisdiction of the Court extends only to public and secular morality. The Court states that our Constitution adheres the benevolent neutrality approach that gives room for accommodation of religious exercises as required by the Free Exercise Clause. This benevolent neutrality could allow for accommodation of morality based on religion, provided it does not offend compelling state interests. The state’s interest is the preservation of the integrity of the judiciary by maintaining among its ranks a high standard of morality and decency. “There is nothing in the OCA’s (Office of the Court Administrator) memorandum to the Court that demonstrates how this interest is so compelling that it should override respondent’s plea of religious freedom. Indeed, it is inappropriate for the complainant, a private person, to present evidence on the compelling interest of the state. The burden of evidence should be discharged by the proper agency of the government which is the Office of the Solicitor General”. In order to properly settle the case at bar, it is essential that the government be given an opportunity to demonstrate the compelling state interest it seeks to uphold in opposing the respondent’s position that her conjugal arrangement is not immoral and punishable as it is within the scope of free exercise protection. The Court could not prohibit and punish her conduct where the Free Exercise Clause protects it, since this would be an unconstitutional encroachment of her right to religious freedom. Furthermore, the court cannot simply take a passing look at respondent’s claim of religious freedom but must also apply the “compelling state interest” test. CONSTITUTIONAL LAW 2 CASE DIGESTS

IN VIEW WHEREOF, the case is REMANDED to the Office of the Court Administrator. The Solicitor General is ordered to intervene in the case where it will be given the opportunity (a) to examine the sincerity and centrality of respondent's claimed religious belief and practice; (b) to present evidence on the state's "compelling interest" to override respondent's religious belief and practice; and (c) to show that the means the state adopts in pursuing its interest is the least restrictive to respondent's religious freedom. The rehearing should be concluded thirty (30) days from the Office of the Court Administrator's receipt of this Decision. German v. Barangan G.R. No. L-68828 March 27, 1985 Facts: 1. In the afternoon of October 2, 1984, petitioners, composed of about 50 businessmen, students and office employees converged at J.P. Laurel Street, Manila, for the purpose of hearing Mass at the St. Jude Chapel which adjoins the Malacañang grounds located in the same street. Wearing yellow T-shirts, they started to march down with raised clenched fists and shouts of anti-government invectives. The marchers were barred by respondent Major Lariosa, upon orders of his superiors and co-respondent Gen. Santiago Barangan, from proceeding any further, on the ground that St. Jude Chapel was located within the Malacañang security area. Despite plea, they were not allowed in the church. 2. Because of the alleged warning given them by respondent Major Lariosa that any similar attempt by petitioners to enter the church in the future would likewise be prevented, petitioners took this present recourse. 3. Petitioners' alleged purpose in converging at J.P. Laurel Street was to pray and hear mass at St. Jude church. At the hearing of this petition, respondents assured petitioners and the Court that they have never restricted, and will never restrict, any person or persons from entering and worshipping at said church. They maintain, however, that petitioners' intention was not really to perform an act of religious worship, but to conduct an anti-government demonstration at a place close to the very residence and offices of the President of the Republic. CONSTITUTIONAL LAW 2 CASE DIGESTS

4. Invoking their constitutional freedom to religious worship and locomotion, petitioners seek the issuance of [1] a writ of mandamus to compel respondents to allow them to enter and pray inside St. Jude Chapel located at J.P. Laurel Street, Manila; and [2] a writ of injunction to enjoin respondents from preventing them from getting into and praying in said church. ISSUE: Whether or not the restriction to petitioners to attend church is a violation of their freedom to religious worship RULING NO. 1. The restriction imposed on the use of J.P. Laurel Street, was established in the interest of national security. Petitioners are not denied or restrained of their freedom of belief or choice of their religion, but only in the manner by which they had attempted to translate the same into action. This curtailment is in accord with the pronouncement of this Court in Gerona v. Secretary of Education. 2. While it is beyond debate that every citizen has the undeniable and inviolable right to religious freedom, the exercise thereof, and of all fundamental rights for that matter, must be done in good faith. As Article 19 of the Civil Code admonishes: "Every person must in the exercise of his rights and in the performance of his duties ... observe honesty and good faith."

FLAG SALUTE EBRALINAG VS. DIVISION SUPERINTENDENT OF CEBU [219 SCRA 256 ; G.R. NO. 95770; 1 MAR 1993] CONSTITUTIONAL LAW 2 CASE DIGESTS

Facts: Two special civil actions for certiorari, Mandamus and Prohibition were filed and consolidated for raising same issue. Petitioners allege that the public respondents acted without or in excess of their jurisdiction and with grave abuse of discretion. Respondents ordered expulsion of 68 HS and GS students of Bantayan, Pinamungajan, Caracar, Taburan and Asturias in Cebu. Public school authorities expelled these students for refusing to salute the flag, sing the national anthem and recite the “Panatang Makabayan” required by RA1265. They are Jehovah’s Witnesses believing that by doing these is religious worship/devotion akin to idolatry against their teachings. They contend that to compel transcends constitutional limits and invades protection against official control and religious freedom. The respondents relied on the precedence of Gerona et al v. Secretary of Education. Gerona doctrine provides that we are a system of separation of the church and state and the flag is devoid of religious significance and it doesn’t involve any religious ceremony. The freedom of religious belief guaranteed by the Constitution does not mean exception from non-discriminatory laws like the saluting of flag and singing national anthem. This exemption disrupts school discipline and demoralizes the teachings of civic consciousness and duties of citizenship. Issue: Whether or Not religious freedom has been violated. Held: Religious freedom is a fundamental right of highest priority. The 2 fold aspect of right to religious worship is: 1.) Freedom to believe which is an absolute act within the realm of thought. 2.) Freedom to act on one’s belief regulated and translated to external acts. The only limitation to religious freedom is the existence of grave and present danger to public safety, morals, health and interests where State has right to prevent. The expulsion of the petitioners from the school is not justified. The 30 yr old previous GERONA decision of expelling and dismissing students and teachers who refuse to obey RA1265 is violates exercise of freedom of speech and CONSTITUTIONAL LAW 2 CASE DIGESTS

religious profession and worship. Jehovah’s Witnesses may be exempted from observing the flag ceremony but this right does not give them the right to disrupt such ceremonies. In the case at bar, the Students expelled were only standing quietly during ceremonies. By observing the ceremonies quietly, it doesn’t present any danger so evil and imminent to justify their expulsion. What the petitioner’s request is exemption from flag ceremonies and not exclusion from public schools. The expulsion of the students by reason of their religious beliefs is also a violation of a citizen’s right to free education. The non-observance of the flag ceremony does not totally constitute ignorance of patriotism and civic consciousness. Love for country and admiration for national heroes, civic consciousness and form of government are part of the school curricula. Therefore, expulsion due to religious beliefs is unjustified. Petition for Certiorari and Prohibition is GRANTED. Expulsion is ANNULLED

FREEDOM TO PROPAGATE RELIGIOUS DOCTRINES American Bible Society v City of Manila GR No. L-9637, April 30, 1957 FACTS: In the course of its ministry, the Philippine agency of American Bible Society (a foreign, non-stock, non-profit, religious, missionary corporation) has been distributing and selling bibles and/or gospel portions thereof throughout the Philippines. The acting City Treasurer of Manila informed plaintiff that it was conducting the business of general merchandise CONSTITUTIONAL LAW 2 CASE DIGESTS

since November 1945, without providing itself with the necessary Mayor’s permit and municipal license, in violation of Ordinance No. 3000, as amended, and Ordinances Nos. 2529, 3028 and 3364. The society paid such under protest and filed suit questioning the legality of the ordinances under which the fees are being collected. ISSUES: Whether or not the ordinances of the City of Manila are constitutional and valid Whether the provisions of said ordinances are applicable or not to the case at bar. RULING: 1. Yes, they are constitutional. The ordinances do not deprive defendant of his constitutional right of the free exercise and enjoyment of religious profession and worship, even though it prohibits him from introducing and carrying out a scheme or purpose which he sees fit to claim as part of his religious system. It seems clear, therefore, that Ordinance No. 3000 cannot be considered unconstitutional, even if applied to plaintiff society. 2. The ordinance is inapplicable to said business, trade or occupation of the plaintiff. Even if religious groups and the press are not altogether free from the burdens of the government, the act of distributing and selling bibles is purely religious and does not fall under Section 27e of the Tax Code (CA 466). The fact that the price of bibles, etc. are a little higher than actual cost of the same does not necessarily mean it is already engaged in business for profit. Thus, the Ordinances are not applicable to the Society.

FREEDOM OF EXPRESSION US v. FELIPE BUSTOS ET AL., GR No. 12592, 1918-03-08 Facts: In the latter part of 1915, numerous citizens of the Province of Pampanga assembled, and prepared and signed a petition to the Executive Secretary through CONSTITUTIONAL LAW 2 CASE DIGESTS

the law office of Crossfield & O'Brien, and five individuals signed affidavits, charging Roman Punsalan, justice of the peace of Macabebe and Masantol, Pampanga, with malfeasance in office and asking for his removal. The justice of the peace was notified and denied the charges. The judge of first instance found the first count... not proved and counts 2 and 3 established. In view of this result, the judge, the Honorable Percy M. Moir, was of the opinion "that it must be, and it is hereby, recommended to the GovernorGeneral that the respondent be removed from his position as justice of the peace of Macabebe and Masantol, Province of Pampanga, and it is ordered that the proceedings had in this case be transmitted to the Executive Secretary." Later the justice of the peace filed a motion for a new trial... the judge of first instance granted the motion and reopened the hearing; "That on or about the month of December, 1915, in the municipality of Macabebe, Pampanga, P. I., the said accused, voluntarily, illegally, and criminally and with malicious intent to prejudice and defame Mr. Roman Punsalan Serrano The Honorable Percy M. Moir found all the defendants, with the exception of Felix Fernandez, Juan S. Alfonso, Restituto Garcia, and Manuel Mallari, guilty and sentenced each of them to pay a fine of P10 and one thirty-second part of the costs, or to suffer subsidiary... imprisonment in case of insolvency. "1. The court erred in overruling the motion of the convicted defendants for a new trial. "2. The court erred in refusing to permit the defendants to retire the objection inadvertently interposed by their counsel to the admission in evidence of the expediente administrativo out of which the accusation in this case arose.

CONSTITUTIONAL LAW 2 CASE DIGESTS

"3. The court erred in sustaining the objection of the prosecution to the introduction in evidence by the accused of the affidavits upon which the petition forming the basis of the libelous charge was based. "4. The court erred in not holding that the alleged libelous statement was unqualifiedly privileged. "5. The court erred in assuming and impliedly holding that the burden was on the defendants to show that the alleged libelous statements were true and free from malice. "6. The court erred in not acquitting the defendants. "7. The evidence adduced fails to show the guilt of the defendants beyond a reasonable doubt This is especially true of all the defendants, except Felipe Bustos, Dionisio Mallari, and Jose T. Reyes." Issues: This appeal presents the specific question of whether or not the defendants and appellants are guilty of a libel of Roman Punsalan, justice of the peace of Macabebe and Masantol, Province of Pampanga. Ruling: Express malice has not been proved by the prosecution, further, although the charges are probably not true as to the justice of the peace, they were believed to be true by the... petitioners. Good faith surrounded their action. Probable cause for them to think that malfeasance or misfeasance in office existed is apparent. The ends and the motives of these citizens to secure the removal from office of a person thought to be venal were justifiable. In no... way did they abuse the privilege. CONSTITUTIONAL LAW 2 CASE DIGESTS

We find the defendants and appellants entitled to the protection of the rules concerning qualified privilege, growing out of constitutional guaranties in our bill of rights. Instead of punishing citizens for an honest endeavor to improve the public service, we should rather... commend them for their good citizenship. The defendants and appellants are acquitted with the costs de officio. So ordered. Principles: The interest of society and the maintenance of good government demand a full discussion of public... affairs. Complete liberty to comment on the conduct of public men is a scalpel in the case of free speech. criticism does not authorize defamation. Nevertheless, as the individual is less than the State, so must expected criticism be born for the common good. Rising superior to any official or set of officials, to the Chief Executive, to the Legislature, to the Judiciary to any or all the agencies of Government public opinion should be the constant source of liberty and democracy. The guaranties of a free speech and a free press include the right to criticize judicial conduct. The administration of the law is a matter of vital public concern. Whether the law is wisely or badly enforced is, therefore, a fit subject for proper comment. If the people... cannot criticize a justice of the peace or a judge the same as any other public officer, public opinion will be effectively muzzled. it is a duty which every one owes to society or to the State to assist in the investigation of any alleged... misconduct. CONSTITUTIONAL LAW 2 CASE DIGESTS

Privilege is classified as either absolute or qualified. With the first, we are not concerned. As to qualified privilege, it is as the words suggest a prima facie privilege which may be lost by proof of malice. A pertinent illustration of the application of qualified privilege is a complaint made in good faith and without malice in regard to the chLracter or conduct of a public official when addressed to an officer or a board having some interest or duty in the matter. Even when the... statements are found to be false, if there is probable cause for belief in their truthfulness and the charge is made in good faith, the mantle of privilege may still cover the mistake of the individual. But the statements must be made under an honest sense of duty; a... self-seeking motive is destructive. The privilege is not defeated by the mere fact that the communication is made in intemperate terms. A privileged communication should not be subjected to microscopic examination to discover grounds of malice or falsity. Such excessive scrutiny would defeat the protection which the law throws over privileged communications. The ultimate test is that of bona fides. As a general rule... words imputing to a judge or a justice of the peace dishonesty or corruption or incapacity or misconduct touching him in his office are actionable. But as suggested in the beginning we do not have present a simple case of direct and vicious accusations published in the press,... but of charges predicated on affidavits made to the proper official and thus qualifiedly privileged Malicious and untrue communications are not privileged. Jose Burgos vs. Chief of Staff G.R. No L-64261 December 26, 1984 Facts: CONSTITUTIONAL LAW 2 CASE DIGESTS

Two warrants were issued against petitioners for the search on the premises of “Metropolitan Mail” and “We Forum” newspapers and the seizure of items alleged to have been used in subversive activities. Petitioners prayed that a writ of preliminary mandatory and prohibitory injunction be issued for the return of the seized articles, and that respondents be enjoined from using the articles thus seized as evidence against petitioner. Petitioners questioned the warrants for the lack of probable cause and that the two warrants issued indicated only one and the same address. In addition, the items seized subject to the warrant were real properties. Issue: Whether or not the two warrants were valid to justify seizure of the items. Held: The defect in the indication of the same address in the two warrants was held by the court as a typographical error and immaterial in view of the correct determination of the place sought to be searched set forth in the application. The purpose and intent to search two distinct premises was evident in the issuance of the two warrant. As to the issue that the items seized were real properties, the court applied the principle in the case of Davao Sawmill Co. v. Castillo, ruling “that machinery which is movable by nature becomes immobilized when placed by the owner of the tenement, property or plant, but not so when placed by a tenant, usufructuary, or any other person having only a temporary right, unless such person acted as the agent of the owner.” In the case at bar, petitioners did not claim to be the owners of the land and/or building on which the machineries were placed. This being the case, the machineries in question, while in fact bolted to the ground remain movable property susceptible to seizure under a search warrant. However, the Court declared the two warrants null and void. Probable cause for a search is defined as such facts and circumstances which would lead a reasonably discreet and prudent man to believe that an offense has CONSTITUTIONAL LAW 2 CASE DIGESTS

been committed and that the objects sought in connection with the offense are in the place sought to be searched. The Court ruled that the affidavits submitted for the application of the warrant did not satisfy the requirement of probable cause, the statements of the witnesses having been mere generalizations. Furthermore, jurisprudence tells of the prohibition on the issuance of general warrants. (Stanford vs. State of Texas). The description and enumeration in the warrant of the items to be searched and seized did not indicate with specification the subversive nature of the said items. NEW YORK TIMES VS SULLIVAN Facts. The Plaintiff was one of three Commissioners of Montgomery, Alabama, who claimed that he was defamed in a full-page ad taken out in the New York Times. The advertisement was entitled, “Heed Their Rising Voices” and it charged in part that an unprecedented wave of terror had been directed against those who participated in the civil rights movement in the South. Some of the particulars of the advertisement were false. Although the advertisement did not mention the Plaintiff by name, he claimed that it referred to him indirectly because he had oversight responsibility of the police. The Defendant claimed that it authorized publication of the advertisement because it did not have any reason to believe that its contents were false. There was no independent effort to check its accuracy. The Plaintiff demanded that the Defendant retract the advertisement. The Defendant was puzzled as to why the Plaintiff thought the advertisement reflected adversely on him. The jury found the ad libelous per se and actionable without proof of malice. The jury awarded the Plaintiff $500,000 in damages. The Alabama Supreme Court affirmed. The Defendant appealed. Issue. Is the Defendant liable for defamation for printing an advertisement, which criticized a public official’s official conduct? Held. No. Reversed and remanded. CONSTITUTIONAL LAW 2 CASE DIGESTS

* Safeguards for freedom of speech and of the press are required by the First and Fourteenth Amendments of the United States Constitution (Constitution) in a libel action brought by a public official against critics of his official conduct. * Under Alabama law, a publication is libelous per se if the words tend to injure a person in his reputation or to bring him into public contempt. The jury must find that the words were published of and concerning the plaintiff. Once libel per se has been established, the defendant has no defense as to stated facts unless he can persuade the jury that they were true in all their particulars. * Erroneous statement is inevitable in free debate and it must be protected if the freedoms of expression are to have the breathing space that the need to survive. * The constitutional guarantees require a federal rule that prohibits a public official from recovering damages for a defamatory falsehood relating to his official conduct unless he proves that the statement was made with actual malice — that is, with knowledge that it was false or with reckless disregard of whether it was false or not. * The Supreme Court of the United States (Supreme Court) holds that the Constitution delimits a State’s power to award damages for libel in actions brought by public officials against critics of their official conduct. In this case, the rule requiring proof of actual malice is applicable. * The Defendant’s failure to retract the advertisement upon the Plaintiff’s demand is not adequate evidence of malice for constitutional purposes. Likewise, it is not adequate evidence of malice that the Defendant failed to check the advertisements accuracy against the news stories in the Defendant’s own files. Also, the evidence was constitutionally defective in another respect: it was incapable of supporting the jury’s finding that the allegedly libelous statements were made of and concerning the Plaintiff. Concurrence. Justice Hugo Black (J. Black) argued that the First and Fourteenth Amendments of the Constitution do not merely “delimit” a State’s power to award damages, but completely prohibit a State from exercising such a power. The Defendant had an absolute, unconditional right to publish criticisms of the Montgomery agencies and officials. Discussion. In order for a public official to recover in a defamation action involving his official conduct, malice must be proved. Without the showing of malice, the Supreme Court felt that a defamation action in this case would severely cripple the safeguards of freedom speech and expression that are guaranteed in the First CONSTITUTIONAL LAW 2 CASE DIGESTS

Amendment of the Constitution and applicable to the States via the Fourteenth Amendment of the Constitution. RESTRICTIONS Gonzales vs. COMELEC, G.R. No. L-28196, November 9, 1967 FACTS: On March 16, 1967, the Senate and the House of Representatives passed three resolutions which aim to: • Increase the number of the House of Representatives from 120 to 180 members (First Resolution). • Call a convention to propose amendments to the Constitution (Second Resolution). • Permit Senators and Congressmen to be members of the Constitutional Convention without forfeiting their seats (Third Resolution). Subsequently, Congress enacted Republic Act No. 4913, which took effect on June 17, 1967. RA 4913 is an Act submitting to the Filipino people for approval the amendments to the Constitution proposed by the Congress in the First and Third Resolutions. Petitioner Gonzales, as taxpayer, voter and citizen, and allegedly in representation thru class suit of all citizens of this country, filed this suit for prohibition with preliminary injunction to restrain COMELEC from implementing Republic Act 4913 assailing said law as unconstitutional. Petitioner PHILCONSA, as a civic, non-profit and non-partisan corporation, assails the constitutionality not only of Republic Act 4913 but also of First and Third Resolutions. ISSUES/HELD: 1. Whether RA 4913 is constitutional – YES. 2. Whether the submission of the amendments to the people of the Philippines violate the spirit of the Constitution – NO. CONSTITUTIONAL LAW 2 CASE DIGESTS

RATIO: 1. RA 4913 is constitutional. The measures undertaken by RA 4913 to inform the populace about the amendments are sufficient under the Constitution. The Constitution does not forbid the submission of proposals for amendment to the people except under certain conditions. 2. The submission of the amendments to the people of the Philippines do not violate the spirit of the Constitution. People may not be really interested on how the representatives are apportioned among the provinces of the Philippines as per First Resolution. Those who are interested to know the full details may enlighten themselves by reading copies of the amendments readily available in the polling places. On the matter of Third Resolution, the provisions of Article XV of the Constitution are satisfied so long as the electorate knows that it permits Congressmen to retain their seats as legislators, even if they should run for and assume the functions of delegates to the Convention. NOTE: The majority voted that the Resolutions and RA 4913 were unconstitutional but they did not reach specific number of votes to invalidate these congressional acts under the 1935 Constitution, which is two-thirds of the Supreme Court.

Social Weather Stations v COMELEC Facts: Petitioner, Social Weather Stations, Inc. (SWS) is a private non-stock, non-profit social research institution conducting surveys in various fields. On the other hand, petitioner Kamahalan Publishing Corporation publishes the Manila Standard, a newspaper of general circulation.

CONSTITUTIONAL LAW 2 CASE DIGESTS

Petitioners brought this action for prohibition to enjoin the Commission on Elections from enforcing Section 5.4 of RA. No.9006 (Fair Election Act), which provides that: “Surveys affecting national candidates shall not be published fifteen (15) days before an election and surveys affecting local candidates shall not be published seven (7) days before an election”. Petitioners argue that the restriction on the publication of election survey results constitutes a prior restraint on the exercise of freedom of speech without any clear and present danger to justify such restraint. They claim that SWS and other pollsters conducted and published the results of surveys prior to the 1992, 1995, and 1998 elections up to as close as two days before the election day without causing confusion among the voters and that there is neither empirical nor historical evidence to support the conclusion that there is an immediate and inevitable danger to tile voting process posed by election surveys. No similar restriction is imposed on politicians from explaining their opinion or on newspapers or broadcast media from writing and publishing articles concerning political issues up to the day of the election. They contend that there is no reason for ordinary voters to be denied access to the results of election surveys, which are relatively objective. Respondent Commission on Elections justifies the restrictions in §5.4 of R.A. No. 9006 as necessary to prevent the manipulation and corruption of the electoral process by unscrupulous and erroneous surveys just before the election. It contends that (1) the prohibition on the publication of election survey results during the period proscribed by law bears a rational connection to the objective of the law, i.e., the prevention of the debasement of the electoral process resulting from manipulated surveys, bandwagon effect, and absence of reply; (2) it is narrowly tailored to meet the "evils" sought to be prevented; and (3) the impairment of freedom of expression is minimal, the restriction being limited both in duration, i.e., the last 15 days before the national election and the last 7 days before a local election, and in scope as it does not prohibit election survey results but only require timeliness. Issue: Whether or not Section 5.4 of RA 9006 constitutes an unconstitutional abridgment of freedom of speech, expression and the press. CONSTITUTIONAL LAW 2 CASE DIGESTS

Held: Yes. It constitutes an unconstitutional abridgement of freedom of expression, speech and the press. To summarize, the Supreme Court held that §5.4 is invalid because (1) it imposes a prior restraint on the freedom of expression, (2) it is a direct and total suppression of a category of expression even though such suppression is only for a limited period, and (3) the governmental interest sought to be promoted can be achieved by means other than suppression of freedom of expression. It has been held that mere legislative preferences or beliefs respecting matters of public convenience may well support regulation directed at other personal activities, but be insufficient to justify such as diminishes the exercise of rights so vital to the maintenance of democratic institutions. Francisco Chavez vs. Raul M. Gonzales and NTC G.R. No. 168338 | February 15, 2008 Facts: As a consequence of the public release of copies of the “Hello Garci” compact disc audiotapes involving a wiretapped mobile phone conversation between then-President Gloria Arroyo and Comelec Commissioner Virgilio Garcillano, respondent DOJ Secretary Gonzales warned reporters that those who had copies of the CD and those broadcasting or publishing its contents could be held liable under the Anti-Wiretapping Act. He also stated that persons possessing or airing said tapes were committing a continuing offense, subject to arrest by anybody. Finally, he stated that he had ordered the NBI to go after media organizations “found to have caused the spread, the playing and the printing of the contents of a tape.” Meanwhile, respondent NTC warned TV and radio stations that their broadcast/airing of such false information and/or willful misrepresentation shall be a just cause for the suspension, revocation and/or cancellation of the licenses or authorizations issued to the said media establishments. Petitioner Chavez filed a petition under Rule 65 against respondents Secretary Gonzales and the NTC directly with the Supreme Court. Issues: (1) Will a purported violation of law such as the Anti-Wiretapping Law justify straitjacketing the exercise of freedom of speech and of the press? (2) Did the CONSTITUTIONAL LAW 2 CASE DIGESTS

mere press statements of respondents DOJ Secretary and the NTC constitute a form of content-based prior restraint that has transgressed the Constitution? Held: (1) No, a purported violation of law such as the Anti-Wiretapping Law will not justify straitjacketing the exercise of freedom of speech and of the press. A governmental action that restricts freedom of speech or of the press based on content is given the strictest scrutiny, with the government having the burden of overcoming the presumed unconstitutionality by the clear and present danger rule. This rule applies equally to all kinds of media, including broadcast media. Respondents, who have the burden to show that these acts do not abridge freedom of speech and of the press, failed to hurdle the clear and present danger test. For this failure of the respondents alone to offer proof to satisfy the clear and present danger test, the Court has no option but to uphold the exercise of free speech and free press. There is no showing that the feared violation of the antiwiretapping law clearly endangers the national security of the State. (2) Yes, the mere press statements of respondents DOJ Secretary and the NTC constituted a form of content-based prior restraint that has transgressed the Constitution. It is not decisive that the press statements made by respondents were not reduced in or followed up with formal orders or circulars. It is sufficient that the press statements were made by respondents while in the exercise of their official functions. Any act done, such as a speech uttered, for and on behalf of the government in an official capacity is covered by the rule on prior restraint. The concept of an “act” does not limit itself to acts already converted to a formal order or official circular. Otherwise, the non formalization of an act into an official order or circular will result in the easy circumvention of the prohibition on prior restraint. Zaldivar vs. Sandiganbayan G.R. No. 79690-707, February 1, 1989 170 SCRA 1 (1989) Facts: The court have examined carefully the lengthy and vigorously written Motion for Reconsideration dated October 18, 1988 filed by counsel for respondent Raul M. Gonzalez, relating to the per curiam Resolution of the Court CONSTITUTIONAL LAW 2 CASE DIGESTS

dated October 7, 1988. We have reviewed once more the Court’s extended per curiam Resolution, in the light of the argument adduced in the Motion for Reconsideration, but must conclude that we find no sufficient basis for modifying the conclusions and rulings embodied in that Resolution. The Motion for Reconsideration sets forth copious quotations and references to foreign texts which, however, whatever else they may depict, do not reflect the law in this jurisdiction. Nonetheless, it might be useful to develop further, in some measure, some of the conclusions reached in the per curiam Resolution, addressing in the process some of the “Ten (10) Legal Points for Reconsideration,” made in the Motion for Reconsideration. Issue: Whether the decision of the SC inviolate the Petitioner’s right to Freedom of Expression. Held: No, The Court penalizes a variety of contumacious conduct including: “any improper conduct tending, directly or indirectly, to impede, obstruct or degrade the administration of justice.” The “clear and present danger” doctrine invoked by respondent’s counsel is not a magic incantation which dissolves all problems and dispenses with analysis and judgment in the testing of the legitimacy of claims to free speech, and which compels a court to exonerate a defendant the moment the doctrine is invoked, absent proof of impending apocalypse. The clear and present danger” doctrine has been an accepted method for marking out the appropriate limits of freedom of speech and of assembly in certain contexts. It is not, however, the only test which has been recognized and applied by courts. The right of freedom of expression indeed, occupies a preferred position in the “hierarchy of civil liberties”. Freedom of expression is not an absolute. The prevailing doctrine is that the clear and present danger rule is such a limitation. Another criterion for permissible limitation on freedom of speech and of the press, which includes such vehicles of the mass media as radio, television and the movies, is the “balancingof-interests test”. The principle “requires a court to take conscious and detailed consideration of the interplay of interests observable in a given situation or type of situation. CONSTITUTIONAL LAW 2 CASE DIGESTS

Under either the “clear and present danger” test or the “balancing-of-interest test,” the court believe that the statements made by respondent are of such a nature and were made in such a manner and under such circumstances, as to transcend the permissible limits of free speech. This conclusion was implicit in the per curiam Resolution of October 7, 1988. It is important to point out that the “substantive evil” which the Supreme Court has a right and a duty to prevent does not, in the instant case, relate to threats of physical disorder or overt violence or similar disruptions of public order. What is here at stake is the authority of the Supreme Court to confront and prevent a “substantive evil” consisting not only of the obstruction of a free and fair hearing of a particular case but also the avoidance of the broader evil of the degradation of the judicial system of a country and the destruction of the standards of professional conduct required from members of the bar and officers of the courts. The “substantive evil” here involved, in other words, is not as palpable as a threat of public disorder or rioting but is certainly no less deleterious and more far reaching in its implications for society. SANIDAD VS COMELEC Facts: On October 23, 1989, Republic Act No. 6766, entitled "AN ACT PROVIDING FOR AN ORGANIC ACT FOR THE CORDILLERA AUTONOMOUS REGION" was enacted into law. The Commission on Elections, by virtue of the power vested by the 1987 Constitution, the Omnibus Election Code (BP 881), said R.A. 6766 and other pertinent election laws, promulgated Resolution No. 2167, to govern the conduct of the plebiscite on the said Organic Act for the Cordillera Autonomous Region. In a petition dated November 20, 1989, herein petitioner Pablito V. Sanidad, who claims to be a newspaper columnist of the "OVERVIEW" for the BAGUIO MIDLAND COURIER, a weekly newspaper circulated in the City of Baguio and the Cordilleras, assailed the constitutionality of Section 19 of Comelec Resolution No. 2167, which provides: Section 19. Prohibition on columnists, commentators or announcers. — During the plebiscite campaign period, on the day before and on the plebiscite day, no mass media columnist, commentator, announcer or personality shall use his column or radio or television time to campaign for or against the plebiscite issues It is alleged by petitioner that said provision is void and unconstitutional because it violates the constitutional CONSTITUTIONAL LAW 2 CASE DIGESTS

guarantees of the freedom of expression and of the press enshrined in the Constitution. Issue : WON the said Section 19 of resolution No 2167 is unconstitutional Held: It is clear from Art. IX-C of the 1987 Constitution that what was granted to the Comelec was the power to supervise and regulate the use and enjoyment of franchises, permits or other grants issued for the operation of transportation or other public utilities, media of communication or information to the end that equal opportunity, time and space, and the right to reply, including reasonable, equal rates therefor, for public information campaigns and forums among candidates are ensured Neither Article IX-C of the Constitution nor Section 11 (b), 2nd par. of R.A. 6646 can be construed to mean that the Comelec has also been granted the right to supervise and regulate the exercise by media practitioners themselves of their right to expression during plebiscite periods. Media practitioners exercising their freedom of expression during plebiscite periods are neither the franchise holders nor the candidates. In fact, there are no candidates involved in a plebiscite. Therefore, Section 19 of Comelec Resolution No. 2167 has no statutory basis. While the limitation does not absolutely bar petitioner's freedom of expression, it is still a restriction on his choice of the forum where he may express his view. No reason was advanced by respondent to justify such abridgement. We hold that this form of regulation is tantamount to a restriction of petitioner's freedom of expression for no justifiable reason. Plebiscite issues are matters of public concern and importance. The people's right to be informed and to be able to freely and intelligently make a decision would be better served by access to an unabridged discussion of the issues, including the forum. The people affected by the issues presented in a plebiscite should not be unduly burdened by restrictions on the forum where the right to expression may be exercised. Comelec spaces and Comelec radio time may provide a forum for expression but they do not guarantee full dissemination of information to the public concerned because they are limited to either specific portions in newspapers or to specific radio or television times.

RENO VS ACLU, D-96-511 JUNE 26, 1997 CONSTITUTIONAL LAW 2 CASE DIGESTS

Facts. At issue is the constitutionality of two statutory provisions enacted to protect minors from “indecent” and “patently offensive” communications on the Internet. The District Court made extensive findings of fact about the Internet and the CDA. It held that the statute abridges the “freedom of speech” protected by the First Amendment of the United States Constitution (Constitution). Issue. Whether the two CDA statutory provisions at issue are constitutional? Held. No. Judgment of the District Court affirmed. Under the CDA, neither parents’ consent nor their participation would avoid application of the statute. The CDA fails to provide any definition of “indecent” and omits any requirement that the “patently offensive material” lack serious literary, artistic, political or scientific value. Further, the CDA’s broad categorical prohibitions are not limited to particular times and are not dependent on any evaluation by an agency familiar with the unique characteristics of the Internet. CDA applies to the entire universe of the cyberspace. Thus, the CDA is a content-based blanket restriction on speech, as such, cannot be properly analyzed as a form of time, place and manner restriction. The CDA lacks the precision that the First Amendment of the Constitution requires when a statute regulates the content of speech. In order to deny minors access to potentially harmful speech, the statute suppresses a large amount of speech that adults have a constitutional right to receive. The CDA places an unacceptable burden on protected speech, thus, the statute is invalid as unconstitutional. Concurrence. The constitutionality of the CDA as a zoning law hinges on the extent to which it substantially interferes with the First Amendment rights of adults. Because the rights of adults are infringed only by the “display” provision and by the “indecency transmission” provision, the judge would invalidate the CDA only to that extent. Discussion. This case brings the First Amendment of the Constitution into the Internet age while prohibiting speech regulations that are overbroad despite their seemingly benevolent goals. CONSTITUTIONAL LAW 2 CASE DIGESTS

Miriam College Foundation, Inc. v CA 348 SCRA 265 December 15, 2000 Facts: The members of the editorial board of the Miriam College Foundation’s school paper were subjected to disciplinary sanction by the College Discipline Committee after letters of complaint were filed before the Board following the publication of the school paper that contains obscene, vulgar, and sexually explicit contents. Prior to the disciplinary sanction to the defendants they were required to submit a written statement to answer the complaints against them to the Discipline Committee but the defendants, instead of doing so wrote to the Committee to transfer the case to the DECS which they alleged to have the jurisdiction over the issue. Pushing through with the investigation ex parte the Committee found the defendants guilty and imposed upon them disciplinary sanctions. Defendants filed before the court for prohibition with preliminary injunction on said decision of the Committee questioning the jurisdiction of said Discipline Board over the defendants.

Issue: WON the Discipline Board of Miriam College has jurisdiction over the defendants. Held: The court resolved the issue before it by looking through the power of DECS and the Disciplinary Committee in imposing sanctions upon the defendants. Section 5 (2), Article XIV of the Constitution guarantees all institutions of higher learning academic freedom. This institutional academic freedom includes the right of the school or college to decide for itself, its aims and objectives, and how best to attain them free from outside coercion or interference save possibly when the overriding public welfare calls for some restraint. Such duty gives the institution the right to discipline its students and inculcate upon them good values, ideals and attitude. The right of students to free speech in school is not always absolute. The court upheld the right of students for the freedom of expression but it does CONSTITUTIONAL LAW 2 CASE DIGESTS

not rule out disciplinary actions of the school on the conduct of their students. Further, Sec. 7 of the of the Campus Journalism Act provides that the school cannot suspend or expel a student solely on the basis of the articles they write EXCEPT when such article materially disrupts class work of involve substantial disorder or invasion of the rights of others. Therefore the court ruled that the power of the school to investigate is an adjunct of its power to suspend or expel. It is a necessary corollary to the enforcement of rules and regulations and the maintenance of a safe and orderly educational environment conducive to learning. That power, like the power to suspend or expel, is an inherent part of the academic freedom of institutions of higher learning guaranteed by the Constitution. The court held that Miriam College has the authority to hear and decide the cases filed against respondent students.

ABS-CBN vs. COMELEC GR No. 133486, January 28, 2000 FACTS: Before us is a Petition for Certiorari under Rule 65 of the Rules of Court assailing COMELEC en banc Resolution No. 98-1419 dated April 21, 1998. In the said Resolution, the poll body “RESOLVED to approve the issuance of a restraining order to stop ABS-CBN or any other groups, its agents or representatives from conducting such exit survey and to authorize the Honorable Chairman to issue the same.” The Resolution was issued by the Comelec allegedly upon “information from [a] reliable source that ABS-CBN (Lopez Group) has prepared a project, with PR groups, to conduct radio-TV coverage of the elections x x x and to make [an] exit survey of the x x x vote during the elections for national officials particularly for President and Vice President, results of which shall be [broadcast] immediately.” The electoral body believed that such project might conflict with the official Comelec count, as well as the unofficial quick count of the National Movement for Free Elections (Namfrel). It also noted that it had not authorized or deputized Petitioner ABS-CBN to undertake the exit survey. CONSTITUTIONAL LAW 2 CASE DIGESTS

On May 9, 1998, this Court issued the Temporary Restraining Order prayed for by petitioner. We directed the Comelec to cease and desist, until further orders, from implementing the assailed Resolution or the restraining order issued pursuant thereto, if any. In fact, the exit polls were actually conducted and reported by media without any difficulty or problem. The solicitor general contends that the petition is moot and academic, because the May 11, 1998 election has already been held and done with. Allegedly, there is no longer any actual controversy before us. ISSUE: Is the “moot and academic” principle a magical formula that can automatically dissuade the courts in resolving a case? RULING: No. The issue is not totally moot. While the assailed Resolution referred specifically to the May 11, 1998 election, its implications on the people’s fundamental freedom of expression transcend the past election. The holding of periodic elections is a basic feature of our democratic government. By its very nature, exit polling is tied up with elections. To set aside the resolution of the issue now will only postpone a task that could well crop up again in future elections. In any event, in Salonga v. Cruz Pano, the Court had occasion to reiterate that it “also has the duty to formulate guiding and controlling constitutional principles, precepts, doctrines, or rules. It has the symbolic function of educating bench and bar on the extent of protection given by constitutional guarantees.” Since the fundamental freedoms of speech and of the press are being invoked here, we have resolved to settle, for the guidance of posterity, whether they likewise protect the holding of exit polls and the dissemination of data derived therefrom.

FREEDOM OF EXPRESSION, LIBEL AND NATIONAL SECURITY CONSTITUTIONAL LAW 2 CASE DIGESTS

POLICARPIO VS MANILA TIMES L-16027, MAY 30,1962 Facts : Plaintiff Lumen Policarpio seeks to recover P150,000.00, as actual damages, P70,000, as moral damages, P60,000 as correctional and exemplary damages, and P20,000, as attorney's fees, aside from the costs, by reason of the publication in the Saturday Mirror of August 11, 1956, and in the Daily Mirror of August 13, 1956, of two (2) articles or news items which are claimed to be per se defamatory, libelous and false, and to have exposed her to ridicule, jeopardized her integrity, good name and business and official transactions, and caused her grave embarrassment, untold and extreme moral, mental and physical anguish and incalculable material, moral, professional and business damages. The defendants are The Manila Times Publishing Co., Inc., as publisher of The Saturday Mirror and The Daily Mirror, which are newspapers of general circulation in the Philippines, and Constante C. Roldan, Manuel V. Villa-Real, E. Aguilar Cruz and Consorcio Borje, as the reporter or author of the first article and the managing editor, the associate editor and the news editor, respectively, of said newspapers Issue : WON the Held : It goes without saying that newspapers must enjoy a certain degree of discretion in determining the manner in which a given event should be presented to the public, and the importance to be attached thereto, as a news item, and that its presentation in a sensational manner is not per se illegal. Newspaper may publish news items relative to judicial, legislative or other official proceedings, which are not of confidential nature, because the public is entitled to know the truth with respect to such proceedings, which, being official and non-confidential, are open to public consumption. But, to enjoy immunity, a publication containing derogatory information must be not only true, but, also, fair, and it must be made in good faith and without any comments or remarks In the case at bar, aside from containing information derogatory to the plaintiff, the article published on August 11, 1956, presented her in a worse predicament than that in which she, in fact, was. In other words, said article was not a fair and true report of the proceedings there in alluded to. What is more, its sub-title — "PCAC RAPS L. POLICARPIO ON FRAUD" — is a comment or remark, besides being false. Accordingly, the CONSTITUTIONAL LAW 2 CASE DIGESTS

defamatory imputations contained in said article are "presumed to be malicious". We note that the news item published on August 13, 1956, rectified a major inaccuracy contained in the first article, by stating that neither Col. Alba nor the PCAC had filed the aforementioned complaints with the city fiscal's office. It, likewise, indicated the number of sheets of stencil involved in said complaints. But, this rectification or clarification does not wipe out the responsibility arising from the publication of the first article, although it may and should mitigate it (Jimenez vs. Reyes, 27 Phil. 52). For this reason, we feel that the interest of justice and of all parties concerned would be served if the defendants indemnify the plaintiff in the sums of P3,000, by way of moral damages, and P2,000, as attorney's fees Espuelas vs People G.R. No. L-2990 December 17, 1951 Facts: On June 9 and June 24, 1947, both dates inclusive, in the town of Tagbilaran, Bohol, Oscar Espuelas y Mendoza had his picture taken, making it to appear as if he were hanging lifeless at the end of a piece of rope suspended form the limb of the tree, when in truth and in fact, he was merely standing on a barrel. After securing copies of his photograph, Espuelas sent copies of same to Free Press, the Evening News, the Bisayas, Lamdang of general circulation and other local periodicals in the Province of Bohol but also throughout the Philippines and abroad, for their publication with a suicide note or letter, wherein he made to appear that it was written by a fictitious suicide, Alberto Reveniera and addressed to the latter's supposed wife translation of which letter or note, stating his dismay and administration of President Roxas, pointing out the situation in Central Luzon and Leyte, and directing his wife his dear wife to write to President Truman and Churchill of US and tell them that in the Philippines the government is infested with many Hitlers and Mussolinis. Issue:

CONSTITUTIONAL LAW 2 CASE DIGESTS

Whether the accused is liable of seditious libel under Art. 142 of the RPC against the Government of the Philippines? Held: Yes. The accused must therefore be found guilty as charged. And there being no question as to the legality of the penalty imposed on him, the decision will be affirmed with costs. Analyzed for meaning and weighed in its consequences, the article written bybthe accused, cannot fail to impress thinking persons that it seeks to sow the seeds of sedition and strife. The infuriating language is not a sincere effort to persuade, what with the writer's simulated suicide and false claim to martyrdom and what with is failure to particularize. When the use irritating language centers not on persuading the readers but on creating disturbances, the rationable of free speech cannot apply and the speaker or writer is removed from the protection of the constitutional guaranty. If it be argued that the article does not discredit the entire governmental structure but only President Roxas and his men, the reply is that article 142 punishes not only all libels against the Government but also "libels against any of the duly constituted authorities thereof." The "Roxas people" in the Government obviously refer of least to the President, his Cabinet and the majority of legislators to whom the adjectives dirty, Hitlers and Mussolinis were naturally directed. On this score alone the conviction could be upheld. Regarding the publication, it suggests or incites rebellious conspiracies or riots and tends to stir up people against the constituted authorities, or to provoke violence from opposition who may seek to silence the writer. Which is the sum and substance of the offense under consideration. The essence of seditious libel may be said to its immediate tendency to stir up general discontent to the pitch of illegal courses; that is to say to induce people to resort to illegal methods other than those provided by the Constitution, in order to repress the evils which press upon their minds.

CONSTITUTIONAL LAW 2 CASE DIGESTS

LOPEZ v. CA G.R. No. 157784 December 16, 2008 FACTS: On 23 March 1968, Juliana executed a notarial will,whereby she expressed that she wished to constitute a trust fund for her paraphernal properties, denominated as Fideicomiso de Juliana Lopez Manzano (Fideicomiso), to be administered by her husband. If her husband were to die or renounce the obligation, her nephew, Enrique Lopez, was to become administrator and executor of the Fideicomiso. Two-thirds (2/3) of the income from rentals over these properties were to answer for the education of deserving but needy honor students, while one-third 1/3 was to shoulder the expenses and fees of the administrator. As to her conjugal properties, Juliana bequeathed the portion that she could legally dispose to her husband, and after his death, said properties were to pass to her biznietos or great grandchildren. Juliana initiated the probate of her will five (5) days after its execution, but she died on 12 August 1968, before the petition for probate could be heard. The petition was pursued instead in Special Proceedings (S.P.) No. 706 by her husband, Jose, who was the designated executor in the will. On 7 October 1968, the Court of First Instance, Branch 3, Balayan,Batangas, acting as probate court, admitted the will to probate and issued the letters testamentary to Jose. Jose then submitted an inventory of Juliana’s real and personal properties with their appraised values, which was approved by the probate court. Thereafter, Jose filed a Report dated 16 August 1969, which included a proposed project of partition. Jose proceeded to offer a project of partition.Then, Jose listed those properties which he alleged were registered in both his and Juliana’s names, totaling 13 parcels in all. The disputed properties consisting of six (6) parcels, all located in Balayan, Batangas, were included in said list. On 25 August 1969, the probate court issued an order approving the project of partition. As to the properties to be constituted into the Fideicomiso, the probate court ordered that the certificates of title thereto be cancelled, and, in lieu thereof, new certificates be issued in favor of Jose as trustee of the Fideicomiso covering one-half (1/2) of the properties listed under paragraph 14 of the project of partition; and regarding the other half, to be registered in the name of Jose as heir of Juliana. The properties which Jose had alleged as registered in his and Juliana’s names, including the disputed lots, were adjudicated to Jose as heir, subject to the condition that Jose would settle the obligations charged on these properties. The probate court, thus, directed that new certificates of title be issued in favor of CONSTITUTIONAL LAW 2 CASE DIGESTS

Jose as the registered owner thereof in its Order dated 15 September 1969. On even date, the certificates of title of the disputed properties were issued in the name of Jose. The Fideicomiso was constituted in S.P No. 706 encompassing onehalf (1/2) of the Abra de Ilog lot on Mindoro, the 1/6 portion of the lot in Antorcha St. in Balayan, Batangas and all other properties inherited ab intestato by Juliana from her sister, Clemencia, in accordance with the order of the probate court in S.P. No. 706. The disputed lands were excluded from the trust. Jose died on 22 July 1980, leaving a holographic will disposing of the disputed properties to respondents. The will was allowed probate on 20 December 1983 in S.P. No. 2675 before the RTC of Pasay City. Pursuant to Jose’s will, the RTC ordered on 20 December 1983 the transfer of the disputed properties to the respondents as the heirs of Jose. Consequently, the certificates of title of the disputed properties were cancelled and new ones issued in the names of respondents. Petitioner’s father, Enrique Lopez, also assumed the trusteeship of Juliana’s estate. On 30 August 1984, the RTC of Batangas, Branch 9 appointed petitioner as trustee of Juliana’s estate in S.P. No. 706. On 11 December 1984, petitioner instituted an action for reconveyance of parcels of land with sum of money before the RTC of Balayan, Batangas against respondents. The complaint essentially alleged that Jose was able to register in his name the disputed properties, which were the paraphernal properties of Juliana, either during their conjugal union or in the course of the performance of his duties as executor of the testate estate of Juliana and that upon the death of Jose, the disputed properties were included in the inventory as if they formed part of Jose’s estate when in fact Jose was holding them only in trust for the trust estate of Juliana. The RCT dismissed the petition on the ground of prescription. The CA denied the appeals filed by both parties. Hence, this petition. ISSUE: Whether an implied trust was constituted over the disputed properties when Jose, the trustee, registered them in his name. HELD: The disputed properties were excluded from the Fideicomiso at the outset. Jose registered the disputed properties in his name partly as his conjugal share and partly as his inheritance from his wife Juliana, which is the complete reverse of the claim of the petitioner, as the new trustee, that the properties are intended for the beneficiaries of the Fideicomiso. Furthermore, the exclusion of the disputed CONSTITUTIONAL LAW 2 CASE DIGESTS

properties from the Fideicomiso was approved by the probate court and, subsequently, by the trial court having jurisdiction over the Fideicomiso. The registration of the disputed properties in the name of Jose was actually pursuant to a court order. The apparent mistake in the adjudication of the disputed properties to Jose created a mere implied trust of the constructive variety in favor of the beneficiaries of the Fideicomiso. PEOPLE’S JOURNAL et. al. vs. FRANCIS THOENEN G.R. No. 143372 December 13, 2005 Facts: On 30 September 1990, a news item appeared in the People’s Journal claiming that a certain Francis Thoenen, a Swiss national who allegedly shoots wayward neighbors’ pets that he finds in his domain. It also claimed that BF Homes residents, in a letter through lawyer Atty. Efren Angara, requested for the deportation of Thoenen to prevent the recurrence of such incident in the future. Thoenen claimed that the article destroyed the respect and admiration he enjoyed in the community. He is seeking for damages. The petitioners admitted publication of the news item, ostensibly out of a “social and moral duty to inform the public on matters of general interest, promote the public good and protect the moral public (sic) of the people,” and that the story was published in good faith and without malice. Issue: Whether or not the news report fall under privileged communication and therefore protected by the constitutional provision on freedom of speech. Held: The right of free speech is not absolute. Libel is not protected speech. In the instant case, even if we assume that the letter written by Atty. Angara is privileged communication, it lost its character when the matter was published in the newspaper and circulated among the general population, especially since the individual alleged to be defamed is neither a public official nor a public figure.

CONSTITUTIONAL LAW 2 CASE DIGESTS

Moreover, the news item contained falsehoods on two levels. First, the BF Homes residents did not ask for the deportation of Thoenen, more so because the letter of the Atty. Anagara was a mere request for verification of Thoenen’s status as a foreign resident. The article is also untrue because the events she reported never happened. Worse, the main source of information, Atty. Efren Angara, apparently either does not exist, or is not a lawyer. There is no constitutional value in false statements of fact. Neither the intentional lie nor the careless error materially advances society’s interest in ‘uninhibited, robust, and wide-open’ debate. Calculated falsehood falls into that class of utterances which “are no essential part of any exposition of ideas, and are of such slight social value as a step to truth that any benefit that may be derived from them is clearly outweighed by the social interest in order and morality… The knowingly false statement and the false statement made with reckless disregard of the truth, do not enjoy constitutional protection” BORJAL VS CA Facts : During the congressional hearings on the transport crisis sometime in September 1988 undertaken by the House Sub-Committee on Industrial Policy, those who attended agreed to organize the First National Conference on Land Transportation (FNCLT) to be participated in by the private sector in the transport industry and government agencies concerned in order to find ways and means to solve the transportation crisis. More importantly, the objective of the FNCLT was to draft an omnibus bill that would embody a long-term land transportation policy for presentation to Congress. The conference which, according to private respondent, was estimated to cost around P1,815,000.00 would be funded through solicitations from various sponsors such as government agencies, private organizations, transport firms, and individual delegates or participants. 2 On 28 February 1989, at the organizational meeting of the FNCLT, private respondent Francisco Wenceslao was elected Executive Director. As such, he wrote numerous solicitation letters to the business community for the support of the conference. Between May and July 1989 a series of articles written by petitioner Borjal was published on different dates in his column Jaywalker. The articles dealt with the alleged anomalous activities of an "organizer of a conference" without naming or identifying private respondent. Neither did it refer to the FNCLT as the conference CONSTITUTIONAL LAW 2 CASE DIGESTS

therein mentioned. Quoted hereunder are excerpts from the articles of petitioner together with the dates they were published Issue : Issue : Whether or not there are sufficient grounds to constitute guilt of petitioners for libel Held : A privileged communication may be either absolutely privileged or qualifiedly privileged. Absolutely privileged communications are those which are not actionable even if the author has acted in bad faith. An example is found in Sec. 11, Art.VI, of the 1987 Constitution which exempts a member of Congress from liability for any speech or debate in the Congress or in any Committee thereof. Upon the other hand, qualifiedly privileged communications containing defamatory imputations are not actionable unless found to have been made without good intention justifiable motive. To this genre belong "private communications" and "fair and true report without any comments or remarks To reiterate, fair commentaries on matters of public interest are privileged and constitute a valid defense in an action for libel or slander. The doctrine of fair comment means that while in general every discreditable imputation publicly made is deemed false, because every man is presumed innocent until his guilt is judicially proved, and every false imputation is deemed malicious, nevertheless, when the discreditable imputation is directed against a public person in his public capacity, it is not necessarily actionable. In order that such discreditable imputation to a public official may be actionable, it must either be a false allegation of fact or a comment based on a false supposition. If the comment is an expression of opinion, based on established facts, then it is immaterial that the opinion happens to be mistaken, as long as it might reasonably be inferred from the facts There is no denying that the questioned articles dealt with matters of public interest. A reading of the imputations of petitioner Borjal against respondent Wenceslao shows that all these necessarily bore upon the latter's official conduct and his moral and mental fitness as Executive Director of the FNCLT. The nature and functions of his position which included solicitation of funds, dissemination of information about the FNCLT in order to generate interest in the conference, and the management and coordination of the various activities of the conference demanded from him utmost honesty, integrity and competence. These are matters about which the public has the right to be informed, taking into account the very public character of the conference itself. Generally, malice can be presumed from defamatory words, the privileged character of a communication CONSTITUTIONAL LAW 2 CASE DIGESTS

destroys the presumption of malice. The onus of proving actual malice then lies on plaintiff, private respondent Wenceslao herein. He must bring home to the defendant, petitioner Borjal herein, the existence of malice as the true motive of his conduct. Cabansag v Fernandez; G.R. No. L-8974; 18 Oct 1957; 102 Phil 152 FACTS: Apolonio Cabansag filed a complaint seeking the ejectment of Germiniana Fernandez from a parcel of land. He later wrote a letter to the Presidential Complaints and Action Commission (PCAC) regarding the delay in the disposition of his case before the CFI Pangasinan. The judge ordered Cabansag and his lawyers to show cause why he should not be held liable for contempt for sending such letter which tended to degrade the court in the eyes of the President (Magsaysay) and the people. After due hearing, the court rendered a decision finding Cabansag and his lawyers guilty of contempt and sentencing them to pay a fine. ISSUE(S): Whether or not Cabansag’s letter created a sufficient danger to a fair administration of justice? HELD: NO. The letter was sent to the Office of the President asking for help because of the precarious predicament of Cabansag. While the course of action he had taken may not be a wise one for it would have been proper had he addressed his letter to the Secretary of Justice or to the Supreme Court, such act alone would not be contemptuous. To be so the danger must cause a serious imminent threat to the administration of justice. Nor can we infer that such act has “a dangerous tendency” to belittle the court or undermine the administration of justice for the writer merely exercised his constitutional right to petition the government for redress of a legitimate grievance. Petition is GRANTED and appealed decision is REVERSED.

CONSTITUTIONAL LAW 2 CASE DIGESTS

In Re: Column of Ramon Tulfo Column of Ramon Tulfo Facts: In Oct. 13, 1989, Tulfo wrote an article in his column in PDI 'On Target' stating that the Supreme Court rendered an idiotic decision in legalizing checkpoints, and again on Oct. 16, 1989, where he called the Supreme Court stupid and "sangkatutak na mga bobo justices of the Philippine Supreme Court". Tulfo was required to show cause why he should not be punished for contempt. Tulfo said that he was just reacting emotionally because he had been a victim of harassmen in the checkpoints, and "idiotic" meant illogical and unwise, and "bobo" was just quoted from other attorneys, and since the case had been decided and terminated, there was not contempts. Lastly, the article does not pose any clear and present danger to the Supreme court. Issue: Whether or not Tulfo is in contempt Held: Yes. 1. At the time Tulfo wrote the article, the checkpoints case had not yet been decided upon, and the Supreme Court was still acting on an MR filed from the CA. 2. Power to punish is inherent as it is essential for self-preservation. Contempt of ocurt is defiance of the authority, justice and dignity of the courts. It brings disrepute to the court. There are two kinds of publications which can be punished for contempt: a. those whose object is to affect the decision in a pending case. b. those whose object is to bring courts to discredit. Tulfo's article constituted both. 3. It should have been okay to criticize if respectful language was used, but if its object is only to degrade and ridicule, then it is clearly an obstruction of justice. Nothing constructive can be gained from them. Being emotional is no excuse for being insulting. Quoting is not an excuse also, because at the end of his article, CONSTITUTIONAL LAW 2 CASE DIGESTS

Tulfo said, "So you bobo justices, watch out!" Also, he said he was not sorry for having written the articles. Tulfo is found in contempt of court and is gravely censured. IN RE EMIL JURADO Facts: Emiliano P. Jurado, a lawyer and journalist who writes in a newspaper of general circulation wrote about alleged improprieties and irregularities in the judiciary over several months. What was particularly given attention by the Supreme Court his column entitled, "Who will judge the Justices?" referring to a report that six justices, their spouses, children and grandchildren (a total of 36 persons) spent a vacation in Hong Kong, and that luxurious hotel accommodations and all their other expenses were paid by a public utility firm and that the trip was arranged by the travel agency patronized by this public utility firm. This column was made amidst rumors that a Supreme Court decision favorable to the public utility firm appears to have been authored by a lawyer of the public utility firm. The Chief Justice issued an administrative order creating an ad hoc committee to investigate the said reports of corruption in the judiciary. A letter affidavit was also received from the public utility, denying the allegations in Jurado's column. The Supreme Court then issued a resolution ordering that the matter dealt with in the letter and affidavit of the public utility company be docketed and acted upon as an official Court proceeding for the determination of whether or not the allegations made by Jurado are true. Held: Jurado's actuations, in the context in which they were done, demonstrate gross irresponsibility, and indifference to factual accuracy and the injury that he might cause to the name and reputation of those of whom he wrote. They constitute contempt of court, directly tending as they do to degrade or abase the administration of justice and the judges engaged in that function. The Court declares Atty. Jurado guilty of contempt of court and sentences him to pay a fine of one thousand pesos (P1,000.00) CONSTITUTIONAL LAW 2 CASE DIGESTS

Free press not to be preferred to an independent judiciary. The court underscores the importance of both the constitutional guarantee of free speech and the reality that there are equally important public interests which need on occasion to be balanced against and accommodated with one and the other. One such public interest is in the maintenance of the integrity and orderly functioning of the administration of justice. There is no antinomy between free expression and the integrity of the system of administering justice. For the protection and maintenance of freedom of expression itself can be secured only within the context of a functioning and orderly system of dispensing justice, within the context, in other words, of viable independent institutions for delivery of justice which are accepted by the general community. As Mr. Justice Frankfurter put it: ". . . A free press is not to be preferred to an independent judiciary, nor an independent judiciary to a free press. Neither has primacy over the other; both are indispensable to a free society." Mr. Justice Malcolm of this Court expressed the same thought in the following terms: 'The Organic Act wisely guarantees freedom of speech and press. This constitutional right must be protected in its fullest extent. But license or abuse of liberty of the press and of the citizens should not be confused with liberty in its true sense. As important as the maintenance of an unmuzzled press and the free exercise of the rights of the citizens is the maintenance of the independence of the Judiciary. Respect for the Judiciary cannot be had if persons are privileged to scorn a resolution of the court adopted for good purposes, and if such persons are to be permitted by subterranean means to diffuse inaccurate accounts of confidential proceedings to the embarrassment of the parties and the court.' (In Re Severino Lozano and Anastacio Quevedo, 54 Phil. 801 at 807 [1930])." Excercise of freedom of speech not to be abused. The Civil Code, in its Article 19 lays down the norm for the proper exercise of any right, constitutional or otherwise, viz.: "ARTICLE 19. Every person must, in the exercise of his rights and in the performance of his duties, act with justice, give everyone his due, and observe honesty and good faith." The provision is reflective of the universally accepted precept of "abuse of rights," "one of the most dominant principles which must be deemed always implied in any system of law." Requirement to exercise bona fide care in ascertaining the truth of the statements when publishing statements which are clearly defamatory to identifiable judges or other public officials. Judges, by becoming such, are rightly regarded as voluntarily subjecting themselves to norms of conduct which embody more stringent standards of CONSTITUTIONAL LAW 2 CASE DIGESTS

honesty, integrity, and competence than are commonly required from private persons. Nevertheless, persons who seek or accept appointment to the Judiciary cannot reasonably be regarded as having forfeited any right to private honor and reputation. For to so rule will be to discourage all save those who feel no need to maintain their self-respect from becoming judges. The public interest involved in freedom of speech and the individual interest of judges (and for that matter, all other public officials) in the maintenance of private honor and reputation need to be accommodated one to the other. And the point of adjustment or accommodation between these two legitimate interests is precisely found in the norm which requires those who, invoking freedom of speech, publish statements which are clearly defamatory to identifiable judges or other public officials to exercise bona fide care in ascertaining the truth of the statements they publish. The norm does not require that a journalist guarantee the truth of what he says or publishes. But the norm does prohibit the reckless disregard of private reputation by publishing or circulating defamatory statements without any bona fide effort to ascertain the truth thereof. FREEDON OF EXPRESSION, MOVIE CENSORSHIP, OBSCENITY AND THE RIGHT TO PRIVACY GONZALES VS KALAW KATIGBAK Facts : In a resolution of a sub-committee of respondent Board of October 23, 1984, a permit to exhibit the film Kapit sa Patalim under the classification "For Adults Only," with certain changes and deletions enumerated was granted. A motion for reconsideration was filed by petitioners stating that the classification of the film "For Adults Only" was without basis. 4 Then on November 12, 1984, respondent Board released its decision: "Acting on the applicant's Motion for Reconsideration dated 29 October 1984, the Board, after a review of the resolution of the sub-committee and an examination of the film, Resolves to affirm in toto the ruling of the sub-committee. Considering, however, certain vital deficiencies in the application, the Board further Resolves to direct the Chairman of the Board to Withheld the issuance of the Permit to exhibit until these deficiencies are supplied CONSTITUTIONAL LAW 2 CASE DIGESTS

Issue : WON the rating made with grave abuse of discretion Held : Roth- Sex and obscenity are not synonymous. Obscene material is material which deals with sex in a manner appealing to prurient interest. The portrayal of sex, e.g., in art, literature and scientific works, is not itself sufficient reason to deny material the constitutional protection of freedom of speech and press. Sex, a great and mysterious motive force in human life has indisputably been a subject of absorbing interest to mankind through the ages; it is one of the vital problems of human interest and public concern. In the Philippine context, E.O. 876 applied contemporary Filipino cultural values as a standard. Moreover, as far as the question of sex and obscenity are concerned, it cannot be stressed strongly that the arts and letters "shall be under the patronage of the State. Given this constitutional mandate, It will be less than true to its function if any government office or agency would invade the sphere of autonomy that an artist enjoys. There is no orthodoxy in what passes for beauty or for reality. It is for the artist to determine what for him is a true representation. It is not to be forgotten that art and belleslettres deal primarily with imagination, not so much with ideas in a strict sense. What is seen or perceived by an artist is entitled to respect, unless there is a showing that the product of his talent rightfully may be considered obscene. On the question of obscenity, therefore, such standard set forth in Executive Order No. 878 is to be construed in such a fashion to avoid any taint of unconstitutionality. To repeat, what was stated in a recent decision in Trinidad- an elementary, a fundamental, and a universal role of construction, applied when considering constitutional questions, that when a law is susceptible of two constructions' one of which will maintain and the other destroy it, the courts will always adopt the former. There can be no valid objection to the controlling standard. There was really a grave abuse of discretion when the Board and its perception of what obscenity is is very restrictive. But, sadly, THERE WERE NOT ENOUGH VOTES TO MAINTAIN THAT THERE WAS GRAVE ABUSE OF DISCRETION. The supporting evidence was in the fact that some scenes were not for young people. They might misunderstand the scenes. The respondents offered to make it GP if the petitioners would remove the lesbian and sex scenes. But they refused. The ruling is to be limited to the concept of obscenity applicable to motion pictures. It is the consensus of this Court that where television is concerned: a less liberal approach calls for observance. This is so because unlike motion pictures where the patrons have to pay their way, television reaches every home where CONSTITUTIONAL LAW 2 CASE DIGESTS

there is a set. It is hardly the concern of the law to deal with the sexual fantasies of the adult population. It cannot be denied though that the State as parens patriae is called upon to manifest an attitude of caring for the welfare of the young. MANUEL LAGUNZAD, petitioner, vs.MARIA SOTO VDA. DE GONZALES and THE COURT OF APPEALS, respondents. G.R. No. L-32066 August 6, 1979 FACTS Petitioner Manuel Lagunzad, a newspaperman, began the production of a movie entitled "The Moises Padilla Story" portraying the life of Moises Padilla, a mayoralty candidate of the Nacionalista Party for the Municipality of Magallon, Negros Occidental and for whose murder, Governor Rafael Lacson, a member of the Liberal Party then in power and his men were tried and convicted. The emphasis of the movie was on the public life of Moises Padilla, there were portions which dealt with his private and family life including the portrayal in some scenes, of his mother, Maria Soto, private respondent herein, and of one "Auring" as his girl friend. Padilla’s half sister, for and in behalf of her mother, Vda.de Gonzales, objected to the "exploitation" of his life and demanded in writing for certain changes, corrections and deletions in the movie. After some bargaining as to the amount to be paid Lagunzad and Vda. de Gonzales, executed a "Licensing Agreement" whereby the latter as LICENSOR granted Lagunzad authority and permission to exploit, use, and develop the life story of Moises Padilla for purposes of producing the picture for consideration of P20,000.00.Lagunzad paid Vda. de Gonzales the amount of P5,000.00. Subsequently, the movie was shown indifferent theaters all over the country. Because petitioner refused to pay any additional amounts pursuant to the Agreement, Vda. de Gonzales instituted the present suit against him praying for judgment in her favor ordering petitioner 1) to pay her the balance of P15,000.00, with legal interest from of the Complaint; and 2) to render an accounting of the proceeds from the picture and to pay the corresponding 2-1/2% royalty there from, among others. Petitioner contended in his Answer that the episodes in life of Moises Padilla depicted in the movie were matters of public knowledge and occurred at or about the same time that the deceased became and was a public CONSTITUTIONAL LAW 2 CASE DIGESTS

figure; that private respondent has no property right over those incidents; that the Licensing Agreement was without valid cause or consideration and constitutes an infringement on the constitutional right of freedom of speech and of the press; and that he paid private respondent the amount of P5,000.00 only because of the coercion and threat employed upon him. As a counterclaim, petitioner sought for the nullification of the Licensing Agreement, Both the trial court and the CA ruled in favor of Vda. deGonzales. ISSUES Whether or not the fictionalized representation of Moises Padilla is an intrusion upon his right to privacy notwithstanding that he was a public figure. Whether or not Vda. de Gonzales., the mother, has any property right over the life of Moises Padilla considering that the latter was a public figure. Whether or not the Licensing Agreement constitutes an infringement on the constitutional right of freedom of speech and of the press. HELD YES, being a public figure ipso facto does not automatically destroy in toto a person's right to privacy. The right to invade as person's privacy to disseminate public information does not extend to a fictional or novelized representation of a person, no matter how public a figure he or she may be. In the case at bar, while it is true that petitioner exerted efforts to present a true-to-life story of Moises Padilla, petitioner admits that he included a little romance in the film because without it, it would be a drab story of torture and brutality. YES, Lagunzad cannot dispense with the need for prior consent and authority from the deceased heirs to portray publicly episodes in said deceased's life and in that of his mother and the members of his family. As held in Schuyler v. Curtis" a privilege may be given the surviving relatives of a deceased person to protect his memory, but the privilege exists for the benefit of the living, to protect their feelings and to prevent a violation of their own rights in the character and memory of the deceased." NO, Lagunzad claims that as a citizen and as a newspaperman, he had the right to express his thoughts in film on the public life of Moises Padilla without prior restraint. The right of freedom of expression, indeed, occupies a preferred position in the "hierarchy of civil liberties." It is not, however, without limitations. One criterion for permissible limitation on freedom of speech and of the press is the "balancing-of-interests test." The principle requires a court to take conscious CONSTITUTIONAL LAW 2 CASE DIGESTS

and detailed consideration of the interplay of interests observable in a given situation or type of situation." In the case at bar, the interest’s observable are the right to privacy asserted by respondent and the right of -freedom of expression invoked by petitioner. Taking into account the interplay of those interests, and considering the obligations assumed in the Licensing Agreement entered into by petitioner, the validity of such agreement will have to be upheld particularly because the limits of freedom of expression are reached when expression touches upon matters of essentially private concern AYER PRODUCTIONS VS. CAPULONG 160 SCRA 861; G.R. NO. L-82380; 29 APR 1988 Facts: Petitioner McElroy an Australian film maker, and his movie production company, Ayer Productions, envisioned, sometime in 1987, for commercial viewing and for Philippine and international release, the historic peaceful struggle of the Filipinos at EDSA. The proposed motion picture entitled "The Four Day Revolution" was endorsed by the MTRCB as and other government agencies consulted. Ramos also signified his approval of the intended film production. It is designed to be viewed in a six-hour mini-series television play, presented in a "docu-drama" style, creating four fictional characters interwoven with real events, and utilizing actual documentary footage as background. David Williamson is Australia's leading playwright and Professor McCoy (University of New South Wales) is an American historian have developed a script. Enrile declared that he will not approve the use, appropriation, reproduction and/or exhibition of his name, or picture, or that of any member of his family in any cinema or television production, film or other medium for advertising or commercial exploitation. petitioners acceded to this demand and the name of Enrile was deleted from the movie script, and petitioners proceeded to film the projected motion picture. However, a complaint was filed by Enrile invoking his right to privacy. RTC ordered for the desistance of the movie production and making of any reference to plaintiff or his family and from creating any fictitious character in lieu of plaintiff which nevertheless is based on, or bears substantial or marked resemblance to Enrile. Hence the appeal. CONSTITUTIONAL LAW 2 CASE DIGESTS

Issue: Whether or Not freedom of expression was violated. Held: Yes. Freedom of speech and of expression includes the freedom to film and produce motion pictures and exhibit such motion pictures in theaters or to diffuse them through television. Furthermore the circumstance that the production of motion picture films is a commercial activity expected to yield monetary profit, is not a disqualification for availing of freedom of speech and of expression. The projected motion picture was as yet uncompleted and hence not exhibited to any audience. Neither private respondent nor the respondent trial Judge knew what the completed film would precisely look like. There was, in other words, no "clear and present danger" of any violation of any right to privacy. Subject matter is one of public interest and concern. The subject thus relates to a highly critical stage in the history of the country. At all relevant times, during which the momentous events, clearly of public concern, that petitioners propose to film were taking place, Enrile was a "public figure:" Such public figures were held to have lost, to some extent at least, their right to privacy. The line of equilibrium in the specific context of the instant case between the constitutional freedom of speech and of expression and the right of privacy, may be marked out in terms of a requirement that the proposed motion picture must be fairly truthful and historical in its presentation of events. MTRCB v. ABS-CBN and Loren Legarda GR No. 155282 (2005) FACTS: An episode of “The Inside Story” entitled “Prosti-tuition,” produced by Loren Legarda was aired by ABS-CBN depicting female students moonlighting as prostitutes to enable them to pay for their tuition fees. Philippine Women’s University (PWU) was named as the school of some of the students involved. MTRCB alleged that the episode besmirched the name of the PWU and CONSTITUTIONAL LAW 2 CASE DIGESTS

respondents did not submit “The Inside Story” to MTRCB for review and exhibited the same without its permission, violating Sec. 7 of PD 1986, Sec. 3, Chapter III and Sec. 7, Chapter IV of MTRCB Rules and Regulations. MTRCB declared that all subsequent programs of the “The Inside Story” and all other programs of the ABS-CBN Ch. 2 of the same category shall be submitted to the Board of Review and Approval before showing. On appeal, RTC ruled that Sections 3 (c) (d), 4, 7 and 11 of P. D. No. 1986 and Sections 3, 7 and 28 (a) of the MTRCB Rules and Regulations are unconstitutional for violating the freedom of expression and of the press guaranteed by the Constitution ISSUE : Whether or not there is compliance with the legal requisites for judicial inquiry so as to proceed with the issue on constitutionality. RULING: NO. There is no need to resolve whether certain provisions of PD 1986 and MTRCB Rules and Regulations contravene the Constitution. No question involving the constitutionality or validity of a law or governmental act may be heard and decided by the court unless there is compliance with the legal requisites for judicial inquiry: 1) proper party 2)actual case or controversy 3) question raised at the earliest possible opportunity and 4) that the decision on the constitutional or legal question must be necessary to the determination of the case itself. The fourth requisite is wanting. MTRCB did not disapprove or ban the showing of the program nor did it cancel respondents’ permit. The latter was merely penalized for their failure to submit the program to MTRCB for its review and approval. Therefore, the issue of constitutionality is not necessary to the determination of the case itself. Soriano vs. La Guardia G.R. No. 164785 April 29, 2009 FACTS: On August 10, 2004, at around 10:00 p.m., petitioner, as host of the program Ang Dating Daan, aired on UNTV 37, made obscene remarks against INC. Two days after, before the MTRCB, separate but almost identical affidavit-complaints were lodged by Jessie L. Galapon and seven other private respondents, all members of CONSTITUTIONAL LAW 2 CASE DIGESTS

the Iglesia ni Cristo (INC), against petitioner in connection with the above broadcast. Respondent Michael M. Sandoval, who felt directly alluded to in petitioner’s remark, was then a minister of INC and a regular host of the TV program Ang Tamang Daan. ISSUE: Are Soriano’s statements during the televised “Ang Dating Daan” part of the religious discourse and within the protection of Section 5, Art.III? RULING: No. Under the circumstances obtaining in this case, therefore, and considering the adverse effect of petitioner’s utterances on the viewers’ fundamental rights as well as petitioner’s clear violation of his duty as a public trustee, the MTRCB properly suspended him from appearing in Ang Dating Daan for three months. Furthermore, it cannot be properly asserted that petitioner’s suspension was an undue curtailment of his right to free speech either as a prior restraint or as a subsequent punishment. Aside from the reasons given above (re the paramountcy of viewers rights, the public trusteeship character of a broadcaster’s role and the power of the State to regulate broadcast media), a requirement that indecent language be avoided has its primary effect on the form, rather than the content, of serious communication. There are few, if any, thoughts that cannot be expressed by the use of less offensive language. Iglesia ni Cristo vs. Court of Appeals, G.R. No. 119673. July 26, 1996 FACTS: Petitioner has a television program entitled “Ang Iglesia ni Cristo” aired on Channel 2 every Saturday and on Channel 13 every Sunday. The program presents and propagates petitioner’s religious beliefs, doctrines and practices often times in comparative studies with other religions. Petitioner submitted to the respondent Board of Review for Moving Pictures and Television the VTR tapes of its TV program Series Nos. 116, 119, 121 and 128. The Board classified the series as “X” CONSTITUTIONAL LAW 2 CASE DIGESTS

or not for public viewing on the ground that they “offend and constitute an attack against other religions which is expressly prohibited by law.” On November 28, 1992, it appealed to the Office of the President the classification of its TV Series No. 128 which allowed it through a letter of former Executive Secretary Edelmiro A. Amante, Sr., addressed for Henrietta S. Mendez reversing the decision of the respondent Board. According to the letter the episode in is protected by the constitutional guarantee of free speech and expression and no indication that the episode poses any clear and present danger. Petitioner also filed a civil case. Petitioner alleged that the respondent Board acted without jurisdiction or with grave abuse of discretion in requiring petitioner to submit the VTR tapes of its TV program and in x-rating them. It cited its TV Program Series Nos. 115, 119, 121 and 128. In their Answer, respondent Board invoked its power under PD No. 19861 in relation to Article 201 of the Revised Penal Code. The Iglesia ni Cristo insists on the literal translation of the Bible and says that the (Catholic) veneration of the Virgin Mary is not to be condoned because it was not in the Bible. The board contended that it outrages Catholic and Protestant beliefs. RTC ruled in favor of petitioners. CA however reversed it hence this petition. ISSUE: Was the Iglesia ni Cristo program constitutionally protected as a form of religious exercise and expression? – YES. RATIO: Any act that restrains speech is accompanied with presumption of invalidity. It is the burden of the respondent Board to overthrow this presumption. Respondent failed to do this. So-called “attacks” are mere criticisms of some of the deeply held dogmas and tenets of other religions. RTC’s ruling clearly suppresses petitioner’s freedom of speech and interferes with its right to free exercise of religion. “Attack” is different from “offend” any race or religion. The respondent Board may disagree with the criticisms of other religions by petitioner but that gives it no excuse to interdict such criticisms, however, unclean they may be. Under our constitutional scheme, it is not the task of the State to favor any religion by protecting it against an attack by another religion. Religious dogmas CONSTITUTIONAL LAW 2 CASE DIGESTS

and beliefs are often at war and to preserve peace among their followers, especially the fanatics; the establishment clause of freedom of religion prohibits the State from leaning towards any religion. The basis of freedom of religion is freedom of thought and it is best served by encouraging the marketplace of dueling ideas. It is only where it is unavoidably necessary to prevent an immediate and grave danger to the security and welfare of the community that infringement of religious freedom may be justified, and only to the smallest extent necessary to avoid the danger. There is no showing whatsoever of the type of harm the tapes will bring about especially the gravity and imminence of the threatened harm. Prior restraint on speech, including religious speech, cannot be justified by hypothetical fears but only by the showing of a substantive and imminent evil. It is inappropriate to apply the clear and present danger test to the case at bar because the issue involves the content of speech and not the time, place or manner of speech. Allegedly, unless the speech is first allowed, its impact cannot be measured, and the causal connection between the speech and the evil apprehended cannot be established. The determination of the question as to whether or not such vilification, exaggeration or fabrication falls within or lies outside the boundaries of protected speech or expression is a judicial function which cannot be arrogated by an administrative body such as a Board of Censors.” A system of prior restraint may only be validly administered by judges and not left to administrative agencies. RADIO BROADCASTS EASTERN BROADCASTING CORP (DYRE) V. DANS JR. 137 SCRA 628; L-59329; 19 JUL 1985 Facts: A petition was filed to reopen the Radio Station DYRE. DYRE was “summarily closed” on grounds of national security. The radio station was allegedly used to incite people to sedition. Petitioner, DYRE contends that they were denied due process. There was no hearing to establish factual evidence for the closure. Furthermore, the closure of the radio station violates freedom of expression. Before the court could even promulgate a decision upon the Issue raised, Petitioner, through its president Mr. Rene Espina, filed a motion to withdraw the CONSTITUTIONAL LAW 2 CASE DIGESTS

petition. The rights of the station were sold to a new owner, Manuel Pastrana; who is no longer interested in pursuing the case. Despite the case becoming moot and academic, (because there are no longer interested parties, thus the dismissal of the case) the Supreme Court still finds that there is need to pass a “RESOLUTION” for the guidance of inferior courts and administrative tribunals in matters as this case. Issues: (1) Whether or not due process was exercised in the case of DYRE. (2) Whether or not the closure of DYRE is a violation of the Constitutional Right of Freedom of Expression. Held: The court finds that the closure of the Radio Station in 1980 as null and void. The absence of a hearing is a violation of Constitutional Rights. The primary requirements in administrative proceedings are laid down in the case of Ang Tibay v. Court of Industrial Relation (69 Phil.635). The Ang Tibay Doctrine should be followed before any broadcast station may be closed. The Ang Tibay Doctrine provides the following requirements: (1) The right to hearing, includes the right to present one’s case and submit evidence presented. (2) The tribunal must consider the evidence presented (3) The decision must have something to support itself. (4) Evidence must be substantial (reasonable evidence that is adequate to support conclusion) (5) Decision must be based on the evidence presented at hearing (6) The tribunal body must act on its own independent consideration of law and facts and not simply accept subordinate’s views (7) Court must render decision in such a manner that the proceeding can know the various issued involved and reasons for decisions rendered. The court stresses that while there is no controlling and precise definition of Due Process, it gives an unavoidable standard that government actions must conform in order that deprivation of life, liberty and property is valid. CONSTITUTIONAL LAW 2 CASE DIGESTS

The closure of the radio station is like wise a violation of the constitutional right of freedom of speech and expression. The court stresses that all forms of media, whether print or broadcast are entitled to this constitutional right. Although the government still has the right to be protected against broadcasts which incite the listeners to violently overthrow it. The test for the limitation of freedom of expression is the “clear and present danger” rule. If in the circumstances that the media is used in such nature as to create this danger that will bring in such evils, then the law has the right to prevent it. However, Radio and television may not be used to organize a rebellion or signal a start of widespread uprising. The freedom to comment on public affairs is essential to the vitality of a representative democracy. The people continues to have the right to be informed on public affairs and broadcast media continues to have the pervasive influence to the people being the most accessible form of media. Therefore, broadcast stations deserve the the special protection given to all forms of media by the due process and freedom of expression clauses of the Constitution. FREEDOM OF ASSEMBLY Primicias v Fugoso 80 PHIL 71 (1948) Facts: An action was instituted by the petitioner for the refusal of the respondent to issue a permit to them to hold a public meeting in Plaza Miranda for redress of grievances to the government. The reason alleged by the respondent in his defense for refusing the permit is, "that there is a reasonable ground to believe, basing upon previous utterances and upon the fact that passions, specially on the part of the losing groups, remains bitter and high, that similar speeches will be delivered tending to undermine the faith and confidence of the people in their government, and in the duly constituted authorities, which might threaten breaches of the peace and a disruption of public order." Giving emphasis as well to the delegated police power to local government. Stating as well Revised Ordinances of 1927 prohibiting as an offense against public peace, and penalizes as a misdemeanor, "any act, in any public place, meeting, or procession, tending to disturb the peace or excite a riot; or collect with other persons in a body or CONSTITUTIONAL LAW 2 CASE DIGESTS

crowd for any unlawful purpose; or disturb or disquiet any congregation engaged in any lawful assembly." Included herein is Sec. 1119, Free use of Public Place. Issue: Whether or Not the freedom of speech was violated. Held: Yes. Dealing with the ordinance, specifically, Sec. 1119, said section provides for two constructions: (1) the Mayor of the City of Manila is vested with unregulated discretion to grant or refuse, to grant permit for the holding of a lawful assembly or meeting, parade, or procession in the streets and other public places of the City of Manila; (2) The right of the Mayor is subject to reasonable discretion to determine or specify the streets or public places to be used with the view to prevent confusion by overlapping, to secure convenient use of the streets and public places by others, and to provide adequate and proper policing to minimize the risk of disorder. The court favored the second construction. First construction tantamount to authorizing the Mayor to prohibit the use of the streets. Under our democratic system of government no such unlimited power may be validly granted to any officer of the government, except perhaps in cases of national emergency. The Mayor’s first defense is untenable. Fear of serious injury cannot alone justify suppression of free speech and assembly. It is the function of speech to free men from the bondage of irrational fears. To justify suppression of free speech there must be reasonable ground to fear that serious evil will result if free speech is practiced. There must be reasonable ground to believe that the danger apprehended is imminent. There must be reasonable ground to believe that the evil to be prevented is a serious one. The fact that speech is likely to result in some violence or in destruction of property is not enough to justify its suppression. There must be the probability of serious injury to the state. Ignacio VS. ELa 99 SCRA436 FACTS: CONSTITUTIONAL LAW 2 CASE DIGESTS

1. Petitioners Fernando and Simeon dela Cruz are members of the Jehovah’s Witness, whose tenets and principles are derogatory to those professed by the Catholic organization. 2. Desiring to hold a meting in furtherance of its objectives, petitioners asked respondent Mayor Norberto Ela of Zambales to give them permission to use the public plaza together with the kiosk. 3. Instead of granting the permission, respondent Mayor allowed them to hold their meeting on the northern part corner of the plaza. 4. The mayor adopted as a policy not to allow the use of the kiosk for any meeting by any religious denomination as it is his belief that said kiosk should only be used “for legal purposes.” 5. Petitioners contend that the action taken by respondent is unconstitutional being an abridgment of the freedom of speech, assembly, and worship guaranteed by the Constitution. ISSUE: Was there a violation of petitioner’s constitutional rights? HELD: The right to freedom of speech and to peacefully assemble, though guaranteed by our Constitution, is not absolute, for it may be regulated in order that it may not be “injurious to the equal enjoyment of others having equal rights, nor injurious to the right of the community or society,” and this power may be exercised under the “police power” of the State, which is the power to prescribe regulations to promote the health, morals, peace, education, good order or safety, and general welfare of the people. The power exercised by respondent cannot be considered as capricious or arbitrary considering the peculiar circumstances of this case. It appears that the public plaza particularly the kiosk, is located at a short distance from the Roman Catholic Church. The proximity of said church to the kiosk has cause some concern on the part of the authorities that to avoid disturbance of peace and order, or the happening of untoward incidents, they deemed it necessary to prohibit the use of the kiosk by any religious denomination as a place of meeting of its members. This was the policy adopted by respondent for sometime previous to the request made CONSTITUTIONAL LAW 2 CASE DIGESTS

by petitioners. Respondent never denied such request but merely tried to enforce his policy by assigning them the northern part of the public plaza. It cannot therefore be said that petitioners were denied their constitutional right to assembly for such right is subject to regulation to maintain public order and public safety. This is especially so considering that the tenet of petitioners’ congregation are derogatory to those of the Roman Catholic Church, a factor which respondent must have considered in denying their request.

REYES VS BAGATSING FACTS: Retired Justice Reyes in behalf of the Anti-Bases Coalition, sought to permit a rally permit from Luneta Park to front gate of the US Embassy in Manila. Mayor Bagatsing denied the petition. He issued City Ordinance No. 7295 to prohibit rallying 500 meter radius around the Embassy. ISSUE: Whether or not the Mayor violated the petitioners' constitutional right. RULING: Yes, the mayor's ordinance which prohibit the petitioners to rally violates their constitutional right to free speech and peaceable assembly. It is settled law that as to public places, especially so as to parks and streets, there is freedom of access. Nor is their use dependent on who is the applicant for the permit, whether an individual or a group. Ruiz v Gordon G.R. No. L-65695 December 19, 1983 Facts: Hector S. Ruiz, Coordinator of Olongapo Citizen's Alliance for National Reconciliation, filed a petition for mandamus against Richard Gordon to be allowed to hold a parade/march from Gordon Avenue to the Rizal Triangle starting at 1:00 P.M. CONSTITUTIONAL LAW 2 CASE DIGESTS

The Court required the respondents to answer. Respondents replied by stating the request for a prayer rally was received in the Office of the Mayor and that respondent had repeatedly announced in his regular program on Sunday over the radio (DWGO) and at the Monday morning flag ceremony before hundreds of government employees that he would grant the request of any group that would like to exercise their freedom of speech and assembly. When interviewed on the matter by the Editor-in Chief of the 'Guardian', he mentioned the fact that he had granted the permit of the petitioner, which interview appeared in the November 22-28, 1983 issue of the said newspaper. Given these, the respondent prayed for the dismissal of the petition. This was complied with. Issue: Can the petition be granted? Held No. Petition dismissed. Ratio: The Reyes case was given some discussion in the course of this petition as to the role of the judiciary in petitions for permits to hold peaceable assembles. "The applicants for a permit to hold an assembly should inform the licensing authority of the date, the public place where and the time when it will take place. If it were a private place, only the consent of the owner or the one entitled to its legal possession is required. Such application should be filed well ahead in time to enable the public official concerned to appraise whether there may be valid objections to the grant of the permit or to its grant but at another public place. It is an indispensable condition to such refusal or modification that the clear and present danger test be the standard for the decision reached. If he is of the view that there is such an imminent and grave danger of a substantive evil, the applicants must be heard on the matter.” Thereafter, his decision must be transmmitted to them at the earliest opportunity. They can have recourse to the proper judicial authority. Free speech and peaceable assembly, along with the other intellectual freedoms, are highly ranked in our scheme of constitutional values. It cannot be too strongly stressed that on the judiciary, — even more so than on the other departments — rests the grave and delicate responsibility of assuring respect for and deference to such preferred rights. CONSTITUTIONAL LAW 2 CASE DIGESTS

As shown both in the manifestation and the answer, this action for mandamus could have been obviated if only petitioner took the trouble of verifying on November 23 whether or not a permit had been issued. A party desirous of exercising the right to peaceable assembly should be the one most interested in ascertaining the action taken on a request for a permit. Necessarily, after a reasonable time or, if the day and time was designated for the decision on the request, such party or his representative should be at the office of the public official concerned. If he fails to do so, a copy of the decision reached, whether adverse or favorable, should be sent to the address of petitioner. Teehankee concurring: The burden to show the existence of such grave and imminent danger that would justify an adverse action lies on the mayor as the licensing authority. There must be objective and convincing, not subjective or conjectural, proof of the existence of such clear and present danger. As the Court stated in its Resolution of October 25, 1983 in the J. B. L. Reyes case, "It is essential for the validity of a denial of a permit which amounts to a previous restraint or censorship that the licensing authority does not rely solely on his own appraisal of what public welfare, peace or safety may require. To justify such a limitation, there must be proof of such weight and sufficiency to satisfy the clear and present danger test. The possibility that subversives may infiltrate the ranks of the demonstrators is not enough." As likewise underscored in the J. B. L. Reyes case, the exercise of the right of peaceable assembly is not to be 4 abridged on the plea that it may be exercised in some other place" (at paragraph 6) and "It is the duty of the city authorities to provide the proper police protection to those exercising their right to peaceable assembly and freedom of expression" (at paragraph 7). J. Conception concurring opinion 9 In order that public officials may not be charged, rightly or wrongly, with dereliction of duty or abuse of powers in the granting or denying of such permits, the following guidelines are deemed necessary: (a) When a peaceful assembly is to be held in a private lot, house, or edifice, only the consent of the owner of the place is necessary. No permit from the government or any public officer is required. (b) When an application to hold a rally, parade, or peaceful assembly has to make use of public places like parks, plazas, and streets, the public authority charged with the duty of granting or denying the permit should also consider the convenience and the right of the rest of the public to use and enjoy these same facilities. CONSTITUTIONAL LAW 2 CASE DIGESTS

(c) Conditions of peace and order in the locality should be carefully considered and precautionary steps taken to prevent vandals, hooligans, provocateurs, and other criminals from turning into a violent one what otherwise should be a peaceful demonstration. MALABANAN VS. RAMENTO [129 SCRA 359; G.R. NO.62270; 21 MAY 1984] Facts: Petitioners were officers of the Supreme Student Council of respondent University. They sought and were granted by the school authorities a permit to hold a meeting from 8:00 A.M. to 12:00 P.M, on August 27, 1982. Pursuant to such permit, along with other students, they held a general assembly at the Veterinary Medicine and Animal Science basketball court (VMAS), the place indicated in such permit, not in the basketball court as therein stated but at the second floor lobby. At such gathering they manifested in vehement and vigorous language their opposition to the proposed merger of the Institute of Animal Science with the Institute of Agriculture. The same day, they marched toward the Life Science Building and continued their rally. It was outside the area covered by their permit. Even they rallied beyond the period allowed. They were asked to explain on the same day why they should not be held liable for holding an illegal assembly. Then on September 9, 1982, they were informed that they were under preventive suspension for their failure to explain the holding of an illegal assembly. The validity thereof was challenged by petitioners both before the Court of First Instance of Rizal against private respondents and before the Ministry of Education, Culture, and Sports. Respondent Ramento found petitioners guilty of the charge of illegal assembly which was characterized by the violation of the permit granted resulting in the disturbance of classes and oral defamation. The penalty was suspension for one academic year. Hence this petition. Issue: Whether on the facts as disclosed resulting in the disciplinary action and the penalty imposed, there was an infringement of the right to peaceable assembly and its cognate right of free speech.

CONSTITUTIONAL LAW 2 CASE DIGESTS

Held: Yes. Student leaders are likely to be assertive and dogmatic. They would be ineffective if during a rally they speak in the guarded and judicious language of the academe. But with the activity taking place in the school premises and during the daytime, no clear and present danger of public disorder is discernible. This is without prejudice to the taking of disciplinary action for conduct, "materially disrupts classwork or involves substantial disorder or invasion of the rights of others." The rights to peaceable assembly and free speech are guaranteed students of educational institutions. Necessarily, their exercise to discuss matters affecting their welfare or involving public interest is not to be subjected to previous restraint or subsequent punishment unless there be a showing of a clear and present danger to a substantive evil that the state, has a right to present. As a corollary, the utmost leeway and scope is accorded the content of the placards displayed or utterances made. The peaceable character of an assembly could be lost, however, by an advocacy of disorder under the name of dissent, whatever grievances that may be aired being susceptible to correction through the ways of the law. If the assembly is to be held in school premises, permit must be sought from its school authorities, who are devoid of the power to deny such request arbitrarily or unreasonably. In granting such permit, there may be conditions as to the time and place of the assembly to avoid disruption of classes or stoppage of work of the non-academic personnel. Even if, however, there be violations of its terms, the penalty incurred should not be disproportionate to the offense. ACOSTA V COURT OF APPEALS Facts: Petitioners are teachers from different public schools in metro manila. On various dates in September and October 1990, petitioners did not report for work and instead, participated in mass actions by public school teachers at the liwasang bonifacio for the purpose of petitioning the government for redress of their grievances. On the basis of reports submitted by their respective school principals that petitioners participated in said mass actions and refused to comply with the return-to-work order issued September 17, 1990 by then Secretary Isidro D. Cariño of the department of education, culture and sports (DECS), petitioners were administratively charged with such offenses as grave misconduct, gross CONSTITUTIONAL LAW 2 CASE DIGESTS

neglect of duty, gross violation of civil service law, rules and regulations and reasonable office regulations, refusal to perform official duty, gross insubordination, conduct prejudicial to the best interest of the service and absence without official leave. Petitioners failed to answer the charges. Secretary Cariño found petitioners guilty as charged and ordered their immediate dismissal from the service. Is hereby meted out the penalty of six (6) months suspension without pay. Petitioners appealed that respondent court of appeals grievously erred when it affirmed the assailed resolutions of CSC. Issues: WON respondent court of appeals grievously erred when it affirmed the assailed resolutions of the civil service commission that wrongly penalized petitioners whose only “offense” was to exercise their consitutional right to peaceably assemble and petition the government for redress of grievances? WON respondent court of appeals grievously erred when it affirmed the assailed resolutions of the civil service commission that wrongly denied petitioners their right to backwages? Decision: No. Petition denied. In Bangalisan v. Court of appeals, petitioners, are being penalized not because they exercised their right of peaceable assembly and petition for redress of grievances but because of their successive unauthorized and unilateral absences which produced adverse effects upon their students for whose education they are responsible. The actuations of petitioners definitely constituted conduct prejudicial to the best interest of the service, punishable under the civil service law, rules and regulations.as aptly stated by the solicitor general, “it is not the exercise by the petitioners of their constitutional right to peaceable assemble that was punished, but the manner in which they exercised such right which resulted in the temporary stoppage or disruption of public service and classes in various public schools in metro manila. For, indeed, there are efficient and non-disruptive avenues, other than the mass actions in question, whereby petitioners could petition the government for redress of grievances. As a general proposition, a public official is not entitled to any compensation if he has not rendered any service. While there recognized instances when backwages may be awarded to a suspended or dismissed public official who is later ordered CONSTITUTIONAL LAW 2 CASE DIGESTS

reinstated, as pointed by petitioners in citing bangalisan, the factual circumstances of the case at bar impel the Court to rule otherwise. Bayan, et al., Vs. Eduardo Ermita, et al., G.R. No. 169838 April 25, 2006 Facts: The petitioners, Bayan, et al., alleged that they are citizens and taxpayers of the Philippines and that their right as organizations and individuals were violated when the rally they participated in on October 6, 2005 was violently dispersed by policemen implementing Batas Pambansa No. 880. Petitioners contended that Batas Pambansa No. 880 is clearly a violation of the Constitution and the International Covenant on Civil and Political Rights and other human rights treaties of which the Philippines is a signatory. They argue that B.P. No. 880 requires a permit before one can stage a public assembly regardless of the presence or absence of a clear and present danger. It also curtails the choice of venue and is thus repugnant to the freedom of expression clause as the time and place of a public assembly form part of the message which the expression is sought. Furthermore, it is not content-neutral as it does not apply to mass actions in support of the government. The words “lawful cause,” “opinion,” “protesting or influencing” suggest the exposition of some cause not espoused by the government. Also, the phrase “maximum tolerance” shows that the law applies to assemblies against the government because they are being tolerated. As a content-based legislation, it cannot pass the strict scrutiny test. This petition and two other petitions were ordered to be consolidated on February 14, 2006. During the course of oral arguments, the petitioners, in the interest of a speedy resolution of the petitions, withdrew the portions of their petitions raising factual issues, particularly those raising the issue of whether B.P. No. 880 and/or CPR is void as applied to the rallies of September 20, October 4, 5 and 6, 2005. Issue: Whether the Calibrated Pre-emptive response and the Batas Pambansa No. 880, specifically Sections 4, 5, 6, 12, 13(a) and 14(a) violates Art. III Sec. 4 of the Philippine Constitution as it causes a disturbing effect on the exercise by the people of the right to peaceably assemble. CONSTITUTIONAL LAW 2 CASE DIGESTS

Held: Section 4 of Article III of the Philippine Constitution provides that no law shall be passed abridging the freedom of speech, of expression, or of the press, or the right of the people peaceably to assemble and petition the government for redress of grievances. The right to peaceably assemble and petition for redress of grievances, together with freedom of speech, of expression, and of the press, is a right that enjoys dominance in the sphere of constitutional protection. For this rights represent the very basis of a functional democratic polity, without which all the other rights would be meaningless and unprotected. However, it must be remembered that the right, while sacrosanct, is not absolute. It may be regulated that it shall not be injurious to the equal enjoyment of others having equal rights, nor injurious to the rights of the community or society. The power to regulate the exercise of such and other constitutional rights is termed the sovereign “police power,” which is the power to prescribe regulations, to promote the health, morals, peace, education, good order or safety, and general welfare of the people. B.P. No 880 is not an absolute ban of public assemblies but a restriction that simply regulates the time, place and manner of the assemblies. B.P. No. 880 thus readily shows that it refers to all kinds of public assemblies that would use public places. The reference to “lawful cause” does not make it content-based because assemblies really have to be for lawful causes, otherwise they would not be “peaceable” and entitled to protection. Neither the words “opinion,” “protesting,” and “influencing” in of grievances come from the wording of the Constitution, so its use cannot be avoided. Finally, maximum tolerance is for the protection and benefit of all rallyist and is independent of the content of the expression in the rally. Furthermore, the permit can only be denied on the ground of clear and present danger to public order, public safety, public convenience, public morals or public health. This is a recognized exception to the exercise of the rights even under the Universal Declaration of Human Rights and The International Covenant on Civil and Political Rights. Wherefore, the petitions are GRANTED in part, and respondents, more particularly the Secretary of the Interior and Local Governments, are DIRECTED to take all CONSTITUTIONAL LAW 2 CASE DIGESTS

necessary steps for the immediate compliance with Section 15 of Batas Pambansa No. 880 through the establishment or designation of at least one suitable freedom park or plaza in every city and municipality of the country. After thirty (30) days from the finality of this Decision, subject to the giving of advance notices, no prior permit shall be required to exercise the right to peaceably assemble and petition in the public parks or plaza in every city or municipality that has not yet complied with section 15 of the law. Furthermore, Calibrated pre-emptive response (CPR), insofar as it would purport to differ from or be in lieu of maximum tolerance, is NULL and VOID and respondents are ENJOINED to REFRAIN from using it and to STRICTLY OBSERVE the requirements of maximum tolerance, The petitions are DISMISSED in all other respects, and the constitutionality of Batas Pambansa No. 880 is SUSTAINED

People vs. Tudtud, GR No. 144037 FACTS: Sometime during the months of July and August 1999, the Toril Police Station, Davao City received a report from a “civilian asset” named Bobong Solier about a certain Noel Tudtud. Solier related that his neighbors have been complaining about Tudtud, who was allegedly responsible for the proliferation of marijuana in their area. Reacting to the report, PO1 Ronald Desierto, PO1 Ramil Floreta and their superior, SPO1 Villalonghan, all members of the Intelligence Section of the Toril Police Station, conducted surveillance in Solier’s neighborhood in Sapa, Toril, Davao City. For 5 days, they gathered information and learned that Tudtud was involved in illegal drugs. According to his neighbors, Tudtud was engaged in selling marijuana. Solier informed the police that Tudtud had headed to Cotabato and would be back later that day with new stocks of marijuana. Solier described Tudtud as big-bodied and short, and usually wore a hat. At around 4:00 p.m. that same day, a team composed of PO1 Desierto, PO1 Floreta and SPO1 Villalonghan posted themselves at the corner of Saipon and McArthur Highway to await Tudtud’s arrival. All wore civilian clothes. About 8:00 p.m., 2 men disembarked from a bus and helped each CONSTITUTIONAL LAW 2 CASE DIGESTS

other carry a carton marked “King Flakes.” Standing some 5 feet away from the men, PO1 Desierto and PO1 Floreta observed that one of the men fit Tudtud’s description. The same man also toted a plastic bag. PO1 Floreta and PO1 Desierto then approached the suspects and identified themselves as police officers. PO1 Desierto informed them that the police had received information that stocks of illegal drugs would be arriving that night. The man who resembled Tudtud’s description denied that he was carrying any drugs. PO1 Desierto asked him if he could see the contents of the box. Tudtud obliged, saying, “it was alright.” Tudtud opened the box himself as his companion looked on. The box yielded pieces of dried fish, beneath which were two bundles, one wrapped in a striped plastic bag and another in newspapers. PO1 Desierto asked Tudtud to unwrap the packages. They contained what seemed to the police officers as marijuana leaves. The police thus arrested Tudtud and his companion, informed them of their rights and brought them to the police station. The two did not resist. The confiscated items were turned over to the Philippine National Police (PNP) Crime Laboratory for examination. Forensic tests on specimens taken from the confiscated items confirmed the police officers’ suspicion. The plastic bag contained 3,200 grams of marijuana leaves while the newspapers contained another 890 grams. Noel Tudtud and his companion, Dindo Bulong, were subsequently charged before the RTC of Davao City with illegal possession of prohibited drugs. Upon arraignment, both accused pleaded not guilty. The defense, however, reserved their right to question the validity of their arrest and the seizure of the evidence against them. Trial ensued thereafter. Tudtud, denying the charges against them, cried frame-up. Swayed by the prosecution’s evidence beyond reasonable doubt, the RTC rendered judgment convicting the accused as charged and sentencing them to suffer the penalty of reclusion perpetua and to pay a fine of P500,000.00. On appeal, Noel Tudtud and Dindo Bolong assign, among other errors, the admission in evidence of the marijuana leaves, which they claim were seized in violation of their right against unreasonable searches and seizures. ISSUE: Whether or not the Tudtud’s implied acquiescence (Tudtud’s statement of “it’s all right” when the police officers requested that the box be opened) be considered a waiver. HELD: NO. The right against unreasonable searches and seizures is secured by Section 2, Article III of the Constitution. The RTC justified the warrantless search of CONSTITUTIONAL LAW 2 CASE DIGESTS

appellants’ belongings under the first exception, as a search incident to a lawful arrest. A search incidental to a lawful arrest is sanctioned by the Rules of Court. It is significant to note that the search in question preceded the arrest. Recent jurisprudence holds that the arrest must precede the search; the process cannot be reversed. Nevertheless, a search substantially contemporaneous with an arrest can precede the arrest if the police have probable cause to make the arrest at the outset of the search. The question, therefore, is whether the police herein had probable cause to arrest Tudtud, et. al. The long-standing rule in this jurisdiction, applied with a great degree of consistency, is that “reliable information” alone is not sufficient to justify a warrantless arrest under Section 5 (a), Rule 113. The rule requires, in addition, that the accused perform some overt act that would indicate that he “has committed, is actually committing, or is attempting to commit an offense.” For the exception in Section 5 (a), Rule 113 to apply, this Court ruled, two elements must concur: (1) the person to be arrested must execute an overt act indicating he has just committed, is actually committing, or is attempting to commit a crime; and (2) such overt act is done in the presence or within the view of the arresting officer. Reliable information alone is insufficient. Thus, herein, in no sense can the knowledge of the arresting officers that Tudtud was in possession of marijuana be described as “personal,” having learned the same only from their informant Solier. Solier, for his part, testified that he obtained his information only from his neighbors and the friends of Tudtud. Solier’s information is hearsay. Confronted with such a dubious informant, the police perhaps felt it necessary to conduct their own “surveillance.” This “surveillance,” it turns out, did not actually consist of staking out Tudtud to catch him in the act of plying his illegal trade, but of a mere “gathering of information from the assets there.” The police officers who conducted such “surveillance” did not identify who these “assets” were or the basis of the latter’s information. Clearly, such information is also hearsay, not of personal knowledge. Finally, there is an effective waiver of rights against unreasonable searches and seizures only if the following requisites are present: (1) It must appear that the rights exist; (2) The person involved had knowledge, actual or constructive, of the existence of such right; (3) Said person had an actual intention to relinquish the right. Here, the prosecution failed to establish the second and third requisites. Records disclose that when the police officers introduced themselves as such and CONSTITUTIONAL LAW 2 CASE DIGESTS

requested Tudtud that they see the contents of the carton box supposedly containing the marijuana, Tudtud said “it was alright.” He did not resist and opened the box himself. Tudtud's implied acquiescence, if at all, could not have been more than mere passive conformity given under coercive or intimidating circumstances and is, thus, considered no consent at all within the purview of the constitutional guarantee. Consequently, Tudtud's lack of objection to the search and seizure is not tantamount to a waiver of his constitutional right or a voluntary submission to the warrantless search and seizure. As the search of Tudtud's box does not come under the recognized exceptions to a valid warrantless search, the marijuana leaves obtained thereby are inadmissible in evidence. And as there is no evidence other than the hearsay testimony of the arresting officers and their informant, the conviction of Tudtud, et. al. cannot be sustained. Silahis Int’l Hotel vs. Soluta FACTS: Petitioner Jose Marcel Panlilio, Vice President for Finance of petitioner Silahis International Hotel, Inc. (Silahis), with his personal secretary, a Bulletin reporter, and a security guard entered the union office located at the hotel basement. The same is with the permission of union officer Henry Babay. Babay was apprised about the suspected illegal activities. During the search they discovered marijuana flowering tops in the union office. An Information indicting the union officers was then filed before the Regional Trial Court (RTC) for violation of Republic Act 6425, as amended by Batas Pambansa Bilang 179 (The Dangerous Drugs Act). The RTC acquitted the accused on the ground that the marijuana tops are inadmissible as evidence. Soluta and his fellow union officers including the union thereafter filed before the RTC a complaint against Silahis, Panlilio and those who cooperated for malicious prosecution and violation of their constitutional right against illegal search. The RTC granted such petition. It ruled that Silahis and Panlilio are jointly and severally liable to pay for damages in favor of Soluta et al. Silahis and Panlilio appealed to the Court of Appeals (CA). On appeal, the CA affirmed the lower court‘s decision. ISSUE: Whether or not Silahis and Panlilio violated the constitutional right of Soluta et al. HELD: CONSTITUTIONAL LAW 2 CASE DIGESTS

As constitutional rights, like the right to be secure in one‘s person, house, papers, and effects against unreasonable search and seizures, occupy a lofty position in every civilized and democratic community and not infrequently susceptible to abuse, their violation, whether constituting a penal offense or not, must be guarded against. The Code Commission thus deemed it necessary to hold not only public officers but also private individuals civilly liable for violation of rights enumerated in Article 32 of the Civil Code. That is why it is not even necessary that the defendant under this Article should have acted with malice or bad faith, otherwise, it would defeat its main purpose, which is the effective protection of individual rights. It suffices that there is a violation of the constitutional right of the plaintiff. In the present case, as priorly stated, Silahis and Panlilio had, by their own claim, already received reports in late 1987 of illegal activities allegedly undertaken in the union office and Maniego conducted surveillance of the union officers. Yet, in the morning of January 11, 1988, Silahis, Panlilio and their companions barged into and searched the union office without a search warrant, despite ample time for them to obtain one, and notwithstanding the objection of Babay. The course taken by Silahis and company stinks in illegality, it not falling under any of the exceptional instances when a warrantless search is allowed by law. Silahis and Panlilio‘s violation of individual Soluta et al.‘s constitutional right against unreasonable search thus furnishes the basis for the award of damages under Article 32 of the Civil Code. As for Silahis et al.‘s contention that property rights justified the search of the union office, the same does not lie. For Soluta et al., being the lawful occupants of the office, had the right to raise the question of validity of the search and seizure. Neither does Silahis et al.‘s claim that they were allowed by union officer Babay to enter the union office lie. Babay‘s account of why Silahis and company went to the union office – to consider Panlilio‘s suggestion to settle the mauling incident is more credible, as is his claim that he protested the search, and even asked if they were armed with a search warrant. While it is doctrinal that the right against unreasonable searches and seizures is a personal right which may be waived expressly or impliedly, a waiver by implication cannot be presumed. There must be clear and convincing evidence of an actual intention to relinquish it to constitute a waiver thereof. There must be proof of the following: (a) that the right exists; (b) that the person involved had knowledge, CONSTITUTIONAL LAW 2 CASE DIGESTS

either actual or constructive, of the existence of such right; and, (c) that the said person had an actual intention to relinquish the right. In other words, the waiver must be voluntarily, knowingly and intelligently made. The evidence shows otherwise, however. Benjamin v Elizalde Facts: Benjamin Victoriano, an Iglesia ni Cristo (INC) member, has been an employee of the Elizalde Rope Factory (ERF) since 1958. He was also a member of the EPWU (Elizalde Rope Workers’ Union). Under the collective bargaining agreement (CBA) between ERF and EPWU, a close shop agreement is being enforced which means that employment in the factory relies on the membership in the EPWU; that in order to retain employment in the said factory one must be a member of the said Union. In 1962, Victoriano tendered his resignation from EPWU claiming that as per RA 3350 he is an exemption to the close shop agreement by virtue of his being a member of the INC because apparently in the INC, one is forbidden from being a member of any labor union. It was only in 1974 that his resignation from the Union was acted upon by EPWU which notified ERF about it. ERF then moved to terminate Victoriano due to his non-membership from the EPWU. EPWU and ERF reiterated that he is not exempt from the close shop agreement because RA 3350, which provides that close shop agreements shall not cover members of any religious sects which prohibit affiliation of their members in any such labor organization, is unconstitutional and that said law violates the EPWU’s and ERF’s legal/contractual rights. ISSUE: Whether or not RA 3350 is unconstitutional. HELD: No. The right to religion prevails over contractual or legal rights. As such, an INC member may refuse to join a labor union and despite the fact that there is a close shop agreement in the factory where he was employed, his employment could not be validly terminated for his non-membership in the majority therein. Further, the right to join a union includes the right not to join a union. The law is not unconstitutional. It recognizes both the rights of unions and employers to enforce terms of contracts and at the same time it recognizes the workers’ right to join or not to join union. RA 3550 recognizes as well the primacy of a constitutional right over a contractual right. People vs. Purisima, No. L -47757-61, January 28, 1980 CONSTITUTIONAL LAW 2 CASE DIGESTS

Facts: Informations were filed to 26 individuals from Manila and Samar, individually and separately, before the Courts of First Instance of Manila and Samar for illegal possession of deadly weapon or violation of Presidential Decree No. 9 pursuant to Proclamation No. 1081 dated Sept 21 and 23, 1973. On the motion to quash by the accused, the three respondent judges: Judge Purisima and Judge Macaren, both of CFI of Manila; and Judge Polo of CFI of Samar, issued in the respective cases filed before them an order to quash or dismiss the informations on a common ground – Lack of essential elements to constitute an offense penalized by PD No. 9. The respondent judges stated that to constitute the said offense, two elements must be present; (1) possession of any bladed, blunt or pointed weapon outside of residence as stated in par 3; (2) and intended to use it to commit or abet subversion, rebellion, etc as stated in the preamble of the said PD. The People, as petitioners, thru the Solicitor General, contended that the prohibited acts need not be related to subversive activities and the intent of the accused are irrelevant since its is a statutory offense and punishing the possession of such deadly weapon is not only to eradicate subversive acts but also criminality in general. The petitioners also argued that the preamble is not an essential part of an act and cannot prevail over the text of the law itself. ISSUE: Whether or not the petitioners’ arguments as to the intention and scope of PD No. 9 (3) correct? HELD: NO. The Supreme Court says that the intention of PD No. 9 (3) is to penalize the acts which are those related to the desired result of Proc. No. 1081 and Gen. Orders Nos. 6 and 7 which are to suppress those who commit or abet lawlessness, rebellion, subversive acts and the like. The preamble of PD No. 9 also clearly concurs to that, though the preamble is not a part of the statute, it is the key to determine what is the intent and spirit of the decree and determine what acts fall within the purview of a penal statute.

CONSTITUTIONAL LAW 2 CASE DIGESTS

Bel Air Village Association vs Virgilio Dionisio Facts: The Transfer Certificate of Title covering the subject parcel of land issued in the name of Virgilio Dionisio, the petitioner contains an annotation to the effect that the lot owner becomes an automatic member of Bel-Air Village Association, the respondent, and must abide by such rules and regulations laid down by the Association in the interest of the sanitation, security and the general welfare of the community. The petitioner questioned the collection of the dues on the following grounds: the questioned assessment is a property tax outside the corporate power of the association; the association has no power to compel the petitioner to pay the assessment for lack of privity of contract; the questioned assessment should not be enforced for being unreasonable, arbitrary, oppressive, confiscatory and discriminatory; the respondent association is exercising governmental powers which should not be sanctioned. Issue: Whether or not the association can lawfully collect dues Ruling: The Supreme Court dismissed the petition for lack of merit. It held that the purchasers of a registered land are bound by the annotations found at the back of the certificate of title covering the subject parcel of land. The petitioner’s contention that he has no privity with the respondent association is not persuasive. When the petitioner voluntarily bought the subject parcel of land it was understood that he took the same free of all ecumbrances except annotations at the back of the certificate of title, among them, that he automatically becomes a member of the respondent association. One of the obligations of a member is to pay certain amounts for the operation and activities of the association. The mode of payment as well as the purposes for which the dues are intended clearly indicates that the dues are not in the concept of a property tax as claimed by the petitioner. They are shares in the common expenses for necessary services. A property tax is assessed according to the value of the property but the basis of the sharing in this case is the area of the lot. The dues are fees which a member of the respondent association is required in hiring security guards, CONSTITUTIONAL LAW 2 CASE DIGESTS

cleaning and maintaining streets, street lights and other community projects for the benefit of all residents within the Bel-Air Village. These expenses are necessary, valid and reasonable for the particular community involved. The limitations upon the ownership of the petitioner do not contravene provisions of laws, morals, good customs, public order or public policy. The constitutional proscription than no person can be compelled to be a member of an association against his will applies only to governmental acts and not to private transactions like the one in question. The petitioner cannot legally maintain that he is compelled to be a member of the association against his will because the limitation is imposed upon his ownership of property. If he does not desire to comply with the annotation or lien in question, he can at any time exercise his inviolable freedom of disposing of the property and free himself from the burden of becoming a member of the association. Aglipay vs. Ruiz Facts: In May 1936, the Director of Posts announced in the dailies of Manila that he would order the issuance of postage stamps commemorating the celebration in the City of Manila of the 33rd International Eucharistic Congress, organized by the Roman Catholic Church. 2. The petitioner, Mons. Gregorio Aglipay, Supreme Head of the Philippine Independent Church, in the fulfilment of what he considers to be a civic duty, requested Vicente Sotto, a member of the Philippine Bar, to denounce the matter to the President. In spite of the protest of the petitioner’s attorney, the Director of Posts publicly announced having sent to the United States the designs of the postage for printing. The said stamps were actually issued and sold though the greater part remained unsold. 3. The further sale was sought to be prevented by the petitioner. He alleged that the provisions of Section 23, Subsection 3, Article VI, of the Constitution were violated in the issuance and selling of the commemorative postage stamps. It was provided therein that, ‘No public money or property shall ever be appropriated, applied, or used, directly or indirectly, for the use, benefit, or support of any sect, church, denomination, sectarian, institution, or system of religion, or for the use, CONSTITUTIONAL LAW 2 CASE DIGESTS

benefit, or support of any priest, preacher, minister, or other religious teacher or dignitary as such, except when such priest, preacher, minister, or dignitary is assigned to the armed forces or to any penal institution, orphanage, or leprosarium.’ Issue: Whether or not the issuance of stamps was in violation of the principle of separation of church and state Held: NO. 1. Religious freedom, as a constitutional mandate, is not inhibition of profound reverence for religion and is not denial of its influence in human affairs. Religion as a profession of faith to an active power that binds and elevates man to his Creator is recognized. In so far as it instils into the minds the purest principles of morality, its influence is deeply felt and highly appreciated. 2. When the Filipino people, in the preamble of the Constitution, implored "the aid of Divine Providence, in order to establish a government that shall embody their ideals, conserve and develop the patrimony of the nation, promote the general welfare, and secure to themselves and their posterity the blessings of independence under a regime of justice, liberty and democracy," they thereby manifested reliance upon Him who guides the destinies of men and nations. The elevating influence of religion in human society is recognized here as elsewhere. In fact, certain general concessions are indiscriminately accorded to religious sects and denominations. 3. There has been no constitutional infraction in this case. Act No. 4052 granted the Director of Posts, with the approval of the Sec. of Public Works and Communications, discretion to issue postage stamps with new designs. Even if we were to assume that these officials made use of a poor judgment in issuing and selling the postage stamps in question, still, the case of the petitioner would fail to take in weight. Between the exercise of a poor judgment and the unconstitutionality of the step taken, a gap exists which is yet to be filled to justify the court in setting aside the official act assailed as coming within a constitutional inhibition. The court resolved to deny the petition for a writ of prohibition. CONSTITUTIONAL LAW 2 CASE DIGESTS

Islamic Da’wah Council of the Philippines, Inc. v. Office of the Executive Secretary FACTS: The IDCP is a non-government corporation that operates under DSWD and renders voluntary services to Filipinos especially in the muslim community. It is also a member of a regional group RISEAP which accredited IDCP to create halal in the Philippines, conduct seminars, orient the manufacturers and issue halal certifications. The petitioner (IDCP-Islamic Da’wah Council of the Philippines Inc.) seek the Supreme Court to declare EO 46, s. 2001unconstitutional and also prays for the prohibition of the respondents of the Office of the Secretary and OMA (Office on Muslim Affairs) to implement such EO on the following grounds that 1. The EO vilolates the separation of church and state doctrine-According to IDCP, they created policies for the halal certification based on their Qur’an and Sunnah and that the product only becomes halal after a religious ritual is performed. The respondents, being a government and not a religious entity could not perform such religious act. 2. The EO violates sec 10 Article 3 of the 1987 Constitution-which impaired their contracts with other parties because after the EO was implemented and among the articles or letters that circulated indicating food manufacturers would violate the EO if they do not get the certification from the respondents, the parties where the petitioners were in contract with stopped getting certifications from them. 3. The EO violates sec 15 and 16 of Article XIII of the 1987 Constitution The respondents however claim that police power towers over freedom of religion and that the act done by the respondent serves to protect public health –for the Filipino Muslim community and even non -muslims who consume halal; The respondents also stated that the lack of a central administrative body may pave way to for schemers in the market who may certify products as halal even if they aren’t just for profit. ISSUE: Whether or not EO 46 is unconstitutional HELD: The Supreme Court declares the EO as null and void and also prohibits the respondents to enforce it. The petition is granted.

CONSTITUTIONAL LAW 2 CASE DIGESTS

Only when there is an imminent threat to the society or the Muslim community can police power impede the exercise of religious freedom. The respondents transgressed the free exercise of religion therefore it encroached on the Muslim’s rights. With regards to the protection of the Muslim community for safe consumption of halal and the protection from deceitful or unscrupulous business practices, there are several governmental agencies like the DOA, DOH and DTI which serve as regulatory bodies to ensure and enforce policies for safe consumption of products, as well as protect consumers from unfair business practices.

Garces Vs Estenzo

Facts: This case is about the constitutionality of four resolutions of the barangay council of Valencia, Ormoc City, regarding the acquisition of the wooden image of San Vicente Ferrer to be used in the celebration of his annual feast day. On March 23, 1976, the said barangay council adopted Resolution No. 5, "reviving the traditional socio-religious celebration" every fifth day of April "of the feast day of Señor San Vicente Ferrer, the patron saint of Valencia That resolution designated the members of nine committees who would take charge of the 1976 festivity. lt provided for (1) the acquisition of the image of San Vicente Ferrer and (2) the construction of a waiting shed as the barangay's projects. Funds for the two projects would be obtained through the selling of tickets and cash donations On March 26, 1976, the barangay council passed Resolution No. 6 which specified that, in accordance with the practice in Eastern Leyte, Councilman Tomas Cabatingan, the Chairman or hermano mayor of the fiesta, would be the caretaker of the image of San Vicente Ferrer and that the image would remain in his residence for one year and until the election of his successor as chairman of the next feast day Funds were raised by means of solicitations0 and cash donations of the barangay residents and those of the neighboring places of Valencia. With those funds, the waiting shed was constructed and the wooden image of San Vicente Ferrer was acquired in Cebu City by the barangay council for four hundred pesos A controversy arose after the mass when the parish priest, Father CONSTITUTIONAL LAW 2 CASE DIGESTS

Sergio Marilao Osmeña refused to return that image to the barangay council on the pretext that it was the property of the church because church funds were used for its acquisition. Because Father Osmeña did not accede to the request of Cabatingan to have custody of the image and "maliciously ignored" the council's Resolution No. 6, the council enacted on May 12, 1976 Resolution No. 10, authorizing the hiring of a lawyer to file a replevin case against Father Osmeña for the recovery of the image (Exh. C or 8). On June 14, 1976, the barangay council passed Resolution No. 12, appointing Veloso as its representative in the replevin case (Exh. D or 9). Later, he and three other persons, Andres Garces, a member of the Aglipayan Church, and two Catholic laymen, Jesus Edullantes and Nicetas Dagar, filed against the barangay council and its members (excluding two members) a complaint in the Court of First Instance at Ormoc City, praying for the annulment of the said resolutions. The lower court dismissed the complaint. lt upheld the validity of the resolutions ISSUE: WON The Barangay Council has the right over the custody of the Relic HELD : The questioned resolutions do not directly or indirectly establish any religion, nor abridge religious liberty, nor appropriate public money or property for the benefit of any sect, priest or clergyman. The image was purchased with private funds, not with tax money. The construction of a waiting shed is entirely a secular matter The wooden image was purchased in connection with the celebration of the barrio fiesta honoring the patron saint, San Vicente Ferrer, and not for the purpose of favoring any religion nor interfering with religious matters or the religious beliefs of the barrio residents. One of the highlights of the fiesta was the mass. Consequently, the image of the patron saint had to be placed in the church when the mass was celebrated If there is nothing unconstitutional or illegal in holding a fiesta and having a patron saint for the barrio, then any activity intended to facilitate the worship of the patron saint (such as the acquisition and display of his image) cannot be branded as illegal. The barangay council designated a layman as the custodian of the wooden image in order to forestall any suspicion that it is favoring the Catholic church. There can be no question that the image in question belongs to the barangay council. The council has the right to take measures to recover possession of the image by enacting Resolutions Nos. 10 and 12. Not every governmental activity which involves the expenditure of public funds and which has some religious tint is violative of the constitutional provisions CONSTITUTIONAL LAW 2 CASE DIGESTS

regarding separation of church and state, freedom of worship and banning the use of public money or property

People vs. Aminnudin FACTS: The PC (Philippine Constabulary) officer received a tip from one of their informers that the accused was on board a vessel bound for Iloilo City and was carrying marijuana. He was identified by name. Acting on this tip, they waited for him in the evening and approached him as he descended from the gangplank after the informer pointed at him. They detained him and inspected the bag he was carrying. It was found to contained three kilos of what were later analyzed as marijuana leaves by the NBI forensic examiner. On the basis of the finding, the corresponding charge was then filed against Aminnudin. ISSUE: Whether or not accused constitutional right against unreasonable search and seizure is violated. HELD: The Supreme Court Held that warrantless arrest allowed under Rule 113 of the rules of court not justified unless the accused was caught in flagrante or a crime was about to be committed or had just been committed. A vessels and aircraft are subject to warrantless searches and seizures for violation of the customs law because these vehicles may be quickly moved out of the locality or jurisdiction before the warrant can be secured. In the present case, from the conflicting declarations of the PC witnesses, it is clear that they had at least two days within which they could have obtained a warrant to arrest and search Aminnudin who was coming to Iloilo on the M/V Wilcon 9. His name was known. The vehicle was identified. The date of his arrival was certain. And from the information they have received, they could have persuaded a judge that there was a probable cause, indeed, to justify the issuance of a warrant. Yet they did nothing. The Bill of Rights was ignored altogether because the PC lieutenant who was the head of the arresting team had determine on his own authority that a search warrant was not necessary.

CONSTITUTIONAL LAW 2 CASE DIGESTS

The evidence of probable cause should be determined by a judge and not law enforcement agents. People vs. Libnao, GR 136860, 20 January 2003 FACTS: On August 1996, intelligence operatives of the Philippine National Police (PNP) stationed in Tarlac, Tarlac began conducting surveillance operation on suspected drug dealers in the area. They learned from their asset that a certain woman from Tajiri, Tarlac and a companion from Baguio City were transporting illegal drugs once a month in big bulks. On 19 October 1996, at about 10 p.m., Chief Inspector Benjamin Arceo, Tarlac Police Chief, held a briefing in connection with a tip which his office received that the two drug pushers, riding in a tricycle, would be making a delivery that night. An hour later, the Police Alert Team installed a checkpoint in Barangay Salapungan to apprehend the suspects. Witness SPO1 Marlon Gamotea, PO3 Florante Ferrer and SPO3 Roberto Aquino were assigned to man the checkpoint. At about 1:00 a.m. of the following day, SPO1 Gamotea and PO3 Ferrer flagged down a passing tricycle. It had two female passengers seated inside, who were later identified as Agpanga Libnao and Rosita Nunga. In front of them was a black bag. Suspicious of the black bag and the two’s uneasy behavior when asked about its ownership and content, the officers invited them to Kabayan Center 2 located at the same barangay. They brought with them the black bag. Upon reaching the center, PO3 Ferrer fetched Barangay Captain Roy Pascual to witness the opening of the black bag. In the meantime, the two women and the bag were turned over to the investigator on duty, SPO3 Arthur Antonio. As soon as the barangay captain arrived, the black bag was opened in the presence of Libnao, Nunga, and personnel of the center. Found inside it were 8 bricks of leaves sealed in plastic bags and covered with newspaper. The leaves were suspected to be marijuana. To determine who owns the bag and its contents, SPO3 Antonio interrogated the two. Nunga stated that it was owned by Libnao. The latter, in turn, disputed this allegation. Thereafter, they were made to sign a confiscation receipt without the assistance of any counsel, as they were not informed of their right to have one. During the course of the investigation, not even close relatives of theirs were present. The seized articles were later brought to the PNP Crime Laboratory in San Fernando, Pampanga on 23 October 1996. Forensic Chemist Daisy P. Babu conducted a laboratory examination on them. She concluded that the articles were marijuana leaves weighing eight kilos. Libnao and Nunga were CONSTITUTIONAL LAW 2 CASE DIGESTS

charged for violation of Section 4, Article II of RA 6425, otherwise known as the Dangerous Drugs Act of 1972, as amended. On 19 November 1998, the Regional Trial Court, Branch 65, Tarlac City, found Libnao and Nunga guilty. For their conviction, each was sentenced to suffer an imprisonment of reclusion perpetua and to pay a fine of two million pesos. Libnao appealed. ISSUE: Whether the warrantless search and seizure made upon Libnao and Nunga was reasonable. HELD: The constitutional guarantee (in Article III, Section 2 of the 1987 Constitution) is not a blanket prohibition against all searches and seizures as it operates only against "unreasonable" searches and seizures. Searches and seizures are as a rule unreasonable unless authorized by a validly issued search warrant or warrant of arrest. Thus, the fundamental protection accorded by the search and seizure clause is that between persons and police must stand the protective authority of a magistrate clothed with power to issue or refuse to issue search warrants and warrants of arrest. Be that as it may, the requirement that a judicial warrant must be obtained prior to the carrying out of a search and seizure is not absolute. There are certain familiar exceptions to the rule, one of which relates to search of moving vehicles. Warrantless search and seizure of moving vehicles are allowed in recognition of the impracticability of securing a warrant under said circumstances as the vehicle can be quickly moved out of the locality or jurisdiction in which the warrant may be sought. Peace officers in such cases, however, are limited to routine checks where the examination of the vehicle is limited to visual inspection. When a vehicle is stopped and subjected to an extensive search, such would be constitutionally permissible only if the officers made it upon probable cause, i.e., upon a belief, reasonably arising out of circumstances known to the seizing officer, that an automobile or other vehicle contains as item, article or object which by law is subject to seizure and destruction. The warrantless search herein is not bereft of a probable cause. The Tarlac Police Intelligence Division had been conducting surveillance operation for three months in the area. The surveillance yielded the information that once a month, Libnao and Nunga transport drugs in big bulks. At 10:00 pm of 19 October 1996, the police received a tip that the two will be transporting drugs that night riding a tricycle. CONSTITUTIONAL LAW 2 CASE DIGESTS

Surely, the two were intercepted three hours later, riding a tricycle and carrying a suspicious-looking black bag, which possibly contained the drugs in bulk. When they were asked who owned it and what its content was, both became uneasy. Under these circumstances, the warrantless search and seizure of Libnao’s bag was not illegal. It is also clear that at the time she was apprehended, she was committing a criminal offense. She was making a delivery or transporting prohibited drugs in violation of Article II, Section 4 of R.A. No. 6425. Under the Rules of Court, one of the instances a police officer is permitted to carry out a warrantless arrest is when the person to be arrested is caught committing a crime in flagrante delicto. PP vs. Nuevas, GR No. 170233, 22 February 2007 FACTS: Police officers Fami and Cabling, during a stationary surveillance and monitoring of illegal drug trafficking in Olongapo City, came across Jesus Nuevas, who they suspected to be carrying drugs. Upon inquiry, Nuevas showed them a plastic bag which contained marijuana leaves and bricks wrapped in a blue cloth. He then informed the officers of 2 other persons who would be making marijuana deliveries. The police officers then proceeded to where Nuevas said his associates, Reynaldo Din and Fernando Inocencio, could be located. Din was carrying a plastic bag which contained marijuana packed in newspaper and wrapped therein. When the police officers introduced themselves, Din voluntarily handed the plastic bag over to them. After the items were confiscated, the police officers took the three men to the police office. Police officer Fami then revealed that when the receipt of the evidence was prepared, all 3 accused were not represented by counsel. He likewise disclosed that he was the one who escorted all the accused during their physical examination. He also escorted all 3 to the Fiscal’s office where they were informed of the charges against them. The 3 were found guilty by the trial court, and the case was automatically elevated to the CA for review. However, Nuevas withdrew his appeal. Thus, the case was considered closed and terminated as to him. The CA affirmed the trial court. CONSTITUTIONAL LAW 2 CASE DIGESTS

ISSUE: Whether or not Din and Inocencio waived their right against unreasonable searches and seizures. HELD NO. The search conducted in Nuevas’ case was made with his consent. However, in Din’s case, there was none. There is reason to believe that Nuevas indeed willingly submitted the plastic bag with the incriminating contents to the police officers. It can be seen that in his desperate attempt to exculpate himself from any criminal liability, he cooperated with the police, gave them the plastic bag, and even revealed his associates, offering himself as an informant. His actuations were consistent with the lamentable human inclination to find excuses, blame others, and save oneself even at the cost of others’ lives. Thus, the Court would have affirmed Nuevas’ conviction had he not withdrawn his appeal. On the other hand, with respect to the search conducted in the case of Din, the Court finds that no such consent had actually been given. The police officers gave inconsistent, dissimilar testimonies regarding the manner by which they got hold of the plastic bag. Neither can Din’s silence at the time be construed as an implied acquiescence to the warrantless search. Thus, the prosecution failed to clearly show that Din intentionally surrendered his right against unreasonable searches. On the other hand, Inocencio’s supposed possession of the dried marijuana leaves was sought to be shown through his act of looking into the plastic bag that Din was carrying. The act attributed to Inocencio is insufficient to establish illegal possession of the drugs or even conspiracy to illegally possess the same. The prosecution failed to show by convincing proof that Inocencio knew of the contents of the bag and that he conspired with Din to possess the illegal items. PP vs Tiu Won Chua, 405 scra 280, 2003 FACTS: Accused Tiu Won Chua a.k.a. Timothy Tiu and Qui Yaling y Chua a.k.a. Sun Tee Sy y Chua were charged and convicted by the lower court for violation of Section 16, Article III of the dangerous drug act of 1972, for their illegal possession of a regulated drug, shabu. They appealed the decision of the lower court questioning the legality of the search warrant and the search and arrest conducted pursuant thereto, and the correctness of the judgment of conviction imposed by the RTC. Accused contends that the defect in the issuance of the search warrant, for it was issued in the name of Timothy Tiu and did not include appellant Qui Yaling, would make the search CONSTITUTIONAL LAW 2 CASE DIGESTS

conducted and consequently, the arrest, illegal. That the evidence presented cannot be serve as basis for their conviction being fruits of an illegal search. ISSUE: (1) Whether or not their was a valid search warrant. (2) Whether or not the court correctly imposed judgment of conviction to the accused. HELD: (1) There are only four requisites for a valid warrant, i.e,: (a) it must be issued upon "probable cause"; (b) probable cause must be determined personally by the judge; (c) such judge must examine under oath or affirmation the complainant and the witnesses he may produce; and (d) the warrant must particularly describe the place to be searched and the persons or things to be seized. A mistake in the name of the person to be searched does not invalidate the warrant, especially since in this case, the authorities had personal knowledge of the drug-related activities of the accused by virtue of the surveillance and test-buy operations of the said authorities. In fact, a "John Doe" warrant satisfies the requirements so long as it contains a descriptio personae such as will enable the officer to identify the accused. Moreover, a mistake in the identification of the owner of the place does not invalidate the warrant provided the place to be searched is properly described. However, the court affirms the illegality of the search conducted on the car for it was not part of the description of the place to be searched mentioned in the warrant. (2) In a prosecution for illegal possession of a dangerous drug, it must be shown that (a) appellants were in possession of an item or an object identified to be a prohibited or regulated drug, (b) such possession is not authorized by law, and (c) the appellants were freely and consciously aware of being in possession of the drug. Since the crime is malum prohibitum, hence, lack of criminal intent or good faith does not exempt appellants from criminal liability. Mere possession of a regulated drug without legal authority is punishable under the Dangerous Drugs Act. In the case at bar, the prosecution sufficiently proved that the packs of shabu were found inside a room in the unit subject of the search warrant, more particularly inside the man’s handbag and ladies’ handbag owned respectively by the accused. As to the penalties imposed, the court did not sustain the trial court’s decision attributing to both appellants the illegal possession of the same amount of shabu. Since no conspiracy was proven, the amount of the shabu from each accused was made the basis of the penalty imposed. Thus, since 234.5 grams CONSTITUTIONAL LAW 2 CASE DIGESTS

of shabu were found inside the man’s handbag, deemed to be owned by Tiu Won, he is guilty of violating Section 16, Article III of R.A. No. 6425, while Qui Yaling, whose handbag contained only 20.3673 grams of shabu is guilty of violating Section 20 thereof. Section 16, in connection with Section 20 (1 st paragraph), provides the penalty of reclusion perpetua to death and a fine ranging from five hundred thousand pesos to ten million pesos where the amount of shabu involved is 200 grams or more. Where the amount is less than 200 grams, Section 20 punishes the offender with the penalty ranging from prision correccional to reclusion perpetua. Padilla vs. CA, G.R. No. 123893, November 22, 2001 FACTS: Respondent Susana Realty, Inc. (SRI), by a deed of absolute sale, sold to the Light Rail Transit Authority (LRTA) several parcels of land located in Taft Avenue Extension, San Rafael District, Pasay City. Under the deed of sale, SRI reserved to itself the right of first refusal to develop and/or improve the property should the LRTA decide to lease and/or assign to any person the right to develop and/or improve the property. Eventually, LRTA and petitioner Phoenix-Omega Development (Phoenix) entered into a Commercial Stall Concession Contract, authorizing the latter to construct and develop commercial stalls on a 90 sq. m. portion of the property bought from SRI. Phoenix, by a deed of assignment, assigned its rights and interests over the remaining property to its sister company, PKA Development and Management Corporation (PKA). Several contracts were executed between and among SRI, PKA and Phoenix, with the sisters company PKA and Phoenix being represented both by Padilla. When the terms of lease were breached, SRI obtained an ejectment order from the court with awards of damage against PKA. When no property could be found to enforce the judgment against PKA, an alias writ was obtained against Phoenix and Padilla which contends that the sister companies are merely an aggregate of persons under one business enterprise, hence, the corporate fiction should be pierced. Hence, this petition. ISSUE: Whether or not PKA and Phoenix committed fraud to justify the piercing of corporate veil. CONSTITUTIONAL LAW 2 CASE DIGESTS

HELD: While both the Regional Trial Court and the Court of Appeals found sufficient basis for the conclusion that PKA and Phoenix were one and the same, and the former is merely a conduit of the other; still the High Court does not consider the application of a writ of execution on a judgment held only against PKA. The general rule is that a corporate is clothed with a personality separate and distinct from the persons composing it. The veil of corporate fiction may only be disregarded in cases where the corporate vehicle is being used to defeat public convenience, justify wrong, protect fraud or defend crime. PKA and Phoenix are admittedly sister companies, and may be sharing personnel and resources, but the Court still find no allegations or proof that their separate corporate personalities are being used to defeat public convenience justify wrong, protect fraud or defend crime. People vs. De Gracia FACTS: Reform the Armed Forces Movement-Soldiers of the Filipino People (RAMSFP) staged coup d’état in December 1989 against the Government. 2. Efren Soria of Intelligence Division, NCR Defense Command, together with his team, conducted a surveillance of the Eurocar Sales Office in EDSA, QC on early morning of December 1, 1989, which surveillance actually started November 30, 1989 at around 10:00 PM. Such surveillance was conducted pursuant to an intelligence report that the said establishment was being occupied by the elements of the RAM-SFP as communication command post. Near the Eurocar office, there were crowd watching the on-going bombardment near Camp Aguinaldo when a group of 5 men disengaged themselves and walked towards their surveillance car. Maj. Soria ordered the driver to start the car and leave the area. However, as they passed the area, then 5 men drew their guns and fired at them, which resulted to the wounding of the driver. Nobody in the surveillance team retaliated for they were afraid that civilians might be caught in the crossfire. Thereafter, on the morning of December 5, 1989, a search team raided the Eurocar Sales Office and confiscated 6 cartons of M-16 ammunition, 5 bundles of C-4 dynamites, M-shells of different calibers, and molotov. Obenia, who first entered the establishment, found De Gracia in the office of a certain Col. Matillano, holding a C-4 and suspiciously peeping through door. No search warrant was secured by the raiding team because, according to them, there was so much disorder considering that the nearby Camp Aguinaldo was being mopped up by the rebel forces and there CONSTITUTIONAL LAW 2 CASE DIGESTS

was simultaneous firing within the vicinity of the Eurocar office, aside from the fact that the courts were consequently closed. ISSUE: Whether there was a valid search and seizure in this case. HELD: YES, there was a valid search and seizure in this case. It is admitted that the raiding team was not armed with a search warrant at that time. It was actually precipitated by intelligence reports that said office was being used as headquarters by the RAM. Prior to the raid, there was a surveillance conducted on the premises wherein the surveillance team was fired at by a group of men coming from the Eurocar building. When the military operatives raided the place, the occupants thereof refused to open the door despite requests for them to do so, thereby compelling the former to break into the office. The Eurocar Sales Office is obviously not a gun store and it is definitely not an armory or arsenal which are the usual depositories for explosives and ammunition. It is primarily and solely engaged in the sale of automobiles. The presence of an unusual quantity of high-powered firearms and explosives could not be justifiably or even colorably explained. In addition, there was general chaos and disorder at that time because of simultaneous and intense firing within the vicinity of the office and in the nearby Camp Aguinaldo which was under attack by rebel forces. The courts in the surrounding areas were obviously closed and, for that matter, the building and houses therein were deserted. Under circumstances, SC considered that the instant case falls under one of the exceptions to the prohibition against a warrantless search. In the first place, the military operatives, taking into account the facts obtaining in this case, had reasonable ground to believe that a crime was being committed. There was consequently more than sufficient probable cause to warrant their action. Furthermore, in the prevailing situation, the raiding team had no opportunity to apply for and secure a search warrant from the courts. The trial judge himself manifested that on December 5, 1989 when the raid was conducted, his court was closed. Under such urgency and exigency of the moment, a search warrant could lawfully be dispensed with.

CONSTITUTIONAL LAW 2 CASE DIGESTS

CONSTITUTIONAL LAW 2 CASE DIGESTS

Related Documents


More Documents from "Katherine Kahlila Raagas"

Constitutional Law 2 Cases
February 2021 12
January 2021 1
El Gran Libro De Ifa
February 2021 2
La Desgracia
February 2021 1